Download as pdf or txt
Download as pdf or txt
You are on page 1of 343

College Panda

The College

SAT
SAT Math
Advanced Guide
Advanced Guide and Workbook
and Workbook
2nd Edition
2nd Edition

Copyright © 2020 The


Copyright College Panda
The College Panda
All rights reserved
A l l rights reserved..

978-1-7331927-2-9
ISBN: 978-1-7331927-2-9

part of this
No part book may
this book be reproduced
may be reproduced without
without written permission from the
written permission the author.
author.

‘SAT is aa registered
*SAT registered trademark
trademark of the College Board,
the College which does
Board, which n o t endorse
does not product.
endorse this product.

For more information, visit


m o r e information, thecollegepanda.com
visit thecollegepanda.com

Discounts available for teachers


Discounts available companies. Please
and companies.
teachers and Please contact thecollegepanda®gmail.com for details.
contact thecollegepanda@gmail.com details.

/
Introduction
Introduction
best way
The best
The to do
way to do well on any
well on any test is to
test is to be
be experienced
experienced with
with the material. Nowhere
the material. Nowhere is is this
this more than on
true than
more true on
the which is standardized
SAT, which
the SAT, standardized toto repeat
repeat the
the same question types
same question types again again. The
and again.
again and purpose of this
The purpose book is
this book
to teach you the
teach you concepts and
the concepts and battle-tested approaches you
battle‐tested approaches need to know
you need know for all these
these questions types.. If
questions types it's
If it's
n in this
o t in
not book, it's
this book, it’s not on the
not on the test. goal is
The goal
test. The is for every question to be
every SAT question be a simple reflex,
a simple reflex, something
something youyou
know
know how to handle
how to instinctively because
handle instinctively because you've seen it so
you’ve seen so many
many times before.
times before.
won’t find any
You won't cheap tricks
any cheap in this
tricks in this book, simply because
book, simply because there aren't any
there aren’t any that consistently. Don't
work consistently.
that work Don’t buy
buy
into idea that
the idea
into the you can
that you improve your
can improve your score significantly without
score significantly without hard work..
hard work

Format of the Test


Format
There are
There t w o math
are two on the
sections on
math sections the SAT. The contains 20 questions
The first contains questions to be
be done 25 minutes
done in 25 without aa
minutes without
calculator. The
calculator. second contains
The second contains 38 questions to be
38 questions be done
done in 55
55 minutes and a
minutes and permitted..
calculator is permitted
a calculator
Some topics
Some only show
topics only up in the
show up calculator section.
the calculator I’ve made
section. I've sure to accurately
made sure the practice
divide the
accurately divide questions
practice questions
non-calculator and
into non-calculator
into and calculator components.
calculator components.

Read this
How to Read
How Book
this Book
For complete understanding,
For aa complete understanding, this book is best
this book best read from beginning
read from beginning to end.
end. That being said,
That being each chapter
said, each chapter was
was
be independent
written to be
written independent of the others as much
others as much asaspossible. After all,
possible. After all, you may already
you may already be proficient in some
be proficient some
topics weak in others.
topics yet weak to jump
others. If so, feel free to jump around, focusing on
around, focusing the chapters
on the that are
chapters that are most relevant to
most relevant
improvement.
your improvement.
your
chapters come
All chapters with exercises.
come with them.. You won’t
exercises. Do them won't master the material
master the until you
material until think through
you think the
through the
questions yourself.
que stions yourself. If you
you get stuck on a question, give yourself
stuck on a question, give yourself a a few minutes
minutes to figure it out. If you’re
figure out. If you're
still stuck,
still stuck, then
then look at the
look at solution and
the solution take the
and take time to fully understand
the time Then circle
understand it. Then the question
circle the number
question number
or make
make aa note somewhere so
note of it somewhere so that
that you
you can the question
redo the
can redo question later. Revisiting questions
later. Revisiting questions you’ve missed is
you've missed
best way
the best improve your
way to improve your score.
score.

the Author
About the
About
Nielson Phu
Nielson graduated from New
Phu graduated University, where
New York University, where hehe studied
studied actuarial He has
science. He
actuarial science. perfect
obtained perfect
has obtained
scores on the SAT and
scores and on the subject test.
math subject
the SAT math test. As aa teacher, he has
teacher, he helped hundreds
has helped students
hundreds of students
throughout Boston
throughout Boston and Hong Kong
and Hong perform better
Kong perform better on
on standardized
standardized tests. Although he
tests. Although he continues pursue
continues to pursue
his interests
his education, he
interests in education, he is now
n o w an engineer in the
an engineer Boston area.
the Boston area.
THE COLLEGE PANDA
COLLEGE PAND A

Contents
Table of Contents
1 Exponents & Radicals
Exponents Radicals 7 .r

exponents
Laws of exponents
Laws
Evaluating
Evalua expressions with
ting expressions exponents
with expo nen ts
Solving equations with exponents
Solving equations with exponents
Simplifying square roots
Simplifying square roots

2 Percent
Percent 15
15
change
Percent change
Percent
Compound interest
Compound interest
Percent word problems
Percent word problems

33 Exponential vs
Exponential Linear Growth
vs.. Linear Growth 23
Linear growth and
Linear growth decay
and decay
Exponentiall growth
Exponentia growth and decay
and decay
Positive negative associa
and negative
Positive and association
tion

4 Rates
Rates 32
factors
Conversion factors
Conversion

5 Ratio Proportion
Ratio & Proportion 38
38
6
6 Expressions
Expressions 44
44
Combining like terms
Combining like terms
Expansion
Expans and factoring
ion and factoring
Combining, dividing
Combining, dividing,, and splitting fractions
and splitting fractions

7 Constructing Models
Constructing Models 52
8 Manipulating &
Manipulating & Solving Equations
Solving Equations 57
Common mistakes to avoid
Common mistakes avoid
isolating variables
Tools for isolating variables
Strategies for solving
Strategies complicated equations
solving complicated equations

9 Equation Solving
More Equation
More Solving Strategies
Strategies 71
Matching coefficients
Matching coefficients
lnfinitely many
Infinitely many solutions
solutions
solutions
No soluti ons
Clearing denominators
Clearing denominators

10
10 of Equations
Systems of
Systems Equations 78
Substitution
Substitution
Elimination
Elimination
Systems with
Systems solutions and
with no solutions and infinite solutions
infinite solutions
problems
Word problems
More complex
More comp systems
lex systems
Graphs of systems
Graphs equations
systems of equations

4
THE COLLEGE
COLLEGE PANDA
PANDA

11 Inequalities
Inequalities 91
H o w to solve
How solve inequalities
inequalities
Inequality word problems
Inequality word problems
Graphs of inequalities
Graphs inequalities

12 Word Problems
Problems 100
13
13 M i n i m u m 8:
Minimum Maximum Word Problems
& Maximum Problems 109
14
14 Lines
Lines 117
Slope and y-intercept
Slope and y-intercept
Equations of lines:
Equations slope-intercept form
lines: slope-intercept and point-slope
form and form
point-slope form
Finding
Finding the
the intersection
intersection of two lines
t w o lines
Parallel and perpendicular
Parallel and perpendicular lineslines
Horizontal
Horizontal and
and vertical lines
vertica l lines

15
15 Interpreting
Interpreting Linear Models
Linear Models 126
16 Functions
Functions 132
132
What is a
What a function?
function?
When is a function
When function undefined?
undefined?
Composite
Composite functions
functions
Finding
Finding the solutions to a
the solutions function
a function
Identifying function graphs
Identifying function graphs
Function transformations
Function transformations

17 Quadratics
Quadratics 146
146
Tactics for finding
finding the roots
roots
Completing
Completing the
the square
square
The vertex
ver tex and vertex form
and vertex form
The discriminant
The discriminant
Quadratic models
Quadratic models

18
18 Synthetic
Synthetic Division
Division 161
Performing synthetic division
Performing synthetic division
Equivalent expressions
Equivalent expressions
The remainder
remainder theorem
theorem

19 Complex
Complex Numbers
Numbers 170
170
20 Absolute Value
Absolute Value 174
21
21 Angles
Angles 180
180
Exterior angle
Exterior angle theorem
theorem
Parallel
Parallel lines
lines
Polygons
Polygons

5
THE COLLEGE PANDA

22 Triangles 187
lsosceles and equilateral
Isosceles and equi lateral triangles
triangles
Right
Right triangles
triangles
Special right triangles
Specia l right triangles
triangles
Similar triangles
Similar
Parallel Lines and
Parallel Proportionality
and Proportionality
Radians
Radians

23 Circles
Circles 207
Area and circumference
and circumference
Arc length
length
Area of a sector
sector
Centra l and
Central inscribed angles
and inscribed ang les
Equations
Equa tion s of circles

24 Trigonometry
Trigonometry 216
Sine, cosine, and tangent
cosine, and tangent
Trigonometric
Trigonometric identities
identities
Evaluating trigonometric expressions
Evaluating trigonometric expressions

25 Reading Data
Reading Data 225
26 Probability
Probability 234
27 Statistics II
Statistics 244
Mean, median,
Mean, and mode
median, and mode
Range
Range and standard
and standard deviation
deviation
Histograms dot plots
and dot
Histograms and plots
Word problems involving averages
problems involving averages
Boxplots
Boxplots

28 Statistics
Statistics II
II 254
Statistical sampling
Statistica l sampling
Using and
Using interpreting the
and interpreting line of best fit
the line
Margin error
Margin of error
Confidence intervals
Confidence intervals
Experimental design
Experimental and conclusions
design and conclusions

29 Volume
Volume 267
30 Answers to the Exercises
Answers Exercises 272

6
Exponents
Ex Radicals
ponents & Ra dicals
Here are
Here are the laws of exponents
the laws exponents you should know:
you should know:

Law
Law Example
Example

xx1z
1
= Xx 331z
1
= 33
F , L

\U_
XO = 11 30_
3D= 11
A 1

111In I 1 111
xX", m 0n
11
· x" = x + 34 35 Z 39
34. 35 = 39

if:
x111
_= : xm‐n
XIII -II
7
27 Z 34
37 = 34
x" 33
4

(XVII)
(x Z _mn
)11=
111 x 11111 (32)-l
(32)4 =: 38
38

my)!” =
(xy)"' : xlllylll
xmym
(2 · 3) 3 3 .33
(2 , 3)3 Z 23 3=2 ·3

I "I Xm 2 3 23

(~)m
(y) =z ;::
w , (~)3
(g) =z ~: 3‐3

W1
'1 1
xA_,-mIn =
7 - 33 -‐44 _ 2_
=
_ g
xm 34
1

7
CHAPTER 1 EXPONENTS & RADICALS

Many students don 't know the difference between

( - 3) 2 and - 32

Order of operations (PEMDAS) dictates that parentheses take precedence . So,

( - 3) 2 = (- 3) · (- 3) =9
Without parentheses, exponents take precedence:

- 32 = - 3 · 3 = - 9

The negative is not applied until the exponent operation is carried through. Make sure you understand this so
you don't make this common mistake . Sometimes, the result turns out to be the same, as in :

( - 2) 3 and - 23

Make sure you see why they yield the same result.

EXERCISE1: Evaluate WITHOUT a calculator . Answers for this chapter start on page 272.

1.
l. t‐1)4
(- 1)4 10. -(- 3)3
‐(‐3)3 50
19. 50

2
2. (‐1)5
(- 1)5 11. -(- 6)2 20. 332
2

3.
3. (‐])lo
( - 1) 10 12. - (- 4)3 3‐22
21. 3-

4. ((21)15
4. - 1) 15 13. 23 X 32 X (- 1)5 22. 53
53

5.(‐-1)8
5. (- 1)8 14. ( - 1)4 X 33 X 22 23. 5‐3
5- 3

6.
6. ‐18
- 18 15. (- 2)3 X (- 3 )4 24. 772
2

7.-‐(‐1)8
7. - ( - 1)8 16. 30 25. 7‐2
7- 2

s.(‐3)3
8. ( - 3)3 17. 6- 1 3
103
26. 10

9.-‐33
9. - 33 18. 4- l 10‐33
27. 10-

8
THE COLLEGE
THE COLLEGE PANDA
PANDA

EXERCISE 2: Simplify
EXERCISE Simplify so that your
so that answer contains
your answer only positive
contains only exponents. Do NOT
positive exponents. a calculator.
use a
NOT use The
calculator. The
first ttwo have been
w o have been done
done for you. Answers
you. Answers for this chapter start
this chapter start on page
page 272.

3x 2 · 2x 3 =
1. 3x2-2x3
1. 6x 5
=6x5 (x2y - 1)3
11. (x2y_1)3
11. 20. ((a‐1 a- 2)2
a- 1 •~a‘2)2

2. 2k‐44 ~4k2
2. 2k- : %
· 4k2 = i
k
12
12.
6u 4
6,‐{1
8112
'8uz
21.
21,
(b- 2) -3. (b3)2
(b‘2)‐3
2 a
, (b3)2
(m2n )3
("1 fl)‘
3. 5x4 · 3x -‐22
3” 5x -3x 13. 21402- - 4u 2v
2u v 2 • ~4uzv
22. ‐‑(mn 2)2
22.
(mnz)2
4. 7m 3 •-‐3m‘3
4. 7m3 -3
- 3111 x22 1
14.
14. X7.
x -3
23. %
23- x- 2
(2x 2)- 3
5. (23r2)’3 I l x
4 mn
3x 24. flm2n3
6. -_ 3a
3a2b 3a ‐ 5b8
b-‐ 3 .• 3a- 15_ W3x4
15.
(x-2)2
3117
3n7 k‐z
k- 2
7. 3
25.
6n 3
7- 55 x1
xi? 25- pk-3
16. 7 I
xi

n:r
c:
8. (azb3)2
(a2b3)2

W4 17.
x7

‘r1 x4
x2 x3.
17 x2. x4
26. (7)
m2 3

9. ( xy4 )
9,(”fl/2)
x3y2
‘ 18. (x 2)- 3 • 2x 3
(.r2)'3-2x3 27 (
2a
x y~z4
x2y3 24 )
27.
10 3 x‘3y'42“5
x -3 y - 4z-5
(- x) 3
10.' -_(_-") ) 2 •- (3111
(2m)2
19. (2m 3 )2
(3m3)2

EXAMPLE If3H2
EXAMPLE 1: If 3x+2 = y, then
then what
what is the
the value j r in terms
value ooff 33x off y ?
terms o ?

A )y+9
A)y B ) y-- 9
B)y ql
C)% D)%
3

Let's avoid
Let’s avoid the trouble of finding
the trouble finding what Here we
what x is. Here notice that
we notice the 2 in the
that the the exponent the only
exponent is the only difference
difference
between
between the
the given
given equation
equation and
and what
what we
we want.
want. 50
So using
using our
o u r laws
laws of exponents,
exponents, let’s
let's extract
extract the
the 2 out:
out:
3X+2:3X.32=y
3x+2 = 3x . 32 = .1/

3xx=: 'i
3
9

Answer ~(D) .
Answer

EXAMPLEz- ff 3"+1
EXAMPLE 2: If = 3‐“7,
3a+l = ra +7 , what the value
what is the value of a
a?7

Here we see that


Here the bases
that the are the
bases are exponents m
The exponents
same. The
the same. muu sstt therefore be equal.
therefore be equal.

a + 1l ==-~ aa+
a+ +7
2aa=: 6
2

au=@J
=l

9
CHAPTER 1 EXPONENTS
CHAPTER EXPONENTS & RADICALS
RADICALS

a
4a
EXAMPLE3zlf2a‐b=4,whatisthevalueof§5
3: U 2a - b = 4, what is the value of b ??
EXAMPLE
2

Realize that
Realize 2.
just 222.
that 4 is just

?:_2h_=F:2
4" (22V 22" :2
4 2. Za‐b

Square roots
Square roots are just fractional
are just exponents:
fractional exponents:
I
xi = .Jx
x?l

But what about xxi?


what about i ? The on top
The 2 on top means
means to square The 3 on the bottom
square x. The means to cube
bottom means cube root
root it:

3/x2

Wecan
We can see this
this more clearly if we
m o r e clearly we break
break it down
down::

g 2 I l
xx33 =
2 (x
(x2)? = {7x2
2 3;-;,
)1 = v x2

order in which
The order we do
which we do the squaring and
the squaring and the cube‐rooting
the cube -rooting doesn’t
doe sn't matter
matter..

2
x? = (xhz : (3/32
The end result
The end just looks
result just prettier with
looks prettier with the cube root
the cube on the
root on the outside.
outside . That we don’t
That way, we don't need the parentheses.
need the parentheses.

EXAMPLE 4: Which
EXAMPLE Which of the following is
the following equal to ~
is equal ?
Vac‐5 ?

A)x
A) .x5- x4
B)x5‐x4
B) CM; on?

1
The fourth
The equates to aa fractional
root equates
fourth root exponent of 3', so
fractional exponent so
4
4
x5=x

(C) .
Answer §).
Answer

10
10
THE COLLEGE PANDA
THE COLLEGE PANDA

SAT will also


The SAT you on
also test you on simplifying
simplifying square
square roots called ”surds").
roots (also called simplify a
"surds"). To simplify a square root, factor
square root,
the number
the inside the square
number inside root and
square root any pairs:
out any
and take out pairs :

v'48 = ✓2-2·2 · 2 · 3 =
JT:\/2~2.2~z- = \J /[I1]-[I}]
- - ' 3 = 2 --32=\2·2v3
/ § = 4= \4v3

In the example above, we
example above, take a
we take a 22 out the first -.
out for the Then we take another
[I}] . Then another 2 out pair -.
second pair
out for the second [I}] .
we multiply
Finally, we multiply the t w o 2’s outside
two 2's outside the square root
square root to get 4. Of course, a quicker route would have looked
course, a quicker route would have looked
like this:
this :
\/4‐
v'48 = / _ - 3 =: 4\/§
= t✓~- 4v3
Here's another example:
Here's another example:

M: \/--~2=2-3\/§:6\/§
backwards, take
To go backwards,
To the number
take the number outside put itit back
and put
outside and back under square root
under the square pair:
as a pair:
root asa

N
6v'2i =: ~ = x/fi
\/6-6- = V72

To simplify
simplify a
a cube such as
cube root such ifi6, take
as W, take out
out any triplets::
any triplets

W=x7-2=2¢/§

EXAMPLE 5: Which anequivalent


following is an
the following
Which of the equivalent form (x2 ) Q
form of (12) ¾, where x > O?
, where 0?

AWE
A)..jx Bm/E
B) x..fi CW? DN/E

Solution
Solution 1:
1:
3
(x2)“ zxz4 ZYZ
s 1
zmz --x=x\/§

Answer §)(B) .
Answer

. . 73 23 3 2 2,1 3 3
. exponent of 3 to the exponent of x in
, each of the
Solution (x2 ) 4 z= x 44 =
Solution 2: Since (x‘)El z x 2- ,, we can compare
we can compare this exponent 5 exponent each the
2
answer choices.
answer choices.
I
Choice
Choice A:A: J?
,Ix== xx2
1 1 I- iI 5
x,/x = x1 • x 2 =x”Z
I 3
ChoiceB:
Choice 8: x\/§=x‘-xz = xx21
= x 1 3
2
Choice
Choice C: W = x‘1
1
I
Choice
Choice D: -1/x=
é/E : xxI1

These results
These results confirm answer is B.
that the answer
confirm that

11
CHAPTER 1 EXPONENTS & RADICALS
& RADICALS

EXERCISE 3: Simplify the radicals or solve for x. Do NOT use a calculator. Answers for this chapter start on
page272.
page 272.

Ju
1. \/1‐2
l. /128
' 10. v128

2. fi)%
2. ll.Sv'2
11 = ./x

3. v'45
3. 1/5 12 3,/x
12. = v'45

/18
4. \/1_8 13 2v'2= v'4x
13.

5 2/fl
5. 2m 144)6
14. = 2/3x

6 3\/7_5
6. 3/75 15

7.J32
7. «3‐2 16 4/3x
16. = 2,/6

v'200
8. m 17 3v8
17. = x/2

9.
9. x/é
v8 18 x,/x
18. = v'2l6

12
12
THE COLLEGE
THE COLLEGE PANDA
PANDA

CHAPTER EXERCISE:Answers for this chapter start on page 272.

A calculator should
should N O T be
NOT used on the
be used
questions.
following questions.
following
~
3
If \/x‐
V}
= x‘
xc for all positive values of xx,, what
positive values what is
yX

__
1I
If aa- 1f =
= 3,
3, what
what is the value of a?
the value a?
the value
the value of c ??

A)
A) - 9
1
B) 9

1
C) 3
D)
D) 9

If
If 3x = 10,
3x = 10, what value
the va
what is the 3"’33 ??
lu e of 3x-
10
2xx
Y =
IfIf 27 3, then
223, then x must equal
must equal
A) ~
3
2
10
10
A ) yy ++ 3
A) B)
39
B
B)) yy-‐ 3 10
10
C ) 33- ‐ yy
C) C) 57
27
D) 3y
3y 27
27
D) fi
10

If y3/55 = 10,
10, what
what is the value of y2
the value y °?
IfIf a and b
a and are positive
bare positive even integers, which
even integers, which of the
the
A ) 440
A) 0
following
following is is greatest?
greates t?
B) 400
B) 400
A)
A) (‐2a)"
(- 2a)b
C) 1,000
1,000
B)
8) ( - 2a) 2b
<‐2a)2"
D) 10,000
C)
C) (2a) 11
an)”
2a 2/J
D) 2112"
D)

The expression
The ifii't,
expression Wx2y4, where
where x > 00 and O,is
and y > 0, is
equivalent
equivalent to which
which of the following?
the following?
If x2 = y3,
x2 = what value
3/3, for what value of 2
z does 32 =
does xX32 9?
= yy9 ?
A)
A) PY
x/Ty
A)) -‐ 1
A
B ) y ,fi
B) y fi
B) 0
1
C) x2 C) 1
D) 2
D) x 2y

13
13
CHAPTER
CHAPTER 1 EXPONENTS
EXPONENTS &
& RADICALS
RADICALS

If jxJx = xa, then what is the value of a? If 2”3


If 2x = k(2x), what
2x+3 -‐ 2X= what is the value of k
the value k??
1 A
A)) 3
A) 2
B) 5
3 C) 7
B) -
B)
4
D)
0) 8
C) 1
C)
4
D)
0) 3
allowed on the following
A calculator is allowed following
questions.
questions.
lH

lfx“‘oxh‘ 30, x > 1,


lf xac • xhc = xx30,x 1 ,and +b =
a n daa+ = 55,, wwhat
h a t iiss 3 4 1
24
k = (53
Jf(S 3) 4k: what is
(5! ))24,, what the value
is the value of
of k
k??
the value
the value of c?
c?
A) - 6
A) 3
2
B) 5 B) 3
C) 6
3
D
0)) 110
0 C) -
4
0) 2

2n+ 3
If 4 = 811+ 5, what .
what is the value
1s the value of
of n ?
11 ?
2n
A) 6 Which of the
Which following is equivalent
the following equivalent to x -;; for
to x7? for
B) 7 all positive
all values of
positive values of x, where
where a and b are
and bare
C
positive
positive integers?
integers?
C)) 8
D
0)) 9 lftix2
A) vbax2
½2n
B) x7x2“
C)
C) (7x1142
tJxa 12

5
0)
D) w,
2"xb
X

Which of the
Which following is equivalent
the following to ((‐2)3
equivalent to - ?
? 2) 3

A)
A) -‐2-\3/Z
2-¼
B)
B) 2-
2· 3/1
</4
C)
C) -‐4-€/i
4· '7'2 If x2y3
If = 10
x 2y 3 = 10 and x 3y 2 =
and x3y2 = 8, what the value
what is the value of
x5y5 ?
x5y5?
D) 4 · v'2
0) 4. 3/2
A
A)) 118
8
B
B)) 220
0
C
C)) 4400
D
0)) 8800

14
14
Percent
Per cent
EXAMPLE 1: Jacob got
EXAMPLE got 50% of the
the questions correct on
questions correct on a 30-question test
a 30-question test and 90% on
and 90% on a
a 50
50 question test.
question test.
percent of all questions
What percent
What questions did Jacob get correct?
did Jacob get correct?

First, let’s find the total


First, let's number of questions
total number questions he
he got
got correct:
correct :

1
X 30 =
50%><30:
50%
2 xX 3300 =: 1155
9
90/ox507mx50‐45
90% X 50 = 1Q X 50 = 45

0 : Z : -.
So
So he 15+
he got 15 + 45 =
z 60
60 questions u t of
correct oout
questions correct of a total of
a total of 30
30 + 50 =
+ 50 ; 80
80 questions: ~~ =
questions : 2‐0 ~ = I 75% I.

EXAMPLE
EXAMPLE 2: 2: The price of
The price of a
a dress
dress is
is increased by 20%,
increased by 20%, then decreased by
then decreased by 40%, then increased
40%, then by 25%.
increased by 25%. The
The
final price
final is what
price is of the
percent of
what percent the original price?
original price?

Here’s
Here's the technique dealing with
technique for dealing with these “series of
these "series percent change”
of percent change" questions. the original
Let the
questions . Let original price
price be
be p.
p.
When increased by 20“/o,
When ppisis increased you multiply by 1.20 because it’s the original price plus 20°/o. When it’s
20%, you multiply by 1.20 because it's the original price plus 20%. When it's decreaseddecreased
by you multiply
by 40%, you multiply by
by .60 because
because 60% is what's
60% is what’s left
left after
after you take away
you take away 40°/o. O u r final
40%. Our price is
final price is then
then
p
p Xx1.20 X 1.25
1.20 X><.60 x .90p
1.25 := .90p

The final price is I90"/o


price is 90% Iof
of the original
original price.
price.

Example
Example 2 shows IMPORTANT percent
shows the MOST IMPORTANT concept by
percent concept by far
far on
on the SAT. Never
the SAT. Never ever calculate
ever calcu late the prices
the prices
at each
each step. String all the changes
step. String changes together to get
together to get the end result.
the end result .
It’s important to know
It's important why this
know why Imagine again
works. Imagine
this works. again that the original
that the original price
price is
is p and we
p and want to
we want it by
increase it
to increase by
20"o. Normally,, we
20%. Normally we would just take
would just p and
take p and add
add 20% of it
20% of it on top:
on top :

p+
p + .20p
.20p

But realize
realize that
that
p+
p .20p == pp(1+.20)
+ .20p :1.20p
( l + .20) = l.20p

15
CHAPTER 2 PERCENT
CHAPTER PERCENT

And now
And want to decrease
n o w we want decrease this
this new price by 40%:
n e w price

1.20p -‐ (.40)(
l.20p (.40)(l.20p) (l.20p)(ll -‐ .40)
: (1.20p)(
1.20p ) = .40) := (1.20p)(.60) (1.20)(.60)p
z (l.20
(1.20p)( .60) = )( .60)p

which proves
which we can
proves we calculate the
can calculate final price
the final directly by
price directly by using this technique.
using this N o w we
technique. Now re set
we're up to
set up tackle the
to tackle the
inevitable compound interest questions on
inevitable compound interest questions on thethe SAT.

EXAMPLE 3: Jonas
EXAMPt:.E has aa savings
Jonas has savings account earns 33 percent
that earns
account that interest compounded
percent interest annually. His
compounded annually. initial
His initial
deposit was
deposit Which of
$1000. Which
was $1-000. of the following expressions
the following expressions gives
gives the value of
the value of the account after
the account 10years?
after 10 years?

A) 100Q(l,30)l0
A)1000(1.30)1° B) 1000 + 30(10)
1000+30(10) C) 1000(1.03)(10)
C) 1000(1.03) (10) D) 1000(1.03)
1000(1.03)10
10

A 33 percent
A interest rate
percent interest compounded annually
rate compounded annually means he earns
means he earns 33 percent
percent onon the
the account once a
account once a year. Keep in
year. Keep
mind that
mind that this
this isn't just 3%
isn't just 3% on original amount
the original
on the amount of $1000. This
This is 3%
3%of whatever's in the
of whatever’s the account at the
account at time,
the time,
including any
including any interest that he's
interest that already earned
he's already in previous
earned in previous years. This is the
years. This meaning of
the meaning of compound
compound interest.
interest.
80 ifif we're
So in year
we’re in year 5,5, he
he would
would earn on the
3°/o on
earn 3% original $1000 and
the original 3% on
and 3°/o the total
on the deposited in years
interest deposited
total interest years 11
through 4.
through 4.
If we
If we try calculate the
try to calculate the total after each
total after each and every year,
and every this problem
year, this would take
problem would take forever. Let’s take
forever. Let's we
what we
take what
learned from Example
learned from apply it here:
and apply
Example 3 and here:

Year 11 total 1000(1.03)‐


total:: 1000(1.03) = 1000(1.03)11
‐ 1000(1.03)
Year2
Year total:: 1000(1.03)(
2 total 1.03) = 1000(1.03)
1000(1.03)(1.03)‐_ 1000003)22
Year3
Year total: 1000
3 total: 1000003)33
(1.03)( 1.03)( 1.03) = 1000(1.03)
1000(1.03)(1..03)(103)
Year4
Year 4 total: 1000(1.03)(1.(o3)
total: 1000(1.03)( 103)(1.03)=1000(1.03)4
1.03)( 1.03)( 1.03) = 1000(1.03) 4

pattern? Each
the pattern?
See the Each year an increase
year is an increase of 3% so it's
3°/o so just 1.03 times
it’s just times whatever value was
the value
whatever the was last year. Note
last year. Note
that we
that we’re doing any
n o t doing
're not any calculations
calculations out. Think of it as
out. Think the price
as the price of a
a dress being increased
dress being increased by 3%
3°/o ten times..
ten times
Therefore, the
Therefore, 10total
the Year 10 1000(l.03)1°,
total is 1000(1.03) 10
, answer §J .
answer ( D ) .

Most of these
Most compound interest
these compound questions can
interest questions be modeled
can be modeled by thethe equation
equation A A= P(1+ r)t,
= P(l r)‘, where
where AA is
the total amount
the total amount accumulated,
accumulated, P is the principal or the
the principal the initial is the
amount, r .is
initial amount, interest rate,
the -interest and t is the
rate, and the
number of times
number received.
interest is received.
times interest

EXAMPLE 4: Jay puts


EXAMPLE puts anan initial deposit of $400 into
initial deposit bank account
into aa bank account that
that earns percent interest
earns 5 percent interest each year,
each year,
compounded annually. Which the following equations gives the total dollar amount,
compounded annually. Which of the following equations gives the total dollar amount, A, in the account
the account
after years?
after t years?

A) A = 400(1.0St)
A) 400(1.05t) B) A =
= 400(0.0St)
4oo(o.051) A = 400(0.0Sl
C) A
C) 4oo(o.05)' D) A = 400(1.05) 1
= 400(1.05)'

After t years,
After years, interest
interest has received t times.
been received
has been times. The rate r is 0.05 and
The rate and the initial amount
the initiaJ Plugging
amount P is 400. Plugging
values into
these values
these formula, we
the formula,
into the we see that the
see that answer is ( D ) .
the answer §J .

16
THE COLLEGE
THE PANDA
COLLEGE PANDA

EXAMPLE 5: This year,


EXAMPLE year, the chickens on
the chickens on aa farm laid 30% less
farm laid less eggs than they
eggs than they did last year.
did last If they
year. If they laid
laid 3,500
year, how
eggs this year,
eggs many did
how many they lay
did they last year?
lay ~t year?

: (.70)
This Year = (.70)(Last
(Last Year)
: (.70)(
3,500 =
3,500 (.70)(Last
Last Year)

I5,0001 = Last Year


Last

change (ak.a.
Percent change
Percent (a.k.a. percent increase/ decrease) is calculated
percent increase/decrease) calculated as follows:
as follows:

_ new value -‐ old


1 value
al
ch
ange -=
,ochange
01
°/o vaollude“13: v ue x 100
n_______________ew
oId v alue

example, if
For example,
For if the price of aa dress
the price dress starts
starts out at 80
out at 80 dollars
dollars and rises to 90 dollars,
and rises the percent
dollars , the change is:
percent change

9 0 ‐ 880
90 0
;80 Xx 100 = 12.5%
12.5%

If percent change is positive,


percent change it’s aa percent
positive, it's percent increase. Negative? Percent
increase. Negative? Percent decrease. important to remember
It’s important
decrease. It's remember
that percent change
that percent is always
change is always based
based onon the original value.
the original value.

EXAMPLE 6: In aparticular
EXAMPLE the number
store, the
a particular store, sold the
number of TVs sold the week Friday was
week of Black Friday was 685. The number
The number
of TVs sold
of sold the following week
the following week was TV sales
was 500. TV sales the
the week following Black Friday
week following Friday were
were what percent less
what percent less
sales the
than TV sales
than week of
the week of Black
Black Friday
Friday (rounded to the
(rounded to the nearest percent)?
nearest percent)?

A) 17%
A)17% B) 27%
B)27% C) 37%
C)37% D) 47%
0)47%

W ~ ‐0.27
500 -‐ 685
500
685
685 ~ - 0 27
.

We put
We put the difference over
the difference Answer ~(B) -.
N O T 500. Answer
over 685, NOT

EXAMPLE 7:
EXAMPLE 7: In
In aaparticular
particular store, number of
the number
store, the of computers
computers sold
sold the week of
the week of Black Friday w.as
Black Friday 470. The
was 470. The
number of computers sold the previous week was 320. Which of the following best approximates
number of computers sold the previous week was 3ZO. Which of the following best approximates the the
increase in computer
percent increase
percent sales from
computer sales from the previous week
the previous week to the
the week Friday?
week of Black Friday?

A) 17°/o
A)17% B) 27°/o
8)27% C) 37%
C)37% D) 47°/o
0)47%

320 ~ O
470 -‐ 320 a: 0.47
47
320 ·
This time, week of
time, the week Friday is
of Black Friday is not ”original” basis
the "original"
n o t the for the
basis for percent change.
the percent Weput
change. We the difference
put the difference
previous week's
over the previous
over week’s number, answer is ~(D) -.
number, 320. The answer

17
17
CHAPTER 2 PERCENT
CHAPTER PERCENT

more examples
A few more examples involving
involving percent:
percent:

EXAMPLE
EXAMPLE 8: The number students at aa school
number of students from 2010 to 2011. IfIf the
decreased 20% from
school decreased oi
number of
the number
enrolled in 201
students enrolled
students 1 was
2011 was k, which
which of the following expresses
the following enrolled
expresses the number students enrolled in
the number of students in 2010
in terms
terms of kk ?
?

A)
A) 0.75k 1.20k
B) 1.20k 1.25k
C) 1.25k D) 1.5k
D) 1.Sk

The answer is
The answer NOT 1.20k. Percent
is NOT change is
Percent change based off of the
is based the original value (from
original value and not
(from 2010) and the new
n o t the value .
n e w value.
be the
Let x be the number
number of students
students in 2010,

.80x =
.80x : k
x=
z 1.25k
1.25k

Therefore,
Therefore, there students in 2010 than
more students
were 25% more
there were than in 2011. Answer (C) .
Answer (@

EXAMPLE
EXAMPLE 9: Among 10th graders
Among 10th graders at
at a school, 40% of the
a school, the students are Red
students are those Red
Among those
Red Sox fans. Among Red
are also Celtics fans.
fans, 20% are
Sox fans, percent of the
What percent
fans. What 10th graders
the 10th the school
graders at the school are both Red
are both Sox fans
Red Sox fans
and fans?
and Celtics fans?

We
We don’t
don't know
know the number of 10th
the number graders at
10th graders at the so let’s
school so
the school suppose that
let's suppose it's 100.
that it's

Red
Red Sox fans = 40'3/o
fans = = 40
40% of 100 =

Celtics
Celtics & Red fans =
Red Sox fans : 20% of 40 =8
40 =

The answer is then


The answer = I8% I
then %~ =
1

strategy in percent
o m m o n strategy
A ccommon percent questions
questions is to make
make up
up aa number represent the
number to represent typically 100.
total, typically
the total,

18
18
THE COLLEGE PANDA
THE COLLEGE PANDA

CHAPTER EXERCISE:Answers for this chapter start on page 276.

following
allowed on the following
A calculator is allowed
questions.
questions.
If
If x is 50% larger
larger than
than 2, and y is
z, and larger than
is 20% larger than
z, then
2, then xxisis what percent larger
what percent larger than
than y
y??
A) 15%
wants to purchase
Reid wants
Reid purchase aa rug
rug that
that has price of
has aa price
$150.00. He has coupon that
ha s aa coupon would reduce
that would the
reduce the B) 20%
rug by
the rug
cost of the
cost by k%. If
If the
the coupon would
coupon would C) 25%
reduce the
reduce the rug
cost of the
the cost what is the
mg by $12.75, what the D) 30%
value of k?
value k?

Veronica has
Veronica bank account
has aa bank that earns
account that m%
earns m%
interest compounded
interest compounded annually. she opened
annually. If she the
opened the
account with
account the expression
with $200, the $200 (x) 1
expression $200(x)‘
represents
represents the amount in the
the amount after t
account after
the account
years. Which of the
years. Which following gives
the following gives x in terms
terms
of m
m??
In March, a city zoo attracted 32,000 visitors to A) 1 +
+ .01m
.Olm
its polar bear exhibit. In April, the number of
B) l1 +
+m
visitors to the exhibit increased by 15%. How
many visitors did the zoo attract to its polar bear C) 1 -‐ m
m
exhibit in April? D) 1 ++ 100m
D) 100m
A) 32,150
A)
B) 32,480
C) 35,200
D) 36,800 A charity organization collected 2,140 donations
last month . With the help of 50 additional
vo lunteer s, the organization collected 2,690
donations this month. To the nearest tenth of a
percent, what was the percent increase in the
Miguel is following a recipe for marinara sauce number of donations the charity organization
that require s half a tablespoon of vinegar. If one collected?
cup is equivalent to 16 tablespoons, A) 20.40/o
A) 20.4%
approximately what percent of a cup of vinegar
B) 20.7%
is the amount required by the recipe?
C) 25.4%
A) 2.3%
D)
D) 25.7%
B) 3.1%
3.10/o
C) 9.4%
D) 12.5%
12.5%

19
19
CHAPTER 2 PERCENT

The discount price of a book is 20% less than the The number
The dishes served
number of dishes served by a restaurant
restaurant
retail price. James manages to purchase the book during
during dinner
dinner was greater than
was 17.5% greater than the
the
at 30% off the discount price at a special book number dishes served
number of dishes during lunch.
served during lunch. If the
If the
sale. What percent of the retail price did James restaurant served
restaurant dishes during
served 940 dishes dinner, how
during dinner, how
pay? many more dishes did the restaurant serve
many more dishes did the restaurant serve
A) 42%
during
during dinner than during
dinner than lunch?
during lunch?

B) 48%
8) 4896
5095
C) 50%
[)) 56%
D) 5696

day, Robert
Each day,
Each eats 40% of the
Robert eats the pistachios
pistachios left
in his that time.
his jar at that end of the
the end
time . At the second
the second
day, 27pistachios
day, 27 pistachios remain How many
remain.. How pistachios
many pistachios in
ln 2010, the
the number houses built
number of houses Town A
built in Town
the jar at the
were in the
were the start
start of the day?
the first day?
was 25 percent
was greater than
percent greater than the number of
the number
houses built
houses built in Town
Town B. If 70 houses
8. If houses were built in
were built
A ) 775
A) 5 Town A during
Town how many
during 2010, how built in
were built
many were
B) 80
8) 80 Town
Town B?
8?
C
C)) 885
5
D ) 995
D) 5

Joanne bought a doll at a 10 percent discount off


the original price of $105.82. However, she had
to pay a sales tax of x% on the discounted price .
If the total amount she paid for the doll was Over a
Over w o week
a ttwo span, John
week span, ate 20pounds
John ate 20 pounds of
$100, what is the value of x? chicken wings
chicken and 15
wings and 15 pounds
pounds of hot dogs.. Kyle
hot dogs
A) 2 ate 20 percent
ate percent more chicken wings
more chicken and 40
wings and
percent
percent more hot dogs
more hot Considering only
dogs.. Considering only
B)
8) 3
chicken wings
chicken wings and
and hot dogs, Kyle ate
hot dogs, ate
C) 4 approximately
approximately x percent weight,
food, by weight,
more food,
percent more
D) 5 than John. What
than John. What is x (rounded
(rounded to the nearest
the nearest
percent)?
percent)?
A)
A ) 225
5
B)
8) 27
27
C
C)) 229
9
D
D)) 330
0

20
20
THE COLLEGE
COLLEGE PANDA
PANDA

ll)

Jane is playing a board game in which she must Omar currently


Omar government bond
holds a government
currently holds bond that
that
collect as many cards as possible. On her first has a
has market value
a market Each year,
value of $900. Each year, the
tum, she loses 18 percent of her cards. On the market value of the
market value bond is expected
the bond expected to be 20%
be 20°/o
second tum, she increases her card count by 36 higher than its market
higher than market value the year
value the year before.
before. If
If
percent. lf her final card count after these two the expression 900(1 +
the expression + p), where
where ppisis a constant,
a constant,
turns is 11, which of the following represents her represents the expected
represents the expected market value of the
market value the
starting card cow1t in terms of n ? bond after 3 years,
bond after what is the value
years, what value of ? p ?
n
11
A)
A)
(1.18)(0.64)
(l.18 )(0.64)
B)
8) (1.18)(0.64)n
(l.18 )(0.64)n
n
C)
C) (1.36)(0.82)
D) (0.82)(1.36)n
(0.82)( 1.36)11

spent xx dollars
Sims spent dollars on
on groceries
groceries in 2015. She
spent more on
34% more
spent 34°/o on groceries
groceries in 2016 than
than in
Due to deforestation , researchers expect the deer she spent
and she
2015, and spent 145°/o more on groceries
145% more groceries in
population to decline by 6 percent every year. If than in 2016. Which
2017 than Which of the following
following
the current deer population is 12,000, what is the expressions represents
expressions represents the amount, in dollars,
the amount, dollars,
approximate expected population size 10 years spent on groceries
Sims spent groceries in 2017?
from now?
A) (2.45)(0.34x)
(2.45)(0.34x)
A) 4800
B)
8) (1.45)(0.34x)
(1.45)( 0.34x)
B)
8) 6460
C)
C) (2.45)(1.34x)
(2.45)( 1.34x)
C) 7240
D) (1.45)(1.34x)
(1.45) (1.34x)
D)
D) 7980

In 2016, County
County A and County B
and County 8 collected
collected the
the
small clothing
A small store sells 3 different
clothing store types of
different types
same amount
same amount of taxes. ln 2017, the
taxes . In the amount
amount of
accessories: 20% are
accessories: scarves , 60°/o
are scarves, 60% are ties, and
are ties, and
collected by County
taxes collected County A decreased
decreased by 25%
the other 40 accessories
the other are belts.
accessories are belts. If half the
half of the and collected by County
and the amount of taxes
the amount taxes collected County B8
are replaced
ties are with scarves,
replaced with scarves, how many scarves
how many scarves
increased by 20%. If County A collected
Cow1ty collected 60
increased
will the store
store have?
have?
million dollars
million dollars of taxes
taxes in 2017, what was the
what was the
amow1t of taxes,
amount taxes, in millions dollars, County
millions of dollars, County B8
collected
collected in 2017?
A
A)) 554
4
B)
8) 78
78
C
C)) 990
0
D
D)) 996
6

21
21
CHAPTER 2
CHAPTER
M PERCENT
PERCENT

Daniel in a
has $1000 in
Daniel has checking account
a checking account and
and
$3000 in a savings account. The checking
savings account. checking
earns him
account earns
account percent interest
him 1 percent interest
compounded annually.. The savings
compounded annually savings account
account
earns him 6 percent
earns him interest compounded
percent interest compounded :
annually.. Assuming
annually Assuming he he leaves
leaves both
both these
these
alone, which
accounts alone,
accounts of the
which of the following
following *
represents how
represents much more
how much more interest Daniel will
interest Daniel will ,
have earned
have savings account
earned from the savings than from
account than from
checking account
the checking after 55 years?
account after years?
3, 000(1.06
A) 3, )5 ‐- 1,
OOO(1.06)5 1,000(1.01)5
000(1.01)5
B) 3,000(1.06)(5)
B) 3,000 (1.06)( 5) -‐1,000(1.01)(5)
1,000(1.01)(5)
C) (3,000(1.06)5
C) (3,000 (1.06) -‐ 3,000
5 3,000)) -‐ (1,000(1.0
(1,000(1.01)5
1)5 -‑
1,000)
1, 000)
D) (3,
D) (3,000(1.06)(5)‐3,000)‑
000(1.06) (5) - 3,000 ) -
(1,000(1.01)(5)
(1,000(1.0 1)(5) -‐ 1,000)

p ( 1 + 1~0)5

The expression gives the population


above gives
expression above of
population of
leopards after
leopards during which
years during
after five years an initial
which an initial
population of P leopards
population grew by rr percent
leopard s grew each
percent each
year. Which of the following
following expressions gives
expressions gives
the percent
the increase in the leopard
percent increase population
leopard population
years?
over these five years?
over

A)
A) (1(Hfi)
+ 1~)5
, .
(1 + 1~)5 - 1
B)
B) (1+fir)
_:____!.=..:..-=- ‐1><100
x 100
(”m)
( 1 + 1~0)5

C)“ll‐fif‐llxloo
)5- 1] x 100
C) [ ( 1 + l ~O

D)(Ll-igafxloo
D) ( 1 + ~)5x 100
1

22
Exponential vs. Linear
vs. Linear
Growth
Growth
The population of ants
The population doubling every
ants doubling every month.
month. A bank bank account earning 55 percent
account earning every year
percent every year.. These
These are
are
examples of exponential
examples exponential growth,
growth, which
which occurs when a
occurs when a quantity periodically by aa factor
grows periodically
quantity grows greater than
factor greater than
1. In
1. In the
the case the ants,
case of the this factor
ants, this is 2.
factor is 2. In
In the
the case bank account,
the bank
case of the it’s 1.05. When
account , it's exponential growth
When exponential growth
happens, we
happens, we can as an
model it as
can model equation that
an equation that looks
looks like
like

y : ab,

is the
where y is
where quantity after
the final quantity after t time
time periods (e.g. years),
periods (e.g. years), a is the
the initial quantity, and
initial quantity, growth factor
b is aa growth
and bis factor
greater than 1. Soif we started
greater than 1. So if we started off with
with 100 model equation would
ants, our model equation would be
ants, o u r be

: 100(2)
y= 100(2)'1

where t is the
where tis number of months
the number that have
months that gone by. And
have gone if oour
And if bank account
u r bank started off with
account started o u r equation
with $200, our equation
would look like
would like
yy =
: 200(1.0Sf
200(1.05)'
t is the
where tis
where the number years that
number of years that have passed.. You've
have passed seen this
You’ve seen already in the
this already the percent chapter..
percent chapter
Graphs exponential growth
Graphs of exponential growth have
have the shape:
following shape:
the following

yll

Notice how
Notice graph creeps
how the graph up slowly
creeps up at first but
slowly at shoots up
then shoots
but then up faster
faster and faster over
and faster That’s exponential
time. That's
over time. exponential
growth.
growth.

23
CHAPTER 3 EXPONENTIAL
CHAPTER LINEAR GROWTH
EXPONENTIAL VS. LINEAR GROWTH

Exponential decay
Exponential decay,, however, the opposite.
however, is the Imagine aa radioactive
opposite. Imagine radioactive substance
substance that
that loses mass over
loses mass time. It
over time. It
loses aa lot
loses lot of its
its mass beginning and
mass in the beginning and then loses it more
then loses more and more slowly
and more astime
slowly as goes by.
time goes

Mass
Mass

It’s worth
It's memorizing the
worth memorizing shapes of these
the shapes these graphs exponential growth
graphs of exponential growth and
and decay. The may test you
The SAT may you
explicitly on
explicitly on them
them..
The equation
The exponential decay
equation for exponential decay is the
the same as the
same as the equation exponential growth:
equation for exponential growth :

ab'1
y = ab

The only difference is that


only difference that the
the growth than 1. Soan
growth factor, b, is less than So an equation
equation that models exponential
that models decay
exponential decay
might look
might look like
like
y31=
: 400(0.6 )1
400(O.6)‘
where y is the
where the mass, grams, of aa radioactive
mass, in grams, radioactive substance
substance t years from now.
years from now. The indicates that
The 400 indicates substance
the substance
that the
currently has
currently has aa mass grams,, and
400 grams
mass of 400 and the 0.6 indicates
the 0.6 at the end
that at
indicates that end of each the substance
each year, the substance is left with
with
mass it started
60% of the mass year with.
started the year other words,
ln other
with. 1n loses 40% of its mass
words, it loses each year.
mass each
Sofar, the
So examples we've
the examples discussed have
we've discussed have been simple. To model
relatively simple.
been relatively model more complicated cases of exponential
more complicated exponential
growth and decay,
growth and asa
such as
decay, such bank account
a bank growing by 3%
account growing 3°/o every
every 2 years
years or a
a radioactive losing half
substance losing
radioactive substance half
mass every
its mass we’ll need
months, we'll
every 9 months, need to use slightly more
use aa slightly advanced exponential
more advanced exponential equation:
equation:

I
yy=abi
= abl
where k is
where is the
the time required for y to increase
time required increase by one factor of b.
one factor 17. Note
Note that t and It must
and k must have
have the same
same
units. So
units. 50 if
if t is in years, then also be in years.
than k should also years.

go over
Let's go
Let's some examples
over some examples so that you
so that understand what
you fully understand what k means.
means.

EXAMPLE 1:
EXAMPLE
M = 400(1.05)§!
= 400(1.05)
equation above
The equation
The models the
above models the mass nanograms, of a
mass M, in nanograms, a particle
particle after years. Based
after t years. Based on the equation,
on the equation,
which of the
which following best
the following best describes the mass
describes the mass of the particle over
the particle over time?
time?

A) It increases
A) increases by 5% every 4 months.
5% every months. B) It increases every 3 years.
5°/o every
increases by 5% years.
C) It increases nanograms every
increases by 5 nanograms months.
every 4 months. nanograms every
increases by 5 nanograms
D) It increases years.
every 3 years.

We have
We have anan exponential
exponential equation with kk == 3
equation with 3 and
and bb == 1.05,
1.05, which means it takes
which means takes 3 years the mass
years for the mass of
particle to increase
the particle increase by aa factor of 1.05. In other the particle's
words, the
other words, increases by 5%
mass increases
particle 's mass every 33 years.
5% every years.
Answer ~(B) .
Answer

24
24
THE COLLEGE PANDA
THE COLLEGE PANDA

EXAMPLEz‑
EXAMPLE2:
M = 400(0
M .6) 31
400035)“
The equation
The above models
equation above models the mass M, in nanograms,
the mass nanograms, of aa particle after t years.
particle after Based on
years. Based equation,
the equation,
on the
which of the
which the following
following best
best describes the mass
describes the the particle
mass of the particle over time?
over time?

It decreases
A) It decreases by 60% every
every 4 months.
months. by 60% every 3 years.
B) It decreases by
C) ItIt decreases
decreases by 40% every
every 4 months.
months. 40%every 3 years.
D) It decreases by 40%

Using y =
Using
’' as a reference,
= abI
ab1ias exponent of the given
the exponent
that the
reference, we see that equation is n
given equation
tt so how are
k'
form of k'
o t in the form
not so how are
we supposed
we supposed to find the value of k?
the value k? We
Wehave use an
have to use arithmetic trick:
an arithmetic

3
M 400(0.6/ 1 =
M == 400(0.6)3’ = 400(o.6)’/“/3’
400(0.6 /f( l / )

N o w we
Now that k =
can see that
we can
1
1,
: 5, which
which means
means the particle’s mass
the particle's mass decreases
1
every 5 year, or
decreases by 11 ‐- 0.6 = 40% every 4
or 41
months. Answer (C) .
months. Answer [ED.
nOn-profit organization
EXAMPLE 3: A non-profit
EXAMPLE organization currently has 50
currently has If the
volunteers. If
50 volunteers. organization is
the organization able to
is able double
to double
the number of volunteers every 8 months, which of the best m
-
the number volunteers every months, which the following equations best models the number of
following equations odels the number of
volunteers, 0,
volunteers, organization will have months from now?
the organization
v, the will have t months from now?
I l
A) v =
= 50(2)§
50(2)-S = 50(2)i
B) vU= 50(2)i V = 50(2)t
C) v 50(2) 1 D) U= 50(2)81
V = 509)!“

1
1
Again,
Again, we’ll use y := abl‘
we'll usey abk asa reference.. Based
as a reference Based on the given
on the information, a =
given information, 50 and
= 50 and b == 2.
2. Since the number
the number
of volunteers increases by the growth
volunteers increases every 8 months,
growth factor every months, k =
and t is in months,
months, and Therefore, the
z 8. Therefore, equation
the equation
that
that best
best models volunteers is v = 50(2)
number of volunteers
models the number 50(2) 5. Answer ~(A) .-
~. Answer

EXAMPLE4: A non-profit
EXAMPLE organization currently
non-profit organization currently has 50 volunteers.
has 50 If the
volunteers . H the organization
organization is able
able to double
double
the number of volunteers
the number volunteers every months, which
every 8 months, which of the following equations
the following equations best models the
best models number of
the number
volunteers, 0,
volunteers, v, the organization will
the organization have t years
will have from now?
years from now?
t
A) v =
= 50(2)é
50(2)lJ = 50(2)%i
B) t)v = 50(2) 7f C) Vv = 500)?¥
= 50(2) D) 0
v== 50(2)8f
;i0(2) 81

This question
question is the same as the one
same as Example 3,
one in Example 3, except years instead
except t is in years months. Because
instead of months. this, the
Because of this, the
answer is n
answer not same. To form the
the same.
o t the equation, t and
correct equation,
the correct and k m u s t have
must the same units, to
have the same units, sowe have convert
so we have convert

= ~5 years.
8
8 months into years,
months into years, which gives k z
which gives = % = years. The equation
equation is then
then
12

u:= 50(2/
V 1(2/ 3 ) = 50(2)37'
sour/(W3) 50(2) ¥

[ED.
Answer (C) .
Answer

25
CHAPTER 3 EXPONENTIAL
CHAPTER LINEAR GROWTH
EXPONENTIAL VS. LINEAR GROWTH

Now
Now let's exponential growth
compare exponential
let's compare growth and decay with
and decay linear growth
with linear growth and As you
and decay. As may already
you may know,
already know,
linear growth
linear can be
growth can by aa line
modeled by
be modeled with aa positive
line with slope. For
positive slope. For example,
example, if Ann has aa piggybank
Ann has piggybank with
with 100
dollars already in it,
dollars already and she
it, and adds 5 dollars
she adds every month,
dollars every the total
month, the amount in the piggybank
total amount piggybank can be modeled
can be modeled
by
by
: St + 100
A =
where A is the
where total amount,
the total amount, t is the number
number of months, and 100 (the
months, and y-intercept) is the initial
(they-intercept) amount.
initial amount.

Unlike
Unlike exponential growth, linear
exponential growth, growth is constant
linear growth and consistent.
constant and consistent. There
There is no slowing down
no slowing speeding
down or speeding
up. The total
up . The goes up by the
total goes the same
same amount time..
each time
amount each
N
Now imagine that
o w imagine that Ann takes 5 dollars
Ann takes dollars out piggybank every
her piggybank
o u t of her every month instead of adding
month instead adding to it. The
The total
total
balance would
balance would decrease by a
decrease by a constant amount each
constant amount each month, resulting in linear
month, resulting linear decay. The total
decay. The amount A in
total amount
the piggybank
the could then
piggybank could be modeled
then be modeled by
A =z 100 -‐ lOt
graph of such
The graph
The an equation
such an equation is aa line with aa negative
line with slope.
negative slope.

Both exponential decay and


exponential decay and linear decay are
linear decay instances of a
are instances a negative association between
negative association w o things.
between ttwo things. As one
one
thing increases,
thing other thing
increases, the other thing decreases example, the number
decreases.. For example, number of absences over the
absences over semester and
the semester and final
exam scores:
exam
Final Exam
Final Exam Score


•••••
• • ••
•• • • •
Number of Absences
Number Absences

When the
When the data points are
data points forming aa smooth
are close to forming line or graph
smooth line shows the negative
that shows
graph that relationship, we
negative relationship, we can
can
that there
say that there is aa strong negative association
strong negative association..

26
THE COLLEGE PANDA
THE COLLEGE PANDA

Both exponential
exponential growth linear growth
and linear
growth and instances of a
are instances
growth are a positive
positive association between two
association between things. As
t w o things. As
one thing increases, the other thing also increases.
one thing increases, the other thing also increases . For example,
example, the number
number of hours spent studying and
hours spent studying and final
exam scores::
exam scores
Final Exam
Final Exam Score



•• • • •• •
••


Hours Studied
Hours Studied
The graph
The graph above positive association
shows a positive
above shows that is quite
association that quite strong.
strong.

27
27
CHAPTER 3 EXPONENTIAL
CHAPTER VS. LINEAR
EXPONENTIAL V5. GROWTH
LINEAR GROWTH

CHAPTER
CHAPTE R EXERCISE:
EXERCISE : Answers this chapter
Answers for this on page
start on
chapter start page 278.

A calculator should NOT be use d on the


followi ng questi ons.
The employees at a new bookstore must stock a
certain number of shelves so that the store is
ready its opening
ready for its opening in ttwo
w o weeks.
weeks. TheThe .
The value of aa house
The value house decreased
decreased by 8°/o
8% from the
from the employees
employees stock shelves at
stock shelves at a
a constant rate
constant rate
previous
previous year consecutive years.
year for n consecutive Which of
years. Which throughout
throughout thethe two weeks. If p(t) is
two weeks. is the
the number
number
the following graphs
the following graphs could model the
could model value of
the value of shelves
shelves left be stocked
left to be after tt days,
stocked after which of
days, which
the
the house over this
house over time period?
this time period? the following
the following statements best describes
statements best describes thethe
A)
A) function p
function p??
A) The
The function an increasing
function p is an increasing exponential
exponential
function.
fw1ction.
B) The function p
The function a decreasing
pisis a decreasing exponential
exponential
function.
hmction.
C) The
C function p is an
The function an increasing linear
increasing linear
Time
Time function.
function .
D The function
D) The is aa decreasing
function p is decreasing linear
linear
B)
B) function.
function .

.-0
.-0
.-0
.-0
.-0
.-0 If the initial population of rats was 20 and grew
to 25 after the first year, which of the following
Time functions best models the population of rats P
with respect to the number of years t if the
C)
population growth of rats is considered to be
exponential?
A) PP==5t
5t ++ 20
20
B) = 20(1.25)'
B) P = 20(1.25)1
C) P =
C) : 20(5)'
20(5) 1

Time
D) P =
D) 5t 2 +
= 5:2 + 20

D)
D)

If the initial population of pandas was 100 and


grew to 125 after the first year, which of the
following functions best models the population
of pandas P with respect to the number of years
Time t if the population growth of pandas is
considered to be linear?
A)) PP == 225t
A 5t++1100
00
B)
B) P =
=100(1.25)'
100(1.25) 1
C) P = 100(1.2)
100(1.2)'1
D) P=20t2+5t+100
D) P = 20t 2 + 5t + 100

28
28
THE COLLEGE
THE PANDA
COLLEGE PANDA

:20(1+%)t
flf(t)t ) =
15
20 ( 1 + 100
)I N = 1, 000(0.97) 41'
A scientist the equation
uses the
scientist uses equation above
above to model
model the
the
The function f above
The function above models temperature, in
the temperature,
models the number
number of bacteria
bacteria NNinin aa petri
petri dish after h
dish after
degrees Celsius,
degrees Celsius, of a metal alloy used
metal alloy used in an
an hours. According
hours. According to the model, the
the model, number of
the number
experiment, where
experiment, where t is the number of seconds
the number seconds bacteria is predicted
bacteria predicted to decrease
decrease by 3°/o every kk
3% every
after began.. Which
experiment began
the experiment
after the Which of the
the minutes. What
minutes. What is the value of kk??
the value
following is the
following interpretation of the
the best interpretation the
number this context?
number 15 in this context?
1
A) -
A) 1
4
4
A) The temperature, degreess Celsius,
temperature, in degree Celsius, of the
the B) 4
metal alloy at
metal alloy at the
the beginning the
beginning of the C
C)) 115
5
experiment
experiment
D) 240
B The increa
B) The increase the temperature,
se in the degrees
temperature, in degrees
Celsius, of the
Celsius, alloy every
metal alloy
the metal every 100
100
seconds during the
seconds during experiment
the experiment
C) The
The percent which the
percent by which temperature, in
the temperature,
degrees Celsius,
degrees alloy
metal alloy
Celsius, of the metal cC = 40.002)“
4(1.002) 21
decreased from each
decreased from second to the next
each second next The equation
The above can
equation above be used
can be model the
used to model the
during the
during experiment
the experiment number of cars,
number millions,, registered
cars, in millions registered in aa
which the temperature,
percent by which
D) The percent temperature, in certain
certain state years after
state tt years According to the
after 2009. According the
degrees Celsius,
degrees Celsius, of the metal alloy
metal alloy model, the number
model, the number of cars registered in the
cars registered the state
state
increased second to the
each second
increased from each the next
next is projected
projected to increase
increase by n%n%every
every 6 months
months..
during the experiment
during experiment What is the
What the value
value of nn ? ?
A) 0.002
B) 0.04
C) 0.2
c u ) == 80(2)
C(t) 80(2)é~ D) 2
To examine how aa certain
examine how certain virus spreads,
virus spreads,
scientists introduced the
scientists introduced virus to cells in aa test
the virus test
tube and found
tube and found that
that the infected cells
number of infected
the number
in the
the test tube grew exponentially
tube grew exponentially over time.
over time.
The function above models
The function C above models thethe number
number of The population
The population of trees
trees in aa forest
forest has
has been
been
infected cells in the
infected the test days after
tube t days
test tube after the
the decreasing
decreasing by 6 percent every 4 years
percent every years.. The
population
population at the beginning
at the beginning of 2015 was was
virus was
virus introduced.. Based
was introduced Based on the function,
on the function,
estimated to be
estimated If P represents
14,000. If
be 14,000. represents the
the
which of the
which following statements
the following statements is true?
true?
population
population of trees
trees tt years which of
after 2015, which
years after
A) The predicted number of infected
predicted number infected cells in the following
the following equations
equations gives the population
gives the population of
the test doubled every
tube doubled
test tube days.
every 5 days. trees over time?
trees over
predicted number
B) The predicted number of infected
infected cells in A) P = 14,000
A) P 14,000(0.06)t
(0.06):J
I

the test tube grew by aa factor


tube grew factor of 5 every
every two
two
B) P =
B) 14,000 +
= 14,000 + 0.94(4t)
0.94(4t)
days.
da ys.
C) The predicted number
The predicted number of infected
infected cells in C) = 14,000 (0.94) 41
P=14,000(0.94)4'
C) P
I
the test tube doubled every
tube doubled every day. D) P
P== 14,000
14,000(0.94)é
(0.94)4°
D predicted number
D) The predicted number of infected
infected cells in
the test tube
the grew by aa factor of 5 every
tube grew day.
every day.

29
29
CHAPTER 3 EXPONENTIAL VS. LINEAR GROWTH

A calculator is allowed on the following


questions. Jamie owes Tina some
Jamie owes some money decides to pay
and decides
money and pay
back in the
her back
her the following
following way. receives 33
Tina receives
way. Tma
dollars
dollars the first day,
the first day, 66 dollars
dollars the second day,
the second day, 18
18
Which scatterplot shows
Which scatterplot shows the
the strongest positive
strongest positive dollars
dollars the third day, and
third day, 54 dollars
and 54 dollars thethe fourth
fourth
between x and
association between
association and y?
y? day. Which of the
day. Which following best
the following describes the
best describes the
relationship
relationship between
between timetime and the total
and the total amount
amount
A) of money (cumulative) Tina
money (cumulative) Tma has
has received from
received from
y
.,. Jamie over
Jamie over the course of these
the course these four days?
four days?

.:..
•,, . ••• A) Increasing
Increasing linear
linear

••
......
.
,, ..
• ••
•••• B)
C)
D)
Decreasing
Decreasing linear
Exponential
linear
growth
Exponential growth
Exponential decay
Exponentialdecay


X

.. -••
B)
y Albert has a large book collection . He decides to
trade in two of his used books for one new book

•......,.
•••
.-··. •
• each month at a local bookstore. Which of the
following best describes the relationship

•• •
..,..
• • • ••

between time (in months) and the total number
of books in Albert's collection?
A) Increasing
Increasing linear
linear
X B) Decreasing
Decreasing linear
linear
C) C) Exponential growth
Exponential growth
y D) Exponential decay
Exponential decay

.,.,,,.
·"

,- .J: A scientist
finds
finds that
every
every hour.
counts 80 cells in aa petri
scientist counts
that each one splits
each one
hour. He
into two
splits into
the function
uses the
He uses
two new
function A(
dish and
petri dish

A ( t) =
and
n e w cells
cells
= cr’
cr1 to
X calculate
calculate the number of cells
total number
the total the petri
cells in the petri
D) dish after t hours.
dish after hours. Which
Which of thethe following
following
assigns
assigns the
the correct values to c and
correct values and r ??
y
A) c = 40,rr =
= 40, = 2

,~ ....
,, J. B)
8) c = 80, r = 0.5
80,r
= 80,r
C) cC =
=
80, r =
: 11.5
.5
·-"~'y.
•• ti'• 80,rr =
D) cc = 80, = 2

• •
X

30
THE
THE COLLEGE PANDA
COLLEGE PANDA

ID,____ _____
.__....____,_ .....
Of the following scenarios, which one would The table below
The table below shows the price
shows the price P, in dollars,
dollars, of
result in linear growth of the square footage of a a barrel of
a barrel cntde oil t days
of crude after the beginning
days after beginning of
store? an
an oil shortage.
shortage.
A)
A) The owner increases
The owner square footage
increases the square footage by
Number of days,
Number days, t Price,
Price, P (dollars)
(dollars)
0.75%
0.75 each year.
% each year.
0 50.00
B) owner increases
8) The owner square footage
increases the square footage by
15 60.51
5% each year.
5% each
30 73.22
C) The owner expands
The owner expands the store by 5%
the store 5%ofof the
the
45 88.77
original square footage
original square footage each year.
each year.
D) The owner alternates
The owner alternates between adding 200
between adding
equation P = m(2)'/
11 11
used to model
m(2) " is used
the equation
If the model the
the
square
square feet one year and
one year and 300 square
square feet the
the
relationship and P, which
between t and
relationship between which of the
next year.
next year.
following could
following could be the values
be the values of m and
and n ?
?
A) m
m = 25 and
and n
11 = 54.38

B) m = 25andn
25and n = =86.12
86.12
C)
C) m = 50andn
50 and n = 54.38
During year of operation,
its first year
During its operation, anan equipment
equipment
D) m
m = and n =
= 50 and = 86.12
supplier carried
supplier items in its
carried 6,400 items its product line .
product line.
For each next 66 years,
each of the next years, the supplier carried
the supplier carried
in its product
product line half
half the number of items
the number items it
had carried
had the previous
carried the previous year. What type of
What type
model is best
model best to model
model thethe number
number of items
items the
the
amount of data,
The amount data, in gigabytes, stored in a
gigabytes, stored a
supplier carried
supplier carried in its product
product line any given
line for any given
database increases by 2% every
database increases every 15 hours. If 16
hours. If
year in its first 7 years
year years of operation?
operation?
gigabytes
gigabytes worth data is currently
worth of data stored in the
currently stored the
A) exponential growth
An exponential growth model
model database, which of the
database, which following functions
the following functions g
B)
B) An
An exponential
exponential decay
decay model
model gives
gives the amount of data,
the amount data, in gigabytes, that will
gigabytes, that will
C) linear growth
A linear growth model
model be stored in the
be stored database t days
the database days from now?
now?
D) linear decay
A linear model
decay model A) 80)
A) 16(2)1s1
g(t) = 16(2)15'
I
B) g<t>=16(1.02)f’-<
B) g(t) = 16(1.02)15
51
C)
C) g(t)=16(1.02)%
g(t)= 16(1.02)s
81
V = 200(i ) D)
D) g(t)=16(1.02)%
g(t) = 16(1.02) '°
V = 1,500t
An analyst
An analyst is evaluating
evaluating how accurate the
how accurate the ttwo
wo
models above
models are in predicting
above are the total
predicting the total number
number
of views,
views, V, an online
V, an video receives
online video days after
receives t days after
itit is released. How many
released. How more views
many more views are
are
predicted
predicted by the linear model
the linear than by the
model than the
exponential model 4 days
exponential model video is
the video
after the
days after
released?
released?
A) 1,400
B) 2,800
C) 3,200
D) 4,000
I

31
31
- ----

Rates
Rates
I’ve found
I've problems to
rate problems
found rate to be pretty polarizing‐some
be pretty students just
polarizing-some students just “get"
"get" them intuitively, others
them intuitively, others get completely
get completely
Most of the
lost. Most rate problems
the rate problems on
on the SAT will bebe pretty straightforward, but
pretty straightforward, ones that
but for the ones that aren't, highly
aren’t, II highly
recommend using
recommend using conversion
conversion factors to set up up the solution ((if
the solution i f you’ve
you've gone through chemistry,
gone through you should
chemistry, you should
know
know what I ' m talking
what I'm talking about).
about) . Conversion are a fool-proof
factors are
Conversion factors fool-proof way approach a lot of these
way to approach problems,
these problems,
but they can
but can bebe slow-going stronger problem
slow-going for stronger I'll be
solvers.. I’ll
problem solvers be covering both the
covering both straightforward, intuitive
the straightforward, intuitive
approaches
approaches and
and the approach throughout
conversion factor approach
the conversion examples in this
throughout the examples chapter..
this chapter

EXAMPLE bicycle manufacturer


EXAMPLE 1: A bicycle manufacturer can produce 20
can produce bicycles per
20 bicycles per hour
hour.. How many hours
H o w many it take
would it
hours would take
the
the manufacturer produce 320
manufacturer to produce bicycles?
320 bicycles?

Easy enough. Wedivide


enough . We total by the rate
the total
divide the + 2020 =
rate to get 320 ...,... : [!§]hours
hours..

EXAMPLE 2:
EXAMPLE 2: A rocket
rocket has 360 gallons
has 360 gallons of fuel
fuel left after 2 hours
left after flight, and
hours of flight, only 100
and only gallons after
100 gallons after 66 hours
hours
flight. It burns
of filght. gallons of fuel
burns n71gallons every hour
fuel for every flight, where
hour of flight, where n is a constant. What is the
constant. What value of n ??
the value

Here, we are
Here, we figuring out
are figuring the rate.
o u t the rate. ln hours of flight, the rocket
In 6 -‐ 2 =z 4 hours burned 360 -‐ 100 =z 260 gallons
rocket burned gallons of
260 ILt:l
fuel. Therefore, rocket bums
the rocket
Therefore, the bums a? =
z ~ gallons of fuel every
5] gallons every hour.
hour .
4

EXAMPLE 3: A box
EXAMPLE at the
box at the supermarket hold 66 oranges
can hold
supermarket can oranges each. Each orange
each. Each orange costs 20 cents
costs 20 that
Given that
cents.. Given
the supermarket
the has a
supermarket has budget of $540
a budget to stock
$540 to oranges, how
stock oranges, how many will the
boxes will
many boxes supermarket be
the supermarket be able to
able to
fill?

If each
If each orange
orange is 20 cents, then aa dollar
cents , then dollar would enough for 5 oranges.
be enough
would be Five hundred
oranges . Five forty dollars
hundred forty would then
dollars would then
be enough for 540 x 5 =z 2700 oranges,
be enough oranges, which would fill 2700 ...,...
which would + 6= : j 450 Iboxes.
boxes.

The examples
examples above were quite
above were straightforward and
quite straightforward and didn 't really
didn’t really call for writing out conversion
writing out factors, but
conversion factors, but
what if
what if we wanted
wanted to use conversion factors
use conversion Example 3?
factors for Example 3? What the solution
would 've the
What would’ve solution looked
looked like?

IOOcents l g~
ange 1l bbox
ox
540 0 W xx 100.deHar
5 4.de-Ha"fs .centS" 1
x x ---- =
‐ 4450
0 boxes
0 m X 66ganges
2 .centS"
l1d o l i a r? X 20 ~ 5 boxes

32
THE COLLEGE
THE COLLEGE PANDA
PANDA

As you
As you can see, conversion
can see, factors are
conversion factors multipliers that
are multipliers that help you go
help you go from one set
from one of units
set of units to
to another They’re
another.. They're
. . . . . . 1 oran e
usually expressed as
usually expressed fractions, and
as fractions, they represent
and they represent either information provided
either information prov1ded by thethe question
question (e.g.Wags)
(e.g. orange)
20 cents
, ._ 100 cents)
cents Wh .. .. f
standard
or stan u n .i t conversmns
d ar d unit conversions e.g. m
((e.g. 100 ) .. When
dollar usmg
en using c o n v e r s 1 o n factors
conversion solve
actors to so 1ve a pro bl em, you
problem, you must
must set
set
1
them up
them up in the right sequence
the right sequence and with the
and with appropriate numerators
the appropriate numerators and denominators.
and denominators.
The rest
The rest of the examples in this
the examples chapter are
this chapter are done with conversion
done with factors to teach
conversion factors how they’re
you how
teach you used, even
they're used, even
though you
though you may be able
may be able to solve the
to solve problems "intuitively".
the problems “intuitively”.

EXAMPLE 4: A car
EXAMPLE can travel
car can mile in 1 minute
travel 1 mile and 15
minute and 15seconds.
second-s. At this rate,
Afthis how many
rate, how miles can
many miles the car
can the ear
travel in 1 hour?
travel hour?

In most
most rate problems, you'll
rate problems, you’ll start with what
start with what the question is asking
the question We need
asking for. We need to convert that 11 hour
convert that to aa
hour to
distance that the
distance that the car travels.. The
car travels The car's
car’s rate every 75 seconds
mile every
rate is 1 mile seconds..

1 r x 6 0 m m 60seeonfis 11 mile = 60 x 60 :
miles = I miles
1 hoof x 60 .mifl1:rtes x 60 _seetmcisx mile 60 x 60 miles mil
r 1wear
hoof 1.rnimrte xlmimifixfiseeerfisz
75 _seetmcis 75 148 es 75
The units
The should cancel
units should as you
cancel as go along.
you go along. If the
the units
units are canceling, chances
are canceling, chances are you're doing
are you're things right.
doing things right.
that the
Notice that
Notice ”miles” unit
the "miles" at the
unit at the end is the
end is the unit we wanted
u n i t we wanted to end
end up with . This
up with. is another
This is sign that
another sign that we’ve
we've
done things
done correctly.
things correctly.

EXAMPLE 5: Tom
EXAMPLE 30miles
drives 30
Tom drives at an
miles at an average rate of 50
average rate 50 miles per hour.
miles per If Leona
hour. If drives at
Leona drives an average
at an average
rate of 40
rate 40 miles per hour,
miles per many more
how many
hour, how more minutes w i l l it take
minutes will her to travel
take her travel the same distance?
the same distance?

We have
We figure out
have to figure how long
o u t how long it takes to drive
takes Tom to drive 30 miles::
30 miles

. hour
1 hour 60 minutes
minutes .
30 miles
m11es><
x .
miles
50 m1
x
1es x
h
1 hour
our
= 36 minutes
2 36 minutes

Leona will take


Leona will take
hour
30 miles x
miles>< ~1 ho_
t x
60 minutes
60
i°~inutes = 45 minutes
miles
4 0 m1 es 1 hour
our : 45 minutes
So,
So,
45-‐ 36
45 =~
36 = E minutes
minutes

prepare for class,


EXAMPLE 6: To prepare
EXAMPLE class, Mr. Chu has
Mr. Chu print a
has to print booklets with
number of booklets
a number with p pages per booklet.
pages per booklet. If
If
every pages cost
every 5 pages cost c0 cents print and
to print
cents to and he spent aa total
he spent total of d dollars,
dollars, how booklets did
how many booklets did Mr. Chu print
Chu print
p,c,
in terms of p, and d ??
c, and

cp 100d
B) 100d 500d
C) 500d D) 35
Sd
A) 500d
A) 500d B) cp C) cp D) cp

,i---:::- 1OOcents‘
100 pages x 11booklet
55 ~
.cetrtS' booklet I 500d booklets
SOOd b kl
d
,.J _
d ~ctrS
r s x lIda-Harxccents
X
.1xrue1r c.centS'
X
..J-J.J...-::° X
ppages
p~
= --
cp
cp
00 ets

CT§]
Answer (C) .
Answer .

33
33
CHAPTER
CHAPTER 4 RATES
RATES

CHAPTEREXERCISEAnswers
: for this chapter star t on page 280.

A calculator should
should N O T be
NOT used on
be used on the
the
following
following questions.
questions.
submarine descends
As a submarine descends into the deep
into the deep ocean,
ocean,
the pressure
the pressure itit m u s t withstand
must withstand increases. At
increases. At an
an
altitude of ‐700
altitude - 700 meters, pressure is
the pressure
meters, the is 50
50 atrn
atrn
diet plan
Tim's diet
Tim’s plan calls for 60 grams of
60 grams protein per
of protein per (atmospheres),
(atmosphe res), andand at an altitude of
an altitude of -‐900
900
day. If
day. If Tim were to meet
were this requirement
meet this requirement byby meters, the
the pressure
pressure is 70 atrn. For every
meters, atrn . For every 10
10
only
only eating certain protein
eating a certain protein bar
bar that
that contains
contains 30
30 meters the
meters submarine descends,
the submarine descends, the pressure itit
the pressure
grams of protein,
grams protein, how many protein
how many bars would
protein bars would faces increases
increases by n, where
where n isis a constant. What
a constant. What
he have to buy
he have buy to last a
a week?
week? is the value
va lue of n ??
A) 0.1
B) 1
C) 2
D
D)) 110
0

electronics company
An electronics company sells
sells computer
computer monitors empty pool
An empty can be
pool can hours if
be filled in 5 hours water is
if water is
monitors
and releases
and releases a model every
e w model
a nnew every year.
year . With
With each
each
pumped
pumped in at 300 gallons
gallons an How many
hour . How
an hour. many
new model,
new the company
model, the increases the
company increases the screen
screen hours would it take
hours would the pool
take to fill the water is
pool if water is
size
size by a constant amount. In
constan t amount. ln 2005, the
the screen
screen
pumped
pumped in at 500 gallons
ga llon s an
an hour?
hour?
size was 15.5 inches.
size was 1n2011, the screen
inches . In screen size was
size was
18.5 inches. Which of the
inches. Which the following
following best
best
describes
describes how
how the size changed
screen size
the screen changed between
between
2005 and
and 2011?
A) The company
A) The company increases
increases the
the screen size by
screen size by
inch every
0.5 inch every year.
year .
The company
B) The increases the
company increases the screen size by 1
screen size
inches every
inches every year.
year.
C) The company increases the
company increases screen size
the screen size by
by 22 If a app les cost d dollars, which of the following
inches every
inches every year.
year .
expressions gives the cost of 20 apples, in
The company
D) The increases the
company increases screen size
the screen size by
by 3
3 dollars?
inches every
inches every year.
20a
A) 303
d
d
B)
B) y20d
a
a
C) 20d
C) 207
20
D>H
D) ad

34
THE COLLEGE PANDA
THE COLLEGE PANDA

During
During a
a raceon
race on a
a circular race track,
circular race track, aa racecar
racecar ‘ Idina can type
Idina can words in 2.5 minutes.
90 words
type 90 minutes. H ow
How
bums
bums fuel
fuel at
at a
a constant rate. After
constant rate. lap 4, the
After lap the many words
many words can
can she type in 12
she type 12 minutes?
minutes?
racecar has
racecar ga110ns left
has 22 gallons tank. After
its tank.
left in its lap 7,
After lap
the
the racecar has 18 gallons
racecar has left in its
gallons left tank.
its tank.
Assuming the
Assuming racecar does
the racecar does not refuel, after
not refuel, after
which lap will
which lap the racecar
will the have 6 gallons
racecar have left in
gallons left
its tank?
tank?
A) Lap 13
Lap 13
B)
B) Lap
Lap 15
15
C)
C) Lap
Lap 16
16 _
D
D) LLap 19
19
) ap A salesman company makes
salesman at aa tea company makes a a $15
commission
commission on on every worth of products
every $100 worth products
_ that he
that he sells.
sells. If
If a
a jar of tea leaves
leaves is $20, how
how
many jars
many jars would
would he have to
he have sell to
to sell make $180 in
to make
commission?
By 1:00 PM,
PM, a total
total of 40 boxes had
40 boxes had been
been commission?
unloaded
unloaded fromfrom a a delivery truck . By 3:30 PM,
delivery truck. PM, a a
total
total of 65 boxes had
65 boxes had been unloaded from
been unloaded from the
the
same truck. If
same truck. If boxes are unloaded
boxes are unloaded from
from the truck
the truck
at a constant rate, what
constant rate, the total
what is the number of
total number
boxes that
boxes will have
that will have been
been unloaded
unloaded fromfrom the
the
truck
truck by PM?
by 7:00 PM?

A train covers 32
train covers 32 kilometers
kilometers in 14.5 minutes.
minutes . If it
If it
continues to travel
continues at the
travel at the same which of the
rate, which
same rate, the
following
following is closest to
is closest distance it
the distance
to the will travel
it will travel
in 2 hours?
hours?
54kilometers
A) 54 kilometers
A calculator is allowed following
allowed on the following B) 265 kilometers
kilometers
questions.
questions. C) 364 kilometers
C) 364 kilometers
D) 928 kilometers
kilometers

A rolling
rolling ball covers a
ball covers a distance
distance of 2400 feet in 4
feet in I1
minutes.
minutes . What
What is the ball 's average
the ball’s speed, in
average speed, _
inches
inches perer second?
second? (1212 inches
inche s == 1 foot
foot)
P ( ) oil leaks
Peanut oil
Peanut out of
leaks out an industrial
of an container at
industrial container at
the rate of 3 liters
the rate liters in 2 hours. the peanut
If the
hours. If peanut oil
oil
costs 8 dollars
costs dollars per
per liter, many dollars’
how many
liter, how dollars' worth
worth
will be lost
will be lost in 11 hours?
hours?
A)
A) $60
B)
B) $96
C) $118
$118
D)
D) $132

35
CHAPTER 4 RATES
CHAPTER RATES

A recipe for soap calls for 1~ cups of lye for Henry drives 150 miles
Henry drives miles at 30 miles
at 30 hour and
per hour
miles per and
then another 200 miles
then another miles at
at 50 miles per
50 miles per hour.
hour .
every~ cup of castor
every 2 cup many cups
H o w many
castor oil. How lye
cups of lye What
What was his average
was his speed, in miles
average speed, miles per
per hour,
hour,
are needed
are needed for aa batch soap that
batch of soap that uses cups of
uses 3 cups for the
the entire journey, to the
entire journey, nearest hundredth?
the nearest hundredth?
castor
castor oil? A) 38.89
4 B) 40.00
A) 15
A) 1
5 C) 42.33
B) 5 D) 43.58
9!4
C) 9g
C)
5
1
D) 11~
D) 113
4
"slow" clock falls behind
A ”slow” behind atat the
the same
same rate
rate
every
every hour. It set to the
It is set time at 4:00
correct time
the correct
A M . When
AM. the clock shows
When the shows 5:00 AM the the same
same
day, the correct
day, the time is 5:08 AM.
correct time AM . When the clock
When the
An 8 inch by 10 inch piece of cardboard costs shows
shows 10:30 AM
AM thatthat day,
day, what
what is the
the correct
correct
$2.00. If the cost of a piece of cardboard is time?
time?
proportional to its area, what is the cost of a A) 11:02 AM
AM
piece of cardboard that is 16 inches by 20 inches?
B) 11:18 AM
AM
A) $4.00 C) 11:22 AM
AM
B) $8.00 D) 12:18 PM
D)
C) $12.00
D) $16.00

Jared
Jared and
and Robert are accountants
Robert are accountants who are tasked
who are tasked
with reviewing
with financial reports.
reviewing financial takes Jared
reports. It takes Jared
9 pikol = 2 large bahar 15 hours, working
15 hours, working at a constant rate, to review
constant rate, review a
report containing
report pages of financial
containing 240 pages financial
400 kulack = 29 pikol
statements. If Robert
statements. If Robert works Jared’s
works at twice Jared 's rate,
rate,
The formulas
The represent the
above represent
formulas above relationships
the relationships how many minutes
how many would it take
minutes would Robert to
take Robert
between some
between some units weight that
unit s of weight were once
that were once review a
review report containing
a report containing 120 pages
pages of financial
financial
used in Indonesia
used weight of 1,000 kulack
Indonesia.. A weight kulack is statements?
statements?
equivalent
equivalent to how
how many large bahar?
many large (Round
bahar? (Round A) 225
your answer
your the nearest
answer to the whole number.)
nearest whole number.) B) 345
C) 450
D) 900

36
36
THE COLLEGE PANDA
COLLEGE PANDA

A flask contains an acidic solution with a


concentration of 7.1 x 1015 hydrogen ions per
mi!Liliter. If 4.8 x 1023 hydrogen ions have a total
mass of 0.8 grams, which of the following is
closest to the concentration, in grams per liter, of
the acidic solution?
5
A) 1.2 ><10_5
A) X 10-

8
B) 1.2 x
><1o‘8
10-
-5
C) 1.5 ><10_5
x 10
8
D) 1.5 ><10_8
D) X 10-

Brett currently spends $160 each month on gas .


His current car is able to travel 30 miles per
gallon of gas. He decides to switch his current
car for a new car that is able to travel 40 miles
per gallon of gas. Assuming the price of gas
stays the same, how much will he spend on gas
each month with the new car?
A) $100
B)
8) $120
$120
C) $130
D)
D) $140

Margaret can buy 4 jars of honey for 9 dollars ,


and she can sell 3 jars of honey for 15 dollars.
How many jars of honey would she have to buy
and then sell to make a total profit of 132
dollars?

37
Ratio Proportion
Ratio & Prop ortion
Let’s that the
Let's say that ratio of cars
the ratio cars to trucks
trucks in parking lot
in aa parking is 5:2
lot is 5:2 (5
(5 to
to 2). Because ratios
2). Because ratios can be written
can be as fractions,
written as fractions,
this
this rratio 15equrvalent
a t i o is to ~.
equivalent to 2 AA ratio of 5:2
r a t i o of 15also
5:2 is equivalent to
also equrvalent to a a t i o of
a rratio of 10:4, the latter
since the
10:4, since latter reduces to the
reduces to the
former.
former.
Ln this
In this context, ratio of 5:2 means
context, a ratio means that every 5 cars,
that for every cars, there are 22 trucks.
there are And assuming
trucks . And assuming that
that there are only
there are only
cars and
cars trucks parked
and trucks parked in the lot, the
the lot, ratio also
the ratio that there
means that
also means are 5
there are for every
cars for
5 cars every 77 vehicles.
vehicles. ByBy the same
the same
token,
token, there are 2 trucks
there are every 7 vehicles.
trucks for every vehicles.

EXAMPLE Minyoung bought


EXAMPLE1: Minyoung croissants and
bought croissants bagels for
and bagels a breakfast
for a event. The
breakfast event. ratio of
The ratio number of
the number
of the of
croissants
croissants she bought to the
she bought number of
the number bagels she
of bagels bought was
she bought was 33 to
to 4. Minyoung bought
[f Minyoung
4. If bought 72 bagels, how
72 bagels, how
many croissants
many did she
croissants did buy?
she buy?

According the given


According to the given ratio,
ratio, Minyoung bought 3 croissants
Minyoung bought croissants for
for every
every 44 bagels. Since she
bagels. Since bought 72
she bought 72 bagels,
bagels, she
she
must
must have bought
have bought
3 croissants
croissants ~ .
x 72 bagels =
72 bagels : ~54 croissants
croissants
bagels
4 bage 1s

EXAMPLE 2: Arfand
EXAMPLE is following
Arfand is following a a recipe for a
recipe for seasoning blend
a seasoning that requires
blend that sea salt,
requires sea salt, black pepper, and
black pepper, and
paprika. According
paprika. According to the recipe, the
the recipe, the ratio of grams
ratio of of sea
grams of salt to
sea salt grams of
to grams of black
black pepper should be
pepper should 1:2, and
be 1:2, and
the ratio of grams
the ratio grams of black pepper to
black pepper to grams
grams ofof paprika
paprika should
should bebe 4:3.
4:3. How grams of
many grams
How many paprika should
of paprika should
Arfand
Arfand useuse to make grams of
make 108 grams of the seasoning blend?
the seasoning blend?

black pepper
Since black
Since pepper is involved
involved in both the given
both the ratios, we
given ratios, we can use itit to
can use to establish
establish aa "common basis”" for
”common basis for
comparison.
comparison. First, First, multiply salt to black
the sea salt
multiply the pepper ratio
black pepper ratio by
by 22 to get 2:4. Why multiply by 2?
to get 2:4. Why multiply by 2? Because Because
o w the
now
n “ 4 " in the
the "4" the ratio lines up
ratio lines with the
up with in the
” 4 ” in
the "4" the black pepper to
black pepper to paprika
paprika ratio. Once they
ratio. Once are lined
they are up , we
lined up, we
can establish
can that the
establish that the ratio between the
ratio between three ingredients is 2:4:3 (sea salt to black pepper to
the three ingredients is 2:4:3 (sea salt to black pepper to paprika).paprika).
According
According to this ratio , 3 grams
this ratio, of paprika
grams of paprika should
should be be used for every
used for every 22++4 + 33 =
4+ : 99 grams of the
grams of blend.. Therefore,
the blend Therefore,
3
should use
Arfand should
Arfand use ~ 5 xx 108 =lliJ
: 36 grams
grams of paprika.
of paprika.

38
38
THE COLLEGE
COLLEGE PANDA
PANDA

Proportion
Proportion
In addition
addition to ratios,
ratios, the will also
the SAT will test you
also test on proportions,
you on proportions, but
but n o t in the
not the way
way that typically learn
you typically
that you learn
them in school
them school (direct indirect proportion).
vs. indirect
(direct vs. proportion). Instead,
Instead, the will give
the SAT will you aa relationship
give you and ask
relationship and you how
ask you how
a change
a change in one variable affects another.
one variable another.
Let’s through a
r u n through
Let's run quick example.
a quick we have
Imagine we
example . Imagine have a
a triangle.
triangle. We
We know
know that
that the area of
the area a triangle
of a triangle is
is

A = ébh

Now let's
Now say we
let’s say triple the
we triple height. What
the height. happens to the
What happens the area?
area?
Well, if we
we triple
triple the height, new height
height, the new is 3h. The
height is The new area is
new area is then
then

1 1
Anew = 517611) 2 3 (ibh) 2 314.0“)

what happened?
See what happened? The
The terms
terms were rearranged so
were rearranged so that
that we could clearly
we could clearly see the e w area
the nnew is three
area is times the
three times the
old area. We put
area. We put the ”3” out in front
"3" out front of the old formula.
the old formula.
This technique
This extremely important
technique is extremely important because saves us
because it saves us time on tough
time on tough proportion problems. We
proportion problems. We could’ve
could've
made up
made up numbers
numbers for the base and
the base and the height and
the height calculated everything
and calculated out, and
everything out, while that’s certame
and while that's certainly aa
strategy you
strategy you should
should have your toolbox,
have in your would’ve taken
toolbox, it would've taken much longer
much longer and left us
and more open
us more open to silly
to silly
mistakes.
mistakes .
Let's do
Let’s do a more examples.
a few more examples.

EXAMPLE The radius


EXAMPLE 3: The radius of a circle is increased
a circle increased by
by 25°/o.
25%. ByBy what does the
percent does
what percent area of
the area the circle
of the circle
increase?
mcrease?

the original
Let the original area be AAM.
area be old· If original radius
the original
lf the is r,
radius is r, then
then the
the nnew
e w radius is 1.25r.
radius is l.25r .

Am, = 7T(1.25r)2 z (l.25)2(7rr2) =1.5625(7rr2) = 1562514,”

We can
can see that area increases
that the area 56.25% I..
increases by j 56.25%
idea is to get a number
The idea
The number in in front of the old
front of formula. In
old formula. this example,
In this that number
example, that out to
turned out
number turned to be 1.5625.
be 1.5625.
Also note
Also note that
that the was wrapped
1.25r was
the l.25r wrapped in in parentheses
parentheses so
so that
that the whole thing
the whole gets squared.
thing gets squared. It would've been
It would’ve been
incorrect have Am”
incorrect to have : rr(l.25
A ,ww = )r 2 because
71'(1.25)r2 we wouldn't
because we be squaring
wouldn’t be squaring the e w radius.
the nnew radius.

EXAMPLE
EXAMPLE 4: 4: The length of
The length of aarectangle is mcreased
rectangle is by 20%.
increased by 20%. The
The width is decreased
width is decreased by
by 20%. Which of
20%. Which of the
the
following accurately describes the change in the area of the rectangle?
following accurately describes the change m the area of the rectangle?

A)
A) Increases
Increases by 10% Decreases by 10%
B) Decreases 10% C) Decreases
Decreases by 4%
4% Stays the
D) Stays
D) the same
same

Originally, A=
Originally, A : lw.
120. Now,
Now,
Anpzu
A, ww Z (1.201)(
= (1201)(0.80u’)
0.80w ) := 0.96171)
0 .96/w = 0.96A01d
0.96A 0 1d

The area
The has decreased
area has decreased by 4%. Answer §J .
Answer (C) . Most think the
students think
Most students answer is
the answer It’s not.
(D). It's
is (D). not.

39
CHAPTER 5 RATIO
CHAPTER RATIO & PROPORTION
PROPORTION

EXAMPLE 5:
EXAMPLES:
= 9q1q2
F_ 94142
- ,2
7‘2

between two
attraction between
The force of attraction particles can
two particles determined by the
be determined
can be formula above,
the formula above, in which F is the
force between them, rr is the
between them, the distance between them,
distance between and q,
them, and q1 and q2are
and 472 are the charges of the
the charges particles. If
two particles.
the two
the distance
the between ttwo
distance between w o charged particles is doubled,
charged particles the resulting
doubled, the resulting force of attraction
attraction is what fraction of
what fraction
the original
the original force?

1 l 1 1
Ali 3); C’s D)1‐6

F 2 94qu z<1>2(9q1qz) _1 (9q1qz)=1p


""“’ (2r)2 2 r2 4 r2 4 0“

Answer ~(B) . Notice


Answer Notice how
how we do nnot like the "9"
constants like
o t let constants " 9 ” in the formula affect the
the formula the result.
result. In getting
ln getting
aa number out front,
number out students often
front, students make the
often make mistake of mixing
the mistake mixing that
that number
number up with numbers that
with numbers were
that were
originally formula.
the formula.
originally in the

EXAMPLE 6: The
EXAMPLE volume of aa cube
The volume tripled. The
cube is tripled. length of each
The length each side have been
must have
side must increased by
been increased
approximately what
approximately percent?
what percent?

A) 3%
A)3% B) 12°/o
B}12% C) 33°/o
C)33% D) 44%
0)44%

we have
N o w we
Now solve backwards
have to solve Keep in mind
backwards.. Keep that the
mind that volume of a
the volume a cube s3 where
cube is V = S3 where ss is the length of
the length
side. Even
each side.
each though this
Even though problem is a little
this problem little different, can still apply
different, we can apply the same process
the same as before:
process as increase
before: increase
each side
each side by some factor and rearrange the terms extract number. Only this time,
some factor and rearrange the terms to extract a number . Only this time, we have to
have use use x.

3
Vm'w :
Vnew = (xs)
(XS)
3 3 _ 3
Vm’u' 2X 5 ‐X Void

3 3 must be equal
Notice how
Notice how wewe were
were still able to extract
still able something out
extract something front, xx3.
out in front, That xx3
. That must be equal to 3 if
if the new
the n ew
volume be triple
volume is to be the old
triple the volume::
old volume

xx3z3
3
=3
= ~ ~ 1.44
x:\3/§z1.44
X

Each side
Each must have
side must have been by approximately
increased by
been increased approximately 44%. Answer
Answer ~(D) .-

40
THE COLLEGE
THE COLLEGE PANDA
PANDA

CHAPTER EXERCISE:Answers for this chapter start on page 282.

A calculator should NOT be used on the


following questions . A calculator is allowed following
allowed on the following
questions.
questions.

ratio of a to b is 7:6, and


The ratio and the
the ratio
ratio of b to c is v2
If a = 28, what
8:5. If what is the value of c ??
the value P =-
= RF
Electric power
Electric power P is related
related to the voltage V and
the voltage and
resistance the formula
resistance R by the formula above.
above. If
If the voltage
the voltage
halved, how
were halved,
were how would
would the electric power
the electric power bebe
affected?
affected?
A) The electric
A) electric power would be
power would be 4 times
times greater.
greater .
B)
8) The electric power would
electric power would be
be 2 times greater..
times greater
C) The electric power
The electric would be
power would be halved.
halved.
D) The
The electric power would
would be quarter of
be aa quarter
2~ : 1I;~ can
electric power
The ratio 2};
The ratio can be
be written
written as
as n
n:: 2. What is
2. What
what was.
what it was.
the value
the value of 11
n ??

Julie has
Julie has a square fence that
a square that encloses her garden.
encloses her garden .
She decides expand her
decides to expand garden by making
her garden making
each side
each the fence 10
side of the percent longer.
10percent After
longer. After
this expansion,
this expansion, the area of Julie’s
the area Julie 's garden will
garden will
have
have increased
increased by what percent?
what percent?
A) 20°/o
A) 20%
The price
The price of Product
Product XXisis 25% greater
greater than the
than the B) 21%
8)
price
price of Product
Product Y. The price
Y.The Product Z is 25%
price of Product C) 22°/o
22%
less than
than the price of product
the price product Y. Y.What the ratio
What is the ratio
D) 25%
the price
of the of Product
price Product X to the price
the price of
Product Z?
Product Z?
A
A)) 3 : 2
B)
8) 4:
4:33
A right
right circular
circular cone hass a base
cone ha radius of rand
base radius r and aa
C
C)) 5 : 2
height of 11.
height 11 If the radius
. If the radius is decreased
decreased by 20
20
D
D)) 5 ::33 percent
percent and
and thethe height
height is increased
increa sed by 1010
percent, which of the
percent , which the following
following is is the resulting
the resulting
percent change
percent change in the volume of the
the volume cone?
the cone?
A) 10% decrease
A) 10°/o decrease
B)
8) 12% decrease
decrease
C) 18.4%
18.4% decrease
decrease
D)
D) 29.6% decrease
decrease

41
41
CHAPTER 5 RATIO & PROPORTION

45°
h
s\/§
s

40
A
A b2 D
D I 45°
5
s
S
area of the
The area
The the trapezoid
trapezoid above can be
above can be found
found
In the triangle
ln the triangle above, the lengths
above, the lengths ofof the sides
the sides
using
using the formula ~%(b1
the formula (bi + b2)h. If lengths
b2)l1.If BC and
lengths BC and relate to one
relate one another as shown. If a new triangle
another as shown. If a n e w triangle
are halved
AD are halved and the height
and the doubled, how
is doubled,
height is how is created
created by by decreasing
decreasing ss such that the
such that the area of
area of
would
would the area of the
the area the trapezoid
trapezoid change?
change? the e w triangle
the nnew triangle is 64 percent of
64 percent of the original area,
the original area,
5 must have
s must been decreased
have been decreased by what percent?
by what percent?
A) The area would
The area would be
be increased
increa sed by 50 percent.
by 50 percent.
B) The
B) area would
The area stay the
would stay the same.
same. A)
A) 8°/o
8%

C) The area would


The area would be
be decreased
decreased by 25 percent.
by 25 percent. B) 20%
D) The area would
The area would be
be decreased
decreased by 50 percent.
by 50 percent. C) 25%
D) 30%

---------- V
T ----------
Calvin has
Calvin has a sphere that
a sphere is four
that is times bigger
four times bigger Questions 10-11 refer to
Questions 10-11 the following
to the following
than
than the
the one Kevin has
one Kevin has inin terms
terms of
of volume. The
volume. The information.
information.
radius Calvin's sphere
radius of Calvin’s how many
sphere is how many times
times
greater in length
greater than the
length than radius of Kevin’s
the radius Kevin's L = 4nd2b
sphere (rounded to the
sphere (rounded nearest hundredth)?
the nearest hundredth)?
The total
The amount of energy
total amount energy emitted
emitted by
by a star each
a star each
A) 1.44 second is
second is called
called its luminosity L,
its luminosity which is
L, which related to
is related to
B) 1.59
1.59 d, its
its distance (meters) away
distance (meters) away from Earth, and
from Earth, 1),
and b, itsits
C) 1.67 brightness measured in
brighb1ess measured watts per
in watts per square meter, by
square meter, by
the formula above.
the formula above.
D) 2.00

If one star is three times as far away from Earth


as another, and twice as bright, its luminosity is
how many time s greater than that of the other
star?
A
A)) 8
B) 9
C
C)) 116
6
D
D)) 118
8

42
42
COLLEGE PANDA
THE COLLEGE PANDA

Astronomers see two equally bright stars, Star A If


If the ratio of y
the ratio y:: 2.4 is
is equivalent
equivalent to 2.7 :: 3.6, what
to 2.7 what
and Star B, in the night sky, but the luminosity of is the value ofy
the value of y??
Star A is one-ninth the luminosity of Star B. The
distance of Star A from Earth is what fraction of A);3
A) 2
the distance of Star B from Earth?

Alfi1
A) 27
B);4
B) 3

1
B) 9
C);
C) 7
3

C) 3
1 D);9
D) 5

2
D) 3

A Box A weighs
weighs 4242 pounds and Box B
pounds and weighs 30
B weighs 30
pounds. ratio of the
The ratio
pounds . The weights of
the weights of Box A A to
to Box
B is equal
equal to the ratio of the
the ratio the weights
weights of
of Box C C to
to
Box D. If
If Box C and
and Box D weigh
weigh a
a total of 180
total of
The student body
The student body at an program
after-school program
an after-school pounds,
pounds, what the weight
what is the weight of Box C,
C, in
in pounds?
pounds?
consists only
consists only of 6th6th graders, 7th graders,
graders, 7th and 8th
graders, and 8th A
A)) 550
0
graders.
graders. The
The ratio 6th graders
ratio of 6th graders to 8th
8th graders is
graders is B) 75
75
17:
17: 28. If a total students attend
total of 110 students the
attend the
program,
program , n 11 of whom
whom are are 7th graders, what
7th graders, what is aa C) 105
possible value of n
possible value ?
11? D) 130

A bookstore ordered an
bookstore ordered initial shipment
an initial shipment of 10 10
paperback and 4 hardcover
copies and
paperback copies hardcover copies
copies of of aa
newly published
newly book. The
published book. store m
The store must order aa
u s t order
second shipment
second shipment with
with the ratio of
same ratio
the same of
paperback
paperback andand hardcover copies as
hardcover copies the initial
as the initial
shipment.
shipment. If the store orders
the store orders 5050 hardcover
hardcover copies
copies
of the
the book
book for the second shipment,
the second shipment, howhow many
many
paperback copies should
paperback copies should the store order?
the store order?

43
43
Expressions
Expressions
33m ~ k
Algebraic expressions
Algebraic just combinations
are just
expressions are numbers and
combinations of numbers x2 +
variables. Both x2
and variables. + y and
and m - k areare examples
examples
2
of expressions. chapter, we’ll
this chapter,
expressions. In this fundamental techniques
some fundamental
cover some
we'll cover allow you
that will allow
techniques that with
deal with
you to deal
questions involving expressions
questions involving quickly and
expressions quickly effectively.
and effectively.

1.
1. Combining Like Terms
Combining Like Terms
When combining like
When combining like terms, the most
terms, the mistake to avoid
important mistake
most important avoid is putting terms together
putting terms look like
that look
together that they
like they
can go
can go together
together but can't.
but can example,, you
' t. For example you cannot
cannot combine b2 + b to make
combine b2+ b3, nor
make £73, can you
n o r can combine a + ab to
you combine
make 2ab. To add
make 21111. add or subtract variables have
the variables
subtract,, the match.
completely match.
have to completely

EXAMPLE 1:
EXAMPLE
20132 -‐ 3a
2(2a 2 2
3a2b2
b -‐ 4b 2
4172) (a22 +
) -‐ (a 5a2 b2 - 10172)
+ 5a2b2 10b2 )
Which of the
Which following is equivalent
the following equivalent to the expression above?
the expression above?
2 b2 -‐ 1st 2 b 2 + 2b 2
6a2 b2
A) -‐6a2b2 B) 3a2 -‐ lla
B) 3a2 11a2b2 18b 2 3a 2 -‐ lla
C) 3,12 11a2b2 + 2172 D) 5a2 + 2a2b2 + 21:2

2 (2a 2
2(2a2 -7 31
2 2 -‐
b
3a12172 2
41,2)
4b ) -‐ (a
<a2
2
+ 2 2
5a2b2
+ 5a b -‐ 10b 2
10172)) = 4a 2 -
: 4a2 6a 2 b2 -‐
6112172 8b2 a 2 -‐ Sa
8b 2 -7 112 b +
2 2
5a2b2 10b 2
+1on2
2 2 + 2b22
= 3a 2 ‐-
: 3112 b + 2b
11112172
11a

IT§].
Answer (C) l.
Answer

44
THE COLLEGE PANDA
THE COLLEGE PANDA
..
v0

2.
2. Expansion Factoring
and Factoring
Expansion and

J
M L ] ;2
EXAMPLE 2: : ~ -.
.r
- 4)(2:c +
2(x -‐ 4){2x + 3)
Which of the
Which the following equivalent to the
following is equivalent expression above?
the expression above?

A) ~ 2 -‐ lOx
A)4x2 10x‐24
- 24 B)4x2+10x‐24
B) 4x2 + lOx - 24 C)4x2+10x+24
C) 4x2 + lOx + 24 D)8x2
D) 8x 2 ‐- 220x
0 x -‐ 224
4

Some people
Some expand using
people like to expand using a method called FOIL (first,
method called outer, inner, last).
(first, outer, If you
last). If you haven't
haven't heard
heard of it,
totally fine. After all,
that’ss totally
that' all, it’s same thing
it's the same asdistributing
thing as distributing each First, we
term. First,
each term. we distribute
distribute the ”2.”
"2."

2(x -‐ 4)(2x = (2x ~- 8)(2x


4)( 2x + 3) = 8)( 2x +
+ 3)

Notice that itit applies


Notice that just one
applie s to just one of the t w o factors
the two Either one
factors.. Either but N
one is fine, but O T both.
NOT both.

(2x -‐ 8)(
(2x 2x + 3) =
8)(2.r 4x 2 + 6x ‐- 16x ‐- 24
: 4x2
= 4x 2 ‐-
: 4x2 10x -‐ 24
10x

[ED.
Answer (A) .
Answer

when it comes to factoring


N o w when
Now factoring and
and expansion,
expansion, there several key formulas
are several
there are formulas you should know:
you should know:

.• (a+b)2
(a + b)2 =a2+2ab+b2
= a2 + 2ab+ b2
0 (a‐b)2
• (a =az‐2ab+b2
- b) 2 = a2 - 2ab+ b2
0 a2 - b2= (a
• 112‐1)2 (a+b)(a‐-b)
+ b)(a - b)
Memorize these
Memorize and backwards.
forwards and
these forwards They show
backwards. They show up very often.
up very often.

3: Which
EXAMPLE 3:
EXAMPLE the following
Which of the is equivalent
following is equivalent to 4.x4‐- 9y2
to 414 9y2 ?
?

A) (2x2 +
A) (2x2 9y)(2x2 -‐ y)
+ 9y)(2x2 8) (4x2 + 3y)
B) (4x2 (x2 -‐ 3y)
syxxz (x 2 + 3y)(4x2
C) (x2 3y)(4x2 -- sy)
3y) (2x2 + 3mm2
D) (2x2
D) 3y)(2x2 -‐ 3y)
By)

Part of what
Part makes for a
what makes a top score is pattern
top SAT score pattern recognition. Once you've
recognition . Once done enough
you 've done enough practice,
practice, you
you should
should
be able
be able to recognize the question
recognize the question above asa difference of t w o squares, a variation of the a2‐
2 b2
2 formula.
above as a difference of two squares, a variation of the a - b formu la. The The
SAT will rarely
rarely test you on those formulas in a straightforward
you on those formulas in a straightfo rward way. Beon the lookout for variations that
Be on the lookout for variations that
match the pattern.
match practice, you’ll
more practice,
pattern . With more you 'll get better and better
better and at noticing
better at noticing them.
them .
Using the formu
Using la a4122 -‐ b2
formula b2 =
= (a + can see that
+ b)(a -‐ b), we can that a = 2x 2 and
= 2x2 and b = By. Therefore,
= 3y. Therefore,

4x4 9y2 =
4x 4 -‐ 9y2 (2x22 + 3y)(2x2
: (2x 3y)(2x2 ‐- ay)
3y)

ITelJ
.
Answer (D) .
Answer

45
CHAPTER 6 EXPRESSIONS
CHAPTER EXPRESSIONS

EXAMPLE 4:
EXAMPLE
4 -‐ 8x
16x4
16x y +
2 2
8x2y2 y4
+ 3;4
Which of the
Which the following equivalent to
following is equivalent to the expression shown
the expression shown above?
above?
4 2
A) (41:2 + f ) ’ (2x -‐ y)
B) (2:
B) y)4 C) (2x
C) (2x +
+ y)2(2x
y)2(2r -‐ y)
y)2 D) (4x + y)2(x ‐ y)2

Using formula (a -‐ b)2 =


the formula
Using the = aa2 Zab +
2 -‐ 2ab 2 (in reverse),
+ bb2 can see that
we can
reverse), we = 4x2
that a = and b =
4x 2 and y2.Therefore,
= 3/2. Therefore,

16x44 -‐ 8x
16x 2
8x2y2y2+ y4 (4x 2 -‐ y2
y4 := (4x2 )2
yz)2

This is not the answer


n o t in the have to take
choices.. We have
answer choices one step
take it one step further and apply
further and a 2 -‐ bb2
the a2
apply the 2 formula to the
formula the
expression inside the
expression inside parentheses..
the parentheses
2 2 2
( “ z-‐ y2
(4x f f)2 x ++
: l=a [(2x wy)( (2x++yy)
a x2x‐-wy fi)] ==( u f a(2x
x ‐- wy)2
2
Answer (C) .
Answer [@]

3. Combining
3. Combining Fractions
you’re adding
When you're
When simple fractions,
adding simple fractions,
11 + 11
-3 + 4-
3 4
the step is to find the
the first step least common
the least multiple of the
common multiple denominators.. We
the denominators Wedo
do this so that
this so we can
that we get a
can get a common
common
denominator.
denominator. In aa lot lot of cases, just the
it’s just
cases, it's product of the
the product the denominators,
denominators, as here, 3 x 4 =
as it is here, z 12.

1 4 3 7
5 + ‘ fi fi ‐ fi
N o w when
Now we’re
when we adding fractions
're adding fractions with expressions in the
with expressions denominator, the
the denominator, the idea
idea is the
the same.
same .

EXAMPLE5:
EXAMPLES: 1 2
1 2
x+2+x‐2
x+2 + x - 2
Which of the
Which the following is equivalent
following is to the
equivalent to expression above?
the expression above?
3x -‐ 22
3x B 3x +
3x + 22 C) 3
3 D) 2
2
A) (x+2)(x' -‐ 22))
A ) (x+2)(x 1)” (x
( x+ 22)(x
) ( x-‐ 22)) ( x+ 22)(x
C ) (x ) ( x-‐ 22)) D ) ((xx++ 22)(x
) ( x-‐ 22))

The common
common denominator
denominator is just product of the
just the product the two denominator s: (x + 2)(
t w o denominators: 2)(xx -‐ 2). So n o w we multiply
So now multiply
the top and bottom of each
and bottom each fraction
fraction by the theyy don
the factor the don’t't have:
have :

11 + 22 _ 11 ix‐2 x- 2 22 .x+2_
x+2 x -‐ 2 + 2 ( x++ 22))
2(x _ ((x
x ‐-22) ) ++2--,--0-
( x ++--'-22))
2(x
-- + -- = -- . -- + -- . - - = ----- + -----'---- = -'----'---,-
x+
+2 ) ( x -‐ 22)) ((xx++ 2)(x
x -‐ 22 _ xx ++ 22 x -‐ 2 x -‐ Z2 x + 2 _ ( x(x++22)(x 2 ) ( x- ‐ 2)
2 ) _ ((xx++ 2)( 2 )x( x- ‐2)2 )
_ 3 x++ 2
3x
‐ ( x(x++22)(
) ( xx -‐ 22))
Answer ( B ) .
Answer [ill).

46
THE COLLEGE PANDA
COLLEGE PANDA

4. Flipping (Dividing)
4. Flipping (Dividing) Fractions
1
- 1
What’s the
What's difference between
the difference between+ and -
and - ??
2
2
3
3

The difference
The difference is where
where the longer fraction
the longer fraction line The first is %divided
line is. The !2 second is 1 divided
The second
divided by 3. The divided by ~-
g.
3
They’re
They 're not the same
not the same..
11
_i_
_L= ; =_ !1 X !1=- !1
_ !1 -'-3
3 ‐2'3‐2x3‐6
3 2· 2 3 6

1 . 2 3 3
--=2 1 -'--=
. 3
1 X-=-
2 2
3
The shortcut
The the fraction
shortcut is to flip the that is in the
fraction that the denominator
denominator.. So,

a _ ac
- g
b- ‐ Fb
c
C

If the
If fraction is in the
the fraction the numerator, then the
numerator, then following occurs:
the following occurs :

a
5_= _1 a
_ b
c _ bcbe

If x > 1,
EXAMPLE 6: If
EXAMPLE 1, which
which of the equivalent to
following is equivalent
the following ‐‐1‐xT-
1 1 ?
?
X

x --‐ 1 + x + 1

__ 2x22
2x B) _______
22 C) x(x - l)(x + 1)
_______
x(x‐1)(x+1) D __
D) ((xx-‐ ll)(x+l)
)(x+1)
A)(x‐1)(x+1)
A) (x - l)(x + 1) B’(x‐1)(x+1)
(x - l)(x + l) C) 2
2 ) 2 2

First, combine the


First, combine the two fractions on the
t w o fractions bottom with
the bottom with the denominator (x -‐ 1)(x +
common denominator
the common + 1).

11 1l _ x +1
X x -‐
X 1l 2
2Xx
x‐1+x+1“'u‐1nx+u + -(u‐1Mx+n
-x-- 1 + -x +-1 = -(x--- 1)-(x-+~1)
x--- 1-)-(x_+_l_) = -(
ux---
‐ 11-)-(
x xx_+_
+l_n)
substitute this back
Next, substitute
Next, and flip it.
back in and

x
X _ x (x
( x-- 1l ))(x
( x + 11)) _ ((xx -‐ 1l ))(x
(x++11))
‐ ‐ 22xx ‐ _ _ ‐ ‐ 22x§ _ _ ‘ ‐ ‐ 22 _ ‑
( x -‐ 11)(x
(x ) ( x + 1l )

Answer ~(D) .
Answer

47
CHAPTER EXPRESSIONS
CHAPTER 6 EXPRESSIONS

5.
5. Splitting
Splitting fractions
fractions

30
EXAMPLE 7: Which
EXAMPLE Which of the followingisequivalent
the following to 306+
is equivalent to : C7
c?

A S5) +c
A) +c
6
? 10+c
B)‐2‐‐
B) 10 +c
2
C ) 5 +c
C)S +c D)5+g
0)5 + ic
Wecan split the
We can split the fraction
fraction into
into two:
two :
30+c_?1)+£_5+£
6 _ 6 6 _ 6

answer is ~( D ) .- This
The answer
The just the
This is just the reverse adding fractions.
reverse of adding fractions.

Note that while


Note that you can
while you split up the
can split the numerators fractions, you
numerators of fractions, cannot do so
you cannot so with
with denominators.
denominators . So,

_ 3 7g_ 3 3
_
-i=-+-
x+y x y
x +y X

3
In fact, you
!J1 cannot break
you cannot break up a fraction like -x i -y any
fraction like further..
any further
x +y

48
THE COLLEGE
COLLEGE PANDA
PANDA

CHAPTER EXERCISE:Answers for this chapter start on page 284.

A calculator should
should N O T be
NOT used on the
be used
following
following questions.
questions.
4 + 8x
Which the following
Which of the following is equivalent
equivalent to
to 4 ~:x
12x
for
forxX76
/; O
0??
Which the following
Which of the following is equivalent
equivalent to 1 ++ Bx
8x
2y +
6x23]
6x 6xy 2 ?
+ 6ch2 A)
A)
3x
A) 6xy (x + y)
A) 6xy(x 4 + 2x
B)
B)
B) 12xy(x ++ y) 3x
6x y2(y + x)
2
C) 6x2y2(y 1
1 + 22xx
C)
D) 12x3y 3
12x3y3 3x
D)) 1l
D

lIff a ,f; !+
0, then 2
76O,then ~ is equivalent
+ 2 equivalent to which
which of the
the
a 4 Which of the
Which following is equivalent
the following equivalent to 3x44 -‐ 3
to 3x 3 ??
following?
following?
3(x 2 + 1)2
A)Mx2+1V
A)
3 + 4a
4a 3(x2 - 1)
B) 3(x2 1)22
A)
A)
4a
4 ++33a
a
C)
C) 30:
3 ( x 3 -‐ ) (X + 1)
1”(35+ 1)
B)
4a
D)
D) 3(X + l )(x + l )(x -‐ 1)
3(x2 +1)(x+1)(x 1)
7
C)
4a
4
D)
a+4 (x +1)2
+ 1)2 +
+2(x +1)(y
2(x + l )(y + 1)+
+ 1) 1)22
( y ++ 1)
+ (y
Which of the
Which following is equivalent
the following equivalent to
to the
the
expression shown above?
expression shown above?
A) (x ++ yy ++1)2
A) (x 1)2
Which of the following is equivalent to B) (x +y +
(x + 2)2
+ 2)2
(x 2 + y) (y + z) ?
x ++yy)2
C)) ( (x F ++22
A) x 2z + 1/ + 1/Z y )2 -‐ x -‐ y
D) (x + y)2
B) x 2y + x 2z + y 2 + yz
C) x2y + y2 + x2z
D) x2 + x 2 z + y2 + yz

49
CHAPTER 6 EXPRESSIONS
CHAPTER EXPRESSIONS

If y I-
If yé0 and xx I-
0 and yéy, which of the
y, which the following
following is
is The expression 8x 2 - ~y 2 can be written in the
xy - x 2
equivalent
equivalent to xy ‐ x;2 ? ? form 8(x -~ cy)(x
form + cy), where
cy)(x + where c is aapositive
positive
xxyy -" yy constant. What
constant. What is the value of c ?
the value

A)‘i‘
A) _ 'f_
X A)
1
A) 16

B) 'f_
i 1
X B) 8
xX
C) g- 1
y C) -
C)
4
xX
D)
D) 7
y D) ~

If x > 1, which
which of the following
following is equivalent
equivalent to 2
xx2(x
(X + 2)(x 2) +
2) ( X -‐ 2) +44
1
-----,----1 -=?7
x‐1+x+5'
x - 1 x +S
--+--
Which of the
Which
expression
following is equivalent
the following
expression above?
above?
equivalent to the
the
2 3
A) 5):
Sx +
+ 77 (x2 ‐- 2)2
A) (x2
A) 2)2
A)
6 B)
B) (x2 + 2)2
(x 2 + 2) 2
B)
6 C)
C) (x
(x --1)2(x
1) 2 (x + 2)2
2) 2
B)
2x + 4 D)
D) (x
(x +1)2(x
+ 1)2 (x -‐ 2)2
2) 2
6
C)
5x + 7
1
D)
D)
30x + 42

1
2+ -
X
-- 1
2- -
X

If x I-
76O, which of the following
0, which following is equivalent
equivalent to
to
the given expression?
the given expression?
2x -‐ 1
A)
A)
2x + 1
2x + 1
B)
B)
2x -‐ 1
4x22 -‐ 1
4x
C) x2
x2
D
D)) -‐ 1

50
THE COLLEGE PANDA
THE COLLEGE PANDA
_\
A calculatoris allowed on the following
A calcu I‑
lator is allowed on the following
questions.
quest ions. ):
X x
--+~
‐ --+2(2 X- x)a
I ‐ “ 2 Which
” H
x- 2
the following
Which of the following is
"
is equivalent
equivalent to
to the
the
3x + 8x2
3x33 + 8x 2 ‐- 4x expression above
expression above for x 752
=/=2 ?
?
X
712‐11x‐7
7x2-11x - 7 A)
A) _L
- x- 2
x‐2
Which of
Which of the following is the
the following the sum of the
sum of the two
two rX
polynomials
polynomials above?
above? B) 2(x
B) -_ T {f - 2)
2)
3 + x 2 - 15x -
A) 3x3+x2‐1Sx‐7
A) 3x 7 C) xX
C) 2(x - 2)
B) 3x3 +
B) 3x 3 1Sx2 -‐ lSx
+15x2 15x -- 77 2(x ‐ 2)
3x
10x55 -‐ 7x -‐ 7
C) 10x D) 3x
D) 2(x - 2)
D) 4 + 3x 3 2
D) 1Sx
15x4 + 3x3 -‐ 15x
15x2 -‐ 7 2(x ‐ 2)

(5a +
+ 3Ja)
WE) -‐ (2a + s fi )
+ sJa)
Whic
Whichh of
of the
the follow is equivalent
ing is
following to the
equivalent to the
expression
expression above?
above?
A)
A) -‐2a\/E
2afa
B) a\/E
B) afa
C 3 -a ‐2/a
C) ) 3a 2 fi
D) 3a
D) 3a+8\/E
+ BJa

9(2y) 26 22 7
2 2 + 2(6y)
If y f=O,what
If yyéo, what is the value
is the MW · B(3y)2
value of ?·
ConstructingModels
Constructing Models
model questions
Constructing model
Constructing questions require you to represent
require you represent real‐life as expressions,
quantities as
real-life quantities equations,, and
expressions, equations graphs .
and graphs. i.
Questions
Question s of this type be found
can be
type can found in several chapter s in this book,
other chapters
several other chapter is specifically focused
but this chapter
book, but
on the ones
on pertain to any of the conventional
't pertain
don't
ones that don conventional model linear, quadratic,
types (e.g. linear,
model types Now
exponential). Now
quadratic , exponential).
that doesn’t mean this chapter’s
doe sn't mean actually quite
questions are actually
Most of the questions
chapter's difficult. Most simple, and
quite simple, and there be
won 't be
there won't .
any nnew
e w concepts here . We’ll
concept s here. two examples
We'll just do two leave the rest to you in
and leave
example s and in the chapter
chapter exercise. l

EXAMPLE 1: At a
EXAMPLE a school, are a grade
there are
school, there levels with
grade levels with b students each grade.
students in each grade - If
If the
th.e school buys n
schooll>µys 71
stickers
stickers to be distributed equally
bedistributed among the
equally among which of the
students, which
the students, following gives
the following thenumber
gives the s ckers
number of stickers
each student
each student receives?
receives?
ab
ab an C) bn
bn rt
A)
A) 7 B) an
B) 7 C) 7 D)~El?
D)
n b a ab

a total of (a)(
The school has a b) =
(a) (b) : ab
abstudents. number of stickers each
student s. To find the number divide
student receives, we divide
each student
number of stickers n by the number
the number student s: ‐”‐.
number of students: Answer (D) .
~: . Answer
ab
ITe:IJ.
EXAMPLE 2: Water was pumped into
was pumped into a tank at a constant rate imtil
constant rate was foll.
until it was full. The tank
tank.was th drained
was then
at
at a
a slower than it had
rate than
slower rate been filled. Which of the
had been following
the following graphs could represent the total amount
could rep ent-thetotal amount of
water the tank
water in the tank versus time?
versus time?
A) B) C) D) 5;
0
s....fil
§ ....
....
s
~
....2
~~
.....
0
s.
§2
-
0
“6
...Iii l‐i
: I
gt at as ~j
533 s1
s“ s;
~~ i” ~it
i
0~ 0~
~ l
Time
Trme Time
Tlllle Time
Time Time
Jhne

Water being
Water be~ng pumped into the
pu~ped into the tank
tank should be represented
should be represented by
by a going up
line going
a line to the
and to
up and the right (positive slope).
right (positive slope) .
Water being
Water bemg drained should be
dramed should be represented
represented by line going
by aa line going down and to
down and the right
to the right (negative slope).. That
(negative slope) leaves
That leaves

~s with
us answers C and
wi~ answers~ D. Since
and D. Since the tank was
the tank drained at
was drained slower rate
at aa slower rate than was filled,
than itit was filled, the
the answer is ~(D)
answer is the
the
lme going
line down is
going down o t as
1snnot as the
steep as
as steep the line going up
line going up..

52

\
THE COLLEGE PANDA
THE COLLEGE PANDA

CHAPTEREXERCISE:Answers for this chapter start on page 285.

A calculator should
should N be used
O T be
NOT used on the
following questions.
following questions.
An intemet service provider
internet service charges
provider charg es a one time
a one time
setup
setup fee of $100 andand $50 each month for
each month for service.
service.
customers join at the
If c customers the same time and
same time and are
are on
carpenter lays
A carpenter bricks per
lays x bricks per hour
hour for y hours
hours the formm months,
service for
the service months, which of the
which of following
the following
and
and then lays ~
then lays g bricks hour for 2y more
per hour
bricks per more expressions represents
expressions represents the total amount,
the total amount, in
2
dollars,
dollars, the provider has
the provider charged these
has charged these
hours. terms of x and
hours. In terms and y, how many bricks
how many bricks did
did
customers?
customers?
he lay in total?
he total?
100c + 50m
A) 100C
A)
A) 2xy
A) 23:31
B) 100c
100i: + 50cm
5
B)
B) gxy
xy C) 150cm
2
C) 5xy
Sxy D) 100m +
100m + 50cm

3
D)
D) gx+3y
2x + 3y

At aa math team competition,


math team there are
competition, there m schools
are m schools
with n students
with from each
students from school. The
each school. The host
host
school wants
school wan ts to order enough pizza
order enough h that
such
pizza suc that
A cheese vendo r current ly has 175 pounds of there are 2 slices for each student.
there are student. If there are
are 8
mozzarella avai lable for sale . If each pound of slices in one
one pizza, which
pizza, whic h of the following gives
the following gives
mozzarella sells for $8.75, which of the following the number of pizzas
the number the host
pizzas the schooll must
host schoo must
functions gives the amount of mozzarella M, in order?
order?
pounds, still available for sale after d dollars mn
nm
worth has been sold? A)
8
d mn
A) M (d) = 175 - - B)
8.75 T4
B) M(d)
B) = 175 - 8.75d
M (d) =175‐8.75d m+2n
m + 2n
C) 8
8.75 8
C ) M(d)
C) M (d) = 1175
7 5-‐ -8 ? D) 2mn
D)
d
D)
D) M(d) =175(8.75)‐d
M (d) = 175(8.75) - d

A retail store has monthly fixed costs of $3,000


and monthly salary costs of $2,500 for each
emp loyee . If the store hires x emp loyees for an
entire year, which of the following equations
represents the store's total cost c, in dollars, for
the year?
A)
A) cc == 3,000+2,500x
3,000 + 2, 500x
B)
B) cc==12(3,000
12(3,000 + 2,500x)
2, 500x)
C) cc = 12(3,
12(3,000) + 2,500x
D) cc== 3,000
3,000 + 12(2,500x)
12(2,500x )

53
CHAPTER 7 CONSTRUCTING
CHAPTER MODELS
CONSTRUCTING MODELS

A calculator is allowed on the following


questions.
A manufacturing plant increases the
temp erature of a chemical compo und by d
degrees Celsius eve ry m minut es. If the
compound has an initial temperature oft Kaiba began commu te by biking
began a 5-mile cormnute biking for 4
degrees Celsius, which of the following miles to a area. He stopped
rest area.
a rest stopped atat the rest area
the rest
expressio ns gives its temperature after x 15 minutes and
for 15minutes and then walked for the
then walked
minutes, in degrees Celsius? remainder
remainder of the commute.
commu te. IfIf Kaiba bikes faster
bikes faster
m x++ t
mx than he
than he walks, following graphs
which of the following
wa lks, which grap hs
A)
A) cou ld represent
could represent his
his commute?
commu te?
d
m
mdd++ t A)
A)
8)
3)
xX "U
-0
QJ
d 2
o:i 5
C)
C) t+‐d‑
t +-
Q)

~ ,...._4
mx
E: 4
F r:~ 33 .
D)
D) t +-
dx
Hi"m w
u Ez
QJ · -

C: -
2
s.is 11
-‐
6"'
D
(I)
0 0-L---+---+----+-➔
15
15 30
30 45
45
Time (minutes)
(minutes)

A cupcake
cupcake store employs
emp loys bakers make boxes
bakers to make 8)
B)
of cupcakes.
cupcakes. Each
Each box contains cupcakes and
con tains x cupcakes and T:
-0
QJ
baker is expected
each baker expected to produce cupcakes
produce y cupcakes o:i
2
Q) 5.
~ ,...._4 .
each day. Which
each Which of the following
following expressions
expressions E? 4
gives
gives the number
number of boxes
boxes needed
needed for all
all the
the m- ~
[r= 2_ 3
QJ ·- 3
cupcakes produced
cupcakes bakers working
produced by 3x bakers working for 4 a5
u E 2
s2 C: -
2
11
days?
days? (I)
6"'
'‐ 0 -"""'--+---+----
o

12x2y
A) 12x2y D 15 30 45
3 Time(minutes)
Time (minu tes)
3y
B)7y
8)
4 C)
C)
12x2 ‘U
-0
C) QJ
y 2
o:i
0.0 5
~ ,...._4
«EA 4
D) 12y
r=
P g~ 33
·;:
~
éé
s2
z211
C: ..::,
U)
-6"' o
0
5 15 3030 45
At
At a shop touri sts, the price
shop for tourists, one souvenir
price of one souvenir Time (minutes)
(minutes)
is a dollars.
dollars . Each additional souvenir purchased
additional souvenir purchased
after the first is discounted
after 40 percent. Which
discounted by 40 percent. Which D)
D)
following equations
of the following equations gives
gives the total cost C,
total cost C, -0
QJ
in dollars,
dollars , of purchasing
purcha sing n souvenirs, where
souvenirs, where o:i 5
> 4
n> > 11?? <1l ,...._
r=~ 3
A) CC == aa +
+(n‐1)(O.4a)
(n - 1)(0.4a) QJ · -
u E 2
B)
8) C = a + (n -‐ 1)(0.6a) ~ - 1
C) C := a + n(0.6a)
n(0.6a) 6"' 0
15
15 30
30 45
45
D) C =
D) = 0.6an Time (minutes)
(min ut es)

54
THE COLLEGE PANDA
THE COLLEGE PANDA

Mike starts driving


Mike starts driving toto work
work and records his
and records his At a video
video game arcade, d dollars
game arcade, dollars can be
can be
distance
distance from
from home, miles, every
home, in miles, every 1010 minutes.
minutes. exchanged
exchanged for p tokens.
tokens. If
If each requires w
game requires
each game w
His distance
His distance from increases slowly
home increases
from home slowly at first
first tokens to play,
tokens to which of
play, which the following
of the gives the
following gives the
due
due to traffic, then increases
traffic, then quickly as
more quickly
increases more as per game,
cost per
cost game, in dollars?
dollars?
clears up.
traffic clears Which of
up. Which the following
of the following graphs
graphs
2 w
could illustrate
could Mike's distance
illustrate Mike’s from home
distance from home A)
A) dp
during
during hishis drive?
drive? dP
A)
A) 8)
B) id
pw
Q.I
E
0
2m
25,c- --r----.--r----,-----.-.
C)
dw
C) di”
.c
2m
20 >--+--~_.,_---+-~------<
§ i 15 f---+---+----+- - ---.------i
J:: :-::: m D) d_p
p
F

Q.I E 10 1--+-----+-~--±-__. m D) dp
u - w
§ 5
(J')

0 10 20 30 40 50 60
Time (minutes)
prepare for landing,
To prepare landing, a
a plane descends so
plane descends so that
that
B)
3) its altitude
altitude decreases
decreases at a constant rate from
constant rate from
Q.I
Q)
E 25 24,500 feet to 17,900 feet in 12 12minutes. Which of
minutes. Which of
o
0 the following
the equations gives
following equations gives the altitude A,
the altitude A, in
in
.:
.c 20
E
E -A feet, of the plane minutes after
plane t minutes descent
its descent
after its
0 ~ 15
é
J:: =
:-::: began, for O fi
began, for0 :S tt :S
s 12?
12?
a sE
Q.I
u -
10
= 17,900 ‐- 550t
A) A =17,900
E@
.....
a(J')
0
5
B)) A =
B :17 , 9 0 0 + 550:
17,900 550t
0 0 100
Q 0 210
0 3200 30
4 0405 50
0 660
0 C) A z= 24, 500 ‐- 550t
24,500
Time (minutes)
(minutes)
D) A == 24, 500 +
24,500 + 550t
550t
C)
Q.I
O.)
E
c0
25
.<:
.c 20
0E- A taxicab charges a dollars for the first mile
... ..!!:! 15
(J')

..... ·- 10 traveled and b dollars for each additional mile. lf


8
u -f:
Q.I E
,_[ a
a particular
particular passenger
passenger traveled more than
traveled more than one
one
E@
.....
(JJ
..<e
5
0
;
I
mile during
mile during a ride that
a ride which of
cost $24, which
that cost of the
the
i5 0 100
D 0 210
0 3200 30
4 0405 50
0 6600 following
following represents the distance,
represents the distance, in
in miles,
miles, the
the
Time (minutes)
Time (minutes) passenger traveled during
passenger traveled the ride?
during the ride?

D)
A)
A)
24‐ba‐b
24 - a - b
D)
b
Q.I

E 25 B) 24‐ba+b
24 - a + b
0
.c 8)
20 -·
'I b
E-
0 ~ 15
J:: :-:::
- I

1 --,- C)
C)
24+l:1‐b
24 + a - b
Q.I E 10 b
u -
;a
..... 5 D) 24b‐a
D)
24 - a
(J')
b
0 00 10
00 10 20
20 30
30 40
40 50
50 60
60
Time (minutes)
(minutes)

55
55
CHAPTER 7 CONSTRUCTING
CHAPTER CONSTRUCTING MODELS
MODELS

Mark started working as an inspector for a large To move into a new


move into studio space,
new studio themm members
space, the members
construction company on June 1, 2019. of an
an art
art club agreed to split
club agreed split the first month’s
month's
According to his contract, his annual salary will rent of r dollars
rent equally among
dollars equally among themselves.
themselves. IfIf k
increase by $15,000 on the first day of June each members fail to pay
of the members pay their which of
share, which
their share,
year. Which of the following graphs could the following
the represents the additional
following represents additional amount,
amount,
model Mark 's annual salary, in dollars, x years in dollars,
dollars, that each of the remaining
that each members
remaining members
after June 1, 2019? must pay
must cover the first month
pay to cover month’s rent?
's rent?
A)
A) rr
A)
A)
a m
111-‐ k

E B)
kr
73 B)
m
m -‐ k
E
b kr(m
kr(m -‐ k)
sro
N
C)
C)
111
111
(V)I )
76 kr
3 D)
D)
m(m ‐- k)
5123456
1 2 3 4 5 6
B)
B)
75
s
T:
E
E‘
N
76
m
a5
f:
5 1 1223344 55 66
C)
E"
s
3
E
E’
2
50°
76
3
E
< 1 12 23 3 44 55 6
D)
D)

'----------+X
1 2 3 4 5 6

56
e

Manipulating &
Manipulati)Jg
Solving Equations
On the SAT,
On there is aa huge
SAT, there emphasis on
huge emphasis on equations.
equations. To get
get these types of questions
these types right, yott
questions right, you must learn how
must learn how
to isolate
isolate the variables expressions you
and expressions
variables and you want. we’ll cover
First, we'll
want. First, several useful
cover several techniques
useful techniquJs in dealing with
dealing with
equations that
equations you may
that you be familiar
already be
may already familiar with.
with. I

1. Don
1. Don’t like terms
't forget to combine like
should be
You should be ruthless finding like
ruthless in finding like terms combining them.
and combining
terms and them. Doing so will simplify
Doing so things and
simplify things and allow you
allow you
figure out
to figure the next
o u t the step.
next step. I

EXAMPLE1: If 2(a + b + 2c + 3d +I) =3a+2b+4c+6d,findthevalueofa.


EXAMPLE1:If2(a+b+2c+3d+1)= 3a + 2b + 4c + 6d, find the value of a.

four variables
same four
The same variables are sides of the
both sides
are on both equation, a,
the equation, b, c and
a,b,c and d. That should tell you
That should you to distribute the
on the
distribute on
left side and then
side first and then combine l i k e terms
combine like simple but
Sounds simple
terms.. Sounds but you
you won
won't't believe how many
believe how students forget to
many hudents
do this, especially
do especially in the middle of aa more
the middle complex problem
m o r e complex problem..

2((aa++ bb + 22c 2 33aa +


6 + 33dd + 11)) = + 22bb + 44c
c + 66d
d

b,c,
The b,
The variables cancel
and d variables
c, and quite nicely.
cancel quite

211+25+M+fid+223a
~ +26+Arf+fad
+ ~ + ¥ +M + 2 = ~ +~ +¥ + M
IT]=
: a

\
\

\ “51
MANIPULATING & SOLVING
CHAPTER 8 MANIPULATING
CHAPTER EQUATIONS
SOLVING EQUATIONS

2. Square
2. equations correctly
Square equations
squaring equations
When squaring
When equations to remove square root,
remove aa square most important
the most
root, the thing to remember
important thing remember is that you’re not
that you're not
squaring individual
squaring elements‐you’re
individual elements squaring the
-y ou're squaring the entire side..
entire side

EXAMPLEZ:
EXAMP1,.'E2:
·"'
\/a_b=a‐b
vab = a- b
If a,>.
U a >. 00 and O,tthe
and b > 0,'. equation above
h;eequation is equivalent
above is equivalent to
to which
which of the following?
the following?

A)ab=az~-b2 B)ab=az+b2 C) 2ab = a 2 - b2


C)2ab=aZ‐b2 D) 3ab = a2 + b2
D)3ab=az~l-b2

·TThe square root


h e square the problem
root in the scream to you
should scream
problem should you that
that the equation should
the equation should be squared. Most
be squared. know
students know
Most students
the square root
the square should be
root should be eliminated, but here's
eliminated, but the common
here’s the mistake they make:
common mistake make:

ab=aZ‐b2
ab = a2 - b2

square each
They square each individual element. However,
individual element. However,this WRONG.When
this is WRONG. modifying equations,
When modifying you must
equations, you apply
m u s t apply
any given
any operation to the
given operation the entire like so:
entire SIDE, like

(fa‐bf =
(M)2 = (a
(a‐- b)2
b)2

helps, wrap
If it helps, wrap each
each side parentheses before
side in parentheses applying the
before applying operation. By the
the operation. way, the
the way, same holds
the same holds true
true for
all other
other operations, multiplication and
including multiplication
operations, including and division When you
division.. When multiply or divide
you multiply divide both
both sides an
sides of an
equation, what
equation, what you're doing is wrapping
actually doing
you’re actually wrapping each side in parentheses,
each side because of the
but because
parentheses, but the distributive
distributive
property, just so
property, it just happens that
so happens thatmultiplying
multiplying or dividing each
or dividing each individual gets you
element gets
individual element you the
the same result. For
same result. For
example, if we
example, we had
had the equation
the equation
x + 22 =
zy
we wanted
and we
and to multiply
wanted to multiply both by 3,
sides by
both sides 3, what we’re actually
what we're doing is
actually doing is

3(x + 2)
3(x 2) = 3(y)

which turns
which out to be
turns out the same
be the as
same as
3x ++ 66 =: 3y
3x 3y
Anyway, back
Anyway, problem:
the problem:
back to the

(M)2
MW = (a -‐ b)
= (a
2
17)?

ab:a2‐2ab+b2
ab = a 2 - 2ab + b2
3ab =
3ab 2
: aa2 + 2
b2
-+- b

is (D) .
answer is~-
The answer

Another common
Another c o m m o n mistake squaring each
mistake is squaring side before
each side the square
before the root is isolated
square root on one
isolated on one side.
side. For example,
For example,
let’s say we
let's we wanted find the
wanted to find solutions to the
the solutions following equation:
the following equation:

Vx+5+lzx

square each
can’t square
We can't each side right away
side right away to get
get rid
rid of the square root.
the square root. We first have
have to move the ”1”
move the "1" from the left
from the
side to the
side theright side:
right side:
x + 5=: x-
v'x+5 x ‐ 1l

58
58
\~ ___ ____ _____ _ _ THE
THE COLLEGE
__ C PANDA
_O_L_L_E_G_E_E
_'AND
__ :A
_

And
And n o w we can
now can square both sides.
square both sides .

52f == ((xx ‐- U1)22


( v x++5)
(Jx

xx+5:x2‐2x+1
+ 5 = x 2 - 2x + 1
00=x2‐3x‐4
= x 2 - 3x - 4
0 := ((x
x ‐-44)(x
) ( x ++ 1l))
x
X ==‐ -1 1,4
,4

So, the solutions are ‐1


solutions are and 4, but
- 1 and but hold We're actually
hold on! We’re not done
actually not yet. When
done yet. When there
there are square roots
are s~uare in the
roots in the
original
original equation, we have
equation, we solutions by
check for false solutions
have to check testing each of our values in the original
by testing each of our values in the original equation. equation.
50
So when hand side
the left hand
when x = -‐ 11,, the J -‐11 +
side is \/ + 5+ + 11 =
z 33 and the right
and the right hand side is
hand side The values
is ‐- 11.. The values don't match,
don't match,
so - 1 is actually
so ‐1 actually n o t a solution.
not When xx =
solution. When = 4,
4, the left hand
the left hand side is \/4
side is J4 ++ 55 ++ 11 := 44 and
and the the right side is
hand side
right hand is 4.
4.
In this case, the values
case, the from both
values from sides match
both sides match soso4 4 is
is a
a solution.
solution.
Why do false solutions
solutions occur? squaring both
Because squaring
occur? Because both sides
sides has
has the effect of
the effect of turning
turning negative
negative values
values into
into
positive
positive ones, which sometimes
ones, which sometimes causes mismatch on both
causes a mismatch sides. If we plug
both sides. If we plug x = - 1 into J x + 5 = xx ‐- 11 from
x = ‐1 into \/ x + 5 = from
above, we
above, see that
can see
we can the left hand
that the side is
hand side is 2
2 and the right
and the right hand side is
hand side One is
is -‐ 22.. One is positive
positive andand the other is
the other is
negative.
negative. Once we square
Once we square both sides, however, this distinction is lost since both sides
both sides, however, this distinction is lost since both sides become 4. become 4.
In summary,
summary, when you’re dealing
when you're with square
dealing with roots in
square roots in an
an equation, square the
equation, square the entire sides, which
entire sides, may require
"'lhich may require
you to move
you something from
move something one side
from one to the
side to other, and
the other, check for
and check by plugging
false solutions by plugging your results
for false solutions your results into
into
the original
the original equation.
equation. I
This may
This like aa lot
seem like
may seem to watch
lot to u t for,
watch oout but for
for, but most of
for most of the questions involving
the questions involving this type of
this type o:fequation
equation on
on the
the
SAT, you
SAT, you can avoid all
can avoid the potential
all the potential pitfalls by plugging
pitfalls by plugging in
in the answer choices
the answer (see tip
choices (see #8) rather
tip #8) solving
than solving
ra~er than
equation algebraically.
the equation algebraically.

3.
3. Square equations correctly
o o t equations
Square rroot correctly
Now, when
Now, it comes
when it to taking
comes to the square
taking the root of
square root both sides,
of both most students
sides, most students forget the plus
forget the plus or minus( (±i )).. Always
or minus Always
remember that an
remember that equation such
an equation asx2
such as z 25
x2 = 25 has
has two solutions:
t w o solutions:

f o z ¢ fi
x = i 5

However,
However, this only applies
this only when you’re
applies when taking the
you're taking the square root to
square root solve an
to solve equation. By
an equation. By definition,
definition, square
square
roots always
roots refer to
always refer the positive
to the root. So,
positive root. \/§ =
So, v'9 = 3,3, NOT And fy'x =
± 3. And
NOT 3&3. : -‐33 is
is not possi~le (except
n o t possible when
(except when
working with non-real
working with numbers, which
non-real numbers, which we'll at in
look at
we’ll look in a future chapter). The plus or minus is only necessary
a future chapter). The plus or minus ls only necessary
when the square
when_the square root is used
root is as a tool
used asa tool to solve an
to solve an equation.
equation. That way, we
That way, get all
we get the possible
all the solutions to
possibl1 solutions the
to the
equation.
equation.

EXAMPLE3:
EXAMPLE 3: If + 3)2
If (x + = 121,
3) 2 = 121, what is the
what is sum of
the sum two possible
the two
of the values of
possible values of x
x??

u+3fi=1m
(x + 3) 2 = 121
tJ(xh ++53)22 =
= i±¢✓121
u1
x ++ 33 =±
= i 11
H
= -‐ 33±i 11
xX = u
Sox could
Sox could be either 8
be either 8 or
or -‐ 14.
14. The s u m of
The sum of those w o possibilities
those ttwo possibilities is
is j ‐6
- 6 I..

59
CHAPTER 88 MANIPULATING
CHAPTER MANIPULATTNG &
& SOLVING EQUATIONS
SOLVING EQUATIONS

4. Cross-multiply when
4. Cross-multiply when fractions
fractions are equal to
set equal
are set each other
to each other
Whenever a
Whenever fraction is
a fraction equal to
is equal fraction,
another fraction,
to another

a _ E
5 C

b_ d
you can
you cross-multiply: ad
can cross-multiply: ad =
: be.

4 10
10 .
EXAMPLE4: If 5xx = ‘3‐', what is
EXAMPLE4: If is the value of
o f x ??
5 3
4
s, _ 1_0
10
5X=3
5 _ 3
12x = 50
12x 50

_ 2:
x_ é
x=l6I

2 1 .
EXAMPLE 5: If
EXAMPLE If ,/:--- _]___ =
‐ 2 ‐ ‐ -‐ ‐- = 0, !Sthe value of
0~what is o f x ??
xX -‐ 44 xx+ +2

2 1
xx2‐ 4 -_ Xx + 22 =
2- 4 _O0
2 _ 1
+2
xx7-2 -‐ 4 _ Xx +
22(x
(x + 2)
2) =
z x2
x2 -‐ 44
2x
2x ++44=
z x2
x2 -‐ 44
00 =
z x2
x 2 -‐ 2x
2x -‐ 88
0O=: ((xx -‐ 44)(x+
) ( x + 22))
x
X =
= 4,
4, -‐2
2

we plug
If we plug these values back
these values back into the original
into the equation, we'll
original equation, we’ll see that x =
see that =4 is a
4 is solution but
a solution = ‐2
but x = - 2 is
is a
a FALSE
solution because
solution because it causes
causes division Therefore, the
division by 0. Therefore, the answer I. As we learned
answer is [±]. learned before, solutions can
before, false solutions can
occur when
occur an equation
when an has square
equation has square roots, but they
roots, but also occur
can also
they can occur when
when there variables
are variab
there are in the
les in denominator
the denominator
of aa fraction. Though you
fraction. Though see false solutions
won’t see
you won't solutions very often on
very often on the
the SAT, it's good practice
it’s aa good practice to always confirm
to always confirm
your results
your results in these
these two cases.
t w o cases.

60
COLLEGE PANDA
THE COLLEGE PANDA

5. Factoring should be in your toolbox


should be toolbox
Some equations
Some equations have variables that
have variables are tougher
that are to isolate
tougher to For a
isolate.. For a lot of these
lot of equations, you
these equations, you will
will have to do
have to do
some shifting
some around to factor out
shifting around the variable you want.
out the variable you want.

EXAMPLEG:
EXAMPLE6: a
a
b = --
=3a+c
3a +e
Which the following
Which of the following expressasainterms
expresses a in terms ofbandc?
of b and e ?

b
bee b
bee 1 -‐ 33bb
C) 1 3b+1
D) 3b +1
A)1‐3b
A) 1 - 3b B) 3b + 1
B)3b+1 C) be
be D) be
be

_ 11
b =- a-
‐3a+c
3a + e
b ( 3 a+e)=
b(3a + c ) : aa
3 a b++be=
3ab b c : aa
b e z aa ‐-S3ab
be= ab
b c : aa(1
be= ( 1 ‐- 33b)
b)
bc
be ‐a
--= a
1 -‐ 33b} ; ‑

what we
See what We expanded
did? We
we did? expanded everything and put
out and
everything out every term containing
put every containing a
a on the right
on the right side. we
Then we
side. Then
were able
were u t a and
able to factor oout and isolate answer is ~(A) -.
isolate it. The answer

EXAMPLE7:
EXAMPLE7:
xx4+3x3+x+3=0
+ 3x3 + x + 3 = 0
4

What
What is one possible real
one possible value of x
real value x for which above is
equation above
the equation
which the is true?
true?

xx4+3x3+x+3zo
4
+ 3x3 + x + 3 = 0
x3(x+3)+(x+3)=0
x3 ( X + 3) + (X + 3) = 0
(x+3)(x3+1)=0
(x + 3)(x 3 + 1) = 0
x :=‐ -3 30orr -‐ 1
Once we
Once factored out
we factored x3
out x 3 from the first two terms, further factoring was possible with the (x + 3) term. How
two terms, further factoring was possible with the (x + 3) term . How
would
would you know to do
you know do this? Experience.
Experience .

61
61
CHAPTER MANIPULATTNG & SOLVING
CHAPTER 8 MANIPULATING EQUATIONS
SOLVING EQUATIONS

6. Treat complicated
6. expressions as
complicated expressions as one unit
one unit

EXAMPLES:
EXAMPLES:
x
x3+x2+x= -‐‐‐-‐-‐1-’-‘‑
MCI-FE)
of the following
Which of following gives min
gives m x?
in terms of x?

A) _(x4+x3+x2)\/;j§
(x4+x3+x2)R B) _ x_312 C) ~(x3+x2+x)(x+-‐)
” ‘ m ‐(x+-( fixl) g ) ”‘M(x4+xa+xz)(x+;) ' " ‘ fi g
A) m =

x
/ x ‐ 1‐
x
D) m =
(x3+x2+x)
D ) m = (x3
(.
+ x2 + x) x +
l)
(x+;)
x
Don’t let
Don't big and
the big
let the complicated expressions
and complicated expressions freak
freak you
you out. these complicated
out. Treat these complicated expressions as one
expressions as unit
one unit
variable, like
or variable, like so:
B
A=~
A *me
fi
Multiply both
Multiply both sides by m.
sides by m.
B
mA =z -C‐
mA
C
Divide both sides
Divide both by A.
sides by
m_1
_ AC
Finally, plug
Finally, plug the
the original expressions back
original expressions back in
in..

1
xJx-.!.
x x‐ ‑
x
nt = X
3 2
(x + x + x) ( x + i)
Answer (D) .
Answer~.

62
THE CKALEGE
cpLLEGE PANDA
PANDA

EXAMPLE 9:
EXAMPLE
(x+1)2+5(x+1)‐24=0
(x + 1)2 + 5(x + 1) - 24 = 0
If xx > 0,
If what real
0, for what real value of xis
value of x is the
theequafiOn above·ttrue?
'E!quation above r u e"'""~
? “1

Treat (x +
Treat + 1) as
as one
one unit and call it A.
unit and

(x+l)2+5(x+1)‐24:O
(x + 1) 2 + S(x + 1) - 24 = 0
A2+5A‐24=0
A 2
+ SA - 24 = 0
( A++ B)(A
(A 8 ) ( A- ‐3)3 )=: 00
+ 11+
((xx+ + B)(x
8 ) ( x++l 1- ‐ 3)
3 )=‐ O0
( x + 99)(x
(x ) ( x-‐ 22)) == 0
x == ‐-99oorr 2

Because the question


Because the stipulates
question stipula 0, the
that x > 0,
tes that the answer is I].
answer is I.

Be comfortable
7. Be solving for expressions,
comfortable solving expressions, rather than any
any one variable
one variable

EXAMPLE 3x +
EXAMPLE 10: If 3x 9y = 9,
+ 9}; 9, wha is the
whatt is value of x +
the value + By?
3y ?

Get in the looking for what


habit of looking
the habit what you
you want before you
want before you solve anything specific.
solve for anything specific. Is any way
there any
Is there to get
way to get
the solving for x and
withoutt solving
answer withou
the answer and y?

Dividing both
Yes! Dividing
Yes! of the
sides of
both sides equation by
given equation
the given by 33 gives + By:
gives x + [I].
3y = I.

EXAMPLE 11: If ~
EXAMPLE 5== 3,
3, what is the
what is value of iY ??
the value
y 2X
1 1 z
C) ~ D) ~3
A) E B) 5 C) 3 D) 52

we have
Here, we
Here, no choice
have no but to so
choice but solve expression.
the express
lve for the We're given
ion . We’re given x over but we
over y but want y over
we want over x. We
We can
can
flip the given equation
the given equation to get
get
I/ 1
!,.._ =-
X 3

Then we can
Then we divide both
can divide side s by
both sides by 22 to obtain
to ob ix
tain the % we’re
we're looking
looking for:
for: ix=
21x
1
: ‐3~
2 3
= 1.
1
= 8“ Answer[EI].
Answer (A) .

63
63
CHAPTER
CHAPTER 8 MANIPULATING
MANIPULATING & SOLVING EQUATIONS
SOLVING EQUATIONS

8. In cases, you
some cases,
In some need to plug
may need
you may answer choices
plug in the answer choices or guess and check
guess and check
When can't find a
you can’t
When you ”mathematical” way
a "mathematical" get the
way to get the answer, you have
answer, you have ttwo
w o options: plug in the
options: 1) plug answer
the answer
choices or 2) guess
choices guess and
and check. Both are
check. Both strategies that
valid strategies
are valid that you shouldn't be afraid
you shouldn't be afraid to use. Not only does aa
use. Not only does
”brute force" approach
"brute force” often tum
approach often turn out be quite
out to be quite efficient, but, for some
efficient, but, questions, it is the
some questions, only way
the only get the
way to get the
answer.
answer .

EXAMPLEIZ:
EXAMPLE 12:
v22~x=x‐2
What is the
What set of all solutions
the set the equation
solutions to the equation above?
above?

A) {- 3, 13}
A){‐3’13} B){- 3,6}
B){‐3/6} C){13}
C) {13} D){6}
D) {6}

We could solve
We could by squaring
solve for x by squaring both
both sides,
sides, but plugging in the
but plugging values from
the values from the answeranswer choices actually
choices is actually
much quicker and
much quicker and easier. We just
easier. We have to see
just have which of the
see which the values (
values (- 3,‐ 3 , 6, or 13) satisfy the equation. When
satisfy the equation. When
x=
= -‐ 33,, the left hand
the left hand side m =
side is y'25 z 5 and
and the right hand
the right side is -‐33 ‐- 2 =
hand side z -‐ 55,, so
so -‐33 is not in the
the solution
solution set.
set.
When x =z 6,
When 6, the left hand
the left hand side /16 == 4 and
side is m and the right hand
the right hand side
side is 6 -‐ 2 == 4, so so 6 is in the
the solution set. At
solution set.
this point,
this point, we can the answer
can tell the is m
answer is~ based on the
based on the available
available choices,
choices, b u t let’s
but let's test = 13
test x = 13 just be sure.
just to be sure.

When x =
When z 13, the
the left hand
hand side \/§ =
side is ,/9 : 3 and
and the right hand
the right hand side 13‐- 2 = 11, so
side is 13 so 13
13is
is not in the solution
the solution
set.
set.

EXAMPLE
EXAMPLE 13:
2 (x3 -‐- 4) = 4x
xx2(x3
If xxisis an
H an integer,
integer, what is one
what is possible solution
one possible solution to
to the equation above?
the equation above?

Assuming we can't
Assuming can’t use a calculator,
use a there is no
calculator, there easy way
no easy way to solve given equation
the given
solve the equation by hand, and there
hand, and there are no
are no
choices to work
answer choices
answer from. A situation
work from. like this
situation like calls for guess
this calls and check.
guess and Typically, you
check. Typically, you want to start
want to with
start with
small
small numbers like xx =
numbers like : -‐ l1,0, 1, and
, O,1, and 2.
Il won’t work through
won't work the guess
through the guess and
and check process here
check process here since
since it’s obvious what
it's obvious need to
you need
what you to do. It turns
do. It out the
turns out the
answer
answer is is l.
[II.

64
THE PANDA
COLLEGE PANDA
THE COLLEGE

EXERCISE1: Isolate the variable in bold . Answers for this chapter start on page 287.

A=7rr2
l. A= nr 2 If t = gax,
22. If ~ax, find
find ax in terms
terms of
oft.t.
2. C ==22nr
7rr
23. If 3x + 6y = 7z, find x + 2y in terms of z.

fig...
3. A = ~bh 24. If x +5 = 2b, find 2x + 10 in terms of b.

4.. V
V :=I lwh
wh ”dz‐fl1 =a,find4tintermsofa.
25.. If a;
25 = a, find 4t in terms of a.

V=7rr2h
p ‐Iz
p-h 2. .
2 p
2
26.. If p + h
6 Ifp+h =z -33, fmdh
find 11
‑ ·
.V:7rr2h I

: a¥2 ++ b§2
7.. c82 = 1 + 2r = -1 , find t m
27. If lltztrzéfindtintermsofr.
27 . If --
1- t 2
. terms o f r.

8..V=s3
V = s3
z, then find x2.1/ terms z.
28. If x”
xY =
: 2, then find xz?’ in terms of 2.
9.. S
S227trh+27tr2
= 2mh + 2m 2

4x411
4x
a -2
2 C If
29. If p(x55 -‐ x14),
= p(x 4 ), what is pin terms
what p in terms of x?
x?
b =d
lO. b _ d x3 - X2

ll.
‘ab1 _ £
b _ d
C
d 30.1f2-‘(x3
30. 1f2x(x 3 ‐ -1)
1)
x
= m(x2
= m(x + 11)-:21
)‐ ‐,w
x2
h a t i s m in
, whatismin
2

terms
terms of xx??
y ==m
12. y +bb
mxx +

”M_x3=
5
5x2 -‐ 3
l,whatisnintermsofx?
31. lf Jx+2 l - x3 = 2...,
n
nx x
whatisnintermsofx?
"IZ‐y2_y1
xz‐xl
If a(b2 +
32. lfa(b2 + 22)) + c = 5(c + 1)3,
= S(c l)3,what isain
what is terms of
a in terms of
m=y‐2_y1 b and cc??
band
xz‐x1
7x2 +
7x2 +33 h . k . f
33. If k(x 2 +
If k(x2 4) +
+ 4) + ky =
z , what is k m
w at 1s terms of
in terms o
15. v02 = 11 + 2as
2 = “22 +
22
a X xandy?
x and y?
16. bE =_y2l
b_y2
34. If ax + 3a + x + 3 = b, what is x in term s of a
andb?
and b?
17. t t=27r\/Z
= 2nJ!j
8

18. A z=mm, ✓p+q


/p+q

I f XX == :X+1,findXintermsonandZ.
19. If : ~, find X in terms of Y and z.
Y+Z

If x(y +
20. If find yin
+ 2) = y, find terms of x.
y in terms

21. If“E‑
~a = 1a 126
b
++1] , find
2c
find a in terms
terms of and c.
ofbband

65
65
CHAPTER 8 MANIPULATING & SOLVING EQUATIONS

CHAPTER EXERCISE:Answers for this chapter start on page 287.

A calculator should
should N O T be used
NOT used on the
following questions.
following questions.
If;= ~m, what is the value of m?

1
A)
A) 6
6
Ifa+b=‐2,then(a+b)3=
If a + b = - 2, then ( a + b)3 =
A
A)) 4 B 2
_
B) 3
) 3
B) O
0
5
C
C)) -‐ 4 C) 3
-
6
D
D)) -‐ 8
D) 6

For what value


For what va lue of n + 4)2
4)2 = (n +
11 is (n ‐- 4)2 4) 2 ?
?
3 x++ 11 =
IIff 3x 8, what is the va lu e of (x + 2) 3 ?
= -‐8,whatisthevalueof(x+2)3?
A)) -‐ 1
A
B) l1
C) 8
D) 125

~ x ~g == 1, what
If %x
a C
what is the value ofb
the value of b -‐ ac
ac?? If k ! i,
‐k 4+ 2 ‐=
2
‐3
" ,where kk i=
where ¢ ‐- z2,, what 's k
w ha t i is ' etterms
kinm rms

o
off xX??
A
A)) -‐ 3
B) 0
A)
A)
12‐2x
12 - 2x
xX
C) 2
B) 12+2x
12 + 2x
D) It cannot
cannot be
be determined from the
determined from the B)
xX
information given.
information given. xX
C)
C) 12+2x
12 + 2x
D
D)) 12x
1 2 x- ‐ 2

If
If 3x = -‐ 223,
3x -‐ 8 _‐_ what is the
3 , what the value
value of 6x
6x -‐ 7?
7?
A
A)) -‐ 5
B
B)) -‐ 221
1 If = 36
3) 2 =
If (x ‐- 3)2 36 and
and x < 0, what
what is the
the value
va lue of
C
C)) -‐ 3300 x2
x2??

D
D)) ‐- 337
7

66
THE COLLEGE PANDA

f=p<(1+ii)n‐1)
J = p ( (1 + r-
1) lIff y >
> Oand
0 and (~)3
(g)3 2
J
= 31/5,
, what the value
what isis the of yy??
value of

The formula
The above gives
formula above gives the
the future value J
future value f of an an
annuity based on
annuity based the monthly
on the monthly payment
payment p, p, the
the
interest rate i, and
interest rate and the number of months
the number months n.
Which of the following gives p in terms
Which the following gives pin terms of J,
f, i,
and
and nn??
fiJi
M
A)
u+nn‐1
(1 + i)" - 1
((11++ i)i ) " ‐ 11
11
B
B) ) ‐ fi ‐
Ji
-
If
2\/x+4
2Jx + 4
= and x > 0, what
= 6 and
.
what is the value
1s the value
3

C)
fJ ‐- i o
off X?
x?
C) (1(1+i)"‐1
+ i)" - 1
D)) Ji
D +11 ‐- ( (1
fi + 1 ++ i i)
) "11

n1 . n
= 2,
If fl- = what is
2, what the value
1s the value of -L ??
Zn
211 2m
2m

A) 1
8 20‐fi=§fi+1o
2
20 - ✓x = 3 ✓x + 10
1
B) - If x > 0, for what value of xx is the
what value the equation
equation
4
1 above
above true?
true?
C) -
2
D) 1

x 2 + Sx - 24 =0
If
If k is a solution the equation
solution of the equation above
above and < 0,
and k < 0,
what is the
what the value
value of lkl
|k| ?
?
IIff x +y + y2 + 16, what
Jx2 +y2
+ y = «X2 what is the value of
the value
xxy?
l/ ?

67
CHAPTER 8 MANIPULATING & SOLVING EQUATIONS

A calculator is allowed on the following


2X - 1 X - 2 questions.
xx ++2 2 _4 T
What
What is the solution set
the solution the equation
set to the above?
equation above?
A) {-
A) {‐ 10,0}
10,O} y+2kx=kx2+5
y + 2kx = kx 2 + 5
B) {-
B) {‐10,
10, -‐ 44}} the equation
In the above, k is a constant.
equation above, constant. If
If y := 23
when x = 3, what
when what is the value
the va lue of k?
k?
C)
C) {0,8}
{0,8}
D)) {-
D { 44,8}
,8} A)) -‐ 6
A
B) 3
C) 6
D) 9
> 0and
IIff xx > (92 -‐2(%)
Oand (if 2(i) -‐ 15
15==o, what isthe
0,whatisthe
value of xx??
value
X + 12
xX = _
x+12 6
If -_
.'
6 =-- 42 ,whatis
,what 1s the
the value of -_
value of x ?7
.
6 ~ X
1
A) 3
3
B
B) ) 2
C) 3
C)
D
D) ) 6

2
x - 4x +3 = 4
x- 1
is the
What is
What solution to
the solution to the equation above?
the equation above?
d = a (~)24
Doctors use Cowling's
Doctors use rule, shown
Cowling's rule, above, to
shown above, to
determine
determine thethe right dosage d, in milligrams,
right dosage milligrams, of
medication for aa child
medication based on
child based adult dosage
the adult
on the dosage
a,in
a, in milligrams,
milligrams, and
and the
the chi age c,
ld 's age
child’s c, in years.
years .
Ben is a
Ben a patient who is in need
patient who need of a a certain
certain
medication.
medication . If aa doctor uses Cowling’s
doctor uses Cowling's rulerule to
prescribe Ben a
prescribe Ben a dosage that is half
dosage that half the
the adult
adult
dosage, what is Ben's
dosage , what age, in years?
Ben's age, years?
(x -‐ 9) =
2 4
xx2(x4 8x44
: 8x
A) 7
0, for what
If x > 0, value of x is the
real value
what real equation
the equation
B) 9
above true?
above true?
C
C)) 11
D
D)) 113
3

68
THE COLLEGE
THE PANDA
COLLEGE PANDA

--------- V ---------
'Y 111
3‐1 ,______ ~......._
___ -J

Questions
Qu estio ns 23-24 refer to the following
following
information.
informa tion . If the
If the masses
masses of both object 1 and
of both and object were
object 2 were
doubled,
doubled, how
how would the acceleration
would the acceleration of the
the ttwo
wo
objects
objects be
be affected?
affected?

1
1
- A)
B)
C)
The acceleration
The acceleration would
The acceleration
The
The acceleration
would stay
stay the
would be
acceleration would
the same.
same .
be halved.
would be
halved .
acceleration would be doubled.
doubled .
D) The acceleration
The be quadrupled
would be
acceleration would quadrupled
(multipled
(multipled by
by a factor of 4).
a factor

2 --------- A ---------

the figure
In the above, ttwo
figure above, objects are
w o objects are connected
connected by aa
string which
string which is threaded through a pulley.
threaded through Using its
pulley. Using lf 3(x ‐- Zy)
2y) ‐- 3z
32 := 0, which of the
0, which the following
following
weight,
weight , object object 1 along
moves object
object 2 moves along a flat surface.
surface. expresses
expresses x in terms and z ??
terms of y and
The acceleration
The acceleration a of the two objects
the two can be
objects can be 2y+32
2y + 3z
determined
determined by the the following formula
following formula A)
A) 3
a _ m2g
ng -‐ w11”mg 1g B
B)) 2 y+
2y +z
a= ---'~--'----=
Ill]] +
m + m z
/112 C
C)) y ++22z2
D) 6y+3z
6y + 3z
where m, and
where m1 are the
and m2 are the masses object 1 and
masses of object and
object
object 2, respectively, in kilograms,
2, respectively, the
kilograms, g is the
acceleration due to Earth's
acceleration due Earth's gravity measured in
gravity measured
m . . .
and 14
-‐ 2 ,, and ISa
11 is a constant
constant known as
known the coefficnent
as the coefficient of
ec2
sec ((xx ++ 1 ) ( x- ‐ 2)
l )(x 2 ) == 77xx -‐ 118
8
friction.
friction.
If x is the
the solution the equation
solution to the equation above, what is
above, what
the value
the value of 7x
7x -‐ 18
18??

Which of the following expresses fl in terms of


the other variables?
a(m]
a(m1 + mg)
m2)
A) /I =
A) y: m1m2g2
"11ng2
_ a(m]
B)) /In=‐
+ m2)
a( m1 + mz)
8
"128
m2g -‐ mm1g
lg

C) ,,-
C) 14Z 111
ng2g -‐¢:7(l:t;
a( m1 +
m1g
+1112)
111
2) Zfi
2./x :=xx ‐- 33
Which the following
Which of the represents all the
following represents the
a(m1 + m2) - m2R possible
possible values of xx that satisfy the
the equation
D 1)-z 4
D) /1 1 1+122;
‐g ‐0m'“ g
111, above?
above?
values of that satisfy equation

A) 1 and
and 9
B) 11 and
and 4
C) 4
D) 9

69
CHAPTER 8 MANIPULATING & SOLVING EQUATIONS

--------- V
~ ---------
4 Questions refer to the
Questions 31-32 refer the following
following
----= 9 information.
information.
x - 6x + 9
2
v z= pP(l-r)
V a‐ W1
Based on
Based the equation
on the equation above, which of the
above, which the
following be the
could be
following could value of x -‐ 33 ??
the value The
The value V of a
value V car depreciates
a car overt t years
depreciates over years
according
according to the
the formula
formula above, where P is the
above, where the
A);2
A) -
3
original
original price
depreciation.
depreciation.
and r is
price and is the
the annual rate of
annual rate of

B);3
8) 2

C);7
C) 3
Which of the following expresses r in terms of
D);9
D) 2 V,P, and t?

A) r __
mr = 1
_I~ V
l - “F

B) r _ I Vp
m
Jx - =
10 = f,/x ‐- ✓-fl
i ~
B)r‐1+\/‐P
= 1+

V
In the
1n the equation
equation above, what is the
above, what value of
the value C)r‐V‐P
C) r = ~_P
- 1
1
_

VTTE?
Jx - 10?
l/‘r
w
/6
A) x/E D
D) )r =r lz--l ‐pT
B 2V2
8) ) h @
c >3./2
C) a fi
D) v'14
\/1‐4
If a car depreciates to a value equal to half its
original price after 5 years, then which of the
following is closest to the car's annual rate of
xy2+ X - y2- 1=0 depreciation?
If th e equation
lf the above is true
equation above true for all
all real values of
real values A
A)) 0.13
o m
y, what
what must the value
must the value of x be?
be? B
8) ) 0.15
o w
C
C) ) 0.16
om
D
D)) o0.22

70
Equation Solving
More Equation
More Solving
Strategies
Strategies
In this
1n chapter, we'll
this chapter, touch on two
we’ll touch equation solving
t w o equation solving strategies
strategies that necessary for certain
are necessary
that are certain types questions
types of questions
involving equations.
involving equations.

1. Matching
1. Matching coefficients
coefficients

EXAMPLE If (x +
EXAMPLE 1: If a)22 =
+ a) 2 + 8x
z xx2 what is the
Bx + b, what value of b
the value b??

It's hard
It’s to see
hard to see anything meaningful right
anything meaningful right away on both
away on sides of
both sides the equation.
of the Solet's
equation. So expand the
let’s expand side first
the left side
and see ifif that
and takes us
that takes us anywhere.
anywhere.
,
(x + aa))2 =
( x+ : (x
(x++aa)(x
) ( . \ '+ : x 22+
+ nn)) = 2ax+n2
+ 2ax + a2

So now
So we have
n o w we have
7 7 ’?
x. r2‘ + + a2“ =
n . t+
+ 22ax z xx‘2 ++8Bx
x++ bb
both sides
For both be equal
sides to be equal to each
each other, coefficients of each
the coefficients
other, the each term
term m
must be equal.
u s t be equal. Let’s
Let's match up.
them up.
match them

x 3 + g x + fi z x2+§x+g
SO,
So,

2a = B
a2 = b

Solving the
Solving = 4 and
equations, a11z
the equations, and b =
: ~-
E.

71
71
CHAPTER
CHAPTER 9 MORE EQUATIONSOLVING
MORE EQUATION STRATEGIES
SOLVING STRATEGIES

Another way
Another way that
that the ”matching
this "ma
th e SAT tests this coefficients"
tchin g coefficient strategy is
s" strategy to phrase
is to th e question
phrase the question in
in the
the context
con text
of infinitely solutions for aa single equation
infinitely many solutions equation (we'
(we’ll talk aboutinfinitely
ll talk about infinitely many
many solutions for a
solution s for a system of
of
equations
equations in the next chapter).
the next chapter). A single equation has infinitely
single equation solutions only
many solutions
infinitely many only when sides of
both sides
when both the
of the
equation
equation are equivalent.
equivalent. For instance,
For instance,
3x ++ 66 =
3x : 3x
3x ++66
has infinitely many
has infinitely solutions because
many solutions no matter
because no what the
matter what value of
the value of x
x is, the equation is
the equation is always true (x
always true = 11 is
(x = is
a solution,, x = 2 is aa solution,
a solution z 3 is a
solution, x = solution, ...
a solution, that the equation
Notice that
. . . ) Notice eq uation boils down to O = O, which,
boils down to O = 0, which,
again, is always
again, always true.
true.
Equations
Equa tion s like 3x + 6 = 3x + 6 are
3x + are a
a bit
bit weird because what
weird because what’s of dealing
point of
's the point dealing with them in
with them in the
the first place?
first place?
Of course
course 3x + + 6 is equal
equal to 3x +
+ 6! But keep
keep inin mind
mind that these equations
that these equa tions are
are nnot meant to
o t meant solved; they’re
be solved;
to be they're
meant to demonstrate
meant demonstrate the concept of infinitely
the concept infinitely many solutions. Let’s
many solutions. take a
Let's take a look atat how concept might
this concept
how this might
appear in an
appear an SAT question.
question.

EXAMPLE2:
EXAMPLE 2:
a(x2-2b)
a(x2 =4x2‐12
- 2b) = 4x2 - 12
In the equation above, a and
equation above, and b are constants. equation has infinitely
copstants. If the equation infinitely many
many solutions, what is the
solutions, what
value of b?
value b?

Just
Just like Example
like in Examp expand the
le 1, we expand the left
left side and match
side and match the
the coefficients so that
coefficients so that both sides are equivalent.
both sides equivalen t.
Only when
Only both sides
when both are equivalent
sides are does the
equiva lent does equation have
the equation have infinitely
infinitely many
many solutions.
solutions.

a(x 2 -‐ 2b) =
a(x2 4x 2 ‐- 12
: 4x2 12
ax22 -‐ 2ab
ax 2ab = 4x22 -‐ 12
: 4x

Comparing the coefficients,


Comparing the coefficients, a = 4 and 2ab = -‐12.
and -‐2ab Now we
12. Now can solve
we can solve for b.

-‐2ab = ‐‐12
2ab =- 12
2(4) b = -‐ 112
-‐2(4)b 2

b = - 12 = (TI]
b=“_‐1§:‑
-8

EXAMPLE
EXAMPLE 3:
kx+3(5 - 2x) ==15
kx+3(5-2x) 15
In the
the equation
equation above, k is a constant.
above, I: constant. If the equation is true
the equation true for
for all
all values what is the
values of x, what value of k ?
the value ?

question is just
This question
This just another way of testing
another way you on
testing you infinitely many
the infinitely
on the many solutions
so lutions concept.
concept.

kkxx +
+ 3(5 2 x) )=
3 ( 5- ‐ 2x : 115
5
kkxx+
+1155-‐ 66xx = =1155
the right
Since the right side
side is just a constant need to cancel
constant of 15, we need cancel out
out the the left
terms on the
the x terms side in order
left side order for both
both
sides to be
sides equivalent. It's
be equivalent. It’s easy
easy to see
see that z ~
that kk = E] does the job. If
does the If it helps,
helps, you think of the
can think
you can the right side
right side
as having a
as having a 0x term. The end result that
Ox term . The end result is no matter
th at no value of x is,
what the value
matter what 15 equals
is, 15 know these
equa ls 15. Yes, I know these
equations
equations are weird, but
are weird, but that’s how you
that's how get infinitely
yo u get many solutions.
infinitely many soluti ons.

72
72
THE
THE COLLEGE
COLLEGE PA NDA
PANDA

N o w the
Now opposite of infinitely
the opposite infinitely many
many solution is no
solutionss is no solutions.
solutions. When an equation
When an hass no
equatio n ha no solution s, there
solutions, are
ther e are
no
no values that satisfy
values of x that equation
illustrate, the eq
satisfy it. To illustrate, uation

3 x++ 6 ==33xx ++110


3x 0

has
has nono solutions there is no
because there
solution s because value
no va that can
lue of x that can ever make 3x
ever make equal to
3x + 6 equal 3x + 10. The
to 3x equation itself
The equation itself
is aa contradiction.
contradiction. This is even
even more we subtrac
obvious if we
more obvious subtractt 3x from both
both sides with 6
we're left with
sides:: we’re 6=z 10, which
which is
is
fundamentally
fundamentally false.
N o w in the equation
Now 3x +
equation 33: + 6 = 3x ++ 10, notice that the x terms
notice that side have
terms on each side the same
have the coefficient of
same coefficient of 3, but
3, but
the cons tant s of 6 and
the constants are different.
and 10 are equation to have
different. For an equation have no solutions,
solutions, the coefficients
coefficients of the x
the x terms
must be the same
must be both sides,
same on both sides, but
b u t the constants must be
constants must be different.
different.

EXAMPLE 4:
EXAMPLE
Box -‐ 4(x +
3cx 1) =
+1) = 2(x ‐- 1)
1)
The equation above has
equation above has no
no solutions, and c is a constant.
solutions, and value of c ‘?
constant. What is the value ?

Expanding
Expanding each side,
each side,

3 c x- ‐ 4(x
3cx 4 ( x++ l1)) = 2(x
2 ( x-‐ 1)
l)
3 c x- ‐4x
3cx 4 x- - 44 =
222xx-‐ 2
= 6x ‐- 2
3cx -‐ 4 =

The constants are different,


constan ts are so we just need
different, so need to get the coefficients
the coefficien ts of
of xx to
to match. Very simply,
match. Very 3c =: 66 and
simp ly, 3c : [f].
and cc = I.

Clearing denominators
2. Clearing denominators

1 1
When solve an
you solve
When you equation like 5xx +
an equation + 5xx = likely first
: 10, aa likely step is
first step to get
is to of the
r i d of
get rid fractions, which
the fractions, are harder
which are harder
2 3
to work with. How
work with. do we
How do we do
do that? By multiplying
that? By multiplying both sides by
both sides by 6. But where
6. But that 6
did that
where did from? 2
come from?
6 come times
2 tim es
3. So
Sothis is what you're actually doing when you multiply both sides
this what you're actually doing when you multiply both sides by 6:by 6:

1 1
2éx-(2-3)+%x-(2~3)=lO-(2-3)
x · (2 · 3) + x · (2 · 3) = 10 • (2 . 3)
3
1 1
£x-(Z-3)+%x~(2-Z)le~(2-3)
1.x· ('/.· 3) + jx · (2 •,3)= 10 • (2 • 3)
3 x+
3x +22xx =
z 6600

We got rrid
We got clearing the
fractions by clearing
i d of the fractions denominators. Here’s
the denominators. takeaway: we
the takeaway:
Here's the can do
we can do the same thing
the same thing even
even
when
when there
there areare va riables in the denominators
variables denominators..

73
73
CHAPTER MORE EQUATION
CHAPTER 9 MORE STRATEGIES
SOLVING STRATEGIES
EQUATION SOLVING

EXAMPLES:
EXAMPLES:
‐3 + ‐ ‐5 - == 2
-+-- 2
xX xx+2
+2
If x is
H is a
a solution the equation
solution to the and ix > 0,
above and
equation above 0, what
what is
is the value of
the value of x ?
?

In the
the same
same way wemultiplied
way we multiplied by
by 2 ·~3 before we can
before,, we multiply
can multipl y by x(x + 2)
by x(x 2) here.
here.

3
·x(x + 2) + x +5 2 · x(x + 2) = 2 . x(x + 2)
xg-x(x+2)+x+2-x(x+2)=2-x(x+2)
3 5
g-x(x+2)+xj(z-le/+47=2X(X+2)
.j(x + 2) + .x-rt •x..(x.-+-2f= 2x(x + 2)
1
3(x+2)+5x=2x2+4x
3(x + 2) + Sx = 2x 2 + 4x
3x+6+5x=2x2+4x
3x + 6 + Sx = 2x 2 + 4x
0:2x2‐4x‐6
0 = 2x 2 - 4x - 6
Ozxz‐Zx‐B
0 = x 2 - 2x - 3
0 :=( x(x‐ -3 3)(x
) ( x ++1l )

z B3 oorr x = -‐1butbecausex
x= l but because x > 0,
O,x =..
x=GJ.
Here’s one final example
Here 's one example that both of the
showcases both
that showcases the strategies thi s chapter.
strategies in this chapter.

EXAMPLEG:
EXAMPLE6:
3x
3x + 55 _ -‐6x2+11x+5
6x2 + llx + 5
xx+1
+1+ a +2 =
axx+ _ (x+1)(ax+2)
(x+l)(ax+2)

the equation
In the equation above, x f:. - ~ and
above, 1:75‐g and a is aa constant. What is the
constant. What value of a ?7
the value
a
A) ‐6
-6 B) -‐22 C) 2 D) 6

Let's clear
Let's clear the denominators by multiplying
the denominators sides by (x +
both sides
multipl ying both + l1)(ax + 2):
)(ax +

3x
3x 55
x+1~~~(x+1)(ax+2)+ax+2(x+1)(ax+2)‐mm(x+l)(ax+2)-‐6x2+11x+5
6x2 + llx + 5
--·(x+ l )(ax + 2)+-- - (x+ l )(ax + 2)= ( l )( ) •(x + l )(ax + 2)
x +l ax + 2 x + ax + 2
3x
3x 5 -‐6x2+11x+5
6x + ll x + 5 2
•-(x(+ x + 1 =) ~ M _ m . ·~ x
5
m
~ -M ( a x ax
+ +22)) ++ ~
•.(,x-l-'11( l ).{ax-+-2T

3 x ( a x++ 22)) + 5S(x


3x(ax 1)) := -‐6x2
( x ++ 1 6x 2 + 11
llx x++ 5
3ax2
+ 6x + Sx + 5 = ‐6x2+11x+5
3ax2+6x+5x+5z - 6x 2 + 1lx + 5

Comparing the coefficients


Comparing the 2 terms,
coefficients of the xx2 Therefore, a =
terms, 3a = -‐ 66.. Therefore, Answer (B) .
= ‐- 22.. Answer [ill].

74
THE COLLEGE
THE PANDA
COLLEGE PANDA

CHAPTER EXERCISE:Answers for this chapter start on page 293.

A calculator should NOT be used on the


following questions.
18x2 - 8 = 2(ax + b)(ax - b)
the equation
In the equation above, and b are
above, a and are constants.
constants .
1 2 Which of the following
Which following could be the
could be value of ab
the value ab?
?
30 (x ~x
30(x3 +3
3x) =
2
6x2 + ~x) : M3
ax + bx 3
cx
bx2 + ex 2
A)
A) 6
equation above,
the equation
In the and ccaare
above, a, b, and constants. If
re constants. B) 9
the equation is
the equation is true all values
true for aJI what is
values of x, what C
C)) 112
2
the va lue of a + b + cc?.7
thevalueofa D
D)) 3366

xx - ~2(33c-+‐8)‐22
1 -(‐1) 4x
(3x + 8) = 2 ( 2 - ~x)

How many
How solutions are
many solutions are there the equation
there to the equation
above?
above?

2 The equation
A) The has no
equation has no solutions.
solutions.
B) The equation
B) The has infinitely
equation has infini tely many
many sosolutions.
lution s.
In the
ln equation above,
the equation above, a and
and b constants.
are constan
bare If
ts. If C) The equation
C) The has exactly
equation has exactly 11solution.
solution.
the
the equation ha s infinitely
equation has infinitely many
many sosolutions,
lutions, what
what The equation
D) The has exactly
equation has exactly 2 solutions.
solutions.
is the lue of ~g-?
value
the va ?

3
A) 4
3xx + aa(3
3 ( 3-‐ 22x)
x ) = 1122 -‐ 77xx
4
B) - the equation
In the equation above,
above, a is a constant. If the
constant. If the
3
C) 6 equation
equation has
has no solutions, what
no solutions, the value
what is the value
of aa??
D ) 112
D) 2
A
A)) -‐ 2
B) 2
C
C)) 4
ax - b = 3(2x + l ) D) 5

In the
ln equation above,
the equation above, a and b are constants
and bare constants.. If
the equation
the equation has no solution, which of the
has no solution, which the
following
following could be the
could be values of aa and
the values and bb ??
A ) aa =
A) n dbb=
z 22 aand = ~- 3 If (2): 5) =
(2x + 3)(ax -‐ 5) 12x22 + bx
= 12x bx -‐ 15
15 for all
all
values what is the
values of x, what value
the va lue of b ??
B)) aa=
B =2a n dbb=
and = 33
C)) aa=: 6 and
C a n db b=: -‐ 3 A) 6
D ) aa == 6 a
D) n dbb== 3
and B) 8
C
C)) 110
0
D
D)) 112
2

75
CHAPTER 9 MORE EQUATION SOLVING STRATEGIES

If (x + 3y) 2 = x 2 + 9y2 + 42, what is the value of IIff n < Oand 4x 2 + m


0 and 4x24» mxx+ (2x+n)2,
+ 9 = (2x what is
+ n)2, what is
x2y2? thevalue
the value o off m +
+ n ??
A) - 15
B
8)) ‐-‐ 9
C
C)) -‐ 3
D
D)) 112
2

6 x = xx ‐-33x(2n
6x x ( 2 n-‐ 11))
If‐+1=
X
1
If ~ + ~ = ~,
x yy
l,whatisxintermsofpandy?
p
P
what is x in terms of p and y ?

In the equation above,


the equation above, n is a constant. If the
constant. If the p- y
A) 19‐3!
A)
equation has infinitely
equation has infinitely many what is
solutions, what
many solutions,
the value
the value of n ?? 8) _EL
3) fly
p+y
A) ‐523
A) -- (C) D PY
j l
p- y
B) ‐531
8) -- D) _EL
D) W
y- p
4
C)
C)
3
5
D)
D) -
3
(x3+kx2‐3)(x‐2)=x4+7x3‐18x2‐3x+6
(x 3 + kx 2 - 3) (x - 2) = x 4 + 7x 3 - 18x 2 - 3x + 6

in the equation
In the equation above,
above, k is a constant.
constant. If the
It the
equation
equation is true values of x, what
true for all values is the
what is the
ab + a a . value of k?
k?
If b- a =
If -ab2- b+
= g + 5 for all values of b, what
all values what is
1s the
the value
value
value of a
a?? A)) -‐ 99
A
B)
8) 5
C) 7
D
D)) 9

1
If~i -‐ ‐y4‐
If - - = 1, what is the
1,what value o
thevalue off x?
x?
X X - 4

76
THE COLLEGE
THE PANDA
COLLEGE PANDA

5 2 ax - b
xx ++33 ' xx -- 22 ‘ (x
( x + 33)(x
) ( x-‐ 22))
The equation above
The equation true for
above is true all x > 2,
for all where a
2, where
and
and b constants. What
are constants.
bare the value
What is the + b ??
value of a +
A) 7
B)
B) 13
13
C
C)) 119
9
D
D)) 221
1

4 + 2 _ 35
35
--+--=--
x -‐ 1l xX + 1
1 _x2‐1 x2 - 1
If x >
> 1, what is the
1, what the solution the equation
solution to the equation
above?
above?

The equation (2x - b) (7x + b) = 14x 2 - ex - 16


is tme for all values of x, where band care
constants. If b > 0, what is the value of c ?
A) - 20
8) 20
C) 28
D) 36

_ 3_ + --3!!.__
=3
n‐l+n+1=
n- 1 11+ 1
If what value
If n > 0, for what value of
of n
11is the equation
is the equation
above
above true?
true?

77
Systems of Equ
Systems Equations
ations
system of equations
A system refers to
equations refers to 22 or equations that
more equations
or more that deal
deal with
with the same set
the same of variables.
set of variables.

-‐5.t
sx ++ yy =-
z ‐77
2}; =
-‐ 33xx -‐ 2y z -‐ 112
2

There are
There are ttwo main ways
w o main solving systems
ways of solving of 22 equations:
systems of equations: substitution and elimination.
substitution and elimination.

Substitution
Substitution
Substitution
Substitution is all
all about isolating one
about isolating variable,
one var either xx or
iabl e, either or y,
y, in
in the
the fastest possible.
way possible
fastest way
Taking the example
Taking the example above,
above, wewe can see that
can see it’s easiest
that it's to isolate
easiest to isolate y in the
y in the first
first equation because it
equation because it has
has no
no
coefficient. Adding
coefficient. 5x to both
Adding Sx both sides,
sides, we get
we get
y=: Sx
5x -‐ 7
We can
We substitute they
then substitute
can then the y in
in the equation with
second equation
the second 5x -‐ 7
with Sx and solve
7 and from there.
solve from there.

-‐ 33xx -‐ 2(5x 7 )=: -~12


2(5x -‐ 7) 12
10x +
-‐ 33xx -‐ lOx + 14 = ‐- 1122
1 4z
3x =
-‐ 113x : -‐ 226
6
x=z 2

Substituting xx =z 22 back
Substituting intoy
back into y=
: Sx
5x -‐ 7,y : 5(2)
7, y = 5(2) -7 77 =
z 3.
3.
solution is x =
The solution
The 2,y z= 3,
z 2,}/ 3, which denoted as
be denoted
can be
which can (2, 3).
as (2,3).

78
THE COLLEGE
THE PANDA
COLLEGE PANDA

Elimination
Elimination
Elimination is
Elimination is about getting the
about getting the same coefficients for
same coefficients one variable
for one across the
variable across the two equations so
t w o equations so that you can
that you can add
add
or subtract the
or subtract the equations, thereby eliminating
equations, thereby that variable.
eliminating that variable.
Using the
Using the same example, we
same example, we can multiply the
can multiply the first equation by
first equation by 2 so that
2 so that the y’s have
the y's have the
the same coefficient (we
same coefficient (we
don't worry about
don’t worry about the sign because
the sign we can
because we subtract the equations).
add or subtract
can add equations).

0x +
-‐ 110x +Z2yy =
= -‐ 114
4
2 y:= -‐ 112
-‐ 33xx -‐ 2y 2

To eliminate we add
eliminate y, we add the equations.
the equations.

10x + 2y
-‐10x 2y == ‐- 114
4
2yy =
-‐ 33xx -‐ 2 : -‐ 112
2

13x =
-‐13x 2 -‐ 226
6

Now, we
Now, we can
can see that x =
see that = 2. This result can
This result be used
can be either of the original
used in either equations to solve
original equations We'll pick
solve for y. We’ll pick
equation.
the first equation.
the

10(2) +
-‐10(2) + 22y
y=: -‐ 114
4
-‐ 2200++ 2y
2 y=-: ‐ 114
4
2yy =
2 =66
yy =
= 33

And finally, we
And we get the same
get the solution as
same solution aswe got using
we got substitution: x =
using substitution: : 2,y = 3.
2, y z 3.
solving systems
When solving
When equations, you
systems of equations, you can
can use method, but
either method,
use either one of them
but one will typically
them will be faster. If
typically be If
you see a variable
you variable with no coefficient,
w i t h no like in -‐ 55xx +
coefficient, like + y =z -‐77 above, likely the
substitution is likely
above, substitution the best
best route.
route . If
matching coefficients
you see matching
you you see that
coefficients or you that it's easy to get
it’s easy matching coefficients,
get matching coefficients, elimination likely the
elimination is likely best
the best
route example above
The example
route.. The above was simple enough
was simple enough for both methods to work
both methods work well (though substitution
well (though substitution waswas slightly
slightly
these cases,
faster). In these
faster). cases, it comes
comes down your personal
down to your personal preference.
preference.

solutions
No solutions
the previous
In the chapter, we
previous chapter, we saw
saw that single equation
that aa single equation has no solutions
has no solutions when both sides
when both the equation
sides of the equation are the
are the
except for the
same except
same the constants.
constants.
In similar fashion, aa system
similar fashion, equations has
system of equations no solutions
has no solutions when the ttwo
when the equations are
w o equations the same
are the except for their
same except their
constants. For
constants. example, the
For example, system
the system

3 x+
3x 2 y=z 5
+ 2y
3 x++ 2y
3x 2 y== -‐ 4

has
has nono solutions
solutions since different constants
the different
since the constants (5(5 vs. result in equations
vs -‐ 44)) result equations that contradict each
that contradict each other.
other . There
There
an x and
isn’t an
isn't and aa y that
that can possibly satisfy
can possibly satisfy both
both equations
equations at the same Note that
time.. Note
same time the system
that the system

3 x+
3x +22yy =
= 55
6 x++4y
6x 4 y=: -‐ 8

also has
also no solution.
has no solution. Why? Because the
Why? Because second equation
the second can be
equation can divided by 2 to get the
be divided contradictory equation
the contradictory equation
we had before.
we had before.

79
79
CHAPTER 10 SYSTEMS
CHAPTER 10 OFEQUATIONS
SYSTEMS OF EQUATIONS

EXAMPLE 1:
EXAMPLE
--‐ax ‐ 1 i j=
ax-12y = 15
15
41: + By =
4x+3y = -‐22
If the system
11 of equations
system 0f above has
equations above no solution,
has no what is the
solution, what value of a ?
the value ?

We get the
must get
We must coefficients to match
the coefficients so that
match so we can
that we compare the
can compare w o equations.
the ttwo equations. To do
do that, we multiply
that, we the
multiply the
equation by
second equation
second by -‐ 44::

123; =
-‐ aaxx -‐ 12y z 15
-‐16x 1211 =
16x -‐ 12y = 8

how the
See how ‐12’s match
the -12's match now? N o w let's
now? Now compare. If aa = ~ then
let's compare. then we
we get
get o t w o contradicting
u r two
our contradicting equations
equations
with no
with no solution.
solution. One
One constant
constant is 15 while the
15 while other is 8.
the other 8.

solutions
Infinite solutions
Infinite
In the
In previous chapter,
the previous chapter, we learned that
we learned single equation
that aa single has infinitely
equation has solutionswhen
many solutions
infinitely many when both the
sides of the
both sides
equation are
equation the same.
are the same.
Similarly, aa system
Similarly, equations has
system of equations infinitely many
has infinitely many solutions
solutions when both equations
when both are essentially
equations are essentially the
the same:
same:

3 x+
3x + 2y
2 y=z 5
3 x+
3x 2 y=
+ 2y = 55

(1, 1),
(1, (3, -‐ 22),
1), (3, (5, -‐ 55)) are
) , (5, all solutions
are all solutions to
to the above, to name
system above,
the system just a
name just a few. Note
Note that
that the system
the system

6x 4 y=: 110
6 x++ 4y 0
3x 2 y=: 5
3 x++2y 5

also has
also infinitely many
has infinitely many solutions.
solutions. The equation can
The first equation be divided
can be divided by
by 2 to
to get
get the equation. They’re
second equation.
the second They're
still essentially the
still essentially the same equation.
same equation.

EXAMPLE2:
EXAMPLE2:
3x 531 =
Bx -‐ 5y 8
= 8
mx -‐ ny = 32
mx 32
In,the
In system of equations
the system and n are
above, m and
equations above, are constants. the system
If the
constants. If system has infinitely many
has infinitely solutions, what
many solutions, what
the value of m + n??
isthevalueofm+n
is

Both equations need


Both equations be the
need to be same for there
the same there to be infinitely many
be infinitely many solutions. We multiply
solutions. We the first equation
multiply the equation by
get the
4 to get right hand
the right sides to match
hand sides match::

12x -‐ 20y = 32
12x
mx -‐ ny
mx 71}; =
= 32
32

N o w we can
Now clearly see
can clearly that m
see that m = 12 and n =
12 and : 20. Therefore + n=
Therefore,, m + I32 I.
: -.

80
THE PANDA
COLLEGE PANDA
THE COLLEGE

Word problems
Word problems
most definitely
You will most definitely rrun into a question
u n into question that asks you
that asks you to translate a
to translate a situation into aa system
situation into system of
of equations.
equations.
Here's a
Here's a classic example:
example:

EXAMPLE
EXAMPLE 3: A groupgroup of 30 students lunch from aa restaurant.
order lunch
students order Each student
restaurant. Each gets either
student gets a burger
either a burger or
or
a salad. Thepriceofaburgeris$5and
asalad. thepriceofasaladis$6.
The price of a burger is $5 and the Ifthegroupspentatotalof$162,how
price of a salad is $6. If the group spent a total of $162, how
many ordered burgers?
students ordered
many students burgers?

Let x be the number


be the number of students who ordered
students who burgers and
ordered burgers be the
and yy be the number who ordered
number who ordered salads.
salads . We then
can then
We can
make w o equations:
make ttwo equations:

x + y z=3300
5xx +66yy ==1162
5 62

Make sure
Make you completely
sure you completely understand
understand how these equations
how these equations were made. This
were made. type of
This type question is
of question is guaranteed
guaranteed
be on the
to be the test.
test.
We’ll use elimination
We'll use elimination to solve
solve this system. Multiply the
system. Multiply first equation
the first equation by
by 6 and subtract:
6 and subtract:

+ 6y == 180
6x+6y
6x
5x + 6y =
5x+6y : 162

x=‑
X=~

18students got burgers.


18 students got burgers.

More complex
complex systems
systems
might encounter
You might systems of equations
encounter systems equations that are a
that are a bit more complicated
bit more complicated than
than thethe standard ones you’ve
standard ones you've seen
seen
above.
above. For these systems, substitution
these systems, and some
substitution and equation manipulation
some equation w i l l typically do the trick.
manipulation will typically do the trick .

EXAMPLE4:
EXAMPLE4:
y + 3x = 0
y+3x
x2 2y22 =
x2 + 2y = 76
If (x,y) is a
If (x,y) solution to the
a solution system of
the system of equations
equations above and y
above and y > 0, what is
0, what value of
the value
is the y?
of y?

In the
ln the first equation, we isolate
equation, we get y =
isolate y to get : -‐ 33x. Plugging this
x . Plugging this into second equation,
the second
into the equation,

xx2+
2 2(‐3x)2
+ 2(- 3x) 2 : 76
= 76
2 + 2(9x2)
xx2 2(9x 2 ) = 76
76
xx2+1sz2 = 76
76
2
+ 18x2
19x2=76
19x 2
= 76
x2=4
x2 = 4
xX =
: i± 2
IIff xx = 2,
2, theny : ‐3(2)
then y = - 3(2) =
= ‐- 66.. IIff xx = -‐ 22,, then = ‐- 33((-‐ 22)) := 6.
thenyy = 6. Becausey > 0,y
Because y > IE].-
: ~
0, y =

81
81
CHAPTER 10
CHAPTER SYSTEMS OFEQUATIONS
10 SYSTEMS OF EQUATIONS

EXAMPLE 5:
EXAMPLES:
xy+2y=2
xy + 23; = 2

(_1 1
(x+2)
x+2
)2++(x+2)
(-1
2 ) - 6= 0
x+2 ‐ 6 _ 0
1

If (x,
If (x, y) is a solution the equation
solution to the what is a
above, what
equation above, possible value
a possible IYI??
value for |y|

Notice + 2)'s
the (x -+-
Notice the 2)'s lying around
around in both equations. This
both equations. hint that
This is a hint there might
that there might be be a clever substitution
clever substitution
somewhere,
somewhere, especially
especially for a
a problem
problem ascomplicated
as complicated as
as this
this one.
one. Isolating
Isolating y in the
the first equation,
equation,

xy + 2y = 22
2) =
y(x + 2) z 22
_ 2
y _ x+2

1 1
From here, 1
From here, z -‐. - . Why
y_ = Why would
would I want
want this 50 I can
form? So
this form? can substitute
substitute - - in the second equation
the second with z.
equation with y__
2 x+2 x+ 2 2
As you do
As you these tougher
do these tougher questions, you m
questions, you keep an
u s t keep
must out for any
eye out
an eye simplifying manipulations
any simplifying such as
manipulations such this
as this
one.
one.
Substituting,
Substituting, we
we get

_1 + (- 11 ) - 6 = 0
’)

((m)1
)2

x + 2 +(x+z)‘6=°
x+2

(f)2+ G)- = o
owe‐6:0 6

g+g~6zfl
y2
4
+ Y-.- 6 = 0
2
f+@‐M=O
y2 + 2y - 24 = 0
((yy+ 4 )== 0
) ( y- ‐ 4)
+ 66)(y
Finally, y =
Finally,y z ‐6
- 6 or
or 4, and IYI
4, and |y| can
can be either /‑6 or 4 /.
be either
H o w will
How will you
you know there's a clever
whether there’s
know whether substitution or ”trick”
clever substitution "trick" you Practice. And
use? Practice.
can use?
you can even then,
And even then,
you won't always
you won’t know for sure.
always know Just keep
sure. Just keep in mind that SAT questions
mind that questions areare designed
designed to be be done without a
done without a
crazy number steps. 50 you
crazy number of steps. So if you feel like you’re running circles
like you're running in circles or hitting a wall, take a step back and
hitting a waJI, take a step back and try
something else.
something else. To get aa perfect score, you
perfect score, must be
you must be comfortable
comfortable with trial and
with trial error.
and error.

EXAMPLE6: If
EXAMPLE = 8,
If xy = = 5,
8, xz = 5, and = 10,
and yz = what is apossible
10, what a possible positive value of xyz ?
positive value ?

trick . Multiply
Here 's the trick.
Here’s Multiply all three equations. Multiply
three equations. Multiply the and multiply
sides, and
the left sides, multiply the right The result
sides.. The
right sides result
is
2y2z2 =
xchyzz2 = 8 -· 5 ~10
- 10

x 2y2z2 =
xzyzzz = 400
Square o o t both
Square rroot sides.
both sides.
/W = ± v'400
= HEW)
x2y2z2
xyz =
= i±220
0

the question
Since the question asks a positive
asks for a value, the
positive value, answer is -.
the answer how we
Notice how
/ 20 /. Notice able to get
were able
we were get the answer
answer
without knowing
without knowing the individual values
the individual values of x,y,
x, y, or 2.
z.

82
THE COLLEGE
THE PANDA
COLLEGE PANDA

Graphs
Graphs
Learning aa bit
Leaming bit about equations and
about equations and their graphs will
their graphs will inform our understanding
inform our understanding of systems equations..
systems of equations
The solutions
The equations are the intersection
system of equations
solutions to a system points of the graphs
intersection points graphs of the equations. Therefore,
the equations. Therefore ,
solutions to a system
number of solutions
the number system of equations equal to the number
equations is equal intersection points.
number of intersection points .
example, the system
Take, for example, equations at
system of equations at the beginning of this chapter:
the beginning chapter:

5x + yy == -‐77
-‐5x
2y = -‐ 112
-‐ 33xx -‐ 2}; 2

We put both
can put
We can equations into
both equations mx +
into y = mx bform
‐+- b form (we
(we won't that here)
show that
won’t show and graph
here) and graph them
them to get the following
get the following
lines..
lines
y

(23)

The
The solution system, (2,3), is the
solution to the system, the intersection point. There
intersection point. There is only one intersection point,
one intersection point, so
so there only
there is only
solution..
one solution
one
What about
What graphs of systems
about graphs that have
systems that have infinite solutions or no solutions?
infinite solutions solutions?
Graphing
Graphing the following system,
the following system, which no solution
has no
which has because its equations
solution because equations contradict each other,
contradict each other,

y - 22xx=: 1l
y -‐ 22xx =
z ‐- 3

weget
we get
y}/

What
What do you
you notice
notice about
about the
the lines?
lines? They have intersection points.
have no intersection They’re parallel.
points . They're Makes sense,
parallel. Makes sense,
right?
right?

83
CHAPTER 10 SYSTEMS
CHAPTER 10 OFEQUATIONS
SYSTEMS OF EQUATIONS

And system with


And for a system with infinite solutions?
infinite solutions?

2y 4 x=
2 y-‐ 4x z 22
y -‐ 22xx = 1
y

It's
It's just one line! Well, actually
just one it’s two
actually it's t w o lines, because they’re
but because
lines, but they're the
the same line, they
same line, overlap and
they overlap and intersect
intersect in
in
an
an infinite number of places.
infinite number Hence, an
places. Hence, an infinite number of solutions.
infinite number solution s.

EXAMPLE
EXAMPLE 7: 7: In the
the xy‐plane, lines y = 3x
the lines
xy-plane, the 3x -‐ 5
5 and = ‐-2x
and y = 2x++1100 intersect at the
intersect at (h,k). What
point (h,k).
the point is
What is
thevalue
the value ofk?
of k ?

As mentioned ear lier, the


mentioned earlier, solutions to aa sys
the solutions system equations are
tem of equations intersection points
the intersection
are the the graphs
points of the grap hs of those
those
equations, and vice
eq uations, and vice versa. So to find the point(s)
versa. So where two graphs
point(s) where intersect, solve
graphs intersect, solve the system consisting of
system consisting
their equations.
equations. 1n
In this problem, that
this problem, that system
system is

y=: 3x
3x -‐ 5
: ‐2x
y =- 2x++ 10
10

Substituting
Substituting the equation into
the first equation into the
the second, we get
second, we get

=‐-22x
3 x-‐ 55 =
3x x ++110
0
Sx = 15
5 x = 1 5
x =3

When So the
When x =z 3, y = 3(3) -‐ 5 = 4. So the ttwo lines intersect
w o lines intersect at (3,4)
(3,4 ) and [i].
and k =z I.

84
THE COLLEGE PANDA
THE COLLEGE PANDA

EXAMPLES:
EXAMPLES:
y=xz‐5x+6
y = x 2 - 5x +6
yy=x+1
=x+1
The system of equations
The system above is graphed
equations above the xy-plane.
graphed in the If the
xy-plane. If the ordered pair (x,
ordered pair y) represents
(x, y) represents an intersection
an intersection
point the graphs
point of the graphs of the w o equations,
the ttwo equations, what one possible
what is one value of y ?
possible value ?

The solutions
The solutions to the system are
the system are the intersection points,
the intersection so let’s
points, so let's solve system. Substituting
solve the system. Substituting the first equation
equation
second, we get
into the second,
into

xz‐Sx+6:x+1
x2 - 5x + 6 = x + 1
xx2‐6x+5:0
2
- 6x + 5 = 0

( x-‐ 11)(x-5)=
(x )(x-5)=0
x ==110orr 55

When x =
When 1,yy = l1 +
= 1, +11 =
: 2. When
When x = 5,yy =: 5 +
: 5, +11 = So the graphs
= 6. So graphs of the w o equations
the ttwo intersect at (1,2)
equations intersect (1, 2)
and (5,6),
and (5,6 ), which means the possible
which means values of y are[}]
possible values are and [I].
and E.

EXAMPLE9:
EXAMPLE9:
y2=x+3
y2 = x+3
y ==| lxl
xl
yy

system of ttwo
A system equations and
w o equations their graphs
and their the xy-plane
graphs in the shown above.
xy-plane are shown How
above. H many solutions
o w many does
solutions does
system have?
the system have?

A) One
A)One B) Two
B)Two C) Three
Three D)Four
D) Four

Simple. The graphs


Simple. graphs intersect w o places
intersect in ttwo places so
so there w o solutions.
are ttwo
there are Answer ~-(B) .
solution s. Answer

85
85
CHAPTE R 10
CHAPTER SYSTEMS OFEQUATIONS
10 SYSTEMS OF EQUATIONS

CHAPTEREXERCISE:Answers for this chapter star t on page 296.

_
should N
A calculator should O T be
NOT be used the
used on the
following
following questions.
questions .
2
2xx+
+ 5Syy =
=2244
x+4y=
x + 4y = 1515
3 x-‐ 5
3x Syy == ‐- 1111 If (x,
If (x, y)
y ) satisfies
satisfies the system of equations
the system above,,
equations above
lX e= ‐1 -B 3yy what is the
what the value
value of x ++ y ??
What is the
What so lu tion (x,
the solution (x,yy)) to sys tem of
the system
to the A
A)) 7
equations
equations above?
above? B) 8
A
A)) ((-‐ s5,2
,2) C) 9
B
B)) ((-‐ z2,1
, 1) D
D)) 110
0
C)
C) (1,0)
(1,0)

_
D) 4,‐1)
D) ((4, - 1)

_
3 x++yy =
3x = ‐- 22xx + 8
-‐ 33xx + 2
2yy = --1100
y++ 22xx =: 2200 If (x, y)
1f a solution
y) is a system of equations
sol u tion to the system equations
6
6xx-‐ 5Sy
y ==1122 above, what
above, w hat is the value
the va lue of xy ??

What
Wh at is the solution (x,
the solution (x,yy)) to the system
the sys tem of A
A)) ‐- 116
6
equations above?
equations above? B
B)) -‐ 8
A ) ((-‐ 77,6
A) ,6) C
C)) -‐ 4
B) (‐6,
B) (- 6, 7) D
D)) 44
C)
C) (6,7)
(6, 7)
D)
D) (7,6)
(7,6 )

y z=aax
x++ bb
y z= ‐-bbxx
3x -‐ 4y =
3x = 21
2] The equations
The equations of ttwo lines in the
w o lines xy-plane
the xy-p lane are
are
4x -‐ 331
4x =
3y = 14
14 shown above, where a and b are constants.
shown above, where and bare cons tants. If the
If the
w o lines
ttwo lines intersect at (2,8),
intersec t at what is the
(2, 8), what value
the va lue
IfIf (x,
(x, y) is a solution to the
a solution system of equations
the system equations of aa?7
above,
above , what
w hat is the value of y -‐ x ?
the value ?
A
A)) 2
A
A)) ‐- 118
8
B) 4
B
B)) -‐ 5
C
C)) 6
C) 5
D) 8
D) 8

86
THE
THE COLLEGE PANDA
COLLEGE PANDA

yy=x2+1
2
= x +1 2
2xx-‐ 44yy=z88
y=z xx -‐ 1l + 22yy = 4
xX +

y How many
How many solutions (x,y)) are
solutions (x,y there to the
are there the
system of equations
system equations above?
above?
A)
A) Zero
Zero
B) One
One
C Twoo
C)) Tw
D) More than ttwo
More than wo

2x -‐ 5};
Sy = a
system of ttwo
A system w o equations and their
equations and graphs in
their graphs
bx + lOy
10y = ‐8
- 8
the xy-plane are
the xy-plane shown above
are shown H o w many
above.. How man y
solutions does
solutions does the system have?
the system have? In
ln the system of equations
the system above, a and
equations above, and b are
bare
A)
A) Zero
Zero constants. If the system has infinitely many
constants . If the system has infinitely many
B) One
One solutions, what
solutions, what is the value of a ??
the value

C ) Tw
C) Twoo A)
A ) -‐ 4
4
Three
D) Three 1
B) 3
-
4
C) 4
D
D)) 116
6

-‐ 5Sxx =
= y +2 2
2 ( 2 x- ‐1)1 )=: 3 -‐ 33yy
2{2x

What solution (x, y) to the


the solution
What is the system of
the system a 2 y=z 5
x + 2y
ax+ 5
equations above?
equations above? 3 x-‐ 66yy ==2200
3x
A ) ({-‐ 22,8
A) ,8)
B ) ( -‐ 11,, 33))
B)
In the
the system equations above,
system of equations above, a
a is a a constant.
constant.
the system
If the one solution,
has one
system has which of the
solution, which the
C ) ((1,
C) 1 ,-‐ 77)) following
following can N O T be
can NOT be the value of aa ??
the value
D) (3,‐17)
(3, - 17)
A)) -‐ 1
A
3
B) 3
B) -
E
4
C) 1
D
D) ) 3

87
CHAPTER 10 SYSTEMS
CHAPTER 10 OFEQUATIONS
SYSTEMS OF EQUATIONS

6y =
4x‐‐y=‐8
1
= -8
4x - - y
3
3x
3x -‐ 63) = 15
15
‐-2x
2x + 4 = -‐ 1100
4yy=
x ++116
y ==44x 6
H
How many solutions
o w many (x,y) are
solutions (x,y) there to the
are there the
What is
What is the
the solution (x,y) to
solution (x,y) to the system of
the system system of equations
system equations above?
above?
equations above?
equations above?
A)
A) Zero
Zero
A)
A ) ((-‐ 22,8)
8) B) One
8) One
8))( (- 1,12
1,12)) C
C)) Tw
Twoo
C)) , (1,20)
( 120) D) More
More than
than ttwo
wo
D)) ( (3,28)
3 , 28)

m
mxx-‐ 66yy =
= 1100
y :=
0 .0.5x
5 x ++114
4 2
2xx-‐ nnyy =
z5

_
x -‐ yy ==‐ -118
X 8
In the
ln system of equations
the system above, m
equations above, m and
and n are
are
According to the
According equations above,
system of equations
the system above, constants. the system
If the
constants. If system has infinitely many.
has infinitely many
what the value
what is the value of y ?
? . . m
so lutions, what
solutions, 15the
what is value of g
the value m ??
11

1
A) 12
A) fi
1
B)§
8) 3
4
C)
C) 5
3

D
D) ) 3

1l 1
3x
y ‘ -a6y
l l ‐= 44
6 x-‐ aayy =
6x =8
:\/E+3
y= vx + 3
[n the
ln system of equations
the system above, aa is aa constant.
equations above, constant.
m_
If the system ha
the system hass no
no solution, what is the
solution, what value
the value fu - y =3
of aa??
IfIf (x,
(x, y) is the solution to the
the solution system of equations
the system equations
1 above, what
above, what is the value of y ??
the value
A) 3

8)
B) 1
C) 3
D) 6

88
THE COLLEGE PANDA
THE COLLEGE PANDA

calculator is allowed
A calculator following
allowed on the following
questions.
questions.
B

A local
and
local supermarket
large jars.
and large
as
jars. Sixteen
w o medium
as ttwo
sells jelly in small,
supermarket sells
small jars
Sixteen small
medium jars one large
and one
jars and
small, medium,
weigh
medium,
jars weigh as much
as
large jar. Four
much
small
Four small
@
0
jars
jars and medium jar have
one medium
and one have the same weight
the same weight
as
as one large jar. How
one large How many
many small jars have
small jars have the
the A game
game of darts points depending
rewards points
darts rewards depending on on
weight one large
weight of one large jar? region is hit.
which region
which hit. There
There are w o regions,
are ttwo regions, A and
and
A)
A) 77 B,as shown above.
B, as shown throws 3 darts,
James throws
above . James darts, hitting
hitting
B) 8 region
region A once and region
once and twice, for a
region B twice, a total
total of
B) 8
18 points. Oleg
18points. Oleg also throws 33 darts,
also throws darts, but
but hits
hits
C) 9
9 regions
regions A and region
twice and
A twice region B once
once for a
a total
total of
D) 10
10 21
21 points. How many
points. How points are
many points are rewarded
rewarded for
hitting region B once?
hitting region once?

On
On aa math with 30questions,
test with
math test 30 questions, 5 points are
points are
rewarded
rewarded for each
each correct and 2 points
answer and
correct answer points
are deducted
are deducted for each
each incorrect James
answer . If James
incorrect answer.
answered the questions
answered all the questions and
and scored 59 points,
scored 59 points,
solving which of the
solving which the following systems of
following systems
equations gives his
equations gives number of correct
his number answers,
correct answers,
x, and his number
and his number of incorrect answers, y, on
incorrect answers, on the
the
math
math test?
test? A restaurant
restaurant has
has two types of tables,
two types tables, rectangular
rectangular
ones that
ones can each
that can people and
seat 4 people
each seat and circular
circular
tables that
tables can each
that can seat 8 people.
each seat people. It
If 144 people
people
A) xX +
+yy = 59
= 59 are enough
are enough to fill all
aU 30 tables at the
tables at the restaurant,
restaurant,
5x -‐ 2y
5x Zy =
= 30
30 how many rectangular
how many rectangular tables
tables does
does the
the restaurant
restaurant
have?
have?
B)
B) x
X + y!f =
+ = 30 A) 12
A) 12
5x 2y =
5x + 2}; = 59
59 B) 16
B) 16

c) x+
+y =
= 30 C)
C) 20
20
C)
2x
2x ‐- 53,
Sy = 59 D)
D) 24
24

D)) xX++yy ==3300


D
5 x-‐ 22yy =: 559
5x 9

89
CHAPTER 10 SYSTEMS OF EQUATIONS

y In the
the xy-plane,
xy-plane, the graph of y =
the graph x 2 ‐- 7x
= x2 7x +
+77
intersects
intersects the graph of y =
the graph : 2x the points
2x -‐ 1 at the points
1) and
(1, 1) and (p,q). What is
(p, q). What value of p
the value
is the p??

_
xx22- ‐2
2x x==yy-‐ 1
1

x = y ‐- 111
1
A system
system of ttwo equations is graphed
w o equations graphed in the
the
xy-plane above. Which the following is the
xy-plane above. Which of the following is the If (x, y) is a
If (x, solution to the
a solution system of equations
the system equations
solution (x,
solution (x,y) the system?
y) to the system? what is one
above, what
above, one possible value of y ??
possible value

A)
A) (0,‐6)
(0, - 6)
8)) (-
B ( ‐ 33,,-‐ 33))

c> (4-3)
C) ( - ~, - 3)

D) (-3, -;)
r» (+2)

1
-
x2 _
x2 y2=_ -11_2
yz
12
2y =
xX -‐ 2y : 00

If
lf the
the ordered (x1,y1)
pairs (x1,
ordered pairs y2) satisfy
and (x2, yi)
Y1) and satisfy
the system
the system of equations
equations above,
above, what are the
what are the
values
values of y1 and y2 ?
and 1/2 ?
1 1
A) --
and -
2 2
1 1
B ‐ ‐ ‐ a and
B) --- n d -- ‐ ‐
) v'12
\/12 12
v'12
1 1
C)
C) ‐i
-- and
4
and E
-
4
1 1
D)
D) --
6
and1
and -
6

90
Just aswe
Just equations and
had equations
as we had of equations,
systems of
and systems we can
equations, we
Inequalities
Inequalities
have inequalities
can have and systems
inequalities and
ll
of inequalities
systems of inequalities..
The only
The only difference
difference is that you m
that you u s t reverse
must sign every
the sign
reverse the you either
time you
every time multiply or
either multiply or divide
divide both sides by
both sides by aa
negative
negative number.
number.
For example,
For example,
+ 33 < 99
2x +
2x
Do we have to reverse
we have the sign
reverse the sign at
at any we would
point? Well, we
any point? subtract by
would subtract by 33 to
to get
get 2x
2x < 6 and then
6 and then divide by
divide by
get x < 3. Yes, we
2 to get we did
did aa subtraction
subtraction but at no
but at no point
point d multiply or
we multiply
i d we
did or divide by aa negative
divide by negative number.
number.
Therefore, the sign
Therefore, the stays the
sign stays the same.
same.
Let’s take
Let's another example:
take another example:
3x ++ 55 < 4x + 4
3x
step is to combine
The first step
The like terms.
combine like Wesubtract
terms. We both sides
subtract both by 4x
sides by 4x toto get
get the x’s on
the x's left hand
the left
on the hand side.
side . We then
We then
subtract both sides
subtract both get the
sides by 5 to get right hand
the right
constants on the
the constants hand side:
side :

3 x-‐ 4
3x 4xx << 4 -7 5
‐ x << ‐- ’ 1l
-x

Notice the sign


that the
Notice that changed yet.
hasn’t changed
sign hasn't yet. Now,
N o w, to get rid of the
get rid negative in
the negative front of
in front of the x, we
the x, need to
we need multiply
to multiply
both
both sides 1. Doing
sides by -‐ 1. so means
Doing so means we need
need to reverse the sign.
reverse the sign.

x>
> ll

concept is the
This concept
This the cause so many
cause of so silly mistakes
many silly mistakes that
that it’s important to
it's important it. Just
reiterate it.
to reiterate working with
Just working with negative
negative
numbers does NOT
numbers does N O T mean you need
m e a n you change the
need to change sign. Some
the sign. Some students see that
students see they're dividing
that they’re negative
dividing aa negative
number and impulsively
number and reverse the
impulsively reverse the sign. Don't do
sign. Don't that. Only
do that. sign when
the sign
reverse the
Only reverse when you you multiply or divide
multiply or divide
both sides by a
both sides negative number.
a negative number.

91
91
CHAPTER 11 INEQUALITIES
CHAPTER 11 INEQUALITIES

EXAMPLE1: Which of
EXAMPLE ofthe following integers
the following integer:s is a solution to the
a solution inequality -3x
the inequality 5 -‐ 77xx -‐ 27
‐ 3 x -‐ 7 $ 27??

A ) -‐ 6
A) B )-‐ 3
B) C )1
C)J D )4
0)4

-‐ 33xx -‐ 7 §s- ‐ 77x


x ‐- 227
7
s - 20
41'5‐20
4x
X
x S
g ‐- S
5

At no
no point we multiply
did we
point did multiply oror divide by aa negative
divide by number so
negative number so there was no
there was to reverse
need to
no need sign. We
the sign.
reverse the We
divided a
divided a negative
negative number, but we
0 , but
number, -‐ 220, did soby
we did so by aa positi
positive number, 4.
ve number,

The only
The answer choice
only answer choice that satisfies x3:S
that satisfies answer (A) .
S -‐55 is -‐ 66,, answer §J .
EXAMPLE
EXAMPLE 2: If --77 $ 2x +
5 -‐-2x + 33 5 15, which
$ 15, following must
the following
which of the be true?
must be true?

A
A)) 55$5 xx $$ 6 B ) -‐ 66_$<x_ $
B) x 5-‐ 5 C)‐6§x$5 D )-5‐ 5$5 xx $5 6
D)

So how do
So how do we solve these
we solve ”two‐inequalities-in-one” problems?
these "two-inequalities-in-one" problems? Well, we can split
we can them up
split them into two
up into inequalities
t w o inequalities
that we can
that we solve separately:
can solve separately:
s ‐2x
-‐77 5 - 2x ++ 3
3
-‐2x s 15
2x ++ 33 5 15
Solving the first inequality,
Solving inequality,

-‐77 3 - 2x ++ 3
$ ‐2x 3
-‐ 1100 s
§ ‐- 22xx
5 ~2 xx
S

Solving second inequality,


Solving the second inequality ,

-‐ 22xx ++33§S 115


5
-‐ 22xx $§ 112
2
x 2 ‐- 66
X ~

Putting result s together,


t w o results
the two
Putting the together, we get -‐66 S xx S
we get 5 5.
5. Answer
Answer ~-(C) .

EXAMPLE 3: To follow
EXAMPLE follow his diet plan, James
diet plan, James must his daily
limit his
must limit sugar consumption
daily sugar consumption to to at most 40
at most grams.
40 grams.
One cookie
One has 5 grams
cookie has sugar and
grams of sugar and one fruit salad
one fruit contains 7 grams
salad contains sugar. If
grams of sugar. ate only
James ate
If James cookies
only cookies
and
and fruit which of the
salads, which
fruit salads, the following inequalities represents
following inequalities represents the
the possible number of
possible number cookies c
of cookies c and fruit
and fruit
salads ss that
salads he could
that he one day
eat in one
could eat day and
and remain his diet’s
within his
remain within sugar limit?
diet's sugar limit?

5 7 5 7
A)§+§<40
A) -+-<
S
40 B)§+§§40
B) - + - ::;40 C ) Sc
C) S c++ 7s
7 s< 4400 D 5 c+
D)) Sc +77ss ::;
$ 4400
C C S

The total
The a m o u n t of
total amount sugar he
of sugar gets from cookies
he gets is Sc.
cookies is 5c. The
The total amount of
total amount sugar he
of sugar he gets from fruit
gets from salads is
fruit salads 75.
is 7s.
50 his total sugar intake
So his total sugar intake for any given day is Sc + 7s, and since it can't be more than 40 grams, Sc + 7s s 40.
any given day is 5c + 75, and since it can’t be m o r e than 40 grams, Sc+ 75 S 40.
Answer ~(D) .
Answer

92
THE COLLEGE PANDA
THE COLLEGE PANDA

From a graphing
From a graphing standpoint,
standpoint, what
what does an inequality
does an inequality look like? What does it mean
What does mean for y > ~x
- x -‐ 1?
1?
y

----

As shown
shown by the shaded
shaded region above, the inequality
region above, inequality y > -‐xx -‐ 1 represents points above
represents all the points above the line
y== ‐x
- x ‐- 1. If
If you have a
you have a hard time keeping
hard time track of what’s
keeping track above a
what's above a line
line and
and what’s just look
below, just
what's below, at the
look at
y-axis. The line cuts
cuts the y-axis into
into two parts.. The top part
two parts y-axis is always
part of the y‐axis always in the "above" region.
the ”above" region. The
bottom part of the y-axis is always
bottom part always in the ”below" region. If the graph
"below" region. graph doesn’t
doesn't show with
intersection with
show the intersection
you can always
the y-axis, you always just draw your oown
draw your vertical line through
w n vertical the graph
through the graph to determine the ”above”
determine the and
"above" and
"below" regions.
“below” regions.
Also note that the line
note that line is dashed.
dashed . Because and NOT y z= ‐x
Because y > -‐xx -‐ 1 and - x -‐ 1, the points
points on the itself do
line itself
the line
o t satisfy
not
n the inequality.
satisfy the inequality . If the equation were y 2::
equation were 2 ‐x ‐ then
- x - 1, then the line would be solid, and points on the line
line would be solid, and points on line
would satisfy
would the inequality.
satisfy the inequality .
y

- X

about a
what about
But what a system inequalities? For example,
system of inequalities? example,

y ::;
5 ‐x- x ++ 44
1
yY >
- ~2 x ‐ 3
-x- 3

When it comes
When comes to graphing, goal is to find the region
graphing, the goal with the points
region with that satisfy
points that both inequalities.
satisfy both inequalities. In this
1 . .
case, we want the points
we want below y = -‐xx +
that are below
points that + 4 but y=
above 31
but above = ~x
Ex ‐- 3.
3. To locate points, we
this set of pomts,
locate this can
we can

shade the regions


shade regions below -x
= ‐x ++ 4 and
below y z and above
1
1x-
above y = Ex -‐ 3 and
.
e g i o n s overlap.
the rregions
where the
and see where overlap .

The overlapping region on


overlapping region the points
contains all the
the left contains
on the points that
that are solutions
solutions to the system.
system.

93
93
CHAPTER 11 INEQUALITIES
CHAPTER 11 INEQUALITIES

N o w if
Now if we
we solved system as
solved the system asifif it were
were a system of equations
a system instead of a
equations instead of inequalities,
system of
a system we would
inequalities, we would
get the intersection point
the intersection the ttwo
point of the lines, which,
w o lines, case, happens
which, in this case, solution with
be the solution
happens to be with the highest value
the highest value
of x. As an exercise, let’s
an exercise, let's find this solution.
find this Substituting the
solution. Substituting ”equation” into
the first "equation" into the second, we
the second, get
we get

11
-‐ xx+
+ 4 == -2x
‐ x ‐- 3
2
-‐ 2
2xx++88=x- :x‐6
-‐ 33xx = ‐- 114
4

= z‐__3
14
- 14
.Xr ‐ z ;:::::
44.66.66
3

At x
x = 4.66, y = -‐4.66 + 4 = -‐0.66
4.66 + (we get
0.66 (we this from
get this the first equation).
from the Therefore, (4.66,
equation). Therefore, (4.66, -‐O.66) is the
0.66) is solution
the solution
with the highest
with the highest value
value of x. There
There are no solutions
are no solutions in which
which x is 5, 6, or larger.
larger.
While finding
While finding the intersection point
the intersection point in this example may
this example have seemed
may have bit pointless
seemed a bit these points
(hahal), these
pointless (haha!), points can
can
be
be very important in the context
very important given situation,
context of a given as finding
such as
situation, such the right
finding the right price maximize profit
price to maximize profit or
figuring out the
figuring out right amount
the right amount of materials construction project.
materials for a construction project.

EXAMPLE4:
EXAMPLE4:
y

II I

lII IV

system of inequalities
following system
The following inequalities is graphed
graphed in the xy-plane above.

y2 2::‐- 33xx + 1
y 22::22xx -‐ 3

Which quadrants
Which quadrants contain solutions to the
contain solutions the system?

Quadrants I and
A) Quadrants II
and JI Quadrants I and
B) Quadrants and IV C) Quadrants
C) Quadrants III
I I I and
and IV D) Quadrants
D) I, II,
Quadrants I, and IV
I I , and IV

First, graph
First, graph the equations, preferably
the equations, preferably with
with your graphing calculator.
your graphing calculator. Then the regions
shade the
Then shade and find the
regions and the
overlapping region.
overlapping region .

.1/

As you
As see, the
you can see, overlapping region,
the overlapping which contains
region, which contains all
all the solutions, is
the solutions, the top
is the region.. It
top region has points
It has in
points in
quadrants I, II,
quadrants Answer ~.(D) .
and IV. Answer
I I , and

94
THE
THE COLLEGE PANDA
COLLEGE PANDA

Ecologists have
EXAMPLE 5: Ecologists determined that
have determined number of
the number
that the frogs y
of frogs must be
y must than or
greater than
be greater equal to
or equal to tluee
three
times number of snakes
times the number a healthy
snakes x for a ecosystem to
healthy ecosystem to be
be maintained
maintained in a particular
in a forest. In
particular forest. In addition,
addition,
the number
the number of frogs and
of frogs and the number of
the number of snakes must sum
snakes must at.least 400.
sum to atwleast

PART1:
PART l! Which the following
Which of the of inequalities
systems of
following systems inequalities expresses
expresses these for a
conditions for
these conditions healthy ecosystem?
a healthy ecosystem?
A)
A) y 2~33xx B)
B) y 2~ 33xx C)
C) y 2~33xx D)
D) y ~ 3x
3,53:
y ‐- xx>400
>400 y‐x2400 y+12400 y+x5400

PART forest currently


the forest
PART2: IfIf the healthy ecosystem,
has a healthy
currently has what is the
ecosystem, what minimum possible
the minimum number of frogs
possible number in
frogs in
the forest?
the forest?

Part 1 Solution:
Solution: The number
number of frogs, y, must
must be
be at
at least
least three
three times the number
number of snakes, x. So,
snakes, x. The
3x. The
50, y 2".
2 3x.
number of frogs
number frogs and
and the number
number of snakes
snakes must
must sum least 400, so
sum to at least soyy +
+ x 2". Answer ~-(C) .
2 400. Answer

Solution: In
Part 2 Solution: these types
ln these types of questions,
questions, the strategy is to look for the
the strategy the graph
minimum in the
the minimum graph of the
the
inequalities. The minimum
inequalities. minimum (or maximum) will typically
maximum) will occur at
typically occur point. To show
intersection point.
at the intersection show you what l1
you what
mean, let's
mean, let’s first put the second
put the inequality in y =
second inequality mx +
= mx + b form.

y 22".
33xx
y 22:‐ -x x++4 400
00

Now we can graph


Now we the inequalities
graph the using aa calculator
inequalities using calculator..
y

The graph
The graph confirms
confirms that y, the
that y, the number
number ofof frogs,
frogs, is at a minimum
is at minimum at the intersection point. After
intersection point. all, the
After all, the
overlapping region (the top region) represents
overlapping region (the top region) represents all all possible solutions and
possible solutions and the
the intersection
intersection point
point is at the
the bottom
bottom
of this region,
of region, representing
representing the solution
solution with
with the minimum number of frogs.
minimum number
We can find
We can find the
the coordinates of that
coordinates of that intersection
intersection point solving a
point by solving a system
system of equations
equations based
based on
on the ttwo
wo
lines.
lines.

= 3x
y= 3x
y=
= -‐xx + 400

Substituting the
Substituting the first equation
equation into
into the second,
second,

3x =
= -‐xx +
+ 400
4x =
: 400
x = 100
X

So, 100
So, is the
100 is the x-coordinate.
x-coordinate. The
Theyy‐coordinate
-coordinate must
must then beyy := 3x
then be 3x = 3(100) =
= 3(100) Given these
: 300. Given these values,
values, the
the
intersection point
intersection point is
is at
at (100,300 and the
(100,300)) and minimum possible
the minimum possible number
number of frogs is j 300 when I
when the forest has
has a
healthy
health y ecosystem.
ecosystem.

95
95
CHAPTER 11 INEQUALITIES
CHAPTER 11 INEQUALIT'IES

CHAPTER EXERCISE: this chapter


Answers for this
EXERCISE:Answers start on page
chapter start page 299.

A calculator
calculator is allowed
allowed on the following
following
questions.
questions.
y

Which the following


Which of the following is a solution to
a solution to the
the
inequality
inequality ‐x
- x -‐ 4
4 > 43:
4x ‐- 14?
14?
------.F------+ X
A
A)) -‐ 1
B) 2
C) 5
D) 8

Which of the following systems of inequalities


could be the one graphed in the xy-plane above?
A) y > 3
IfIf 3Zx‐4> 1
~ x - 4 > Ex‐10,whichofthefollowing
~x - 10, which of the following
A )y>3
yy >
> xx
mustbetrue?
must be true?
B) y < 3
A)) x <<224
A 4 B) y < 3
y <x
B)) x >>224
B 4 y<x
C < ‐-224
C) ) x < 4 C)) y
C y< < 33
D
D)) x >> ‐-224
4 yy >
> xx

D
D)) yy >> 3
y<
<x

Jerry estimates that


Jerry estimates that there marbles in a jar.
are m marbles
there are
Harry, who knows
Harry , who knows the actual
actual number
number of marbles
marbles
the jar, notes
in the notes that the actual
that the actual number,
number, n, is
within 10 marbles
within (inclusive) of Jerry's
marbles (inclusive) Jerry's estimate.
estin1ate .
Which of the
Which following inequalities
the following inequalities represents
represents
the relationship
the between Jerry’s
relationship between estimate and
Jerry's estimate the
and the
actual
actual number
number of marbles
marbles in the
the jar?
n+
A)) 11
A + 10 5 mm '.S
1 0'.S 5nII ‐- 110
0
B
B)) mm-‐ l10
O '.S
S nn S'.S
nmH+ - I10
O
C
C)) 11
n '.S
Sm m S'.Sl 1011
On
m
D)
D) ‐-10_n_10m
< n < 10m
10 <- -

96
R E ‑ THE COLLEGE
THE COLLEGE PANDA
PANDA

manufacturer produces
A manufacturer produces chairs
chairs for aa retail
retail store
store ‘ yy
according
according to the M =
the formula, M = 12P +
formula, 12P + 100, where
where
M is the number of units
the number units produced
produced and and PP is
is the
the
retail price
retail price of each chair. The number
each chair. number of units
units
sold by the
sold the retail store is given
retail store given by
N= = ‐- 33PP + 970, where
where N is thethe number
number of of units
units f'--------1--------.i X

sold
sold and retail price
and P is the retail each chair.
pri ce of each What
chair. What
the values
are all the
are values of P for which
which the number of
the number of
units produced
units produced is greater
greater than or equal
than or to the
equal to the
number
number of units sold?
unit s sold?
A) P2
A) P ~ 58
58 The graph
The the xy-plane
graph in the could represent
above could
xy-plane above represent
B) P
B) P5:S 58
58 which of
which the following
of the systems of
following systems of inequalities?
inequalities?
P 2:::::
C) P 55
55 A)
A) y
y 2:::::
3
3
:S55
D) P g y 5 ‐3
-3:s
yS
B) y
B) :S 33
y :::::
y z ‐- s3
If
If n is an
an integer and 3(n ‐- 2) > ‐4(n
integer and - 4(n -‐ 9),
9), what
what C)
C) x 2 3
X:::::
3
least possible
the least
is the value of n
possible value n?? r < _3
X '.S- 3

D) x S
:S 3
xX 22 -‐33

work, Harry
To get to work, must travel
Harry must miles by bus
travel 8 miles bus
and 16miles
and 16 train everyday.
miles by train bus travels
The bus
everyday . The travels
at an average speed
an average miles per
speed of x miles hour and
per hour and the
the
train travels
train travels at an average
at an speed of y miles
average speed miles per
per
hour . If
hour. Harry's daily
lf Harry’s never takes
commute never
daily commute takes more
more
than 1 hour,
than which of the
hour, which following inequalities
the following inequalities
represents the possible
represents the average speeds
possible average speeds of thethe bus
bus
and train during
and train during the
the commute?
commute?
8 16
A)§+ES1
A) -+-:S
y
l
X x y
16 8
B>E+§g1
B) -+-:S
X y
l
x y
yy
C)8+16‐1
C) _ xX
- +-‐<
8 16 -
<1
D
D)) Bx+
8 x + 16y
1 6 y:S
§ 11

97
11 INEQUALITIES
CHAPTER 11
CHAPTER INEQUALITIES

An ice
An ice cream distributor contracts
cream distributor contractsout
out toto two
two 3
different companies to
different companies to manufacture
manufacture cartons
cartons of
y2§x+2
Y -> -2 x + 2
ice
ice cream Company A can
cream.. Company can produce
produce 8080 cartons
cartons y g::;‐-Z2xx -‐ S
5
each and Company
hour and
each hour Company B can produce 140
can produce
each hour.
cartons each
cartons The distributor
hour. The needs to
distributor needs to fulfill
fulfill Which of the
Which the following
following graphs the xy-plane
graphs in the xy-plane
an order
an order of over 1,100 cartons
over 1,100 cartons in 10 hours of
10 hours could represent
could represent the system of inequalities
the system inequalities above?
above?
contract time. It contracts
contract time. hours to
out x hours
contracts out A)
A)
Company A and
Company the remaining
and the hours to
remaining hours
Company B. Which
Company Which of the following inequalities
the following inequalitie s
all possible
gives all
gives values of xx in the
possible values context of
the context
this problem?
this problem?
140
A)
80
~> >1,100
1100
X +
A) ‘x‐ + 10
10‐- Xx I

B) 140x +
+ 80(10 -‐ x) >
>1,100
1, 100 B)
B)
80x +
C) 80x 140(10 -‐ x) > 1,100
+140(10 1,100
D) 80x +
+ 140(x 10) > 1,
140(x -‐ 10) 1,100
100

y >>1l5 +aa
5 xx + C)
C)
y< +bb
<55xx +
In the system of inequalities
the system above, a and
inequalities above, b are
and bare
constants. lf (1,
constants . If 20)} is a solution
(1,20 solution to the
the system,
system,
which of the
which following could
the following be the
could be the value of
value of
b -‐ aa ?
A) 6
A) D)
D)
B) 8
C ) 110
C) 0
D ) 112
D) 2

98
THE COLLEGE PANDA
THE COLLEGE PANDA

20 9 .
Tina works
works no more than
no more than 30 hours at
30 hours at a nail salon
a nail salon If - < - 2x + 4 < - , what 1s one possible
each week. She
each week. can do
She can do aa manicure
manicure in 2020 minutes
minutes 3 2
value of x - 2 ?
and a
and a pedicure
pedicure in 30 minutes . Each
30 minutes. manicure
Each manicure
earns her $25 and
earns her and each pedicure earns
each pedicure earns her
her $40,
and she
and she must earn at least
must earn least $900 to cover
cover her
her
expenses.
expenses. IfIf during
during one week, she
one week, she does enough
does enough
manicures m and
manicures and pedicures cover her
pedicures p to cover her
expenses,
expenses, which the following
which of the following systems
systems of of
inequalities describes her
inequalities describes working hours
her working hours and
and her
her
earnings?
earnings?
A)
A) 3m +
+ 2p ::;
5 30
25m + 40];
40p 2 900
Joyce wants to create
Joyce wants create a rectangular garden
a rectangular that
garden that
has an
has area of at least
an area least 300 square and a
meters and
square meters a
B) 2m +
+ 3p S
::; 30
perimeter
perimeter of at least 70
at least the length
meters . If the
70 meters. length of
25m +40p
25m + 40p 2 900 the garden
the meters long
garden is x meters the width
and the
long and is yy
width is
meters long,
meters long, which
which ofof the following systems
the following systems of of
C)
C) flm
-+-<
p
3 +2E<_30
30 inequalities represents Joyce’s
inequalities represents Joyce's requirements?
requirements?
3 2-
A) xy 2 70
25m+40p2900
25m + 40p 2 900
x+
+ y 2 300
300
D)
D) 3-+-2_900
_"1
-+->
Ill p 900
E>
3 2- B
B)) x xy
y 221 150
50
25m + 40p ::; 30
25m+40p§30 xX + y.1/22770
0

C
C) ) x xy
y 223 300
00
xX + y.1/22770
0

If k 5::; x 5 3k+
::; 3k + 12, which of the
which of following must
the following must D
D) ) x xy
y 223 300
00
be true?
be true? xX +
+ y.1/223355
I. x ‐- 12
12 5::; 3k
lII.
l. k 2 -‐ 66
III.
111. x -‐ k 220
A) lonly
I only IfIf a
a < b, which
which of the following
of the must be
following must be true?
true?
B) n d lIIl only
B ) Il aand only I. a2 < b2
a2 < b2
C)
C) llII and
and IlII I I only
only 1],
II. 2a2a << 2b
II, and
D) I,I, 11,
D) and 111 III I ] . -‐ b <
I111. < -‐ a
A) 11
II only
only
B n d lIIl only
B)) lIaand only
C) II and
and III] only
[ I only
D I , and
D)) II,, III, and IIII
ll

99
For many
For students, solving
many students, solving word
Word
WordProblems
Problems
word problems
problems isis a frustrating experience.
a frustrating
12
They require
experience. They you to
require you translate the
to translate the
question
question before you can
before you can even the math.
even do the The examples
math. The and exercises
examples and the exercises
the in this chapter will show you
chapter will show you how
how
to handle
handle the range of word
the full range problems that
word problems tested. You will
are tested.
that are develop an
will develop instinct for
an instinct for translating words
translating words
math, setting
into math,
into the right
setting the right variables, and finally
variables, and solving for the answer.
finally solving Experience is the
answer. Experience guide.
best guide.
the best

EXAMPLE 1: The
EXAMPLE The sum integers is 72. What
consecutive integers
three consecutive
sum of three the largest
What is the largest of three .integers?
these three
of these integers?

most important
The most
The important technique solving word
technique in solving problems is to let a
word problems be one
variable be
a variable of the
one of things
the things you don't
you don’t
know. In this
know. problem, we don't
this problem, any of the
know any
don’t know three integers, so
the three integers, so we let the smallest one
let the smallest one bebe x.
x. It doesn't
It doesn’t
which number
m a t t e r which
matter number we set as as long
as x, as as we're
long as consistent throughout
we’re consistent the problem.
throughout the problem.
So if xis
So if x is the
the smallest, o u r consecutive
then our
smallest, then integers are
consecutive integers are

x, x +
x,x 1,x +
+ 1,x + 2

Because they sum


Because they we can
sum to 72, we an equation:
make an
can make equation:
+ ((xx ++ 11)) + ((xx + 22)) =: 7722
xx +
3x + 33 z=772
3 x+ 2
3 x=
3x : 669
9
xX :=223
3

Because x is the
Because xis the smallest,
smallest, our three consecutive
o u r three integers must
consecutive integers must then be 23, 24, and
then be 25 I(the
and I25 largest) .
(the largest).
would the
what would
But what solution have
the solution have looked like if we
looked like had let
we had let x be largest integer?
be the largest Our
integer? O u r three integers would've
three integers would’ve
been
been
x -‐ 2,x
2,.\‘ -~ 1,x
1, x

A n d our
And equation would’ve
o u r equation would've been
been

(x - 2) + (x - 1) + x = 72
3 x-‐ -3 =
3x ‐ 772
2
3xr =
3 ‐ 775
5
rX ‐=2255

A n d because
And because x was set to be the
was set largest of the
the largest the three integers in this
three integers we're already
scenario, we’re
this scenario, already at the answer!
the answer!

100
THE COLLEGE
THE PANDA
COLLEGE PANDA

The lesson
lesson here
here is that you should
that you think about
should think which unknown
about which unknown you want to
you want set as
to set as the variable. Often
the variable. Often times, that
times, that
unknown will
unknown be what
will be what the question
question is asking for.
is asking for. Other times, itit will
Other times, will bean
be an unknown
unknown you you specifically
specifically choose to
choose to
make the problem
make easier to set up and
problem easier solve. And
and solve. And sometimes,
sometimes, as was the
as was in Example
case in
the case Example 1,1, itit doesn’t
doesn 't matter
matter
which unknown you
which unknown you pick; you’ll end up
you'll end with the
up with answer with
same answer
the same with the amount of
same amount
the same of effort.
effort.

EXAMPLE 2:
EXAMPLE 2: One
One number is 3 firms
number is times another If they
number. U
another number. sum to
they sum to 44, what is
44, what the larger
is the larger of
of the
the two
two
numbers?
numbers?

In
1n this problem,
problem, we want to
we want to set xx to be the
to be smaller of
the smaller of the numbers. That
two numbers.
the two That way,
way, the two numbers
the two can be
numbers can be
expressed
expressed as
as
xx and
and 3x
3x
If we
we let xx be
be the larger
larger of
of the
the two, we would
two, we to work
have to
would have work with
with

xX
x.r an
andd ‑3
3
and fractions
and are yucky.
fractions are
Setting
Setting up oour
u r equation,
equation,

xX + 3x
3x =
: 44
44
4x = 44
4x = 44
xX =
= 11

Becareful‐we’re not done


Be careful - we're not The question
done yet! The the larger
for the
asks for
question asks larger of
of the two, sowe
the two, so we have to multiply
have to multiply x
x by
by 3
3 to
to
get I-.33 I.

EXAMPLE
EXAMPLE 3: What
What is
is a number such
a number that the
such that square of
the square the number
of the is equal
number is equal to 2.7% of
to 2.7% reciprocal?
its reciprocal?
of its

Let the number we're looking


number we’re looking for bex.
be x.
2 1
X =
x2 = .027 x1
X -
X

Multiply sides by
both sides
Multiply both by x
x to
to isolate
isolate it.
it.
xx33 =
= .027
Cube
Cube root both sides.
root both sides .
x:‑
X= []]

101
CHAPTER 12 WORD
CHAPTER 12 WORD PROBLEMS
PROBLEMS

EXAMPLE4:-Albert
EXAMPLE4: Albert is 7 years
years older
older than Henry. In 55 years,
than Henry. years, Albert will be
Albert will twice as
be twice old as
as old How
Henry. H
as Henry. old
o w old
is Albert now?
now?

Let x be Albert's age


be Albert’s We could've assigned
o w. Wecould’ve
age nnow. assigned x to be
be Henry’s
Henry's age,
age, but as we mentioned
but aswe earlier, assigning
mentioned earlier, the
assigning the
variable to be
variable be what the question
what the is asking
question is asking for is typically the faster route. Now at this point, some you might
typically the faster route. Now at this point, some of you might
be thinking of assigning
be thinking another variable
assigning another Henry’s age.
variable to Henry's age. While
While that would certainly
that would certainly work, would only
work, it would add
only add
more steps
more steps to the solution. Try to stick
the solution. stick to one variable unless
one variable the question
unless the question clearly more.
calls for more.
clearly calls
If
If Albert years old
Albert is x years then Henry
o w, then
old nnow, Henry m be x -‐ 7 years
u s t be
must old .
years old.
Five years
Five years from
from nnow, Albert will
o w, Albert be x +
will be + 5 and Henry will
and Henry be x ‐- 2 years
will be old .
years old.

x
X + 55 ==22(x
+ ( x -‐ 22))
x + 5 = 22xx -‐ 4

we
EXAMPLE5:
EXAMPLE5: JakeJake can
can runmm6060 yards per minute.
yards per Amy can
minute. Amy UO yards
u n 120
can rrun per minute
yards per minute for the first
for the 10minutes
first 10 minutes
then slows
but then
but slows down
down to 20 20 yards per minute
yards per thereafter. If they
minute thereafter. start running
they start at the
running at same time,
the same time, after how
after how
many minutes t w
many minutes i l l both
will Jake and
both Jake and Amy have run
Amy have r u n the same distance,
the same assuming t > 10
distance, assuming ?
10 ?

The problem
The problem already
already gives
gives us
us aavariable with. We
work with.
variable t to work Wewant equate Jake's
want to equate distance rrun
Jake’s distance u n with Amy’s..
with Amy's
Jake’s
Jake's distance:
distance: 60t
Amy’s distance:
Amy's 120(10) +
distance: 120(10) + 20(t
20(t -‐ 10)
10)

= 120(10 ) + 20(t - 10)


60t::120(10)4-20(t‐»10)
60t
60t = , 2 o o+
: 11,200 + 20t
2 0 ; - 200
40tn =
M = 11,000
bm)
tf =z 25fi

After I2sIminutes,
After minutes, they will have
they will have rrun same distance.
the same
u n the distance.

EXAMPLE6: At a
EXAMPLE pharmaceutical company,
a pharmaceutical company, research must be
equipment must
research equipment shared among
be shared the scientists.
among the scientists.
microscope for every
one micromope
There is one one centrifuge
scientists, one
every 4 scientists, for every
centrifuge for 3 scientists,
every 3 one
and one freezer
scientists, and for
freezer for
every scientists. If there
2scientists.
every 2 total of 52
there is a total pieces of research
52 pieces research equipment company, how
equipment at this company, how many scientists
many scientists
are there?

Let x be the number


be the number of scientists.
scientists. Then number of microscopes
U1enumber
Then the microscopes IS i,
. X
number of centrifuges
the number
is 1, the
.
is 5, and
centrifuges IS and i,
, x

. x
the
the number
number of freezers IS ~
freezers is 5..
X X x
X
+ - =
-Z4 ++ -3 + _ 552
2
2
Multiply
Multiply both sides by 12
both sides get rrid
12 to get i d of the fractions,
the fractions,

3x 4xx + 66xx z=552


3 x+ 4 2 ~• 12
l2
1
13x3 x=: 6624
24

x:‑
102
THE
THE COLLEGE PANDA
COLLEGE PANDA

EXAMPLE 7:
EXAMPLE 7:A group of friends
A group friends wants
wants to split the
to split cost of renting
the cost renting a cabin equally.
a cabin friend pays
each friend
equally. If each pays $130,
they will
they will have
have $10 too much. If each
too much. each friend pays $U0,
friend pays $120, they will have
they will have $50 too little. How
too little. How much does it cost
much does cost
to rent the -cabin?
rent the cabin?

t w o unknowns
have two
We have problem.. We'll
unknowns in this problem We’ll let number of people
let the number group be
people in the group be n and
and the
the cost of
renting aa cabin
renting be c.
cabin be c. From the information
From the given, we
information given, we can come
come up
up with w o equations
with ttwo sure you
(make sure
equations (make you see the
the
reasoning behind
reasoning behind them):

13011 -‐ 10 =
13011 z cC
120n +
+ 50 = cC

equation, 13011
In the first equation, represents the
130n represents the total
total amount group pays,
amount the group but because
pays, but because that’s 10 dollars
that's 10 dollars too much,
much,
need to subtract
we need subtract 10
10to the cost
arrive at the
to arrive second equation,
rent, c. In the second
cost of rent, equation, 120n represents total amount
represents the total amount
group pays,
the group
the pays, but
but this time
time it's 50 dollars
it’s 50 little, so
dollars too little, so we need to add
we need 50 to arrive
add 50 at c.
arrive at 6. Substituting from
Substituting c from
the equation into
the first equation into the second,
second, wewe get
get

120n + 50 = : 130n
130" -‐ 10
-‐ 1lOn
0 n=-
: ‐ 660
0
= 6
n=

So there
So there are group. And
friends in the group.
are 6 friends And

c:=
C 13011‐10=
13011 1 3 0· 6- 6- ‐ 10
- 10 = 130 1 0== 770

renting the cabin


The cost of renting is I-.770 1-
cabin is

EXAMPLE 8:
EXAMPLE 8: Of the jellybeans in
the 200 jellybeans in aa jar, 70% are green and
are green and the rest are
the rest red. How
are red. many green
How many jellybeans
green jellybeans
be removed
must be
must removed soso that 60% of
of the
the remaining jellybeans are green?
remaining jellybeans green?

answer is NOT
The answer N O T 20. You can't take 10% of the
just take
can’t just the green jellybeans away
green jellybeans because as
away because as you do that,
you do that, the total
total
number of jellybeans
number jellybeans also
also goes down. We first find that
down. We that there are % x
there are :a
x 200 = 140 green jellybeans.. We
green jellybeans need to
We need
remove x of them
remove that 60% of what's
so that
them so green:
what’s left is green:

jellybeans left _ 60“/¼


green jellybeans
green
-------
totaljellybeans
total jellybeans left ‘
= 6O°oo

140‐x_
140 - X 66
2 0 0- ‐Xx _
200 - 110
0
multiplying,
Cross multiplying,

10(140 -‐ x) = : 6(200 -‐ x)
1,400
1, 10x := l,1,200
400 -‐ lOx 200 -‐ 6x
200 == 4x
= 50
x _‐_ 50
X

IsoIgreen jellybeans need


green jellybeans to be
need to be removed This type
removed.. This type of word problem
of word with percentages
problem with very common
is very
percentages is common in
in
chemistry and is typically
chemistry and known as
typically known asa problem..
”mixture" problem
a "mixture"

103
CHAPTER
CHAPTER 12 WORD
WORD PROBLEMS
PROBLEMS

Our next example


Our next example is the area / perimeter word
the classic area/perimeter problem.
word problem.

EXAMPLE 9: A rectangle
EXAMPLE rectangle has
has aa width is 3
that is
width that 3 inches shorter than
inches shorter than its length. It
its length. If the area of
the area of the
the rectangle is
rectangle is
108 square
square inches, what is the
inches, what perimeter, in inches,
the perimeter, inches, of the rectangle?
of the rectangle?

Em I
If
If we the length
we let the be I, then
length be/, the width
then the w is/
width w is I -‐ 3. Since
Since aa rectangle’s
rectangle 's area is equal
area is equal to the length
to the length times the width,
times the width,
we can set up
we can following equation:
the following
up the equation :

w=
llw : 108
1(1‐
/(/ - 3) = 108
12 31 ‐ 108 z= o
12‐- 31- 0
( 1-‐ 112)
(/ 2 )(/( 1
+ +9)9 =
):o
0

length of a
the length
Since the a rectangle has to be
rectangle has positive, /I =: 12. The width
be positive, width is then /I -‐ 3
is then 3 =z 12
12‐- 3
3 =z 9. Finally, the
9. Finally, the
perimeter 21+
perimeter is 2/ + 2w
210 = + 2(9) =: I-.42 1.
: 2(12) +
Never forget
Never that the perimeter
forget that a rectangle
perimeter of a rectangle is twice the length
twice the length plus
plus twice the width.
twice the width . I’ve many
seen too many
I've seen
just add
students just
students the length
add the length and
and the width without thinking
width without thinking it through.
through .

EXAMPLE
EXAMPLE 10: When Alex and
When Alex Barry work
and Barry separately from
work separately from each other, Alex
each other, Alex can paint a
can paint house in
a house in 6
6 days,
days,
Barry can
and Barry can paint a house 12days. Assuming
house in 12 Assuming that they each work at a constant
that they each work at a constant rate, how many days
days
w it take
i l l it
will Alex and
take.Alex Barry to paint a house if
and Barry they work together?
if they together?

the typical
This is the
This typical “work‐rate” problem that
"work-rate" problem involves two
that involves who work
individuals who
t w o individuals work at different rates.
at different general
The general
rates . The
approach is to use
approach use the formula W = rt,
the formula where W
r t , where W is
is the
the amount of work done, r is the overall rate
amount of work done, r is the overall rate at which at which
work is being
work done, and
being done, and t is the time spent. The key
the time spent. The key thing thing to note is
note thatthat the overall rate, r, can be found
overall rate, r, can be found by by
summing up
summing the individual
up the individual rates.
rates.
11
can paint
Since Alex can paint a house days,, his
house in 6 days his rate house per
rate is 5 of a house day. Since Barry
per day. can paint
Barry can a house
paint a house in 12
in 12
6
1
days, his
days, rate is ‐ of a
his rate a house per day.
house per
112
2
. . 1 1 _ 2 1 _ 3 _ 1 _
Working
Working together,
together, they can pamt 6 +
can pamt + 12 =‐ 12 + 12 =‐ 12 = 4 of
of a per day.
house per
a house we can
N o w we
day. Now use W
can use W=‐ rt,
rt,
6 12 12 12 12 4
where W =
where = 1 (i.e.
(Le. 1 house) and r =
house) and z ~, find the
31, to find the time will take
time it will take them
them to paint one
to paint house..
one house

W=
z rrtt
1
1l z= ‐- t
4
4 =: 1t
Therefore, it will take
Therefore, take Alex and Barry [±]days
and Barry days to paint one
to paint one house. This answer
house. Thjs makes sense
answer makes because ifif Alex
sense because Alex
finish a
can finish
can a house days by himself,
house in 6 days then should take less than days Barry working
himself, then it should take less than 6 days if Barry is workjng alongside
it 6 if is alongside
him.
him.

104
THE COLLEGE
COLLEGE PANDA
PANDA

CHAPTER EXERCISE:Answers for this chapter start on page 301.

A calculator should N O T be
should NOT used on the
be used
following
following questions.
questions.
A rectangular has a
monitor has
rectangular monitor a length
length of xx inches
inches
and a
and width that
a width one-third of its length.
that is one‐third length. If the
the
perimeter
perimeter of the monitor is 48
the monitor 48 inches, what is the
inches, what the
Which the following
Which of the represents the square
following represents square of value
va lue of x?
x?
the sum of x and
the sum and y,
y, decreased the product
decreased by the product of
x and
and y
y??
A) x2
A) +y2
x2+ y 2 -‐ xy
xy
2y 2 -‐ xy
B) xx2y2 Jr};

C) (x + y) 2 - ((xH+yy))
C) (x+y)2‐
D) (x + y)2 -‐ xy
D)

Susie buys
Susie pieces of sa
buys 2 pieces salmon,
lmon, each weighing x
each weighing
On
On aa 100 cm ruler, lines
cm ruler, drawn at
are drawn
lines are at 10, X, and
and pounds, and 11 piece
pounds, and piece of trout,
trout , weighing
weighing y
distance between
The distance
98 cm. The between the lines at X and
the lines and pounds, where x and
pounds, where and y are
are integers. salmon
integers . The salmon
three times
98 cm is three the distance
times the between the
distance between per pound
cost $3.50 per and the
pound and the trout
trout cost per
cost $5 per
lines
lines at X and
and 10 cm. What is the
What the value value of X ?? pound.
pound . If the
the total cost of the
total cost was $77, which
the fish was which
of the following
following could be the
cou ld be lue of y?
value
the va y?
A
A)) 4
B) 5
C) 6
D) 7

If 5 is added to the square root of x, the result is A 20% nickel alloy was
nickel alloy made by
was made by combining
combining 22
9. What is the va lue of x + 2? grams
grams of a nickel alloy with
a 35% nickel with 6 grams
grams of an
an
x% nickel alloy. What
x % nickel What is the value
value of xx ?
?

A grocery store sells tomatoes in boxes of 4 or 10.


If Melanie buys x boxes of 4 and y boxes of 10,
where x 2: 1 and y 2: 1, for a total of 60
tomatoes , what is one possible value of x ?

105
CHAPTER 12 WORD
CHAPTER WORD PROBLEMS
PROBLEMS

allowed on the following


A calculator is allowed following
questions.
questions. At a Hong Kong learning center, i of the
1
studen ts take
students debate, 2 of the
tak e debate, studen ts take
th e students tak e
6
If 8 + 5x is
’ twice
' x -‐ 5,, what
h is' the value off x ?
8 + 5x 15tw1ce x 5 w at 15the value 0 x
? ..
~1 of the
and §
writing, and
writlng, the students
,
take scrence.
students take The
science . The
A
A) ‐6- 6
) math. If 33students
take math.
rest take
rest 33 studen ts take math, what
take math, is
what is
B) -3
B) ‘3 the total
the number of students at the learning
total number students at the learning
center? 7
C) _§7 center.
C) 3
A
A)) 660
0
D) -‐22
D) B) 66
B) 66
C
C)) 7722
_ D ) 7 8
D) 78

IfIf 75°/o
75% of is the
of 68 is the same of n, what
same as 85% of what is the
the
vvalue
a l u eofdn?
"? _

Ian has 20football


Ian has 20 footba ll cards, and Jason
cards, and Jason has 44
has 44
baseball cards . They
baseball cards. They agree
agree to trade such that
trade such that
Jason
Jason gives Ian 2baseball
gives Ian 2 baseball cards every card
cards for every card
Ian gives to Jason.
Ian gives Jason. After how many
After how suc h trades
many such trades
will Ian and
will Ian Jason each
and Jason each have an equal
have an equa l number
number of
cards?
cards?

| | _ A ) A) 9
9
B
B)) 110
0
The Pirates
The Pirates w o n exactly
won exactly 4 of their 15 games.
their first 15 games . C)
C) 1111
They then played
They then played N remaining
remaining games
games andand won
won D) 12
D) 12
all of them.
all them. If they
they w o n exactly
won half of all
exactly half all the
the
games they played,
games they what is the
played, what value of N ??
the value

IfIf 3 is subtracted from 3 times


subtracted from the number
times the number x, the
the
result
result is 21. What
What is the
the result when 8 is added
result when added to
half of x ?
A) 1
B) 5
| | _ B ) C) 8
5
Alice and Julie
Alice and start with
Julie start with the number of
same number
the same D) 12
D) 12
pens.
pens. After
After Alice
Alice gives
gives 16of
16 of her
her pens
pens to Julie,
Julie,
Julie
Julie then
then has times as
w o times
has ttwo many pens asAlice
as many pens as Alice
does. H
does. How many pens
o w many did Alice
pens did have at the
Alice have the
start?
start?

106
THE COLLEGE
COLLEGE PANDA
PANDA

At aa store,
three
the price
store, the
times the
three times
price of a
price of
dollars less
a tie is k dollars than
less than
the price a shirt. If a shirt costs $40
a shirt. If a shirt costs
Mark and
Mark and Kevin
Kevin o
own i i
w n %and
and g of the books on
the books on aa
shelf, respectively.
shelf, respectively . Lori the rest
owns the
Lori owns rest of the
the
and
and a tie costs $30, what
tie costs what is the value of k?
the value k? books than Mark,
more books
books. If Kevin
books . If w n s 9 more
Kevin oowns than Mark,
how many
how many books
books does Lori own?
does Lori own?

A wooden board in the


wooden board shape of a
the shape a rectangle has
rectangle has
a
a length
length that twice its
that is twice width. If
its width. If the area of the
the area the
its grand
A bakery
bakery gave out coupons
gave out coupons to celebrate
celebrate its grand
board is 128 square
board square feet, what is the
feet, what the length,
length, in
opening.
opening. Each coupon was
Each coupon worth either
was worth either $1, $3,
feet, of the
feet, the board?
board?
or $5. Twice
Twice as many $1coupons
as many were given
$1 coupons were out
given out
as$3
as $3 coupons, and 3 times as many $3 coupons
coupons, and 3 times as many $3 coupons
were given
were u t as$5
given oout as $5 coupons.
coupons. The total value
The total value of
all the coupons
all the given out
coupons given many
H o w many
was $360. How
out was
$3
$3 coupons were given
coupons were out?
given out?
A
A)) 440
0
B) 45
45
C)
C) 48
48
D
D)) 554
4
Alex, Bob, and
Alex, and Carl all collect
Carl all collect seashells.
seas hells . Bob has
has
half as
half as many seashells as
many seashells Carl. Alex
as Carl. Alex ha three
hass three
times as
times many seashells
as many seashells as
as Bob. If
If Alex and
and Bob
together
together have seashells, how
have 60 seashells, many seashells
how many seashells
does
does Carl
Carl have?
have? A water tank is connected to two pipes, Pipe A
and Pipe B. It takes 4 hours to fill the tank when
A
A)) 115
5
only Pipe A is in use, and it takes 6 hours to fill
B) 20
20 the tank when only Pipe Bis in use . If it takes 111
C
C)) 330
0 minutes to fill the tank when both Pipe A and
D
D)) 4400 Pipe Bare in use, what is the value of m ?

107
CHAPTER
CHAPTER 12 WORD
WORD PROBLEMS
PROBLEMS

Yoona runs steady rate


runs at a steady yard per
rate of 1 yard second..
per second Terry is hired
hired to pave
pave aa parking
parking lot and finishes
lot and finishes
Jessica runs
runs 4 times
times as
as fast. IfIf Jessica
Jessica gives
gives Yoona 1
a head start of 30 yards
head start how many
race, how
yards in a race, many 3 of the
the parking
parking lot before Andy
lot before Andy is hired to
is hired to work
work
3
yards must Jessica
yards must Jessica rrun catch up to Yoona?
u n to catch alongside him . They
alongside him. work at
each work
They each at a
a constant
constant
rate, but Terry works
rate , but as Andy
as fast as
works twice as Andy does.
does.
The equation
equation 9 9 (ix
( +
1 + i)1 )
2
2 can be used to
x = 2 can
3 be used to
the total
find the number of days
total number days x it would
would have
have
taken Terry to pave
taken pave the entire parking
the entire by
lot by
parking lot
himself. Which of the
himself. Which the following
following is the
the best
best
interpretation
interpretation of the number 9 in the
the number the equation?
equation?
A) The number of days
The number days it would have taken
would have taken
Terry and and Andy
Andy to pave
pave the
the entire parking
entire parking
Nicky owns a house that has a patio in the shape lot ifif they
they had worked together
had worked from the
together from the
of a square . She decides to renovate the patio by start.
start.
increasing its length by 4 feet and decreasing its number of days
B) The number will take
days it will take Terry and
and
width by 5 feet. If the area of the renovated patio Andy
Andy to pave the remainder
pave the remainder of thethe parking
parking
is 90 square feet, what was the original area of lot working
working together.
together .
the patio, in square feet? number of days
The number
C) The days it would take Andy
would take Andy to
pave
pave the the remainder the parking
remainder of the parking lot
lot if
if hehe
were working
were working alone.
alone .
D) The number of days
The number days it would
would take Terry to
take Terry
pave
pave the the remainder the parking
remainder of the lot ifif he
parking lot he
were working
were working alone.
alone.

108
Minimum
Minim Maximum
um & Maxim
Word Proble
Word
um
Problems
ms
13
Minimum and
Minimum and maximum word problems
maximum word require aa bit
problems require bit of and an
of logic and an understanding
understanding of and inequalities
rates and
of rates inequalities
(chapters
(chap ters 4 and One of the most
and 11). One common issues
most common students
issues studen have is that they’re
ts have is that they 're unsure of whether to
unsure of whether round
to round
examples in this
down. The examples
up or down. chapter will address
this chapter address this issue and illustrate
issue and illustrate the strategies you’ll need to
strategies you' ll need to
solve types of problems.
solve these types problems.

Corinne is aa graphic
EXAMPLE 1: Corinne
EXAMPLE designer who
graphic designer earns $275 for
who earns for every logo she
every logo designs.. What
she designs is the
What is the
minimum number
minimum logos she would
number of logos would have design to earn
have to design earn at least $4,000
at least $4,000 ?

0
earn at least
To earn
To least $4,000, would have
Corinne would
$4,000, Corinne have to design least 4,
design at least T
\~ x 14.5 _
logos. That's
14.3 logos.
:::::: 14 logos
That’s 14 logos and half aa
and half
because
logo. But becau implied that
it’s implied
se it's logo cannot
that a fraction of a logo be designed
cannot be designed and sold, we have
and sold, have to round up to
round up to
ii logos.
OI) logos.
When a whole
When whole number
number answer is implied,
answer is implied, the m i n i m u m generally
the minimum generally requires we round
that we
requires that up.
round up.

EXAMPLE pallet truck


EXAMPLE 2: A pallet truck can
can move up to 3 tons
move up single trip.
tons in a single trip. If the truck
If the truck is
is to used to
be used
to be to move
move
320-pound pallets, what
320-pound pallets, what is the number of whole
maximum number
the maximum pallets the
whole pallets truck can
the truck can move
move in single trip?
in aa single trip?
ton = 2,000 pounds)
(1 ton pounds)

A) 6
A)6 B) 18 C) 19 D) 106

6, 000
6,000
Since 3 tons is equivalent
equivalent to 3 x 2, 000 =: 6,000 pounds,
2,000 pounds, the truck can move
move : 18.75 pallets.
= pallets. However,
However,
320
the question specifically states
question specifically whole pallets,
states whole sowe
pallets, so have to
we have to round
round down to DI]
down to E pallets
pallets.. If
If we
we rounded up, the
rounded up, the
weight would be
weight would above what
be above truck can handle.
what the truck handle.
When a whole
When number answer
w h o l e number implied, the maximum
a n s w e r is implied, maximum generally requires that we
generally requires we round
round down.
down.

109
CHAPTER 13 MINIMUM
CHAPTER WORD PROBLEMS
MAXIMUM WORD
MINIMUM & MAXIMUM PROBLEMS

EXAMPLE 3: H If one tray of flatbread


one tray can be
flatbread can made from
be made from 8 of flour
cups of
8 cups 6 cups
and 6
flour and cups of greek yogurt,
of greek what
yogurt, what
is‘the
is number of whole
maximum number
the maximum whole trays flatbread that
trays of flatbread can be
that can from 150
made from
be made 150 cups of flour
cups of flour and 100 cups
and 100 cups
of greek
gteEtkyogurt?
yogurt?

In these
In types of questions,
these types one of the
questions , one resources (either
the resources flour or greek
(either flour yogu rt) will
greek yogurt) be used
will be before the
up before
used up other,
the other,
and that
and that resource
resource will limit the amount that can be produced. the best way to approach
will limit the amount that can be produced. Therefore, the best way to approach these
Therefore, these
questions is to consider
questions consider each separately.
resource separately.
each resource
10
we only
IfIf we only consider
consider the flour requirement,
the flour l~O = 18.75 trays
requirement, % flatbread can
tra ys of flatbread If we
made . If
be made.
can be only consider
we only the
consider the
100
greek yogurt requirement,
greek yogurt l~O ~
requirement, ‐6‐ 2 16.7 trays flatbread can
trays of flatbread be made.
can be the amount
Since the
made . Since that can
amount that be produced
can be produced
from the
from greek yogurt
the greek yogurt is less
less than
than the
the amount beproduced
can be
that can
amount that produced from the flour,
from the the greek
flour, the a limiting
is a
yogurt is
greek yogurt limiting
factor.
factor . There isn't enough
There isn’t enough of it to use up the
use up the flour, so we're
flour, so we’re limited of flatbread . As a result,
trays of flatbread. As a result, the
limited to 16.7 trays the
maximum whole number
maximum whole number of trays
trays that can be
that can made is [!I].
be made -. Remember that
Remember round down
we round
that we down when finding the
when finding the
maximum.
maximum.

EXAMPLE4:
EXAMPLE 4:
C= 18tw +
= 18tw + 1,050
1,050
An appliance manufacturer uses
appliance manufacturer uses the equation above
the equation calculate the
above to calculate total cost
the total cost C, C, in dollars, of
in dollars, producing
of producing
a shipment
a shipment of t toasters weigh w
each weigh
that ea-ch
toasters that pounds. If
to pounds. If the manufacturer can
the manufacturer spend no
can spend no more than $21,000
more than $21,(X)0
producing the
producing shipment of toasters,
next shipment
the next the weight
and the
toasters, and weight of each toaster w
each toaster will be 6
i l l be what is
pounds, what
6 pounds, the
is the
maximum number toasters that can be produced
maximum number of toasters that can be produced for the next shipment?
the next shipment?

Let's solve
Let’s this question
solve this by setting
question by setting up an inequality:
up an inequality:
c ~
C 5 21,000
21,000
18110 +
lBtw 1,050 5
+ 1,050 ::S:21,000
21,000
18t(6) ::S:
18t(6) 5 19,950
19,950
10815
108t 19,950
::S:19,950
t5 184.72
::S:184.72

it's implied
Since it’s
Since are produced
toasters are
that toasters
implied that numbers , the
whole numbers,
produced in whole maximum number
the maximum that can
number that can be produced
be produced

for the next shipment


the next I184 1.
shipment is -.

110
THE COLLEGE
THE PANDA
COLLEGE PANDA

EXAMPLE 5:
EXAMPLE 5: A deck of48cards
Adeck consists of
of 48 cards consists only red cards andblackcards.
ofonlynedcards Ifthenumberofredcards
and black cards. If the number of red cards
less than
is less than twice
twice the number of black
the number black cards, what is the
cards, what the minimum possible number
minimum possible number ofof black
black cards in the
cards in the
deck?
deck?

Let r be
Let be the number of red
the number red cards and b be
cards and the number
be the number of black cards. Using
black cards. Using these variables, we
these variables, set up
can set
we can up a
a
system that
system consists of an
that consists an equation an inequality.
and an
equation and inequality.

r+b=
: 448
8
r <<22b
b

Since the
Since the question asking about
question is asking about the black cards,
the black o u r goal
cards, our should be
goal should be to i d of
get rrid
to get of rr so that we
so that end up
we end with an
up with an
inequality in terms
inequality terms of bonly.
b only. To do we isolate
so, we
do so, isolate rr in
in the equation to
the equation get rr = 48‐
to get 48 - b.b. N o w we
Now we substitute 48‐- bb
substitute 48
for r in the
the inequality:
inequality :

4 8-‐ bb <<22b
48 b
488 < 33b
4 b
l166 < b

Based on
Based on this resulting inequality,
this resulting inequality, the minimum possible
the minimum number of
possible number black cards
of black is [ill.
cards is-.

EXAMPLE 6: An art
EXAMPLE teacher needs
art teacher buy aa total
needs to buy of 36paintbrushes
total of 36 paintbrushes for a
a painting
painting class. Each paintbrush
class. Each paintbrush
be either
must be
must an acrylic
either an acrylic brush, which costs
brush, which costs $5, or a watercolor brush, which costs
watercolor brush, which costs $3. If no
no more than
more than
$150 can be spent on
can be spent on the paintbrushes, what
the paintbrushes, what is the
the minimum number of
minimum number of watercolor brushes the
watercolor brushes art teacher
the art teacher
can buy?
can buy?

Let a be the number


be the acrylic brushes
number of acrylic and w be
brushes and the number
be the number of watercolor brushes.. Now
watercolor brushes Now we
we can
can set up aa
set up
system of an
system an equation and an
equation and inequality just aswe did in Example 5.
an inequality just as we did in Example 5.

a+w
a+ w ==336
6
5 a+
5a + 33w
w §:S1150
50

Our goal is to
Our goal to get inequality in terms
the inequality
get the w only,
terms of w so let’s
only, so let's first isolate a in
isolate a in the to get
equation to
the equation get a
a=z 36
36‐- w.
w. Now
Now
we can
we substitute for a
can substitute a in the
the inequality:
inequality :

5(36 w) +
5(36 -‐ w) + 3w $ 1150
3w :S 50
180 -‐ 510
5w + 310
3w 5150
:S 150
180 ‐- 2w 5
:S 150
‐2w 5 ‐30
- 2w :S - 30
w 215
w ~ 15

Based on
Based on this resulting inequality,
this resulting inequality, the
the minimum number of watercolor
m i n i m u m number watercolor brushes
brushes that can be
that can be bought is lli].
bought is-.

111
111
CHAPTER 13 MINIMUM
CI-LAPTER & MAXIMUM
MINIMUM & WORD PROBLEMS
MAXIMUM WORD PROBLEMS

EXAMPLE 7: Shahar
Shahar collects baseball
baseball cards
cards that sold in
are sold
that are regular packs
in regular premium packs.
and premium
packs and Two rare
packs. Two rare
cards can be found inevery
cardscanbefound in every regular pack and three rare cards can be found in every premium pack. If
regularpackand threerarecardscanbeformd ineverypremium pack. If
Shahar
Shahar wants add at least
wants to add least 30 rare cards to
30 rare cards his collection
to his by buying
collection by buying no
no more 12 packs
than 12
more than of baseball
packs of baseball
cards,
cards, what the least
what is the number of
least number packs he
premium packs
of premium could buy?
he could buy?

Again, let's set up


Again, let’s up a
a system with r as
system with as the number of
the number of regular
regular packs and pas
packs and pas the number of
the number premium packs.
of premium packs .

+ pp §:S1122
r+
2rr + 3
2 3pp 2
~ 330
0
Since we have a
we have system of ttwo
a system w o inequalities,we just do
can't just
inequalities, we can’t what we
do what did in
we did Examples 5
in Examples and 6.
5 and Instead, we
6. Instead, we need
need
on the following
to rely on following trick: inequalities
inequalities can be together if
added together
be added their signs
if their point in
signs point in the same direction.
the same direction.
Note that
Note that inequalities
inequalities should never be
should never be subtracted
subtracted from another; only
one another;
from one think in
only think terms of
in terms addition. Soto
of addition. So to get
get
signs to point in the
the signs the same direction, we can multiply the first inequality
same direction, inequality by 2 . This will
by - 2. This will switch the
‐ switch the sign
sign
and get
and get the
the coefficients match up.
coefficients of r to match up .

-‐ 22rr -‐ 22p
p2~ ~24
- 24
2r+3p
2r + 3p 2~ 30
30

Now we
Now add the inequalities
can add
we can together to
inequalities together to get

p ~> 6
p 6

Based on this result,


Based minimum possible
result, the minimum value of
possible value of pis [ill.
p is [E].
Another valid
Another valid way
way to approach problem is guess
approach this problem guess and check . For example,
and check. example, we with p
start with
we can start p ==0 0 and
and
rr = Given those
z 12. Given values, is 2r +
those values, + 3p at least not, repeat
least 30? If not, repeat the process with p z 1 and r = 11, and
process with p = 1 and r = 11, and etc. etc.
Soon enough,
Soon enough, you’ll arrive at p =
you'll arrive z 6. Guess
Guess and
and check turns be quite
u t to be
turns oout quite efficient in
in many so don’t
cases, so
many cases, don't
give up
up on
on it too early.

112
THE
THE COLLEGE PANDA
COLLEGE PANDA

CHAPTER EXERCISE:Answers for this chapter start on page 304.

A calculator is allowed following


allowed on the following
questions.
questions.
A gift shop
shop held
held aa weekend
weekend salesale with
with the goal of
the goal
selling
selling at
at least worth of greeting
least $8,000 worth greeting cards
cards
and gift boxes.
and gift Each greeting
boxes . Each was sold
card was
greeting card sold for
Katherine has
Katherine 28classroom
has 28 calculators that
classroom calculators that each
each $5, and
and each gift box
each gift box was sold for $7. If no more
was sold more
require
require aa set
set of 44 batteries.
batteries. If her school
If her supplies
school supplies than 400 gift
than boxes were
gift boxes were sold during the
sold during sale due
the sale due
her with batteries
her with packs of 6, what
batteries in packs what is the least
the least to limited
limited inventory, what is
inventory, what is the minimum
the minimum
number
number of packs needed to
packs needed to provide every
provide every number of greeting
number greeting cards shop could
the shop
cards the could have
have
calculator with
classroom calculator
classroom complete set
with aa complete set of sold to meet
sold meet itsits goal?
goal?
batteries?
batteries?
A) 1,040
B) 1,160
C) 1,280
D) 1,400

To restock
restock supplies,
supplies, a nail salon
a nail purchases
salon purchases
Martha is
Martha is working
working on design project
on aa design project that
that toolkits
toolkits that
that each include 80
each include nail files and
80 nail and 150
requires 16
requires 16ounces glue. The
ounces of glue. glue gun
The glue gun she is
she is nail
nail buffers. the nail
buffers. If the salon needs
nail salon needs to restock at
restock at
using comes preloaded with a glue stick that
using comes preloaded with a glue stick that least nail files
least 1,800 nail and at
files and least 4,000 nail
at least nail
provides
provides 2.5 ounces glue. The
ounces of glue. only additional
The only additional buffers, the minimum
what is the
buffers, what number of toolkits
minimum number toolkits
glue sticks Martha
glue sticks c a n purchase
Martha can purchase are that
ones that
are ones the salon
the can purchase?
salon can purchase?
provide 1.75 ounces
each provide
each glue.. Assuming
ounces of glue Assuming thatthat
glue sticks
the glue
the sticks can only be
can only be purchased
purchased in whole
whole
numbers, what
numbers, what is the
the minimum
minimum number glue
number of glue
sticks Martha
sticks Martha must purchase for
must purchase for her project?
her project?
A ) 6
A)
B)7
B)
C)8
C)
D
D)) 9
One liter
One equivalent to approximately
liter is equivalent approximately 33.8
ounces. Mark
ounces. Mark hashas plastic
plastic cups
cups that each hold
can each
that can hold
12 ounces of liquid
12 ounces liquid.. At most, many of these
how many
most, how these
plastic could aa ttwo
cups could
plastic Cups w o liter bottle of soda
liter bottle soda fill?
A)5
A)
B
B)) 6
C
C) ) 7
D
D) ) 8

113
CHAPTER 13 MINIMUM & MAXIMUM WORD PROBLEMS

In one hour, Jason can install at least 6 windows During


During a week-long fishing
a weekolong fishing trip, Ashleigh
trip, Ashleigh
but no more than 8 windows. Which of the caught nine less
caught nine less than
than three times the
three times number of
the number
following could
following bea
could be possible amount
a possible time, in
amount of time, Naomi caught.
fish Naomi caught. If they caught at
they caught at least
least 45
45 fish
hours,
hours, that to install
takes to
Jason takes
that Jason install 100 windows
windows in combined, what is the
combined, what minimum number
the minimum number of fish
fish
a home?
home? that Naomi could
that Naomi have caught?
could have caught?
A) 12
A) 12
B) 16
B) 16
C) 17
C) 17
D) 18
18

IN I’ ' ' F V s

1 fluid ounce =
fluid ounce = 29.6 milliliters
milliliters
A jar is filled
filled with pebbles, white
black pebbles,
with black white pebbles,
pebbles,
cup =
1 cup fluid ounces
= 16 fluid ounces and jade pebbles.
and jade The number
pebbles. The jade pebbles
number of jade pebbles is
greater than half
greater than half the number of black
the number pebbles,
black pebbles,
chemistry teacher
A chemistry planning to
teacher is planning to nm
r u n aa class
class
and
and the number of white
the number white pebbles
pebbles is less
less than
than
experiment in which
experiment which each student must
each student must measure
measure
twice the
twice number of black
the number black pebbles. If there
pebbles . If there are
are
out 100 milliliters
out 100 vinegar in a
milliliters of vinegar graduated
a graduated
32
32 jade pebbles in the
jade pebbles what is the
the jar, what maximum
the maximum
cylinder. The
cylinder. The class is ljmited
class is to using
limited to using 66 cups
cups of
number of white
white pebbles
number pebbles that
that could be in the
could be the jar?
jar?
vinegar. Given the
vinegar. Given information above,
the information what is
above, what
the maximum number
the maximum number of students
students who
who willwill be
be
able to participate
able this experiment?
participate in this experiment?

Giovanni works as
Giovanni works as aa waiter
waiter atat an Italian
an Italian
restaurant. For every table that he serves,
restaurant. For every table that he he
serves, he
earns tip on
15% tip
earns a 15% on the bill. During
the bill. During lunch, he
lunch, he
served 12 tables
served and each
tables and each table an average
had an
table had average
bill of $25. If each
bill of table during
each table dinner will
during dinner will have
have
an average
an average bill what is the
bill of $45, what the least number
least number
tables Giovanni
of tables Giovanni must serve during
must serve dinner to
during dinner
at least
earn at
earn least $180 for the day?
the day?
A) 3
A)
B
B)) 116
6
C
C)) 118
8
D ) 2200
D)

114
THE COLLEGE
THE COLLEGE PANDA
PANDA

pharmacy produces
A pharmacy produces a certain medication
a certain medication in aa part of aa marketing
As part campaign, aa restaurant
marketing campaign, restaurant is
daytime variety
daytime variety and
and aa nighttime variety.. A bottle
nighttime variety bottle offering
offering 44 free
free tacos burrito a
every burrito
tacos for every a customer
customer
of the daytime variety
the daytime variety contains
contains 2 ow1ces the
ounces of the buys. If the
buys. If the restaurant
restaurant would normally sell
would normally sell the
the
ingredient and
active ingredient
active and 66 ounces flavored syrup
ounces of flavored syrup.. tacos for $2.60 each,
tacos each, what is the
what is the minimum
minimum
bottle of the
A bottle nighttime variety
the nighttime contains 33
variety contains number
number of burritos customer would
burritos aa customer would have to
have to
ounces of the
ounces active ingredient
the active and 55 ounces
ingredient and ounces of buy
buy to receive at least
receive at least $140 worth tacos for
worth of tacos
flavored syrup. The
flavored syrup. The pharmacy currently has
pharmacy currently no
has no free?
more than
more ounces of the
than 385 ounces ingredient
active ingredient
the active
and no
and no more than 850 ounces
more than ounces of flavored syrup
flavored syrup
available.. If at
available least 65
at least bottles of the
65 bottles daytime
the daytime
variety must
variety must bebe filled, what is the
filled, what the maximum
maximum
number bottles the nighttime variety
number of bottles of the nighttime variety that
that
can be filled?
can be filled?
A ) 778
A) 8
B) 85
C) 92
C) 92
Ava is decorating two-tier and
decorating two-tier three-tier
and three-tier
D) 106 wedding cakes. takes her 20 minutes to
wedding cakes. It takes her 20 minutes to
each two-tier
decorate each
decorate wedding cake
two-tier wedding cake and
and 3535
minutes decorate each
minutes to decorate wedding
three‐tier wedding
each three-tier
cake . If
cake, needs to decorate
If Ava needs least 14
decorate at least
wedding cakes today,
wedding cakes and she
today, and she can spend no
can spend no
banquet hall
A banquet hall has
has aa maximum seating capacity
maximum seating capacity more than
more hours doing
than 6 hours doing so, what is the
so, what the
people.. For
of 168 people For aaparticular
particular event,
event, the banquet
the banquet maximum three-tier wedding
number of three-tier
maximum number wedding cakescakes
manager must
manager m u s t use an arrangement
use an arrangement of shortshort she
she can decorate today?
can decorate today?
tables
tables and long tables
and long ensure that
tables to ensure that there
there is
enough seating
enough seating to meet that capacity.
meet that Each short
capacity. Each short A
A)) 4
table
table seats
seats 4 4 people
people andand each long table
each long table seats 8
seats 8 B)5
B)
people. If no more than 32 tables can be placed
people . lf no more than 32 tables can be placed C
C) ) 6
inside the banquet
inside the what is the
hall, what
banquet hall, the maximum
maximum
D
D) ) 8
number of short
number short tables can be
that can
tables that used?
be used?
A ) 110
A) 0
B)
B) 14
14
C) 18
C) 18
D)) 2222
D

115
CHAPTER 13 MINIMUM & MAXIMUM WORD PROBLEMS

Lianne wants to make a seasoning that consists C 10On 0/


= 100n %
of 75% sea salt and 25% black pepper . If sea salt _n+w
n+w
0

costs $2 per pound and black pepper costs $8 per The formula
formula above can be
above can used to determine
be used determine the
the
pound, and Lianne can spend no more than $210 volume
volume percent concentration
percent concentration C of an ethanol
an ethanol
on these ingredients, what is the maximum solution containing
solution containing 11 ounces of ethanol
n ounces ethanol and
and w w
number of pounds of seasoning that she wiU be ounces of water.
ounces water . A chemist use the
wants to use
chemist wants the
able to make? formula
formula to create
create an solution with
ethanol solution
an ethanol with aa
A
A)) 442
2 volume
volume percent
percent concentration
concentration of no more than
more than
B)
B) 50
50 16%. If
If the chemist will mix 10
chemist will 10 ounces ethanol
ounces of ethanol
and x cups
and cups of water
water to create the desired
create the desired
C
C)) 556
6
solution, what
solution, minimum possible
the minimum
what is the value of
possible value
D
D)) 6600 x, assuming that x is aa whole
assuming that whole number?
number?
cup =
(1 cup = 8 ounces)
ounces)

A toy company ships its


company ships products in small,
its products small,
medium, and large boxes. Last month,
medium , and large boxes . Last month, the
the
company shipped a
company shipped total of 250 boxes,
a total boxes, of which
which
70 were medium
70 were boxes. The
medium boxes. number of large
The number large
boxes shipped
boxes shipped was more than
was more than the sum of the
the sum the
number of small
number boxes shipped
small boxes shjpped and the number
and the number
of medium
medium boxes shipped. What
boxes srupped. greatest
the greatest
What is the
possible number of small
possible number small boxes the company
boxes the company
shipped last
shipped last month?
month?

t
.
i
116
Lines
Lines are
Lines just functions
are just the form
functions in the f(x) =
form of J(x) : mx ++ b, which why they
which is why they are often referred
are often referred to as
as linear functions .
linear functions.
cover functions
We'll cover functions asas a whole in aa future
a whole chapter; we’re
future chapter; we're covering lines
covering Lines first because they present some
because they present some
concepts
concepts that don’t apply
that don't other functions.
apply to other these concepts
tests these
The SAT tests
functions . The so frequently
concepts so they deserve
that they
frequently that deserve their
their
o chapter. Let's
w n chapter.
own dive in!
Let’s dive
Given any
Given w o points
any ttwo points (x1,_1/1)
(x1,yi) and (Md/2)) on aa line,
and (x2,Y2 line,

, rise
rise 1/2 2 -~ y1
Slope
SIope of Line =: -4 =z -y
o f line · -- “ll‐1
rrun
un .\'
X2 -~ x1
3 XJ

The slope
The slope is a measure of the
a measure steepness of aa Line-the
the steepness bigger the
line‐the bigger the slope,
slope, the
the m steep the line
o r e steep
more rise is the
The rise
line is. The the
distance between the
distance between they.'1 coordinates
coordinates and the distance between the coordinates. slope 2 means
u n is the
the rrun
and the distance between the x coordinates. A slope of means

goes 2 uunits
the line goes up for every
n i t s up every 11 unit to the
u n i t to right, or 2 uunits
the right, down
nits d n i t to
every 1 uunit
o w n for every to the lleft. slope of -‐ ~5
e f t , A slope

means the
means goes 2 units
line goes
the Line units down every 3 units
down for every to the right,
units to right, or
or 2 units
units up
up for every
every 3 units
units to the left.
to the left.
Lines with
Lines positive slope
with positive slope always
always go up and
go up and to the right as
the right graph above.
the graph
as in the above.

yV

rise

run

Lines with
Lines negative slope
with negative slope go and to the
down and
go down right::
the right
y

117
CHAPTER 14
CHAPTER 14 LINES
LINES

EXAMPLE1:
EXAMPL£1~
y

(a.b)

shown in the
line shown
The line the xy-plane
xy-plane above passes through
above passes the origin
through the point (a,
and point
origin and (a,b), where a > b. Which
b), where Which of
the following could
thelollowing bethe
could be slope of the
the slope line?
the line?

A)‐-%
A) --
1
2
B);~4
B) C ) 11
C) D);
D)~
2

..
First,
F1rst, .. tthat
notice
notice h at tthe .. positive.
lope 1s
h esslope lS .. .. Th
posnhve. Theesslope,
rise ·_ 1
Iope, -rise
ru_', 1s so equal
15aalso
b
equa I to E'
-b .
runI] a
y

Since b, ~g is always
Since a > b, less than
always less 1. For
than 1. example, if
For example, if aa =z 5 and
and b = 3, the slope
3, the slope would be ~-
would be 2 The
The only choice
only choice
a 5
that’s both
that's positive
both po sitive and than 11 is
less than
and less is answer
answer (B) .[NJ.
EXAMPLE 2:
EXAMPLE 2: Line
Line m passes through
m passes through points and (3,1:
points (k, 7) and 4). If
(3, k -‐ 4). the slope
If the line m
slope of line m is 3, what
what is the
the
value of k?
value k?

( k- 4)
e(k
S l o =p ----
Slope z3 ‐-43k)-‐T7 == 33
1 == 33(3
k -‐ 111 ( 3 -‐ kk))
k - 1111 =
2 99 ‐- 33kk
4 k=
4k : 2200

k:‑

118
118
THE COLLEGE PANDA
THE COLLEGE PANDA

EXAMPLE 3:
EXAMPLE 3:If
If a line has
a line a slope
has a slope of
of~g and through the
passes through
and passes point (1,
the point (1, ‐- 22), which of
) , which the following
of the following points
points
also lies
also lies on
on the line?
the line?

A)
A) ((-2,
4 , --5)
5) B)(-‐2,‐1)
B) (-2,-1) cue‐11)
C) (4, -1) D) (4,10)
D) (4, 10)

A slope i
1
slope of 5 means
means 11up every 33 to
up for every to the right, or
the right, or 11 down every 3
for every
down for 3 to left. If
the left.
to the If we
we go3
go 3 to the left,
to the the point
left, the point

we get to
we get to on the line
on the is ( -‐ 22,, -‐3).
line is If we
3). If we go
go3 to the
3 to right, the
the right, the point we get
point we get to
to on the line
on the is ((4,
line is 4, -~1), [I§J.
answer (C) .
1), answer
In this
this case, we got
case, we got to the answer pretty
the answer quickly, but
pretty quickly, but if we
we hadn’t,
hadn't, we would have
we would moving right
continued moving
have continued or
right or
left until
left until we
we fotmd
found an answer choice
an answer that matched
choice that On the
matched.. On the SAT, it shouldn't ever take
shouldn't ever too long
take too long to at the
arrive at
to arrive the
answer for aa question
answer question like
like this
this..

addition to slope,
In addition you also
slope , you need to know
also need know what and y intercepts
what x and intercepts are. x-intercept is
The x-intercept
are. The is where graph
the graph
where the
crosses the
crosses the x-axis. Likewise, the y-intercept is where
Likewise, they-intercept the graph
where the crosses the
graph crosses y‐axis..
the y-axis
Let's say
Let’s say we the line
have the
we have line
2x + By
2x =
3y = 12
12
To find the x‐intercept, set
the x-intercept, equal to 0.
set y equal

2x + 3(O) =
+ 3(0) = 12
2x =
2x : 12
12
x =
: 6

The x-intercept
The x-intercept is 6.
the y-intercept,
To find the y-intercept, set equal to 0.
set xx equal 0.

2(0) + 3y == 12
12
3y := 12
3]; 12
y =z 4

The y‐intercept is 4.
They-intercept

EXAMPLE 4:
EXAMPLE 4: If the line ax
the line +
ax + 3y = 15, where aa is a
15, where has an
constant, has
a constant, x‐intercept that
an x-intercept is twice
that is twice the value of
the value of
the y-intercept, what is
the y-intercept, what is the value of aa ??
the value

set x =
First, set
First, = 0Oto find the
to find y-intercept:
they-intercept:
a ( 0 )++3y
a(O) 3 y=: 115
5
3
3yy = 115
5
yy =
=5
The y-intercept is 5,
They-intercept 5, which means the
which means x-intercept must
the x-intercept must be
be 5
5xx2
2= 10. Plugging
= 10. Plugging in
in x
x z 10, y
= 10, y= 0,
= 0,

a(10) +
a(lO) +3(0) : 15
3(0) = 15
10a = 15
a =~

119
CHAPTER 14 LINES
CHAPTER 14 LINES

All lines
All can be
lines can expressed in slope-intercept
be expressed slope-intercept form:

y = mx
mx +
+ bb

where
where m m is
is the slope and
the slope and b is the y‐intercept. So
they-intercept. So for
for the line y =
the line = 2x
2x ‐- 3,
3, the slope is 2 and
the slope the y‐intercept
and the y-intercept is
-‐ 33::
y

all lines
While all
While lines can be expressed
can be slope‐intercept form,
expressed in slope-intercept sometimes it’ll
form, sometimes it'll take some work
take some to get
work to there . If you’re
get there. you're
given slope and
given aa slope and aa y-intercept,
y-intercept, then
then of course really
it’s reall
course it's easy to get
y easy get the equation of the
the equation the line.
line . But what if
But what
we’re handed
we're slope and
handed aa slope point instead
and aa point slope and
instead of aa slope and a a y-intercept?
y-intercept? ThenThen it’ll be more
it'll be convenient to use
more convenient use
point-slope form:
point-slope form:
yy ~- yY1i =m m(x
( x-‐ xxi)
i)
(x1,y1)
where (x1,y
where given point.
the given
1 ) is the point. For example, let’s
For example, let's say we want
say we want to find the
the equation
equation of aa line has aa slope
that has
line that slope
and passes
of 3 and through the
passes through the point
point (1, -‐ 22).
) . The equation of the
The equation line is then
the line then

yy -‐ ( (-‐ 22)) =
= 33(x
(x~- 11))

Once it’s in point-slope


Once it's we can
form, we
point -slope form, expand and
then expand
can then shift things
and shift around to get
things around get to slope-intercept if we
form if
slope-intercept form we
need to.
need
= 33(x
y -‐ ((-‐ 22)) = ( x-‐ 1)
l)
= 33xx -‐ 3
y +2 =
=33xx ~- 55
y=

EXAMPLE 5:
EXAMPLES:
y
,
........ ,_
............ 'J
.........
,_
,_
.........
,.,. "' i-....
I"
-14. - SI 2 - 1P
-X
'
Which of the
Which following could
the following be the
could be the equation of the
equation of line shown
the line shown in the xy-plane
in the above?
xy-plane above?

A ) y == ‐- Z2x+3
A)y x+3 B)y
1
B)y=%x+3
= -2 x + 3 C)y=‐%x+3
C)y
1
=--x+ 3 D ) y=
D)y = 2x
2 x-‐ 33
2

To get
To the equation
get the equation of of the line y =
the line mx +
: mx + b, we
we need
need to the slope
to find the slope m and the
m and y-intercept b,
they-intercept The line
b. The line crosses
crosses
the y-axis at 3, sob = 3. The line goes downward from left right, down
the y‐axis at 3,so b : 3. The line goes downward from left to right, down 1 for every 2 to right, so the slope
every to the right , so the
the slope
1
m is -‐ ~.
5. Therefore, equation of the
the equation
Therefore, the line is y =
the line z -‐ ;%x+
x + 3. Answer
Answer ~-(C) .

120
THE COLLEGE
THE PANDA
COLLEGE PANDA

EXAMPLE6: A line
EXAMPLE6: through the points
line l1passes through ( - 22,3)
points (- and (3, 13).
,3) and 1;3),What is the y‐intercept
y~interceptof Imel?
line I ?

yz‐y1 = 1 133-‐ ‐ 33 =: 2
Slope
5 = 1 Y2- Y1 =
=
oPe X2 x 2-‐ xX11 33 ‐- ( -‐ 22)) 2
Using point-slope
Using point form, our
-slope form, line is
o u r line
13 = 2(x -‐ 3)
y -‐ 13
Note that
Note we could've
that we could’ve used point (-
other point
used the other ( ‐ 22,,3).
3). The result
result will
will ttum
u r n out be the
out to be the same.
same.

y -‐ 13
13 =: 2(x
2(x -‐ 3)
3)
=22xx ‐- 66++113
y= 3
y=
: 22x
x++ 7

After
After putting
putting the equation
equation into slope‐intercept form,
into slope-intercept we can
form, we can easily
easily see that
that they -intercept is [z].
the y-intercept ..

There are aa few more


There are things you
more things need to know
you need about lines.
know about lines.

Two lines parallel if they


lines are parallel they have
have the same slope.
same slope.
y

Two lines
Two lines are perpendicular if
are perpendicular if the
the product
product of
of their
their slopes
slopes is In other
is -‐ 11.. In other words,
words, ifif one
one slope
slope is
is the
the negative
negative
1
reciprocal of the
reciprocal the other
other (e.g. and -‐ E).
(e.g. 22 and
2 ).
y

121
CHAPTER 14
CHAPTER LINES
14 LINES

EXAMPLE 7: Line
EXAMPLE m has
Line m has a slope of ~g and
a slope passes through
and passes point (4,
the point
through the 3). If line
(4, 3). 1:is
line n perpendicular to line
is perpendicular line
m and
m through the
passes through
and passes the same
same point (4,3), which
point (4,3), the following
which of the following could
could bebe the
the equation line n ?
equation of linen?

3 3
A)y= -- 32 x + 9
A)y=‐§x+9 B)y=‐§x‐‐3
B)y= - -x
2
- 3 C)y=‐?2‐x+6
C)y =- -x + 6
2
D)y=‐%x+9
D)y
3
=- -x + 9
2

3
it's perpendicular
Because it's
Because m, linen
line m,
perpendicular to line line Hmust
must have slope of -~‐5'
have aa slope ~- Using point-slope form,
Using point-slope form,

3
y‐3z‐g(x‐4)
y - 3 =-
2
(x - 4)

yz‐gx+6+3
y =-
3
2x + 6 + 3
3
yy=‐%x+9
=- x+ 9
2
Weget
We equation into
get the equation slope-intercept form
into slope-intercept form to see that the
see that answer is ~-( D ) .
the answer

Finally,you’ll need to know


Finally, you'U need equations of horizontal
know the equations horizontal and vertical lines.
and vertical equation of the
The equation
lines . The vertical line
the vertical line that
that
passes through
passes 0) is x = 3.
(3,0)
through (3, 3.
y
x=3

+-+--+---+---+--+--+--+--f-1-+ X

equation of the
The equation line that
horizontal line
the horizontal passes through
that passes (0, 3)) is y =
through (0,3 : 3.
3.

yll

y=3

+-+--+---+---+--+---+--+---t---,1--+ X

122
COLLEGE PANDA
THE COLLEGE
THE PANDA

CHAPTER EXERCISE:Answers for this chapter start on page 307.

A calculator
calculator should
should N be used
O T be
NOT on the
used on the
following
following questions.
questions.
In the xy-plane,
ln the xy-plane, points (‐3,5) and
points (-3,5) (6, 8) lie
and (6,8) on
lie on
line 1.
line Which of
I. Which of the following points is also
following points is also onon
lline
i n e ]I??
What is the equation
What equation ofof the line parallel
the line to the
parallel to the
y-axis and
y~axis and 3 units to the
3 units right of
the right of the y-axis?
the y-axis? (0, 6)
A) (0,6)
A)
A)) Xx=
A = -‐ 3 B) (3,8)
B ) x=3
B) x=3 C)
C) (9,10)
(9, 10)
C)) yy =
C = ‐- 3 D) (12,11)
D) (12,11 )
D)) yy== 3
D

y
y
(5, n)

(-1, 1)

+----, 0+-----. X

Figure not
Note: Figure
Note: n o t drawn to scale.
drawn to scale.
- -6
the figure
In the figure above,
above, the slope of
the slope line through
the line
of the through
t w o plotted points is The graph of LineI is shown in the xy-p lan e
the
the two plotted points is~%- What
What is
is the value
the value
above. Which of the following is an equation of
o
off n
n?? a line that is parallel to line I ?
A) 9
B) 4
A)y=‐§x+2
2
A) y =-- x + 2
3
C) 3 2
B)y=§x+10
B) y = x + 10
3
7
D) 3
3 C)yzgx‐4
3
C) y = - x - 4
2
D)) yy =
D =33xx-~ 11

In the xy-p lane, a


xy-plane, line has
a line anx-intercept
has an of -‐22
x-intercept of
and a
and a y-intercept
y-intercept of -‐ 44.. What
What is
is the slope of
the slope of the
the
line?
line?
A)) -‐ 2
A

B) ~§11
B)
2
1
C) -5
C)
2
D) 2

123
CHAPTER 14 LINES

y y

+---+-----+---<>----+----- X
-3 -2 - 1 O 1 2 3 Line I/ in the xy-coordinate
xy-coordinate system
system above be
can be
above can
represented by the
represented the equation mx + b.
equation y = mx b. Which
Which
In the
the xy‐plane
xy-plane above,
above, the graph of the
the graph linear
the linear of the
the following must be
following must be true?
true?
function J
function f is perpendicular
perpendicular to the graph of the
the graph the
linear
linear function (not shown).
function g (not If the
shown). If graphs of
the graphs offf A) mb
A) >0
mb >
B) mb < 00
B) mb
and g intersect
and the point
intersect at the point (1,~),
(1, 2), what
what is the
the
mb =
C) mb = 0
value of gg(‐1)
value (- 1) ?
? mb =
D) mb : 1

The line y = -‐ 22xx -‐ 22 is perpendicular line I. If


perpendicular to line/. If
these two lines have the same y-intercept, which
of the following could be the equation of line / ?
A
A)) yy=z ‐- Z
2xx-‐ z2
allowed on the following
A calculator is allowed following B
B) ) yy== 2x
2 x- ‐ 2
questions.
questions.
1
C)
C) .v yz‐ix‐Z
=-- 2 x - 2
1
y
D)y=%x‐2
D) y = - x - 2
2
m

The slope line/ I is ~%and


slope of line its y‐intercept
and its y-intercept is 3.
What is the equation
What equation of the line perpendicular
the line perpendicular to
line I that
line goes through
that goes 5) ??
(1,5)
through (1,
A
A)) yy =-
= ‐ 22xx + 3
What is the slope
What slope of the
the line m in the
line m figure
the figure B
B) ) yy =-
= ‐ 2x
h+ +7
above?
above?
1 11
11
A)
A) - 2 C)
C) y y:_§x+7
= - 2x + 2
1 1 9
B
B) ) ‐ ‑
2 D)
D) yy‐§x+§
= 2x + 2
1
C) -
C)
4
1
D) 2

124
THE COLLEGE
THE COLLEGE PANDA
PANDA

2
line with
A line slope of g passes
with aa slope through the
passes through the In the xy-plane, the line with equation
3
points (1,4)
points (1, (x, 10).
and (x,
4) and 10). What value of x?
the value
What is the x? -1y1
ax ‐ 531 := 8, where
.
where a 15a constant, passes
is a constant, passes
A) 4
A) through
through the point (2,6).
the point What is the
(2, 6). What the
B) 6 x-coordinate the x-intercept
x-coordinate of the x-intercept of the
the line?
line?
C) 8
C)
D ) 1100
D)

D Average speed,
speed , 5
s Number of
Number
Day
ay per hour)
(miles per hour) calories burned,
burned , c
a
Monday
Monday 7.2 616 y = bx + c
Thursday
Thursday ”‑ 6.8 584
d
Friday
Friday 7.9 672 y = ‐- xX +
+cC
y e
Saturday
Saturday 8.5 720
The equations
The two perpendicular
equations of two lines in the
perpendicular lines the
On certain days of the
certain days the week, Elaine runs
week, Elaine r u n s for an
an
xy-plane are
xy-plane shown above,
are shown where a,b,c,d,
above, where and e
a, b, c, d, and e
hour on
hour on aa treadmill.
treadmill. For each day that
each day that she ran in
she ran
a .
the last week,
the last week, the table above
the table above shows
shows the average
the average are
are constants. If O
constants. If b < 1, which
0< 2 which of the
the
speed sat
speeds which she
at which she ran,
ran, in miles per hour,
miles per and
hour, and following must be
following must true?
be true?
the number of calories
the number calories c she
she burned during the
burned during the
run.. If
run If the
modeled
relationship between
the relationship between cc ands
linear function,
modeled by aa linear
and s can
which of the
function, which the
be
can be
A)‘1<‐1
d
A) - < - 1
e 8
following functions best
following functions best models the
models the d
relationship?
relationship? B)_1<fl<0
B) - 1 < - < 0
e
L’

c(s) =
A) c(s) 30s + 400
z305+400
A)
C)0<§<1d
C) 0 <-<
e
l
B ) c(s)
B) c(s) = 660s
0 $ + 2210
10
d
C) c(s) =
C) c(s) 80s + 40
2805+40 D)5>1
D) - > 1
e
8
D ) c(s) =
D) 2 990s
0 5-‐ 330
0

If m
If and b are
m and real numbers
are real and m
numbers and m > 0 and
and b > 0,
0,
then the line whose equation
then the line whose equation is y = mx +
= mx + b
contain which
cannot contain
cannot the following
which of the following points?
points?
A) (0,1)
A) (0, 1)
B)
B) (1,1)
(1,1)
C
C)) ((-‐ 11,1)
,1)
D)
D) (O,‐1)
(0, - 1)

125
Interpreting Linear
Interpreting Linear
15
Models
Models
On the you will
SAT, you
the SAT, w i l l encounter linear model
encounter linear questions that
model questions that are
are a direct the previous
extension of the
direct extension chapter
previous chapter
about lines. You’ll
about lines. have to interpret
You'll have interpret the meaning of the
the meaning numbers in these
the numbers real world context,
these models within a real world context,
models within a
applying
applying your understanding of slope
your understanding slope and do so.
y‐intercept to do
and y-intercept

EXAMPLE 1: The value


EXAMPLE V, in dollars,
value V, home from 2006 to 2015 can
dollars, of aa home can be esmn~ted by the
be estimated equation
the equation
V== 240, 000 ‐- 5, 000T,
240,000 where Tis
OOOT,where T is the number
number of years
years since 2006.

PART 1: Whichof
PART following best
Which of the following describes the
best describes meaning of the
the meaning the number equation?
the equation?
number 240,000 in the
A) The value
value of the home
home in 2006
B) The value
value of the home
home in 2015
C) The average
C} The value of the
average value the home from 2006 to 2015
home from
D) The increase value of the
the value
increase in the home from 2006 to 2015
the home

following best
PART 2: Which of the following best describes meaning of the number
the meaning
describes the number 5,000 in the equation?
the equation?
A) The number homes sold
number of homes year
each year
sold each
B) yearly decrease
The yearly decrease in the value of the home
the value home
C) The difference between the
difference between value of the
the value home in 2006 and
the home and in 2015
D) The yearly decrease in the
yearly decrease the value the home
value of the home per square foot
per square

Solution: Many
Part 1 Solution: these questions
Many of these que stions w give you
i l l give
will you an equation in y =
an equation mx +
: mx form. The
+ b form. -intercept b will
Theyy-intercept will
typically
typically designate an initial
designate an initial value, when x =
value when
value, the value : O.In y-intercept is 240,000 and
they-intercept
case, the
0. In this case, 240,000 describess
and it describe
the value of the
the value when T =
home when
the home : 0, zero years after
zero years 2006, which,
after 2006, which, of course, Answer ~-(A) .
course, is 2006. Answer

Solution: Again,
Part 2 Solution: Again, we’re dealing with
we're dealing an equation
w i t h an equation of the form y := mx
the form mx ++ b. The slope m
The slope always designates
m always designate s
rate, the
a rate, the increase decrease in y for each
increase or decrease increase in x. In this case,
each increase the slope
case, the slope is - 5, 000, which
‐ 5 , 000, which means the
means the
value of the
value the home decreases by 5,000 for each
home decreases year that
each year goes by. Answer
that goes Answer ~(B) -.

126
THE COLLEGE
THE PANDA
COLLEGE PANDA

important that
It’s important
It's you don
that you don’t get tricked
' t get choosing a
into choosing
tricked into rate that
a rate that looks right but
looks right ultimately
but ultimatel doesn't fit the
y doesn't the
context set by the
context set variables xx and
the variables and y (in this case,
( i n this and V). Answer
case, T and A ) is wrong
Answer ((A) wrong because we’re n
because we're dealing
o t dealing
not
with the
with number of homes
the number homes sold;
sold; we're dealing with
we’re dealing with the value of a
the value a home. Answer (D) is wrong
home. Answer wrong because
because thethe
numbers in the
numbers equation aren
the equation aren’t on aa per-square-foot
't on per-square‐foot basis. Always be
basis. Always aware of the
be aware the variables you’re working
variables you're working
with.
with.

EXAMPLE 2- The
EXAMPLE 2:
where x .is
where
maximum height
The maximum
the amount
is the
plant h, in inches,
height of aa plant
amount of fertilizer,
can bedetermined
inches, can
used to
grams, used
fertilizer, in grams, to grow
be determined by
the plant.
grow the plant .
the equation
by the equation h =
4
x:
“5+ 6
6,,

PART According to
PART 1: According the equation,
to the gram of fertilizer
more gram
one more
equation, one would increase
fertilizer would increase the height of a
maximum height
the maximum a
plant by how
plant many inches?
how many inches?

PART 2: To raise
PART the maximum
raise the maximum height plant by
height of aa plant by exactly
exactly one inch, how
one inch, many more
how many grams of fertilizer
more grams fertilizer
be used
should be
should growing the
used in growing plant?
the plant?

Solution: This question


Part 1 Solution: question is essentially asking for the
essentially asking change in hh for every
the change every 11 unit increase in x. This
unit increase This is
. . 4 .
the slope. From
the slope. From the equation,
equation, we can
can see that the
see that slope is
the slope 5’ or I-.0.8 I. To make
IS ~, even clearer,
make this even can put
clearer, we can the
put the

equation into y =
equation into mx +
: mx + bb form by splitting
form by splitting up
up the fraction: hh =
the fraction: : Ex + 2. Note
~x + t
that when
Note that we’re dealing
when we're with
dealing with
changes and y, the
changes in x and y‐intercept b is irrelevant
they-intercept irrelevant because it’s a
because it's constant that’s
a constant always there.
that's always there.

Part 2 Solution: Because this


Solution: Because question is
this question asking for the
is asking the change
change in x for every
every l1 unit increase in h, the
unit increase reverse of
the reverse
need to rearrange
Part 1, we need
Part equation so
rearrange the equation so that we have
that we have x in terms
terms of h.

4x+6 =2hh
4x + 6
s5
+ 66 ==5Shh
4 x+
4x
4 x=
4x =5Shh-‐ 6
S 3
sz-lh‐§
X= -h- -
4 2

Now
N o w we can
can see that x increases
see that Z, or [ill
increases by ~, when
when h increases
increases by 1. The
The answer
answer is just the slope
just the slope of oour
u r new
new

equation. A shortcut
equation. shortcut for this type of question
this type question is to take the reciprocal
take the reciprocal of the slope of the
the slope original equation.
the original equation. The
The
4 . S
recrproca l of 4
' proca 1
reci 5 is 5.
1s 4.
5 4

EXAMPLEB:
EXAMPLE 3:
T == 665
5 -‐ 66m
m
A can soda is put
can of soda into aa freezer.
put into freezer. The temperature T of the
The temperature degrees Fahrenheit,
soda, in degrees
the soda, Fahrenheit, can be found
can be found byby
using the equation
using the where m is
above, where
equation above, is the
the number
number of minutes
minutes the
the can been in the
has been
can has What is the
freezer. What
the freezer. the
decrease in the
decrease temperature of the
the temperature the soda, degrees Fahrenheit,
soda, in degrees Fahrenheit, for every minutes the
every 5 minutes left in the
can is left
the can the
freezer?
freezer?

slope of -‐66 represents


The slope change in the
the change
represents the temperature for every
the temperature every 1 minute
minute the
the can the freezer. So
can is left in the So
every 5 minutes,
for every temperature of the
the temperature
minutes, the the soda S x 6 = I30 Idegrees
decreasess by 5
soda decrease degrees Fahrenheit.
Fahrenheit.

127
CHAPTER 15
CHAPTER 15 INTERPRETING LINEAR MODELS
INTERPRETLNG LINEAR MODELS

CHAPTER EXERCISE:
CHAPTER Answers for this
EXERCISE: Answers chapter start
this chapter on page
start on page 309.

A calculator should NOT be used on the


following questions.
A membership
membership website video tutorials
offers video
website offers on
tutorials on
programming.
programming. The number of members,
The number members, m,m,
subscribed
subscribed to the can be
site can
the site estimated by the
be estimated the
The water
The water level feet, in aa large
level h, in feet, large aquarium
aquarium equation
equation mm = 500 + 200n, where nn is the
200n, where number
the number
can be modeled by h =
can be modeled by h z 100 T 3d' where d
100 - 3d, where d is
is the
the of videos
videos available
available on
on the
the site. Based on
site . Based on the
the
number of days that have passed
number of days that have passed since the since the equation,
equation, which the following
which of the following statements
statements is
aquarium was last
a q u a r i u m was last refilled Based on
refilled.. Based the model,
on the model, true?
true?
how does the
how does water level
the water change each
level change day?
each day? i
A) For every
A) For one additional
every one additional video,
video, the site
the site
A) Decreases by
A) Decreases 3 feet
by 3 feet gains
gains 500 e w members.
500 nnew members .
B) Increases
B) by 3
Increases by feet
3 feet B) The site initially
The site initially made
made 200 videos available
videos available
Decreases by 100 feet
C) Decreases to members.
members .
D) Increase feet
Increasess by 100 feet C) The was able
site was
The site able to get
get 500 members
members
without any
without any available videos..
available videos
D) The site gains 500 new members for every
‘ ' ' D) T h eadditional
200 S i ’ e g a mvideos
additional videos membe’sm’eve’y
S m Wavailable
available on
on the
the site.
site.

The number
The loaves of bread
number of loaves remaining in aa
bread b remaining
bakery day can
each day
bakery each can be estimated by the
be estimated the
equation b = 200 -‐ 18h,
equation 18h, where the number
where h is the number
hours that
of hours have passed
that have since the
pass ed since store’s
the store's s= 10 -‐ 2h
5 = 10 211
opening.. What
opening is the
What is meaning of the
the meaning the value 18in
value 18 in A recipe
recipe suggests honey tea with
sweetening honey
suggests sweetening with
equation?
this equation?
this sugar. The equation
sugar. The equation above can be
above can be used to
used to
bakery sells all
The bakery
A) The all its loaves of bread
its loaves 18
bread in 18 determine
determine the
the amount sugar s,
amount of sugar in teaspoon
5,in teaspoons, s,
hours.
hour s. that should be
that should added to a
be added beverage with
a tea beverage with h Ii
B) bakery sells
The bakery
B) The 18 loave
sells 18 bread each
of bread
loavess of each teaspoons Of
teaspoons of honey.
honey. What
What is the meaning
is the meaning ofof the
the 22
hour.
hour. in the equation?
in the equation?
C) The bakery sells
The bakery total of 18
sells aa total loave s of bread
18 loaves bread A)
A) For every teaspoon
For every teaspoon of
of honey
honey in
in the
the
each day..
each day beverage, two
beverage, more teaspoons
two more teaspoons ofof sugar
sugar
should be added .
There are
D) There 18 loave
are 18 bread left
loavess of bread left in the
the ShOUId be added.
bakery at the
bakery at end of each
the end day..
each day B) For
B) every teaspoon
For every teaspoon of
of honey
honey in
in the
the
beverage, two fewer
beverage, two teaspoons of sugar
fewer teaspoons sugar
should
should be added.
be added.
For every
C) For teaspoons of honey
two teaspoons
every two honey in the
the
beverage,
beverage, one
one more teaspoon of sugar
more teaspoon sugar
should be added.
should be added.
D) For
For every
every ttwo teaspoons of honey
w o teaspoons honey in the
the
beverage, teaspoon of sugar
fewer teaspoon
one fewer
beverage, one sugar
should
should be added.
be added.

128
THE COLLEGE PANDA
THE COLLEGE PANDA

The monthly salary of a salesperson at a used car h ==1100


0 0 -~ 44tt
dealership is determined by the expression The equation
The equation above
above can can be used to model
be used model the
the
1,000 + 2, 0OOxc, where x is the salesperson's number hours h until
number of hours a gallon
until a gallon of milk
milk held
held at
at
commission rate and c is the number of cars sold a
a temperature
temperature ofoft,t, in degrees Celsius, goes
degrees Celsius, goes sour.
sour.
by the salesperson. Which of the following Based on
Based on the which of the
model, which
the model, the following
following is
statements is the best interpretation of the best interpretation
the best
the interpretation of the number 4 in the
the number the
number 2,000 in the context of this problem? equation?
equation?
A) The average price
The average price of a
a used at the
car at
used car the °C will
increase of 11°C
A) An increase make a gallon
will make gallon of
dealership
dealership milk
milk go sour 4 hours
go sour hours faster.
faster.
B) The base
The monthly salary
base monthly salary of a
a salesperson
salesperson at
at B) An increase
increase of 1°C will
1°C make 4 gallons
will make gallons of
the dealership
the dealership milk sour 1 hour
milk go sour hour faster.
faster .
C) The average monthly
The average commission earned
monthly commission eamed increase of 4°C will
C) An increase make a gallon
will make gallon of
by each
each salesperson the dealership
salesperson at the dealership milk
milk go sour 1 hour
go sour hour faster.
faster.
D) The average
The average number cars sold
number of cars the
sold by the 4° C will
increase of 4°C
D) An increase make a gallon
will make gallon of
dealership each month
dealership each month milk
milk go sour 4 hours
go sour hours faster.
faster.

p = 2, OO0s+ 15, 000


p=2,0005+15,000 An antique
antique lamp sold at
was sold
lamp was auction.. The
an auction
at an The
state government
A state the equation
uses the
government uses above to
equation above price the lamp,
price p of the dollars, during
lamp, in dollars, during the
the
estimate the
estimate average population
the average population p for a town
a town auction can be
auction can modeled by
be modeled the equation
by the equation
withs 5 schools.
with Which of the
schools. Which the following best
following best p = 900 ‐- 10t, where t is the
lOt, where number of seconds
the number seconds
the meaning
describes the
describes the number
meaning of the number 2,000 in the
the left the auction. According to the model, what
left in the auction. According to the model, what
equafion?
equation? the meaning
is the meaning of the the equation?
the 900 in the equation?

The average
A) The average number
number of students each
at each
students at A) The starting
A) The auction price
starting auction the lamp
price of the lamp
school
school in a
a ttown
own B) The final auction price the lamp
B) The final auction price of the lamp
B) The
The average number of schools
average number schools in each
each C) The
C) the price
increa se in the
The increase price of the lamp per
the lamp per
ttown
own second
second
C) The estimated
C) The increase in a
estimated increase a town’s
town 's D) The time it took
The time the lamp,
auction off the
took to auction lamp, in
population
population for each additional school
each additional school seconds
seconds
D) estimated population
The estimated
D) The population of a
a town
town
without any schools
without any schools

= l.30x
y= 1.30x‐- 1.50
A bank
bank teller the equation
uses the
teller uses equation above
above to
exchange
exchange US U.S. dollars into euros,
dollars into where y is the
euros, where the
amount and
euro amount
euro and x is the U.S. dollar
the US. amount.
dollar amount.
Which of the
Which following is the
the following best interpretation
the best interpretation
of the
the 1.50 in the
the equation?
equation?
A
A) The bank charges
The bank euros to do
charges 1.50 euros do the
the
currency exchange.
currency exchange.
B) bank charges
The bank
B) The charges 1.50 U
U.S. dollars to do
S . dollars the
do the
currency exchange.
currency exchange.
C) One
C) U.S. dollar
One US. dollar is worth euros.
worth 1.50 euros.
D) One
D) One euro is worth
euro is U S . dollars.
worth 1.50 U.S. dollars.

129
CHAPTER 15 INTERPRETING
CHAPTER 15 INTERPRETING LINEAR MODELS
LINEAR MODELS

calculator is allowed
A calculator following
on the following
allowed on
questions.
questions.
Which of the following is the best interpretation
of the y-intercep t in the contex t of this problem?
A) The price of each cake
t_2x+9
t=--
2x + 9
B) The cost of making each cake
_ 55
C) The daily costs of running a bakery
The equation above
The equation models the
above models time t, in
the time
seconds, it takes
seconds, takes to load
load aa web
web page with x
page with D) The daily cost of making the cakes that
images. Based
images. on the
Based on model, by how
the model, many
how man y weren't ab le to be sold
seconds does
seconds does each image increase
each image the load
increase the time
load time
of a web page?
ofawebpage? _

What does
What does it mean that (5, O)is
mean that 0) is aa solution
solution to the
the
equation
equation of the
the line?
line?
A) The
The bakery needs to sell 55 cakes
bakery needs per day
cakes per to
day to
cover its daily
cover daily expenses.
expenses .
B) Each
Each cake be sold
must be
cake must sold for at least 5 dollars
at least dollars
---------T---------7 to cover
cover the cost of making
the cost making iit.t,
Questions 11-13
Questions 11'13 refer
refer to
to the
the following
following C)
C) It costs 55 dollars
It costs dollars to
to make
make each cake.
each cake.
information.
information. D) Each day, the
Each day, the bakery
bakery gives the first 5 cakes
gives the cakes
away
away for free.
Daily Profit
Daily Profit (y)
600 -1----,----,-1 ------ --------- A ---------
500
400 - L -
300 -l- ---r----"'-------"'-1 ;-;,"'--~-
200 +- -,----,------ ----;;;,

100 +- --,----,------,-- ~- ~--
1---4----+--+--+~~+--+----t---;,- Cakes (x) T = 56 + Sh
-100 7- - 8--- 9--
-200 To w a r m up
warm up his room, Patrick
his room, turns on
Patrick turns the
on the
-300 . heater. The temperature
heater . The temperature T of his room, in
his room,
-400
-500 degrees Fahrenheit,, can
degrees Fahrenheit can be
be modeled the
modeled by the
-600 equation
equation above, where h is the
above, where the number
number of hours
hour s
-700 ·
the heater
since the
since started running.
heater started Based on
running . Based on the
the
model, what is the
model, what the temperature increase, in
temperature increase,
The relationship
The between the
relationship between daily profit
the daily profit y, in
degrees Fahrenheit,
degrees Fahrenhei t, for every 30 minutes
every 30 minutes the
the
bakery and
dollars,, of aa bakery
dollars and the
the number cakes sold
number of cakes sold
heater
heater is turned on?
turned on?
by the bakery is graphed
the bakery graphed in the xy-plane above
the xy-plane above..

What does
What slope of the
the slope
doe s the the line represent??
line represent
price of each
The price
A) The cake
each cake
The profit
B) The profit generated from each
generated from cake sold
each cake sold
C) The
C) daily
The dail profit generated
y profit from all the
generated from the
cakes that
cakes that were sold
were sold
D) The
The number cakes that
number of cakes need to be
that need sold to
be sold
make aa daily
make daily profit dollars
profit of 100 dollars

130
130
THE COLLEGE PANDA

2yy-‐ xX ==114
2 4
Alice owns a
Alice owns pet frog
a pet would like
but would
frog but like to add
add
turtles
turtles to the same tank.
the same The local
tank . The local veterinarian
veterinarian
the equation
uses the
uses above to determine
equation above determine the total
the total
amount of water
amount watery,y, in gallons,
gallons, that should be
that should be
held in the
held tank for x turtles
the tank turtles to thrive
thrive alongside
alongside
Alice's frog.
Alice’s Based on
frog. Based the equation,
on the which of the
equation, which the
following must
following must be
be true?
true?
1,
I. One additional gallon
One additional gallon of water can support
water can support
ttwo more turtles.
w o more turtles .
III. One additional
I , One turtle requires
additional turtle t w o more
requires two more
gallons
gallons of water.
water.
III. One
One more turtle requires
more turtle an additional
requires an half
additional half
a
a gallon
gallon of water.
water.
A) II
ll only
only
B) I I only
IIII only
C)
C) I and
and II only
only
D) I and I I I only
and III only

C = 1.5 + 2.5x

A local
local post the equation
uses the
post office uses above to
equation above
determine
determine the cost C, in dollars,
the cost mailing a
dollars, of mailing a
weighing x pounds.
shipment weighing
shipment increase of 10
pounds . An increase 10
dollars in the
dollars the mailing cost is equivalent
mailing cost to an
equivalent to an
increase of how
increase many pounds
how many the weight
pounds in the weight of
the shipment?
the shipment?
A) z2
B) 2.5
C) 4
D)
0) 5

131
131
Functions
Functions
function is aa machine
A function takes an
that takes
machine that input, transforms
an input, transforms it, and spits
it, and an output.
u t an
spits oout output. In math, functions are
math, functions are
denoted by Jf ((x),
denoted x ) , with being the
with x being input. So
the input. the function
50 for the function

f(.\') : x2 +1
every input
every squared and
input is squared and then
then added the output.
get the
o n e to get
added to one important to understand
It's important
output. It's understand that
that x is aa
completely
completely arbitrary label‐it’s
arbitrary label just aa placeholder
- it's just placeholder for the input. In fact, II can
the input. in whatever
put in
can put whatever II want
w a n t as
as the
the
input, including values
input, including with x in them:
values with them:

(2x) =
ff<2x> (2.102 +
: (2x)2 +11

Jf(a):a2'+1
(a) = a 2 + 1
Jf ((b
b++11)) =
: (b
(17+ 1)2+1
+ 1)2+ 1
) =
fW(*) r (*)2+
( * ) ‘ * 1l
f(Panda)) =
/(Panda : (Panda )2 +
(Panda)2 +11

Notice
Notice the careful use
the careful parentheses.. In the
use of parentheses equation, for example,
the first equation, (2x)2 is not
example, (2.1')2 n o t the same as
the same as 2x2 Wrap the
2x2.
. Wrap the
input parentheses and
input in parentheses you’ll never
and you'll go wrong
never go wrong..

I f J(x)
EXAMPLE 1: If
EXAMPLE f(x) =
: (11+ 1)‘, then
(x + l)X, value off(0)
what is the value
then what of J(O) + ff(l) + f/(2)
(1) + ( 2 ) + f/(3)?
(3) ?

Just plug
Just the inputs.
plug in the inputs .

fJ((O)
0 )++ Jf ((1)
l ) f+f J( 2
(2)) ++fJ( (3)
3) =z(0+1)“+(1+1)1+(2+1)2+(3+1)3
(0 + 1)0 + (1 + 1) 1 + (2 + 1)2 + (3 + 1)3
:1“+2‘+32+4~‘
= 1°+ i + i + 43
: 11f+22¢+99++664
= 4

z fi_. f f
=~

132
132
COLLEGE PANDA
THE COLLEGE PANDA

EXAMPLE
EXAMPLE2: 4
4
f(x) = xx2‐10x+25
f(x) 2 -1 0x+25

what value
For what
For value of x is the function f above
the function aboveundefined?
undefined?

we can't
Because we
Because divide by 0,
can’t divide O,a function is
a function undefined when
is undefined when the denominator zero. Setting
denominator is zero. the
Setting th e denominator
denominator
to zero,
zero,

xx22 -‐ lOx + 25 z= 0
10x +

5)2z= 0o
(x ‐- 5)2

x
X =z [fil
The function f is undefined
The function when x = 5.
undefined when 5.
would
This wou
This be aa good
ld be time to talk
good time about domain
talk about domain and range::
and range
•0 Domain:
Domain: The
The set of all
set of all possible input values
possible input (x) to
values (x) to a
a function (values that
function (values that don’t lead to
don 't lead to an
an invalid
invalid
operation or an
operation output)..
undefined output)
an undefined
0 Range: The
• Range: all possible
The set of all output
possible ou (y) from
values (y)
tput values from a function.
function .
In Example 2, x z= 5 leads
Examp le 2, (x) being
leads to Jf (x) being undefined. However, all other
w1defined. However, values
other val give real
ues of x give real number outputs.
number outputs.
Therefore,
Therefore, the domain of
the domain offf is all real
real numbers except 5.
numbers except To verify,
5. To we can
verify, we take aa look
can take at the
look at graph
the grap h of
off:f:

0 5

As
As you can see,
you can graph has
the graph
see, the has no y-value
no y-va when x =
lue when = 5.
5. In fact, x =
In fact, = 5 is
is like an invisible
like an invisible line
line that
that the graph
the graph
approaches but
approaches but never
never crosses We call these
crosses.. We these lines vertical asymptotes.
lines vertical asymptotes . To summarize,
summarize, the function f has
the function has one
one
verticall asymptote
vertica equation x =
with equation
asymptote with = 5.
5.
might’ve also
You might've also noticed
noticed that
that the graph never
the graph goes below
never goes below the x‐axis. ano ther line
It's another
x-ax is. It’s line that the graph
that the approaches
graph approaches
but crosses.. The
never crosses
but never x-axis, in this
The x-axis, this case,
case, is horizontal asymptote.
is aa horizontal asymptote . The The function
function f hashas one horizontal
o n e horizontal
with equation
asymptote with
asymptote equation y = 0.
Because
Because there are no
there are no points on the
point s on graph that
the graph have a
that have y‐value of O
a y-value Oor below, the range
or below, of f is
range of is all positive
positive
real
real numbers. Put mathematically,
numbers. Put f (x) > 0.
mathematically, f(x) 0. By
By the
the way, thiss makes
way, thi of the
Because of
sense. Because
makes sense. the square in the
square in the

denominator of ff(x)
denominator of (x) =
= xm2 _ l~x + 25 = fi,
(x ~ 5 ) 2 , you
you always
alwa ys get
get a positive output
a positive output for
for any value
any va lue of
of xx in the
in the

domain.
domain .
Let’s
Let's summarize.
summarize. To find the domain,, start
the domain start with all real
with all real numbers and then
numbers and then exclude
exclude the for which
values of xx for
the values which
function is invalid
the function
the invalid or undefined. example,
For examp
undefined . For le, the domain of y := fl.jx is
the domain is xx ~ 0 because
2 0 because we
we can't take the
can't take the
square
square root negative numbers
root of negative numbers..
the range,
To find the graph the
range , graph the function on your
function on your calculator and figure
calculator and figure o u t the
out possible values
the possible of y,
values of taking note
y, taking of
note of
any horizontal
any asymptotes.
horizontal asymptotes.

133
CHAPTER
CHAPTER 16
16 FUNCTIONS
FUNCTIONS

EXAMPLE 3:I f f ( x -‐ 1)
EXAMPLE3:If/(x 1) =
= 6x and g(x) == xx +
6xandg(x) 3,what is the value (2))?
+ 3,whatisthevalueof/(g off(g(2)) ?

Whenever you
Whenever you see composite
composite functions
functions (functions of other
(functions of other functions), star t from
functions), start the inside
from the and work
inside and your way
work your way
out. First,
out. First,
g( 2) = 2 + 3 = 5
Now we have
Now we have to
to figure value of
the value
out the
figure out of f/(5).
(5).
Well, we can plug
we can plug in
in x
x := 6 f ( x -‐ 1)
into J(x
6 into 1) == 6x
6x to
to get
getf(5) = 6(6)
f (5) = = I-.36 I.
6(6) =

EXAMPLE 4:
EXAMPLE
the value
Functions/ f and
4: Functions
value of k ?7
and g are defined
g are byf(x)
defined by f(x) = xx+1
= and g(x)
+ 1 and g(x) = i·
= §.1ff(g(f(k))) = 10,
If f(g(f(k))) = is
what is
10, what

Again,
Again, we start from
we start from the and work
inside and
the inside way out:
our way
work our out:

k )=: k + 11
f ((k)

g(k+1):k%1
g(k + 1)
= k+1
2

1c;1) = k ;1+ 1

Finally,
Finally,

k+l
-I%1+1210
2- + 1 = 10

k%1:9
k+1 =
2
9

kk + 1l 2=118
8

k=E
k= [ill
As we've
we 've mentioned, function takes
mentioned, a function takes anan input
input and returns an
and returns an output. these input
Well, these
output. Well, input and output pairs
and output allow
pairs allow
us
us to graph
graph any function asa set of points in the xy‐plane, with the input as x and the output
any function as a set of points in the xy-plane, with the input as x and the output as y. 1n fact,asy. In fact,
2
y = xx2++ 11is the same
is the as f (x) = x2+
same asf x2 + 1. Both ff(x)
1. Both and yy are
(x) and are the same thing‐they’re
the same used to
thing-they're used to denote the output.
denote the output.
The only
The use y is that
reason we use
only reason that it’s
it's consistent with the
consistent with y-axis being
the y‐axis the y‐axis.
being the y-axis.
Anytime
Anytime ff(x) used in a graphing
(x) is used question, think
graphing question, of it
think of it as
as the
they.y. 50
So ifif a
a question (x) >
that fJ(x)
states that
question states > 0, allyy
0, all
values are positive
values are positive and
and the graph is always
graph is above the
always above x-axis. It’s extremely important that you learn to
the x-axis. It's extremely important that you learn to think think
of points
points on a graph
on a as the
graph as inputss and
the input and outputs
outputs of
of a
a function.
function .

134
THE
THE COLLEGE PANDA
COLLEGE PANDA

EXAMPLE 5:
EXAMPLES:
y
;

I/\
J \
- 1- -· I \
I \ X
'O
\ I \
\ I \

The graph of
The graph f (x) is
of J(x) is shown in the
shown in xy-plane above.
the xy-plane above‘ For value of
what value
For what of x (x) at
xisis fJ(x) its maximum?
at its maximum?

Again, when it
Again, when comes to
it comes to graphs, think of
graphs, think (x) as
offf (x) as they
the y.. So we're looking
So we’re looking for
for the point on
the point on the graph with
the graph with the
the
highest y‐value, the
highest y-value, ”peak”" of
the "peak the graph.
of the point is
That point
graph. That (5, 4),
is (5, and so
4), and so the x-value is ..
the x-value is I}].

EXAMPLE the function


EXAMPLE6: If the function with equation yy =
with equation = ax + 33 crosses
a):22 + crosses the point (1,
the point 2), what
(1,2), what is the value
is the value of
of a
a??

Remember - a point
Remember‐a is just
point is an input
just an input and
and an output, an
an output, an xx and
and a
a y. Because (1,
y. Because 2) is
(1, 2) a point
is a the graph
on the
point on graph of
of the
the
function, we
function, can plug
we can plug in 1 for x and
and 22 for y.

2
2 =:a(1)2
a(1 )2++3
3
2 = aa + 3

a z j

function y = 2+
EXAMPLE 7: If the
EXAMPLE the function = xx2 + 2x -‐ 4 contains
contains the point (m,2m)
the point and m
(m, 2m) and m>> 0,
0, what is the
what is value
the value
of m?
ofm?

It's important
It's important n o t to get
not get intimidated by all
intimidated by all the The question
variables. The
the variables. gives us
question gives point on
us aa point on the graph, so
the graph, let's
so let's
plug
plug it
it in.
in .

yy =
: x2
x2 +
+2x
2x -‐ 4
4
2m m2 +
2m = m2 + 2m
2m-‐ 44
00 = m2
m 2 -‐ 4
4

From here
From we can
here we that m = ±
can see that :l:2. question states
The question
2. The that m > 0,
states that 0, so
so m
m=: l.@].

zeros, roots, and x-intercepts


The zeros,
The x-inter~epts of aa function
function are all just
are all just different
different terms the same
for the
terms for thing‐the
same thing values
- the values
of x that
that make (x) =
make ff(x) = 00 (or y =
= 0). Graphically, they
O). Graphically, refer to
they refer to the values of
the values of x
x where the function
where the crosses
function crosses
the
the x-axis.

135
CHAPTER 16
CHAPTER 16 FUNCTIONS
FUNCTIONS

EXAMPLE8:
EXAMPLES:
y
~ I I
I
,
-- ' .,_:.
I
I
I

. '\ I

I
I
.."
"" '\
'\
' I
I
I

--
X
'
I " ...../ I

I
I
I
I

The graph off (x) = x3 - 2x2- Sx + 6 is shown in the xy-plane above.

PART 1: How many distinct


How many zeros does
distinct zeros does the function ff have?
the function have?

PART 2: If
If k is a constant such that
constant such f (x) =
that J(x) = k has solution, which
has 1 solution, the following
which of the following could
could be the value
be the value
of kk??
A)
A) ‐3
-3 B) 1 C) 5 D)
0)99

Part 1 Solution:
Part The graph
Solution : The crosses the
graph crosses x-axis three
the x-axis so ff has
times, so
three times, has ffidistinct
distinct zeros From the
zeros.. From graph, we
the graph, can
we can
see that
see that these are -‐ 22,, 1, and
zeros are
these zeros and 3.

Part 2 Solution: This question


So lution: This question is quite involved, so
quite involved, so don't panic ifif you
don’t panic lost during
you feel lost the explanation.
during the Read
explanation. Read
all
all the way through
the way through and then go back the bits that were confusing. promise you'll be able make sense of
and then go back to the bits that were confusing . I promise you'll be able to make sense
everything.
everything.
To truly understand
understand this question, first realize
this question, constant is just
that a constant
realize that just a function.
function. No matter the input,
matter the we always
input, we always
get the
get the same output. In this
same output. this question,
question, wewe can write it as
can write asyy = k or g(x) = k. So say k = -‐ 33.. What
let's say
So let’s does
What does
y z= ‐3
- 3 look like? A horizontal
look like? line at
horizontal line at -‐ 33!!

Now when a
Now when a question
question asks the solutions
asks for the = k, it’s
so lutions to fJ (x) = merely referring
it's merely referring to thethe intersection points of f (x)
intersection points
and the horizontal
and the line y =
horizontal line k. In general,
= k. a question
general , if a functions equal
w o functions
sets ttwo
question sets equal to eacheach other, f(x) =
other, J (x) = g(x), and
and
asks
asks you about the
you about the solutions, it's referring
solutions, it’s referring to the intersection points.
the intersection After all,
points. After it's only
all, it’s only at the points
intersection points
the intersection
that the value
that the value of y is the same for both
the same both functions. this particular
functions. In this case, g(x)
particular case, g(x) just
just happens
happens to be be a constant
a constant
function, g(x ) =
function , g(x) : k.
The number of solutions
The number equivalent to the
solutions is equivalent the number intersection points.
number of intersection So ifif k =
points. 50 : ‐3
- 3 as
as shown above, there
shown above, there
be 3 solutions
must be
must as represented
solutions to fJ (x) = -‐ 33,, as represented by the intersection points.
the 3 intersection The solutions
points. The solutions themselves are the
themselves are the
x-values those points.
x-values of those points . We can estimate them
can estimate them to be 2.2, 1.6, and
be -‐2.2, and 2.6.

136
THE
THE COLLEGE PANDA
COLLEGE PANDA

Getting back
Getting original problem,
the original
back to the we have
problem, we have to choose a
to choose ak
k such
such that there is
that there is only one solution.
only one Now we’re
solution. Now we're
thinking
thinking backwards. Instead of being
backwards. Instead being given
given the constant, we
the constant, have to
we have to choose
choose it.it. Where might
Where might we place aa
we place
horizontal line
horizontal line so
so that only one
there’s only
that there's one intersection point? Certainly
intersection point? o t at
Certainly nnot - 3 because
at ‐3 just showed
we just
because we how
showed how
that would
that result in 3 solutions.
would result solutions .
looking back
Well, looking back at the graph,
at the graph, we could place
we could one just
place one just above or just
above 88 or just below Horizontal lines
below -‐ 44.. Horizontal at these
lines at these
values would
values would intersect w i t h fJ (x) just
intersect with just once. Looking at
once. Looking at the answer choices,
the answer choices, 9
9 is the only
is the one that
only one meets our
that meets our
Answer ~(D) .-
condition. Answer
condition.

Let’s take a
Let' stake a moment part 1.
revisit part
moment to revisit In part
1. In part 1, we found
1, we found the number of
the number intersection points
of intersection between fJ (x)
points between and
(x) and
the
the x-axis. realize that
But realize
x-axis . But that the
the x-axis is just the
is just horizontal line
the horizontal y = 0. In counting the number of intersection
line y = 0. In counting the number of intersection
points between f (x) and
points between the horizontal
and the horizontal line
line yy = 0, what
= 0, what youyou were
were really doing is
really doing is finding number of
the number
finding the of
solutions to f (x) =
solutions = 0.
If you
you didn’t grasp everything
didn't grasp everything in this example
in this example the time through,
first time
the first through, it’s Take your
ok. Take
it's ok. and go
time and
your time through
go through
it again, making sure
again, making you fully
sure you fully understand each of
understand each the concepts.
of the concepts . The SAT will
The SAT will throw quite aa few
throw quite few questions at
questions at
related to the
you related
you zeros of
the zeros of functions
functions asas well as the
well as solutions to
the solutions to fJ (x)
(x) == g(x).
g(x) .
Hopefully by
Hopefully by now, you’re starting to see constants ashorizontal lines.
now , you're starting to see constants as horizontal lines. 50 instance, if
for instance,
So for if fJ (x)
(x) >> 5, 5, that means
that means
entire graph
the entire
the graph of
offf is above the
is above horizontal line
the horizontal line y =z 5.5. Thinking
Thinking of of constants
constants in this way
in this way w help you
i l l help
will you on
on a
a
lot of SAT graph
lot graph questions.
question s.

EXAMPLE 9:
EXAMPLE Which of
9: Which of the following could
the following be the
could be graph of
the graph of y
y = xx3+ 2x 2 +
3 + 2x2 + xx ++ 11??
A) m
B) C)

#‘x $3: %x
~ q
yy y y

D)
yy

Although the
Although the given function looks
given function complicated and
looks complicated and you might be
you might tempted to
be tempted graph itit on
to graph on your
your calculator,
calculator, this
this
is the easiest question
the easiest ever! A
que stion ever! Alll l you have to
you have to do
do is find a
is find a point
point that’s certain to
that's certain be on
to be on the graph eliminate
and eliminate
the graph and
the graphs
the graphs that don 't have
that don’t that point.
have that So what’s
point. So an easy
what 's an point to
easy point to find and test?
find and test?

Plug
Plug in x = 00 to
x z get y =
to get = 1.
1. N o w which
Now graphs contain
which graphs the coordinate
contain the (0, 1)?
coordinate (0, 1)? Only
Only graph
graph ~-. -
By
By the way, numbers
the way, like 00 and
numbers like and 11 are particularly good
are particularly for finding
good for finding ”easy”
"easy" points to use
points to this strategy.
for this
use for strategy.

137
CHAPTER
CHAPTER 16
16 FUNCTIONS
FUNCTIONS

Function Transformations
Function Transform ations
Function transformations
Function transformations areare changes
changes we make to
we can make equation of
the equation
to the of a function to
a function to ”transform”
"transform" its graph in
its graph in
specific ways.
ways . The transformations you might encounter on the SAT are reflection
transformations you might encounter on the SAT are reflection across the x-axis, vertical
across the x-axis, verticaJ
shift, horizontal
shift, horizontal shift, and absolute
shift, and absolute value, We’ll cover
value. We'll cover the first three
the first three here and discuss
here and discuss absolute value transformations
absolute value transformations
in the absolute value chapter.
absolute value chapter.
Let’s
Let's start the example
with the
start with example function (x) =
function fJ(x) x2 +
z x2 + 2x, whose graph
2x, whose graph looks
looks like
like
y

To reflect the graph


graph of fJ(x)
(x) across x-ax is (flip
across the x-axis (flip itit upside down),, multiply
upside down) multiply ff(x)
( x ) by
by -‐ 11.. The
The resulting
resulting equation,
equation,
312 ~f (x) z ‐x2 ‐
y = - f(x) = - x 2 - 2x, produces
produces the reflected graph.
reflected graph.
yy =
= -‐ f(x)
fl fl
y
·. I

To shift
To graph of fJ(x)
shift the graph ( x ) up, add a constant
up, add example, y 2= fJ(x)
( x ) . For example,
constant to ff(x). ( x ) ++ 22 =z x2
x2 +
+ 2x + 22 produces
2x + produces aa
graph that is 2 units
graph that above the graph
units above graph of
of ff(x).
(x).
shift the graph
To shift graph of fJ(x) down, subtract
(x) down, subtract a constant
constant from ff(x). For example,
(x). For example, yy = (x) -‐ 2
= fJ(x) 2 z= x2 + 2x
x2 + 2x -‐ 2
2
produces
produce s aa graph that is 2 units
graph that units below
below the graph of f (x).
the graph of J(x).

yy = fl n) ++22
= f(x y = f(x) - 2
y y

138
THE COLLEGE
COLLEGE PANDA
PANDA

graph of fJ((x
the graph
shift the
To shift x ) to the [1units,
the left by a substitute x +
units, substitute in for
+ a in for x. For example,
x. For = fJ((xx +
example, yy = + 1)
1) =
=
(x + 1)
1)22 + 2(x + 1)
1) produces
produces a that is
graph that
a graph is 11 unit
unit to
to the left of the graph of f(x).
the left of the graph of J (x).
To shift
shift the graph o
the graph offff ((x)
x ) to the
the right
right by a11units, substitute x -‐ a
units, substitute a in
in for x. For example,
x . For = fJ((xx -‐ 1) =z
example, yy =
)2 +
(x -‐ 11)2 + 2(x -‐ 1)
1) produces
produces a a graph
graph that is 11 uunit
that is n i t to the right
to the right of the graph
of the graph ofoff(x).
J (x).
y = f (x + l ) y = J (x - 1)
y y

Here's that I like to use


Here’s a trick that make sense
use to make horizontal shifts:
sense of horizontal shifts :

For horizontal shifts,


For horizontal find out
shifts, find what value
out what of xx makes
value of the substituted
makes the substituted expression equal to
expression equal to 0. This value
0. This value will
will
tell you
tell you what
what the horizontal shift
the horizontal shift is. For instance,
is. For when we
instance, when we have ‐ 1),
have J(x - 1), what value of x makes x
f (x what value of x makes x -‐ 1
1
equal to 0? x = 1. So J(x - 1) is 1 unit to the right off (x ). What about J(x + 4)? Well, x = - 4 makes x + 4
equalto 0?x = 1.Sof(x ‐ 1)is1 unit tothe rightoff(x). What aboutf(x+4)? Well,x = ‐4makesx+4
equal to 0. Since
equal to Since the value is negative,
the value negative, the graph is
the graph is shifted
shifted 4 units
units to
to the
the left. what about
And what
left. And f(3x -‐ 2)?
about /(3x 2)?
What’
What's5 the
the horizontal
horizontal shift relation to
shift in relation f (x)? A value
to f(x)? = ~
value of x = 2 makes equal to
makes 3x -‐ 22 equal to 0,so horizontal
the horizontal
0, so the
2
is ~3 units
shift is
shift units to
to the right.
the right.

Note that
Note horizontal and
that horizontal and vertical
vertical shifts are commonly
shifts are referred to astranslations.
commonly referred as translations. And the graph
And the of a
graph of a transformed
transformed
hmction is often
function called an
often called image of the
an image graph of the
the graph original function.
the original function .

EXAMPLE 10: In the


EXAMPLE the xy‐plane, the graph
xy-plane, the graph of the function g
the function the graph
g is the graph of
of f/ translated
translated 55 units the left
to the
units to left
and 33 units
and units downward.
downward. If function f is
the function
lithe is defined (x) = (x -‐ 3)
defined by fJ(x) 3)22 +
+ 1,
1, which
which of the following
of the following defines
defines
g(x)?
g(x)?
A)g(x) = (x - 8)2 - 2
A)g(x)=(x‐8)2‐2 B)g(x)=(x+2)2‐2
B)g(x) = (x + 2)2 - 2 C)g(x)=(x‐8)2+4
C)g(x) = (x - 8)2 + 4 D)g(x)=(x+2)2+4
D)g(x) = (x+2) 2 +4

A translation
translation of 5 units
units to the
the left and downward means
units downward
and 3 units means that
that

3 0x )) =
g( J (x + 5) -‐33
=f(-Y+5)
: ( (x
= + 11-_ 33 == (x+2)2~2
( x + 55)) -‐ 33)) 22+ (x + 2) 2 - 2

Answer ~(B) .
Answer

summarize, for a function


To summarize,
To f (x),,
function J(x)
o (x)) results
• -‐ Jf (x in a
results in a reflection across the
reflection across x-axis
the x-axis
x) +
• fJ((x
0 + a results
results in an upward
in an upward shift units; Jf(.r)
shift of a units; (x) ‐- 11results in
a results in a
a downward
downward shift of a
shift of a units
units
• f (x +
0 + a) results
results in a horizontal shift of a units
horizontal shift to the
units to the left; a) results
left; f (x ‐- (1) results in
in a horizontal shift
a horizontal shift of a units
of a w1its to
to
the right
the right

139
CHAPTER 16 FUNCTIONS
CHAPTER 16 FUNCTIONS

CHAPTEREXERCISE:Answers for this chapter start on page 311.

A calculator should NOT be used on the


following questions.
y

X y
0 20
1 21
3 29

The
The table above displays
table above displays severa on the
points on
severall points the
graph of the
graph the function f in the
function fin xy-plane. Which
the xy-plane. Which of
the following
the could bef
following could (x) ?
be J(x)?
A) f (x)) = 20x
A) J(x
8) (x)) z= xx ++20
B) fJ(x 20 The graph
The graph of the function f is shown
the function shown in the
the
C)
C) fj (x)
(X) = = xX -‐ 20
20
xy-plane above. If =
xy-plane above. If f (a) f (3), which of the
J(a) f(3), which the
following could be
following could of aa ??
value of
the value
be the
D)
D) f(x)
J(x )==x2 x ++20
2
20
A)) -‐ 4
A
B
8) ) -‐ 3
C
C)) -‐ 2

y D) 1

J(x)

In the portion of the xy-plane shown above, for


how many values of x does J(x ) = g(x)?
A) None
B) One
The function f is graphed in the xy-plane above .
C) Two For how many values of x does J(x) = 3?
D) Three
A) Two
8) Three
C) Four
D) Five

140
THE COLLEG
THE COLLEGEE PANDA
PANDA

For which of the following functions is it true IfIf fJ(x)


(x) =
= x2, which of
for which
x 2 , for the following
of the lues of
values
following va of
that J(-3) ==f(3)
thatf(‐3) J(3)?? c isf
is ((c)
c) < cc??

M
A) f m ==-X2;
J(x) A)E1
A) 1
2
x3 B
8) ) 1
B)fu%=§
8) J(x) =
3 3
C) ~
C)z
C) J(x) =3x2+1
C) f(x) = 3x 2 + 1 2
D) f(x)
0) + 22
f(x) = xx + D
D) ) 2

The hmction f is defined by J(x) = 3x + 2 and IfIf the graph of the


the graph function fJ has
the function has x-intercepts
x-intercepts atat
the function
the function g is defined by g(x)
defined by = J(2x)
g(x) = f(2x) -‐ 1.
l. -‐33 and
and 2, and
and ay-intercept
a y-intercept at 12, which
at 12, which ofof the
the
What is the
What the value of g( 10)
value of lO) ?
? following
following could define f ?
could define ?
A) ( x )‐=‐ (x
A) fJ(x) (x + 3)2(x -‐ 2)
(+3)2<x z)
B)) f ((x)X =) ((xH+
B + 33)(x 2)22
X ‐ )-( 2)
C ( X ) = (x
C)) fJ(x) + 2)
3)2 (X+2)
( X-‐ 3)2(x
= ‐(x( X- 3)(x
D) ) fJ ((x) =
D 3)(X + 2)22
+ 2)

2 J(x) = x 2 + 1
If J(x) = 16 +x x for all x 7é0,
-:fa0, what is the
what is value
the value
2 g(x)=x 2 - 1
off (- 4)? g(X)=x2‐1
A) - 8 The functions
The functions f and
and g are above. What
defined above.
are defined is
What is
8) -4 value off(g(2))
the value
the of f(g(2))? ?
C) 4 A) 3
D) 8 B) 5
C
C)) 110
0
D
D)) 117
7

X 0 1 2
J(x) -2 3 18
Several values
Several values of thethe function
function fJ are given in
are given in the
the
the xy-plane,
In the which of
xy-plane, which the following
of the following
translations
translati on s of graph of
the graph
of the of y 2x 2 ‐- 2
y = 2x2 results in
2 results in
table above.
table above . If (x) = ax2
If fJ(x) ax2 ++ bb where
where a a and
and b are
bare
constants, what value of (3) ?? the graph ofy
the graph of y == 2x2
2x +
2
+ 44??
constants, what is thethe value offf (3)
A
A)) 223
3
A)
A) A translation
translation 2 units downward
2 units downward
B)
B) 39
39
B)
8) A translation
translation 6 units upward
6 units upward
C)
C) 43
43
C) A translation
translation 2 units
units to the left
to the left
D
D)) 556
6
D) A translation
translation 6 units to
6 units the right
to the right

141
141
CHAPTER 16 FUNCTIONS

y [n the xy-plane,
in the graph of the
the graph
xy-plane, the function f
the function
A
reaches maximum value
its maximum
reaches its value at the point (3,f (3)
the point (3) ).
The function
The g(x) =
defined by g(x)
function g is defined (x) +
= ff(x + 7.
7. At
which of the
which the following points in the
following points the xy-plane
xy-plane
does the graph
does the reach its
graph of g reach its maximum value?
maximum value?
(10, f (10) + 7)
A) (10,f(10) 7)
(f (3), f (3) ++ 7)
B) (f(3),f(3)
B)
(3,f( lO))
C) (3,f(10))
C)
D) (3,f(3) ++ 7)
D) (3,](3)

graph of the
The graph function f and
the function line segment
and line segment EAB
are shown in the
are shown xy-plane above.
the xy-plane For how
above . For many
how many
values of x between
values between -‐33 and
and 3 does f(x) =
does J(x) = c?
c? (x) = ✓xx -
fJ(x) 2
‐ 2
function f is defined
The function
The above for all x 2:
defined above 2 2.
2.
Which of the
Which the following is equal
following is equal to
to
f(18) -‐ f f(1
f(18) ( 11l ))??
A)
A) f (3)
f(3)
B)
B) ff(5)
(5)
C)
C) f(6
f(6))
D)
D) ff(7)
(7)

X f(x)
f<x>
-4 3
- 2 5
The function f is defined by J(x ) = (x - 3)2,
0 2
and the function g is defined by
2 16
g(x) = x2 + 4x + 4. The graph of g in the
3 4
xy-plane is the graph off shifted k tmits to the
4 8
left. What is the value of k ?
The table above gives some values for the
function f. If g(x) = 2f(x), what is the value of
kif g(k) = 8?
A) 2
B) 3
C) 4
D) 8

142
142
THE COLLEGE PANDA
THE COLLEGE PANDA

A calculator is allowed on the following


questions.
Ifg(c)
If g(c) := 5, what is
5, what is the of f(c)
value of
the value J(c)??
A)) -‐ 22
A
B) 3
_ xx +
+1l
y=
‐ x ‐ 11
x - C) 5
D) 6
Which of the
Which the following points in
following points xy-plane is
the xy-plane
in the is
NOT on
NOT the graph
on the graph of y
y??
1
A
A) (- 2, 3)
8) (- 1, 0)
1 .
C) (0, - 1) l f J(x)
If and éfla)
f ( x ) = -‐ 33xx + 55 and J(a ) = 10, i sthe
what 1s
10, what the
2
D) (1,2 ) value
va lue of a
a??
A
A)) -‐ 8
B
B)) -‐ 5
C) 5
the function
Let the be defined
function g be defined by g(x) =
by g(x) : @.
ffx. If D
D)) 8
g(a) == 6, what
what is th value of a ??
thee value
A) 3
B)
8) 6
C) 9 X J(x)
D)
D) 12
12 0 -4
1 -8
--------~-------
V - 2 3
Questions 20-21 refer to
Questions 20-21 following
to the following 3 6
information.
information. 4 7
X f(x) g(x) 5 2
- 2 3 4 6 4
- 1 5 2 7 5
0 - 2 -3 Several values
Several values of the
the function
function f areare given
given in the
the
1 3 5 table above. If the function
table above . If the function g is defined
defined by
2 6 7 g(x)
g(x) == f(2x
J(2x ‐- 1), what is
1), what is the value ofg(3)
the value of g(3)??
3 7 1
A)
A) 2
The functions
The functions f and
and g are defined
3 are values
the six values
defined for the B)
8) 6
of x shown the table
shown in the table above.
above .
C) 5
D)
D) 7

What is the value of J(g( - 1)) ?


A) - 2
B) 3
C) 5
D) 6

143
CHAPTER 16 FUNCTIONS
CHAPTER 16 FUNCTIONS

f(x) :=44xx -‐ 3
J(x) IIff f(x)
f ( x ) is aa linear
linear function such that
function such f(2) :::;
that /(2) f(3)
S /(3),
g(x) = 33x+5
g(x) x+5 f(4)
/(4) 2 ~ f(5),
/(5), and f(6) = 10, which
and /(6) which of
of the
the
following
following must must bebe true?
true?
The functions
The functions f and
and g are defined above.
are defined Which
above. Which
of the
the following is equal
following is equal to
to f (8)
(8) ?
?
A ( 3 ) < f(O)
A) ) f/(3) f ® ) <<f/(4)
@)
f(O)
B) f(0 >=
B) = 0
A) g(l)
C
C) ) fl
f(O)m >>110 0
B) g(3)
D) ( 0 )=
D ) ff(O) ‐‐ 10
10
C) g(5)
D) g(B)

y -- Y = g(x) -

1
+-----::Q +----------1----+ X
1

'
•--~ .
I •

! ····- I ..

The graph of the function g is shown in the


xy-plane above, and the function f (not shown)
graph of f(x)
The graph
The f (x) is shown
shown in the xy-plane
the xy-plane is defined by f (x) = x3 . If g is defined by
above.
above. If g(x) =
If g(x) = (x ++ 3)(x - l ), which
3)(x ‐ 1), for which of the
the g(x)
g(x) := fJ(x
(x +
+a)+ where a and
a) + b, where and b are constants,
bare constants,
following values of x is
following values ( x ) > g(x)
is ff(x) g(x)?? what is the
what the value
value of a
a++ bb??
A
A)) ‐- 3 A
A)) ‐- 5
B
B)) -‐ 2 B)) -‐ 1
B
C) 1 C) 1
D) 2 D
D)) 5

In the xy-plane, the graph of the function g is the


image of the graph of the function f after a

translation of 11;~ units


translation units to the right. Which
the right. Which of the
the
following
following defines g(x) ?
defines g(x) ?
A g u )==J(3x
A) ) g(x) f 6 r-‐ 2)
E
( x ) = fGX+fl
B)) gg(x)
B f(3x + 2)
( x )=
C)) gg(x)
C = ff(2x
o ‐- w3)
D) =
D ) g u ) = f o ++%
g(x) f(2x 3)

144
THE COLLEGE PANDA
THE COLLEGE PANDA

w”

y y

graph of the
The graph
The function y =
the function 2
= 9 -‐ xx2 shown in
is shown
the xy-plane above
the xy-plane What is the
above.. What length of E
the length AB ?? The function f(x) = x3 + 1 is graphed in the
A) 3\/§
A) 3v'2 xy-plane above. If the function g is defined by
B ) 3v'10
B) 3m g(x) = x + k, where k is a constant, and
J(x) = g(x) has 3 solutions, which of the
C) 9
following could be the value of k?
D ) 9v'10
D) 9m
A)) -‐ 1
A
B) 0
C) 1
D) 2
y

[n the
1n the xy-plane, function y = ax
the function
xy-plane, the ax +
-+- 12, where
12, where
passes through
constant, passes
a is a constant, the point
through the point ((‐a,a).
- a, a) .
> 0, what
If a > what is the value of a ??
the value

The function f is graphed in the xy-plane above.


If the function g is defined by g(x) = f(x) + 4,
what is the x-intercept of g(x)?
A) - 3
B) - 1
C) 3
D) 4

145
Just as lines
Just as were one
lines were group of
one group functions that
of functions that have
have their
their own
Quadratics
Quadratics
o w n properties,
17
quadratics are
properties, quadratics another. A
are another. quadratic
A quadratic
is a
is function in the
a function the form
form
f(x) =
J(x) axz2 + bx
r - ax b x+
+c
which the
in which highest power
the highest is 2.
power of x is 2. The
The graph of aa quadratic
graph of quadratic is
is a
a parabola.
parabola .
To review
review quadratics, we’ll walk
quadratics, we'll through aa few examples
walk through examples to
to demonstrate
demonstrate the properties you
various properties
the various to
need to
you need
know.
know .

QUADRATIC 1:
QUADRATIC
f(x) 2 -- 44x
f(x) = xx2 x ‐- 21
21

The Roots
The Roots
The roots
The refer to the
roots refer values of x that
the values that make x) =
make Jf ((x) They’re also
: 0. They're also called x‐intercepts and
called x-intercepts and sosolutions. We’ll
lutions . We'll
mainly use the term
mainly use the this chapter,
“ r o o t ” in this
term "root" chapter, but the other
but the other term
termss are just as
are just as common.
common. DonDon’t forget that
't forget they all
that they all
mean the
mean the same thing.. Here
same thing Here,, we we can
can just factor to
just factor find the
to find roots:
the roots:

xx2~4x‐21
2 - 4x :00
- 21 =
) ( . r+
( . r-‐ 77)(x
(x 3 ) =2 0
+ 3)
x
X =
: 7,
7, -~33

The
The roots are 7 and
roots are 3. Graphically,
and -43. this means
Graphically, this means the quadratic crosses
the quadratic the x-axis
crosses the at xx =z 77 and
x-axis at and x.r =: -A3.
3.

Roots
Sum and Product of the Roots
The Sum
We already
We found the
already found the roots, so their
roots, so their ssum just 77 + ( -‐ 33)) z 4
u m is just =
4 and product is
their product
and their is just
just 77 xx -‐33 =
z -‐ 221. This
1 . This
really easy,
was really
was so why
easy, so why do we care about these
care about these values?
values? Because
Because sometimes you’ll have
sometimes you'll have to find the the sum the
or the
s u m or
product the roots
product of the roots without knowing the
without knowing the roots themselves. H
roots themselves. o w do we do that?
How do we do that?
. . . b
Given a
Given quadratic of the
a quadratic : ax
form y =
the form + bx + c, the
axz2 + the sum of the
s u m of the roots is equal
roots IS to -‐ ~E and
equal to the product
and the of the
product of the roots
roots
a
. equal
,
is
1s equa 1to -E.
c
c.
a

146
THE COLLEGE PANDA
THE COLLEGE PANDA

In our
our example,
example, a =
: l,Lbb =
: -‐ 44,, c = -‐21.So,
21. So,

Sumz‐é=‐‐:4
= - -- 4 = 4
b
Sum = - -
a l
1

‐ ‐-_21fl = ‐- 2211
C
Product = -E = ---
Product
a l
1

how we were
See how able to determine
were able determine these values without
these values knowing the
without knowing roots themselves?
the roots themselves? The we
that we
roots that
The roots
found earlier just
found earlier just confirm our values.
confirm our values.

The
The Vertex
The the midpoint
vertex is the
The vertex midpoint of a
a parabola.
parabola .

vertex
vertex

The x-coordinate of the


The x‐coordinate always the
vertex is always
the vertex midpoint of the ttwo
the midpoint roots, which
w o roots, can be
which can found by
be found averaging them.
by averaging them.
vertex is at x =
+ (-( ‐ 33))
7+
Because
Because the roots are
the roots 7and
are 7 the vertex
and -‐ 33,, the z
2
= When x =
z 2. When 2, J (x) := (2)
: 2,f(x) (2)22 -‐ 4(2) 21 := ‐- 225.
4(2) ‐- 21 5.
Therefore,
Therefore, the vertex is at (2, -‐25).
the vertex that the
Note that
25). Note maximum or m
the maximum i n i m u m of a quadratic
minimum quadratic is always at
is always at the vertex.
the vertex.
ln this
in it's a minimum
case, it’s
this case, minimum of ‐25. - 25.

Form
Vertex Form
Just asslope-intercept
Just as slope-intercept form mx + b)
form (y = mx+ b)is one way
is one way ofof representing
representing aa line,
line, vertex form is
vertex form is one way of
one way of representing
representing
a quadratic
a quadratic function We’ve already
function.. We've already seen t w o different
seen two ways quadratics
different ways be represented,
can be
quadratics can namely standard
represented, namely standard
form (y =
form (1; ax2 +
: ax2 + bx
bx++ c) and
and factored form (y =
factored form : (x -‐ a a)(x
)(x -‐ b)). Vertex form looks likey
form looks like y = a(x -‐ h)2
= a(x + k.
h)2 +
get a
To get quadratic function
a quadratic function into
into vertex
vertex form we have
form,, we have toto do
do something called completing
something called completing the the square. Let's walk
square. Let’s walk
through it step-by‐step:
through step -by-step:
yy z= x2
x2 -‐ 4x
4x -‐ 21
21
the middle
See the middle term?
term? The That’s
The -‐ 44.. That 's the key. The
the key. The first step is to divide
step is divide it by 2
it by to get
2 to get -‐ 22.. Then
Then write the
write the
following:
following :
2) 2 ‐- 21
y = (x ‐- 2)2
where we put
See where put the
the ‐- 22? The first part
? The part is done.
done . N o w the
Now second step
the second step is to take that -‐22 and
take that square it.
and square get
We get
it. We
4.
yy:= (x‐2)2‐21‐4
(x - 2)2 - 21 - 4
See where we put
where we the 4?
put the Wesubtracted
4? We at the
subtracted it at end. The
the end. The vertex
vertex form is then
form is then

y z= (x -‐ 2)2 -‐ 25

To recap,
recap, divide coefficient by
middle coefficient
divide the middle by 22 to
to get
get the
the number
number inside the parentheses.
inside the Subtract the
parentheses. Subtract square of
the square of
that
that number the end.
at the
number at end .

147
CHAPTER 17 QUADRATICS
CHAPTER 17 QUADRATICS

Completing
Completing the the square takes some
square takes some time and practice,
time and so if
practice, so if you
you didn't catch all of
didn 't catch this, first
of this, prove to
first prove to yourself
yourself
that it is indeed
that the same
indeed the quadratic by
same quadratic by expanding
expanding the Then repeat
result. Then
the result. repeat the process of
the process of completing
completing thethe square
square
yourself.
yourself. If you’ve been taught
you've been taught a slightly different
a slightly way, feel free to
different way, We'll do
use it. We’ll
to use many m
do many o r e examples
more in this
examples in this
chapter.
chapter.
N o w why
Now why do do we care about
we care vertex form?
about vertex form? Well, look at the
look at the numbers! It’s called
numbers! It's vertex form
called vertex for aa reason.
form for The
reason. The
vertex (2, -‐25)
vertex 25) can be found
can be just by looking
found just at the
looking at numbers in
the numbers the equation.
in the equation. But we we already found the
already found the vertex,
vertex,
you say! Yes, that’s
you true, but
that 's true, had to find the
but we had the roots to do
roots to do so
so earlier,
earlier, and finding the
and finding roots is
the roots o t always
is nnot always so
so
allows us
form allows
easy. Vertex form us to
to find the vertex without
the vertex knowing the
without knowing roots of
the roots of a quadratic. It’s also very
a quadratic. It's also very muchmuch
tested
tested on the
the SAT!
SAT!
One final n
One o t e ‐ o n e of
note-one the most
o f the common mistakes
most common mistakes students make iiss ttoo look
students make look a
att y = 2)22 -‐ 2
= ((xx -‐ 2) and think
255 and think the
the
vertex is at
vertex is (‐ 2,
at (- 2, -25) of (2,
instead of
‐ 2 5 ) instead (2 -‐ 225).
5 ) One of thinking
pattern of
One pattern thinking II use
use to avoid this
to avoid mistake is
this mistake is to
to ask, What
ask, What
value of x would
value would makemake the expression inside
the expression inside the parentheses equal to zero?
parentheses eq11al zero? Well, x: x =2 2 would make xx ‐- 2
would make 2 equal
equal to 0.
to 0.
Therefore,
Therefore,the vertex is at xx‐ =
the vertex ‐ 2. T h i sis
This is the
the same type of
same type thinking you
of thinking you would use to
would use get the
to get solutions from
the solutions from the
the
factored form y‐‐
factored form y = (x ‐- a)(xa)(x -‐ b).
b).

The Discriminant
The Discriminant
If a quadratic
If a quadratic is in the
the form ax2 + bx + c,
form ax2 the discriminant
then the
c, then discriminant isis equal to b2‐
equal to b2 - 4ac. As we
4ac. As we '’ ll explain later,
l l explain the
later, the
discriminant'
discriminant IisSa component of
a component the quadratic
of the quadratic formula we explain
Before we
formula. Before significance, let’
its significance,
explain its let'ss calculate the
calculate the
discriminant u r first example,
discriminant for oour example,
f(x) = xx2‐4x‐21
f(x) 2
- 4x - 21

Discriminant =
Discriminant b2 - 4ac
= b2‐ : ((‐4)2
4ac = - 4 )2 -‐ 4(1)(‐21)
4(1) ( - 21) =
: 100
Now, what does
Now, what the discriminant
does the mean? Well, the
di scriminant mean? the value the discriminant
value of the doess not
discriminant doe matter. What
n o t matter. matters is
What matters is
the sign
sign of the
the discriminant‐whether
discriminant -w hether it's it’s positive, negative, or zero.
positive, negative, zero. In
In other words, we
other words, we don’t care that
don't care that it’s
it's
100, we just care
we just that it's
care that positive.. Letting
it’s positive Letting D be short for discriminant,
be short discriminant,

W h e nDD > 00,,


When W h e nDD =
When z O0,, W h e nDD < O
When 0,,
y y y

-----0-+----- X

there
there are w o real
are ttwo real roots
roots ((two solutions)
t w o solutions). tthere one real
h e r eis one real root.
root. there
there are
are no real roots.
no real roots.

The Quadratic
The Quadratic Formula
Formula
As we’ve seen, the
we've seen, the roots are the
roots are the most
most important aspect of a
important aspect a quadratic. Once you
quadratic . Once have the
you have roots, things
the roots, things like
like
ve rtex form
vertex and the
form and discriminant are
the discriminant are n ashelpful.
o t as
not helpful. Unfortunately,
Unfortunately, the roots aren't always
roots aren’t easy to
always easy to find or work
find or work
with. That’s
with. That's when vertex form,
when vertex discriminant, and
the discriminant,
form, the and the u m / product of the
the ssum/product the roots get us
can get
roots can to the
us to the answer
answer
faster.
if we
But if we m u s t find the
must the roots, always one
there is always
roots , there way to
surefire way
one surefire to do o ‐ t h e quadratic
do sso-the quadratic formula.
formula.

_ ‐b:|:\/b2‐4ac
- b± Jb 2 - 4ac
x=
_T 2a

forax2+bx+c:0.
for ax2 + bx + c = 0.

148
THE
THE COLLEGE PANDA
COLLEGE PANDA

purpose of learning,
For the purpose let's apply
learning, let’s apply the
the quadratic formula to
quadratic formula our example,
to our example,

f(x) = x 22‐- 44xx ‐- 2211


J(x)

According
According to the
the formula,
formula, the roots //ssolutions
the roots are
olutions are

x_
= -‐(‐4)i,/(‐4)?‐4(1)(‐21) _44 ± i/100
m _= 44 x± 1100 _= ?or _
x
‐ T ‐ 2(1)T ‐ ‐ ‐ 27 0 2 r ‐ 33
(- 4) ± J( - 4)2 - 4(1)( - 21) =

These are the


These are the same
same values that we
values that got through
we got factoring..
through factoring
Notice
Notice that the discriminant,
that the b2 -~ 4ac,
discriminant, 172 is tucked
4116, is tucked under the square
under the root in
square root the quadratic
in the quadratic formula.
formula. H does this
o w does
How this
help us
help us understand what we
understand what know about
we know about thethe discriminant?
discriminant?
Well, when b2 - 4ac > 0,
when b2‐ 0, the takes effect
the"“ :±l :"" takes we end
and we
effect and end up
up with w o different
with ttwo roots. When
different roots. b2 ‐- 4ac
When 172 4ac =
= 0,
0, the
the
does not
”" ±i "” does have an
n o t have an effect
effect since
since we're essentially adding
we’re essentially adding andand subtracting
subtracting 0, 0, both of which
both of give us
which give us the same
the same
root.. When
root b2 ‐- 4116
When [72 4ac < O, we're taking
0, we’re taking the square root
the square root of a negative
of a negative number, which is
number, which is undefined
undefined and gives us
and gives us
no real roots
no real roots (we'll about imaginary
talk about
(we’ll talk imaginary numbernumber in later chapter).
in aa later chapter).
Hopefully, the quadratic
Hopefully, the formula helps
quadratic formula helps you understand where
you understand the discriminant
where the discriminant and
and its
its various meanings come
various meanings come
from. Understanding
from. Understanding this
this connection will help
connection will remember the
you remember
help you the concepts.
concepts.
Now
Now that we’ve taken
that we've taken you on aa thorough
you on tour through
thorough tour properties of
the properties
through the of quadratics,
quadratics, we’ll go through
we'll go through a
a few
few
more examples
more examples to illustrate some important
illustrate some variations, but
important variations, we'll do
but we’ll do so
so at
at a much faster
a much pace.
faster pace.

QUADRATIC 2:
QUADRATIC 2:
(x) =
ff(x) ‐x22 ++ 66xx‐- 1100
: -x

Roots
The Roots
This quadratic cannot be
quadratic cannot be factored.
factored. A
And if we
fact, if
n d in fact, we look
look at the discriminant,
at the discriminant,

ac =
b1922 -‐ 44ac (6)22 -- 44(-1)(
: (6) 0 ) := ‐-44
( ‐ 1 ) ( ‐-110)

it's negative,
it’s negative, which means there
which means no real
are no
there are roots or solutions.
real roots The graph
solutions. The graph of the quadratic
of the quadratic makes
makes this
this even
even
more
more clear:
y

When the coefficient


When coefficient of the x2
the x 2 term the parabola
negative, the
term is negative, parabola is
is in
in the shape of
the shape an upside-down
of an upside-down ”"U."
U.”

149
CHAPTER 17 QUADRATICS
CHAPTER 17 QUADRATICS

The Sum
The and Product
Sum and Roots
Product of the Roots

x ) = -‐x2
Jf ((x x 2 +66xx -‐ 10
10
b
Sum= ‐9 = ‐i =6
Sum = - - = - - = 6
a
a
6
-‐11
C - 10
P r o d u c=t : -5 =
Product _‐10 :=110
z -- 0
a -‐11

already determined
what!? We already
Wait, what!? determined that
that there
there were no roots
were no H o w can
roots.. How there be
can there be a and aa product
sum and
a sum product of roots
roots
that don’t exist?
that don't quadratic doesn't
the quadratic
exist? Well, the doesn’t have any real roots,
have any but it does
roots, but imaginary
have ima
does have roots. The
ginary roots. values
The values
above are
above the sum
are the product of these
and product
sum and imaginary roots
these imaginary roots.. We'll cover imaginary
We’ll cover numbers in a
imaginary numbers later chapter.
a later chapter.

Vertex Form
Because the roots
Because imaginary, we
are imaginary,
roots are we can't
can't use
use their midpoint to find
their midpoint find the
the vertex. these cases,
vertex . In these we must
cases, we get the
must get the
quadratic in vertex
quadratic form. We'll
vertex form. We’ll have complete the
have to complete square..
the square

yyz‐x2+6x‐10
= - x 2 + 6x - 10
multiply everything
First, multiply
First, everything by negative
negative 11 to get
get the out of the
negative out
the negative x 2 term.
the x2 Having the
term . Having negative there
the negative makes
there makes
things needlessly
things complicated.. We'
needlessly complicated We’ll multiply everything
ll multiply back by -‐11 later.
everything back later .

-‐y:x2‐6x+10
y = x 2 - 6x + 10

Divide the
Divide the middle
middle term
term by 2 to get -‐33 and square this
and square result to get
this result get 9. Remember that we put
Remember that put the
the -‐33 inside
inside the
the
parentheses with
parentheses with x and subtract the 99 at
and subtract at the
the end.
end. Putting these pieces
Putting these pieces in place,
place,

= (x‐3)2+10‐9
-‐ y =( x - 3) 2 + 10 - 9
-‐yy =( 3)22 +
= (xx -‐ 3) +11

N o w multiply
Now everything by
multiply everything by -‐11 again,
again,
y= 3)22 -‐ 1
: -‐ ((xx -‐ 3)
Now it's
Now that the
easy to see that
it’s easy the vertex at (3,
vertex is at (3, -‐ 11)).. And
And because the graph
because the graph is an
an upside-down "U," -‐11 is the
upside-down ”U,” the
maximum value of Jf(x).
maximum value (x).

QUADRATIC 3:
QUADRATIC 3:
f(x) 2x2+5x‐3
f ( x ) = 2x 2
+ 5x - 3

The Roots
The Roots
Wecan
We can factor this quadratic to get
this quadratic get

2x22 +
2x + 5x -‐ 3 =
= 0
((2x
2x -‐ 1l ) ( Xx + 33)) = O
0
x
X =
z 0.5, -‐33

The roots
The roots are you don't
and -‐ 33.. If you
are 0.5 and don’t know
know how we factored
how we this, unfortunately
factored this, teaching factoring
unfortunately teaching factoring from the
from the
ground
ground upup is not
n o t within scope of this
the scope
within the book. Don't
this book. be afraid
Don’t be afraid to look
look up factoring lessons
up factoring lessons and drills online
and drills online
and in your
and textbooks. It's
your textbooks. It’s an essential skill
an essential skill to have Just know
have.. Just that every
know that every method
method out involves a
there involves
o u t there a little
little
trial and error. And if you’re ever stuck, the quadratic formula is always
trial and error. And you're ever stuck , the quadratic formula always an option . an option.

150
THE
THE COLLEGE
COLLEGE PANDA
PANDA

The Sum
The Sum and
and Product the Roots
Product of the Roots

b 5
Sum a
Sum = -‐B = -‐§ =
a
: -‐2.5
2
2
2.5
-3
Product
C

a
a=
Product = E = _‐3 = -‐ l1.5
22
.5

The Vertex
The
Averaging the
Averaging the two
t w o roots find the
roots to find the x-coordinate the vertex,
x-coordinate of the vertex,

0 5+
0.5 + (-
( ‐ 33)) =_ -‐2.5
2.5 =_ _ 25
22 _ 2 2 _ 1.25 1.

Plugging this
Plugging this into f (x) to find
into f(x) the y-coordinate,
find they-coordinate,

ff(‐1.25) 2(‐1.25)2
(- 1.25) = 2( 5( - 1.25) -‐ 3
- 1.25)2 + 5(‐1.25) :
3 = -‐6.125
6.125

The vertex
The vertex is atat (- . 2 5 , -‐6.125).
( ‐ l 1.25, Because the
6.125). Because quadratic opens
the quadratic opens upward
upward in the shape of aa "U,"
the shape the minimum
” U , ” the minimum
value f ( x ) is
value of J(x) is -‐6.l25.
6.125.

Vertex Form
y= 2x22 +
: 2x + 5x -‐ 3
2
everything by 2. Before
divide everything
First, divide
First, completing the
Before completing the square, always make
square, always the coefficient
sure the
make sure coefficient of xx2 is l. We'll
We'll
multiply the
multiply the 22 back later..
back later
y 2 5 3
2=X + 2X- 2
25
Divide the
Divide the middle
middle term get 2 and
term by 2 to get~ square this
and square result to get
this result 1‐6. We
get~!. We put the~2 inside
put the inside the parentheses with
the parentheses with
25
x and subtract the
and subtract at the
the 3‐2 at the end.
end.
16

-"‐<+?)2
E‘ x 4 2 16
z_
!!..= (x+ §Y_9
~)2
- 4916
22‘(x+4 4 16

Multiplying by 2,
Multiplying

I/
·3“
=2(x+ ~)
‐2(x+§)2‐9
4
2 - 49
8
yi, = 2(x +
+125)2
1.25) 2 -‐ 6.125

This is consistent
This with the
consistent with the vertex above..
found above
vertex found

The Discriminant
The Discriminant
For the
For the sake completeness, let's
sake of completeness, calculate the
let's calculate the discriminant. Hopefully, it will
discriminant. Hopefully, confirm the
will confirm that this
the fact that this
quadratic has two
quadratic has t w o distinct real roots
distinct real roots..
y=_ 2x22+
‐ 2x 5x -_ 3
+ 5x 3

Discriminant =
Discriminant 2 - 4ac := (5)
: b62‐ (5)22 -~ 4(2)(‐3)
4(2)(-3) = 49
The discriminant
The positive, which
discriminant is positive, confirms the
which confirms the fact that this quadratic
that this quadratic has
has ttwo real roots.
w o real roots.

151
151
QUADRATTCS
CHAPTER 17 QUADRATICS
CHAPTER

QUADRATIC 4:
QUADRATIC
f(x) ==4x2-12x+9
f(x) 4x2 - 12x + 9

Roots
The Roots
The
Wecou
We could but let's
factor this, but
ld factor use the
let's use quadratic formula
the quadratic instead.
formula instead.

x _ -‐bi\/b2‐4ac
b ± Jb 2 - 4ac _ -‐(‐12):t\/(‐12)2‐4(4)(9) 1 2±1 \/6
(- 12) ± J( - 12)2 - 4(4)(9 ) _ 12 J6 _ 3-3
x =------
_ 2a
2a ' 2(4) _ 8 ‘:'z2

As you can
As you can see, discriminant is 0 and
see, the discriminant quadratic has
and the quadratic has just one
one root, 2
root,~-
2

The Sum
Sum and Product Roots
Product of the Roots

b - 12
Sum = -‐~- = -‐‐ -
Sum z 3
=
a 4
C 9
Product = - = -
Product
a 4

If we only have
we only have one
one root, how is it that
root, how we can
that we have aa sum
can have and a
sum and product of two
a product Why are
roots? Why
t w o roots? are they
they different
different
from the
from the one root
root we found?
found?
Here's thing. While we
Here’s the thing. we may
may say a quadratic has just
a quadratic just one root, it really
one root, really has
has ttwo roots that
w o roots are the same.
that are same.
2 term, is expected to have two roots. When they're the same, we just refer to
all, aa quadratic,
After all,
After quadratic, with an an xx2 term, expected have t w o roots. When they’re the same, we just refer to
them as
them as one
one..
9
Soour
So ” t w o " roots
o u r "two" are ~3 and
roots are g. If we
and ~. we add we do
them, we
add them, do indeed 3, and
indeed get 3, and if we
we multiply
multiply them, we do
them , we do get ~.
Z.

The Vertex
The
When quadratic has
When aa quadratic just one
has just root, the
one root, x‐coordinate of the
the x-coordinate vertex is the same
the vertex same as
as the
the root.
root. That's because aa
That’s because
quadratic is tangent
quadratic when it has
tangent to the x-axis when has one
one root.
root.

0 Q
2
3
The y-coordinate is, of course,
They-coordinate course, 0. Therefore, the vertex
Therefore, the at (5,
vertex is at ( ~, O).
0). The minimum
minimum value of f (x)
value off (x) is 0.
0.

152
THE COLLEGE
THE PANDA
COLLEGE PANDA

Vertex Form
y= 4x22 -‐ 12x +
= 4x +9
2
First,
First, divide everything by 4.
divide everything 4. Before completing the
Before completing square, always
the square, always make
make sure the coefficient
sure the coefficient of xx2 We’ll
is 1. We'll
multiply the 44 back
multiply the later.
back later.
y1 : 22 _ 9
3X + -2
-4 = Xx - 3x+4
4 4
. .
D1v1de
Divide the
.
middle term
the middle by 22 to get
term by
3
get -‐ ~5 and
and square
square this result to get
this result 1. We
get ~.
9 the -‐%~ inside
put the
We put the parentheses
inside the parentheses
2 4 2
and subtract
with x and
with the ~g at
subtract the the end.
at the end.

The constants
The cancel out.
constants cancel out.

Multiplying
Multiplying by
by 4,
4,

This is consistent
This the vertex
with the
consistent with vertex found above.
found above.

Wow! WeWejust
just covered pretty much
covered pretty everything you
much everything need to
you need know about
to know quadratics.. Unfortunately,
about quadratics Unfortunately, we're
we’re not
not
quite
quite done
done yet as there
yet as there are
are aa few tough question variations
tough question variations that you should
that you be exposed
should be exposed to.

EXAMPLE 1: In the
EXAMPLE xy-plane, the
the xy-plane, parabola with
the parabola y = x2
equation 31=
w i t h equation 5x +
x 2 ‐- Sx + 6 intersects
intersects the line y =
the line = 3x -‐ 10
at point
at (a,b).
point (a, is the
What is
b). What value of b?
the value b?

This question type


This is aa question type that
that we already covered
we already covered in the systems of equations
the systems equations chapter,
chapter, but reviewing it
we’re reviewing
but we're
again here
again here because
because it will help you
will help you understand
understand the next few examples.
the next examples. The
The core concept is
core concept is that whenever
that whenever
you have to find
you have find the
the intersection point(s) of two
intersection point(s) graphs, solve
t w o graphs, solve the
the system
system consisting their equations.
consisting of their equations. The
The
solutions
solutions to the system are
the system are the
the intersection points. Here,
intersection points. we have
Here, we have

x 2 -‐ 5x
yy = x2 Sx +
+66
y = 3x
3x -‐ 10
10

Substituting the second


Substituting the equation into
second equation the first,
into the first,

3x‐10zx2‐5x+6
3x - 10 = x 2 - Sx + 6

00 == x¥2 -‐ &
Bx+
+m16

00 =
: ((x
x ‐- 44)F2
x=4
x=4

To find the y, we
find they, plug x =z 44 into
we plug into either
either of the
the original equations: y = 3(4)
original equations: 3(4) -‐ 10
10 =z 2.
2. Therefore,
Therefore, the point of
the point
intersection is at
intersection at (4, = [I].
and b =
(4, 2) and I.

153
CHAPTER 17
CHAPTER 17 QUADRAncs
QUADRATICS

EXAMPLE 2: How man ' the gra h of =


EXAMPLE2: How manyy times does the _ _ 2 . ‑
xy-plane? cannot use acalculator.)
xy-plane?(You cannotuse a calculator.) p Ofy
graph 6x + 3
y _ - xx2 + 6x 3 mm the line
intersect the y = 10in
line 31= the
10 in the

We’re dealing with the intersection of two graphs again. Sowhat do we do?">
We:re deal~g with the intersection of two gra hs a ain. So wh
Substituting the second eq
the second th fj
ti~ . g the at do the system
solve the
We solwg
we do . We consisting Of
system C°"5i5ti"8 of
the" equations, Substih1ting rst, we
their eguat10ns.
ua on into
equation into e first we get
get

1 0=: -‐x2+6.r+3
10 x + 6x + 3
2

00z
= -‐x2 + 6x
x2 + 6x‐- 7
7
Now
Now we couldgo
we could go ahead finish solving
and finish
~,ead and solving this to find
this to find the intersection point(s)
the intersection just like
point(s) just like we did in
we did in the previous
the previous
example, but there’s a faster way. For
example, but theres a faster way. For the
the purposes
purposes of
of this
this question,
question, we
we don
don ’t
't care
care where
where the intersection
intersection points
points
are.
are. We
We just
just want
want to know
know how
how many
many there
there are.

Sound familiar?
Sound familiar? We
We can use the
can use discriminant to do
the discriminant do that.
that.
2
Discriminant: b2‐
Discriminant = b (6)22 ‐- 4
: (6)
- 4ac = ) ( ‐ 77)) := 8
( ‐ 11)(-
4(-

discriminant is positive,
The discriminant which means
positive, which means there solutions to the equation
are 2 solutions
there are equation we up above.
we set up there are 2
above . If there
solutions the equation
solutions to the above, there
equation above, must be
there must be [I} intersection
intersection points.
points . To summarize,
summarize, we didn’t bother finding
didn't bother finding
two values
the two values of x. They could’ve been x
could've been = 2 and
and x = 100 for all we care, and the intersection points might've
and the intersection points might’ve
been (2,5)
been (100,6).
and (100,6
(2,5 ) and ). It doesn’t matter. What
doesn't matter. mattered was
What mattered was that were ttwo
there were
that there of them,
w o of we used
and we
them, and the
used the
discriminant determine that.
discriminant to determine that. If the discriminant were 0, there
discriminant were would
there would only be one intersection point. And ifif
be one intersection point. And
the discriminant than 0, there
were less than
discriminant were would be
there would points
intersection points.
no intersection
be no .
Make you understand
sure you
Make sure question.. Feel free to go
understand this question and figure out
back and
go back where the
out where points
intersection points
the intersection
actually (Hint: It's
are (Hint:
actually are not fun.
It's not You’ll need
fun . You'll the quadratic
need the formula . That’s
quadratic formula. why the
That 's why discriminant was
the discriminant so
was so
helpful).
helpful).

EXAMPLES:
EXAMPLE3:
y -‐ kk =
= 0O
yy=x2‐3x+l
= x2-3x + 1
Jn the
in system of equations
the system above, k
equations above, k is For which
constant. For
a constant.
is a of the
which of following values
the following values of k does
of It does the system
the system
of equations have no
equations have real solutions?
no real solutions?

A) ‐22
A)- B) -‐11 C)O D) 1

we get y =
First, we
First, equation and
z k from the first equation substitute this
and substitute into the
this into equation ,
second equation,
the second

kk = x2
x 2 -‐ 3x +1l
3x +

O=z x22-‐ 33xx + ((11-‐ kk))


O

If the system
If system of equations has no
equations has real solution,
no real then the
solution, then above should
equation above
the equation have no
should have real solution
no real The
solution.. The
discriminant should be
discriminant should than 0.
be less than

Discriminant: b2 -‐ 44ac
Discriminant = 172 (‐3)2
a c = (- 4(1)(1 -‐ k) = 9 ‐- 4 +
3)2 -‐4(1)(1 z 55 +
4k =
+4k 4k
+4k

Now
N o w we test the answer
each of the
test each answer choices to see
choices to see which one results
which one results in 5+
in 5 + 4k being negative.
4k being answer
2, answer
Only -~2,
negative. Only

~ produces a negative
produces a negative discriminant.
discriminant.

examples we
The examples we've done so
've done some of
showcase some
far showcase
so far the toughest
of the you might
questions you
toughest questions see on
might see SAT. Go
the SAT.
on the back
Go back
and make
and you understand
sure you
make sure them .
understand them.

154
THE COLLEGE
THE PANDA
COLLEGE PANDA

EXAMPLE biologist uses


EXAMPLE 4: A biologist uses the function p(
the function - 10Qn2 +
n) = ‐100n2
p(n) + 1, 000n to model
1,000n model the population seagulls
population of seagµlls
on aa beach where 1 ~
number n, where
beach in year number 5 n ::;
S 10. Which of the equivalent forms of p(n)
the following equivalent displays
p(n) displays
the maximum
the population of seagulls
maximum population seagulls and
and the number of the year in which
the number population reaches
which the population that
reaches that
maximum as
maximum as constants
constants or coefficients? ‘

A) p(n) = -‐4n(25n
A) 4n(25n -‐ 250) B) p(n)
B) 10(10n2 -‐ 100n)
p(n) = -‐10(10n2 10011) C) p(n) = -‐100(n
C) lOO(n-‐ 5)2 + 2,500
D) p(n) = -‐100(n
D) 7)22 +
lOO(n- 7) +4,900
4,900

Anytime you
Anytime you see
see a quadratics question that deals
quadratics question deals with maximum or minimum
with the maximum minimum of a function
function output
output (i.e.
(i.e.
y-value), either figure
the y-value), either figure out the vertex or look
look for vertex form. After all,
After the vertex
all, the where the
vertex is where maximum
the maximum
or minimum
minimum occurs fact, the
occurs.. In fact, the answer
answer is is either (C) or (D) because
either (C) because those only ones
are the only
those are form..
vertex form
ones in vertex
Furthermore, with a little
Furthermore, with little calculation,
calculation, it's easy to see that
it's easy that (D) does
does not expand to be
n o t expand the original
be the equation,, so
original equation so
the answer
the answer is (C).
However, for learning
However, purposes
learning purpo (and for the
ses (and tougher questions),
the tougher I’ll show
questions), I'll you how
show you how to do this question
do this question in ttwo
wo
different ways
different We can
ways.. We the vertex
can find the using the
vertex using average of the roots
the average roots and
and then reverse engineer
then reverse form..
vertex form
engineer vertex
we can
Or we can transform
transform the equation into
the equation into vertex
vertex form directly.
form directly.

Solution 1: To find the roots,


Solution we set
roots, we the equation
set the equal to O
equation equal 0 and
and factor,

100n 2 + 1,000n
-400,12 0
1,000n = 0
lO0n(n -‐ 10) =
-‐100n(n z 0
n=
: 0,10
0, 10

The roots
roots are
are O which means
and 10, which
0 and the x-coordinate
means the x-coordinate of the vertex Now we
vertex is 5. Now we can plug 5 into
can plug into p(n) to find
find
the y-coordinate..
they-coordinate
100(5) 2 +
: -‐100(5)2
p(5) = 1, 000(5) =
+1,000(5) : 2,500
2,500

So the
So the vertex (5,2500).
vertex is at (5, 2500). Now remember what
N o w remember what vertex looks like: y =
form looks
vertex form h)22 +
: a(x -‐ h) + k. Given
Given our values,
o u r values,
we have
have
p(n) = 5)22 +
= a(n -‐ 5) + 2,500
2,500
We now
n o w need what a is. To do that,
need to find what we need
that, we another point
need another point to work
work with it’s easy
with.. Well, it's that p(n)
easy to see that
passes through the
passes through the point (0, O).
point (0,0 Plugging that
). Plugging that in,
in,

o
0= a(0 -‐ 5)2 +
z a(O + 2,500
2,500
0= 2511 +
2 25a + 2,500
2,500
25a = 2,500
-‐25a 2,500
a == -‐100
100

Finally, p(n) = -‐100(n


Finally, 5)22 + 2,
lO0(n - 5) 2,500. [@[J
Answer (C) .
500. Answer .

155
CHAPTER 17
CHAPTER 17 QUADRATICS
QUADRATICS

Solution 2: This
Solution This second method involves
second method completing the
involves completing square to get the vertex
the square form directly.
vertex form First, divide
directly . First, divide
everything
everything by ‐100
- 100 to ensure coefficient of 1n122 is 1.
the coefficient
ensure the l.

= ‐100n2
p(n)) =
p(n + 1,00011
- 100n2 + 1,00011

POI)
p(n) =
‐‐_100 _
‐n n2_
2 - lOn
lOn
- 100

Do you remember what


you remember what to do next? If we wrote
next? If the constant
wrote the constant 0 at the end,
end, the ”middle" would be
term would
"middle" term lOn.
be -‐10n.
Divide 0 by 2 to get
the ‐- 110
Divide the get -‐55 and square that
and square get 25. The
that to get - 5 belongs
The ‐5 the parentheses
inside the
belong s inside with n and
parenthe ses with and the
25 gets subtracted
25 subtracted at the end.
at the end .
p01)
p(n) =
W _ (n
_ 25
5)2 ‐- 25
(n -‐ 5)2
- 100
N o w we
Now can multiply
we can multiply everything back by ‐100.
everything back - 100.

p(n) = ‐100(n 5)22 +


- l00 (n -‐ 5) + 2,500
2,500

And again,
And again, we prove
prove that answer is (C) .
the answer
that the .
[@IJ

156
THE COLLEGE
THE PANDA
COLLEGE PANDA

Review:
Review:
Given aaquadratic
Given quadratic of the form, y =
the form, ax2 + bx
= ax2 bx++ c,c,
The also called
roots, also
The roots, solutions and
called solutions and x-intercepts,
x‐intercepts, can
can be
be found the following
found in the ways:
following ways:
0 Factoring
• Factoring
0
• Graph on the
Graph on calculator (look
the calculator for the
(look for x‐intercepts)
the x-intercepts)
-‐ bb± ✓bi2 - 4ac
m
o The quadratic
• The formula x =
quadratic formula 2a
211

Sum of the
Sum = - -ab
Roots = ‐5
the Roots

C
Product of the
Product the Roots = -2
Roots =
a

The discriminant D =
ThediscriminantD =bb2‐4ac
2 - 4ac

0 When D >
• When > 0,
0, there
there are
are ttwo real solutions.
w o real solutions.
When D = 0,
•0 When there is one
0, there real solution.
one real solution.
0 When D < 0,
• When 0, there no real
are no
there are real solutions.
solutions.

find the
To find vertex,
the vertex,
0
• Take
Take the average of the
the average roots to get
the roots get the x-coordinate. Then
the x-coordinate. plug that
Then plug value into
that value the quadratic
into the quadratic to get
to get
y-coordinate.
the y-coordinate.
the
•0 Put
Put the quadratic in vertex
the quadratic by completing
form by
vertex form square .
completing the square.
2 is positive 1 by dividing everything by a.
the coefficient
Ensure the
1. Ensure coefficient of xx2 positive 1by dividing everything by

-a = X + -E
y =x2+ b
2
a
x+
x +-
C
a

b b2 b
Divide the
2. Divide
2. the middle
coefficient of the
the coefficient term bx to getZa
middle term 2a. Square
get 2. Square that result to
that result g e4a1,22
to get t 2‐ .. PPutu 2a
2a
t ‐

inside
inside the parentheses with
the parentheses and subtract
with x and ! 2
subtract £72 at the end.
the end.

z_(.+£)2+£_i
a
a_ 2a
+:. - ~
(x+ _!:_)2
¥_=
a 4a
211 a 4:122

3. Multiply
3. everythingby
Multiply everything by a.
b 2
2
b2
2
yy=a(x+‐‐2) c ‐ ‐b ‑
= a ( x +-b ) + c--
2a 4a

4. IIt's
t ’s unnecessary
unnecessary to memorize
memorize these
these steps
steps with variables. Practice
the variables.
with the on quadratics
Practice on actual
with actual
quadratics with
numbers. remember what
do remember
However, do
numbers. However, vertex form looks like: y = a(x - h )2
what vertex form looks like: y : a(x‐- h)2 + k. + k.

Whenever you're
Whenever you're asked the minimum
asked for the or the
minimum or maximum of aquadratic,
the maximum the vertex.
find the
a quadratic, find vertex.

157
CHAPTER 17 QUADRATICS
CHAPTER 17 QUADRATICS

CHAPTER EXERCISE: Answers for this chapter start on page 314.

A calculator
calculator should
should N O T be
NOT used on the
be used the
following questions.
following questions.
3x2+10x=8
3x 2 + lOx = 8
If
If a and
and b are
are the w o solutions
the ttwo the equation
solutions to the equation
In the
the xy-plane,
xy-plane, whatwhat is the distance between
the distance between the
the above and
above > b, what
and a > what is the b2 ?
value of If2
the value ?
w o x-intercepts
ttwo x-intercepts of thethe parabola
parabola 4
yy = x2x2 ‐- 3x
3x ‐- 10
10?? A) 9

A) 3 2
B) 3
B) 5
C) 4
C)
C) 7
D)
D) 16
16
D
D)) 110
0

Whatare solutions tox2+4x+2=0?


the solutions
What are the to x 2 + 4x + 2 = 0 ?
fJ(x) =m[(x‐m)2‐l]
(x) = m [(x - m) 2 - 1]
In the hmction ff defined
the function above, m
defined above, a positive
m is a positive
A = z- 2‐ ±Z vf2
A) ) x x i fi constant. The graph
constant. The the xy-plane
offf in the
graph of xy-plane is a
B)) x x= 2z±e2v12
B n/i parabola.
parabola . Which the following
Whkh of the statements
following statements
C
C) ) Xx=-
= ‐ 22 i±22y12
\/§ the parabola
about the
about parabola is true?
true?
D
D)) xx== ‐- 44 i±Z2,/2
\/§ A) [Its
t s minimum at (m,
occurs at
minimum occurs ( m, ‐- 11).
).
B) Its minimum
B) Its at (m, ‐m).
occurs at
minimum occurs - m).
C)
C) Its
Its maximum at (m,
occurs at
maximum occurs (nz, ‐- 1).
1).
D) Its
D) Its maximum at (m,
occurs at
maximum occurs (m, -‐ m ).
m).
lIff a < 1l and
and 2a2 7a + 3 =O,whatisthevalue
2a 2 ‐- 7a+3 = 0, what is the value
o
off a a??

yy = ‐- 33
yzx2+cx
2
y = x + ex

In system of equations
the system
ln the above, 0is
equations above, c is a
a constant.
constant.
which of the
For which
For the following values of c does
following values does the
the
system of equations
system have exactly
equations have w o real
exactly ttwo real
solutions?
solutions?
What
What is the u m of the
the ssum the solutions
solutions of A)) -‐ 4
A
(2x‐3)2
(2x - 3) = 4x + 5?
2 =4x+5?
B) 1
C) 2
D) 3

158
THE COLLEGE
THE COLLEGE PANDA
PANDA

A calculator is allowed on the following


questions. P= m 2 ‐- 100m -‐ 120,000
= m2 120,000
The monthly
The monthly profit mattress company
profit of a mattress company can can
be
be modeled
modeled by by the
the equation
equation above, where P
above, where Pisis
the following
which of the
At which points does
following points line
the line
does the the profit,
the profit, in dollars, and m is the
dollars, and number of
the number
with equation y =
with equation z 4 intersect the parabola
intersect the parabola mattresses
mattresses sold. What is the
sold. What minimum number
the minimum number
y = (x + 2)2
2)2 -‐ 55 in the
the xy-plane?
xy-plane? the company
mattresses the
of mattresses company must sell in a
must sell given
a given
A) (-
A) (‐1,4) ( ‐ 55,, 4 )
and (-
1,4 ) and month so that it does not lose money during
month so that does not lose money during that that
month?
month?
B) (1,4
B) (1,4)) and (‐5,4)
and (- 5,4 )
C) (1,4)
C) (1,4 ) and (5,4)
and (5, 4)
D) (-
D) (‐11,4) (7,4))
and (7,4
11, 4) and

E(x) =
E(x) 50x22 ‐- 8003:
= 50x B00x ++ 10,000
10,000
The function
The function above
above models the relationship
models the relationship
between the total
between the monthly expenses
total monthly expenses E, in dollars,
dollars,
restaurant and
of a restaurant and the number of tables
the number tables x in its
its
dining area, where
dining area, O5 x::;
where O::; x 5 25. What does the
What does the
number represent in the
number 10,000 represent the function?
function?
(3, -8)
The maximum
A) The number of tables
maximum number tables that can fit
that can
in the
the dining
dining area
area
Which of the following equation s represents the
B) The
The average monthly expenses,
average monthly expenses, in dollars,
dollars,
parabola shown in the xy-plane above?
for each table in the
each table dining room
the dining room
A) y = (x - 3)2 - 8 C) The total monthly
The total monthly expenses,
expenses, in dollars
dollars,,
8) y =(x+ 3)2+ 8 when
when there zero tables
are zero
there are the dining
tables in the dining
C) y = 2(x - 3)2 - 8 area
area
D) y= 2(x+ 3)2- 8 total monthly
The total
D) The monthl y expenses,
expenses , in dollars,
dollars,
when the
when the number
number of tables the dining
tables in the dining
area maximized
area is maximized

For
For what
what value of t does
value oft equation vv = St
the equation
does the 2
St -‐ tt2
result in the
result value of v ?
maximum value
the maximum
f (x) = - x 2 + 6x + 20
The
The function above.. Which
defined above
function f is defined the
Which of the
following equivalent
following forms of f (x)
equivalent forms displays the
(x ) displays the
maximum value of f as
maximum value as a constant or coefficient?
a constant coefficient?
A
A)) ff (x)
( x )=: ‐-(( xx‐-3 3)2
) 2 ++ 11
11
B ) f ( x ) : ‐ ( x ‐
B) f (x) = -(x - 3) + 293 ) 22+ 2 9
C) f(x)=‐(x+3)2+11
C) f(x) = - (x + 3)2 + 11
D)
D) ff(x):‐(x+3)2+z9
(x)=-(x+ 3)2+ 29

159
CHAPTER 17 QUADRATICS
CHAPTER 17 QUADRATICS

yy =
= ‐- 33 g(x) = - 3x 2 + 18x
y=ax2+4x‐4
y = ax2 + 4x - 4 The function
The function 3
g above gives the
above gives the data
data transfer
transfer
speed, in megabytes
speed, megabytes per over a
second, over
per second, network
a network
In the system of equations
the system above, a
equations above, a is aa constant.
constant. connection minutes after
connection x minutes after a file transfer
transfer was
was
which of the
For which
For the following values of a
following values does the
a does the initiated. The graph
initiated. The graph of y = = g(x) the xy-plane
g(x) in the xy-plane
system of equations
system have exactly
equations have one real
exactly one real has x-intercepts
has at x = 0
x-intercepts at and x = c.
0 and c. Which
Which ofof the
the
solution?
solution? following
following is the best interpretation
the best interpretation of the value
the value
cC
_7
A)) -‐ 4
A of 2 ?.
2
B
B)) -‐ 2
initial data
The initial
A) The transfer speed
data transfer over the
speed over the
C) 2 network
network connection
connection
D
D)) 4 B) The
The maximum data transfer
maximum data speed over the
over the
B) transfer speed
network
network connection
connection
C) The time at which
The time which the
the data transfer speed
data transfer speed
the network
over the
over network connection was at its
connection was
f ( x ) = x 2‐ 24x +180 highest
highest
D) The time at which
The time the file transfer
which the transfer
For a
For a manufacturer x‐ray machines
manufacturer of x-ray machines,, the
the cost
completed
completed
per unit, thousands of dollars,
per unit, in thousands dollars , can be modeled
can be modeled
by the
the function above, where
function f above, the weekly
where x is the weekly
number of units
number units produced. How many
produced. How many units
units
should
should the manufacturer produce
the manufacturer produce each week to
each week
minimize the
minimize the cost per unit?
cost per unit? y = a(x - 3)(x - k)
In the quadratic equation
the quadratic above, a
equation above, and k are
a and are
If the
constants. If
constants. graph of the
the graph the equation
equation in the
the
xy-plane
xy-plane is a
a parabola
parabola with
with vertex (5, -‐32),
vertex (5, 32), what
what
is the
the value
value of a ?
?
A) 2
B) 5
C) 6
D) 8
J(x) : -‐4x2
f(x) = 4x 2 + 2 x
22x
The function ff above
The function above gives the data
gives the data transfer
transfer
speed, megabytes
speed, in megabytes per
per second, over a
second, over a network
network
connection minutes after
connection x minutes a file transfer
after a transfer was
was
initiated.
initiated . The f(x) in the
graph of y = J(x)
The graph xy-plane
the xy-plane the xy-plane,
In the xy-plane , the line y =
the line = 2x + intersects the
+ b intersects the
has
has x-intercepts at x =
x-intercepts at and x =
= 0 and = b. Which
Which ofof the
the parabola
parabola y = = x2
x 2 + bx + 5 5 at the point (3, k). If b
at the point (3,k). If b
following the best
following is the best interpretation
interpretation of b b?? constant, what
is a constant, what the value
is the value of k ?
A) The
The initial
initial data transfer speed
data transfer speed over the
over the A) 0
network
network connection
connection B) 1
B) The maximum data
The maximum data transfer speed over
transfer speed the
over the C) 2
network
network connection
connection D) 3
C) The time at
The time at which the data
which the transfer speed
data transfer speed
over the network
over the network connection
connection w its
a s at its
was
highest
highest
The time
D) The time at which the
at which the file transfer
transfer
completed
completed

160
Synthetic
Synthetic Division
Division
18 •
e

Synthetic division involves


Synthetic division dividing one
involves dividing polynomial by another
one polynomial the same
another in the way you
same way divided numbers
you divided in 3rd
numbers in 3rd
grade.
grade .

1 8 R 2 xx 2 2++ 3x
3 x 7 22 R ‐- 1]
R
3156
3 5 6 xv1|Fx3+2x2~5x+1
x - 1 Ix 3
+ 2x2 5x + 1

l’ll teach you


I'll teach long “mathematical”
you the long "mathematical" way way first, but
but then direct you
then direct towards several
you towards shortcuts that
several shortcuts will get
that will get
through almost
you through
you synthetic division
any synthetic
almost any question on
division question without using
on the SAT without using the long way. These
long way. questions
These questions
rarely and if they do,
up, and
show up,
rarely show they’ll show
do, they'll show upup only
only once
once..
Let’s retrace
Let's retrace the steps 56by
dividing 56
steps of dividing by 33 so
so you how the same
can see how
you can applies to
same logic applies synthetic division.
to synthetic division.
First, we see that
First, into 5 once.
that 3 goes into on top
put aa 1“1on
once . We put and a 1 x 3 =
top and z 3 below then subtract
below the 5. We then subtract to get 22
to get
and bring
and bring the 6 dOWn.
down.
l
3156
6
3
26
26

how many
N o w how
Now times does
many times does 3 go into and a 3 x 8 =
put an 8 up top and
times.. So we put
into 26? 8 times 24 below
z 24 the 26.
below the 26.
Subtracting,
Subtracting, we get 2.
l 8
3156
3
2 6
2 4
- 2-

point, there
At this point,
At digitss to bring
m o r e digit
are no more
there are bring down
down and does nnot
and 3 does o t go into n t o 56
Therefore,, 3 goes iinto
into 2. Therefore eighteen
56 eighteen
times
times w i
with t h a remainder
remainder of two
t w o . This result
result can
can be
be written
written in the following
following form:

56 = 18~
3 3

where 18
where 18is the quotient,
is the remainder,, and
quotient, 2 is the remainder divisor .
and 3 is the divisor.

161
CHAPTER
CHAPTER 18
18 SYNTHETIC
SYNTHETIC DIVISION
DIVISION

process of dividing
The process
The dividing a a polynomial
polynomial is essentially
essentially the same. To
the same. you how
show you
To show synthetic division
how synthetic division works,
works, let’s
let's
divide xx3+
divide 2x22 -‐ Sx
3 + 2x 5x + ll by
by x -‐ l.1.
H many times
o w many
How does x -‐ 1
times does l go into 3? xx22 times.
into xx3? times. Why?
Why? Because x2 =
Because xx xx x2 x3. The
= x3. The goal
goal is
is to
to match 3. We
match xx3. We
don’t care about
don ' t care the -‐11 during
about the this ”fitting
during this Now, (x
step. Now,
i n " step.
"fitting in" (x ‐- 1)
1) xx x2
x2 = xx3‐
3- x This is
2 . This
x2. is what we put
what we put below
below
the dividend.
dividend.

xx22
xx -‐ l1x
j x3
3 ++ 22xx2 2- ‐ 5Sxx ++ 11
xx33 ‐ x x2
2

Finally,
Finally, we subtract like we
we subtract do in basic
we do number division.
basic number division. Notice that we
Notice that we must subtract each
must subtract each element, so the -~x2
element, sothe x2
becomes +
becomes +x2,
2
x , yielding 2
yielding 3x2.
3x . Unlike long division
Unlike in long division with numbers, all the remaining
with numbers, terms from the
remaining terms dividend
the dividend
should be brought
should be down for each
brought down step in synthetic
each step synthetic division.
division.

x2
x2
xX -‐ l11 ++ 22xx22- ‐ 5Sxx ++ 11
xx33
x 3 ‐ x x2
x3 2
3 x2 2- ‐ Sx
3x 5 x++ 11
Next step. 2 ? 3x o u r goal
Next step. H o w many
How many times into 3x
does x -‐ 1 go into
times does 3x2? 3x times.
time s. Remember
Remember our each step
goal at each step is to get
the same
the exponent and
same exponent and the
the same coefficient as
same coefficient as the
the term with the highest
term with power . We put
highest power. put the +3x up
the +3x up top and
top and
3x 3x 2 -‐ 3x
3x x (x -‐ 1) = 3x2 3x on the bottom.
on the bottom .

x 2++ 33xx
x2

xx -‐ 1
l jxx33 ++ 22xx2 2- ‐ 5
Sx x +
+ 11
xx33 -‐ x x2
2
3 x2 2 ‐ Sx
3x 5 x++ 11
3 x2 2 ‐ 33xx
3x

And
And just like last subtract each
time, we subtract
last time, each term,
term, nnot just the
o t just then bring
the first. We then down the
bring down 1.
the 1.

x 2 ++ 33xx
x2
x ‐ 111
X - xx33 ++ 22xx2 2- ‐ 5
Sx x +
+ 11
x 3 ‐ xx2
x3 2
3 x2 2- ‐ Sx
3x 5 x++ 11
3x 2
3x2‐3x - 3x
-‐ 22xx ++ 11

162
THE COLLEGE PANDA
THE COLLEGE PANDA

We're almost
We’re almost done.
done. How How many
many times does xx -‐ l1 go
times does go into
into -‐ 22xx + 1? -‐22 times
+ 1? times.. So
So a - 2 goes
a ‐2 up top
goes up and
top and
1) =
-‐22 x (x -‐ 1) = -‐ 22xx + 2 goes
goes on bottom.
the bottom.
on the

x 2+ + 3x
x2 3 x ‐ 22
xx -‐ 1 x33 ++ 22xx2 2 ‐ 5
l lx 5xx ++ 1
1
‘%
xr :1 ‐ x2
x2

3
3xx
2 2 ‐ 5x
5 x++ 11
3x 2
3x2‐3x
3x
‐ 22xx ++ 1
1
‐ 22xx ++ 2
Subtracting,
Subtracting, we get ‐1
we get - 1 at
at the
the end.
end .

x2 + 3x 2
X -
3
1 1x + 2x 2 5x + 1
3 : 3 ‐ xx2
x3 2
3
3x x
2 2 ‐5x
5 x+ + 11
3x 2
3x2‐3x
3x
‐ 22xx ++ 1
‐ 22xx ++ 2
-‐11

. . 56
56 2 . .
know we're
We know we’re done when we end
done when constant. And
With a constant.
end up with And Just
just as
as we can express ‐3‐ as
can express 185,
as 18~, fraction,
m1xed fraction,
aa mixed
we can express
we can express 3

x33++22xx 22_- 5x + 1 as x2+3x‐2‐


5 x+ 2 11
as X + 3x - 2 - --
x -‐ 1l x -‐ 1l
Notice where
Notice each component
where each placed. The
component is placed. The quotient
quotient is written in front.
u t in
written oout front. The remainder, -‐ 11,, is
The remainder, the
is the
numerator of the
numerator fraction and the divisor, x ‐ 1, is the denominator. These placements are exactly the
the fraction and the divisor, - l, is the denominator. These placements are exactly the same as same as
in long
long division
division with actual numbers.
with actual Get used
numbers . Get to seeing
used to seeing synthetic
synthetic division
division results in this
results in this format.
format.
another thing
Here’s another
Here's that’s the
thing that's the same. The result
same. The of our
result of long division
o u r long division with numbers
with numbers

1 8 R 2
3156
3 5 6
means that
means = 3 x 18
56 =
that 56 18 + 2.
The same
The meaning applies
same meaning applies to synthetic division
our synthetic
to our division result.
result .

x3+2x2‐5x+1=(x‐1)(x2+3x‐2)‐1
x 3 + 2x 2 - 5x + 1 = (x - l ) (x 2 + 3x - 2) - 1
Dividend =
Dividend Quotient x Divisor
= Quotient Divisor + Remainder
Remainder

Hopefully you've
Hopefully you've been
been able to grasp
able to synthetic division
grasp synthetic division mmore intuitively through
o r e intuitively the comparison
through the with regular
comparison with regular
long
long division.
division. A
Alll l the parts relate
the parts relate to each other in
each other in the same way.
the same Let's dive
way. Let's dive into more examples
some more
into some where
examples where
we
we can show you
can show some shortcuts.
you some shortcuts.

163
18 SYNTHETIC
CHAPTER 18
CHAPTER SYNTHETIC DIVISION
DIVISION

6 5
EXAMPLE 1: The
EXAMPLE expression (2:25
The expression equivalent to which
x - is equivalent which of the
the following?
following?
x+ 2
17 7 6 -‐ 5
C) 6-5 5
_2_
A’6“m
A)6 -~
x+2
B)6+
B)6+x+2
x+2 07 2 9’6‘52
0)6 - -

Using synthetic division,


Using synthetic division,

6
x
X + 22 I6x
+ 6x ‐ 5
6x +
6x + 12
‐ 17
17

The quotient is 6 and


The quotient and the remainder is -‐ l17.
the remainder We can
7 . We result as6
this result
write this
can write xl‐ZZ'-An
as 6 -‐ ~ swer~-(A) .
Answer
x +2 ~
how would
N o w how
Now we approach
would we approach this
this question without using
question without synthetic division?
using synthetic division?

. 6
6xx-‐ 5 6 ( 2 )- ‐ 5
6(2) 77
w e ccan
We a nplug
p1u g in
l n numbers t hat we
n u mb ers that m ak eup
w e make Let ’ssayx
up.. Let's ' he n --x + 2 = ---2 + 2 = ‐-4 .
say x == 22.. ' IThen
x+2 2+ 2 4
7
look for an
n o w look
We now an answer choice that
answer choice when x =
gives 2 when
that gives z 2. We can rule out
can rule out (C) and right away
and (D) right since they
away since they
4
don't give
don't i·
Plugging x =
give 2. Plugging z 2 into gives
answer (A) gives
into answer

_17_617 2417 7
x+2 _ 4 4 4 4

This confirms
This that the
confirms that the answer indeed (A)
answer is indeed strategy of making
This strategy
(A).. This up numbers
making up and testing
numbers and testing each
each answer
answer
choice can be
choice can much faster
be much faster than synthetic division.
than synthetic division.

7
EXAMPLBthensz+4isdividedbyx‐1,theresultisA+;‐_7:‐1.WhatisAintermsofx?
EXAMPLE 2: When 3x 2 + 4 is divided by x - 1, the result is A + -- . What is A in terms of x ?
x- 1
A ) 3 x-‐ 4
A)3x B)3x‐3
B)3x-3 C ) 3 x++ 3
C)3x D)3x+4
D)3x+4

Using synthetic division,


Using synthetic division,

3x + 3
x
X -‐ 1 I3x
3x22 + 4
3x22 -‐ 3x
3x
3x + 4
3x ‐ 3
7

you followed
If you along, you
followed along, you should've noticed it got
should’ve noticed little clunky
got aa little when we subtracted
clunky when subtracted the and brought
the -‐ 33xx and brought
the 4 down.
the down. That's because the
That’s because the dividend, 3x22 +
dividend, 3x + 4, has no
4, has term. Still, the
no x term. process is the
the process same: subtract
the same: subtract and
and
bring the
bring remaining terms
the remaining down.
terms down.

164
THE COLLEGE PANDA
THE COLLEGE PANDA

quotient is 3x
The quotient
The 3x ++ 3 and the remainder
and the 7. The
remainder is 7. result can
The result be expressed
can be as 33x
expressed as
2
x +
4 : 3x + + 7
+14
x ‐- 1
= 3x + 3 + _ __ Now
x -‑ 1
L1.
Now

it's easy to see


it’s easy that A =
see that 3x +
= 3x + 3, answer ~-(C) .
3, answer

Again, we
Again, we could've
could’ve done
done thi question by
thiss question making up
by making up numbers. = 2,
lf x =
numbers. If 2, then
then

3x22 +
3x +44 _ 3(2)
3(2)2+4
2
+4
- xx---‐ 1
1- = - 2
2---‐ 1
1- = 16
216

If we
If we didn't know the
didn’t know the answer
answer was we would
(C), we
was (C), test each
would test answer choice
each answer with xx = 22 until
choice with until we
we got 16, but
got 16, since
but since
we do
we do know, we’ll test
know, we'll (C) first for confirmation.
test (C) Letting A =
confirmation. Letting 3x +
= 3x + 3,
3,

7 7
3X+3+m
+ 3 + --
3x
x- 1
‐3(2)+3+m
= 3(2) + 3 + --
2- 1
~ ‐99++77=: 116
= 6

confirmed.
Answer confirmed.
Answer

2 4 1
EXAMPLEB:Hfl\eexpressim-5’:2‐Jt_‐__4ii‐liswritteniritheform5x+6+‐B-,whereBisaconstant,
EXAMPLE 3: If the expression Sx - ~ + is written in the form Sx + 6 + __!!_, where B is a constant,
x- 2 xx-‐ 2
whatisthe
what value o
is the value off B ?

Based on where
Based on where it is, represents the
is, B represents remainder of the
the remainder the division.
division.

5 x+
5x + 6
x -‐
X 2 I5x
5x2‐ 4x
2
4x +1
+ 1
5x2‐10x
5x 2
lOx
6x x ++ 1
6
6x ‐ 12
13

13
13 r:;---:;-i
Wecan
We write the
can write result of this
the result this division
division as Sx + 6 + --
as5x from which
x _ 2,, from = L.llJ.
which B = -.
x- 2
last example
This last
This perfect for demonstrating
example is perfect demonstrating aa shortcut
shortcut called
called the remainder theorem, which
remainder theorem, allows us
which allows us to get
get
the remainder without
the remainder without going through synthetic
going through synthetic division.
division.
In Example
1n 3, we
Example 3, we divided
divided Sx 4x +
5x22 -‐ 4x + 11by
by xx -‐ 2.
2. Whenever
Whenever a a polynomial
polynomial is divided by a
divided by a monomial, which is
monomial, which is
something in the
just something
just ax + b,
form of ax
the form b, the remainder can
the remainder be found
can be found by plugging in to the
by plugging the polynomial
polynomial the value
the va lue
of xx that
that makes
makes the
the monomial equal to
monomial equal to 0. The process sounds
The process m o r e complicated
sounds more than it is,
complicated than so let’s
is, so show how
let's show how
done..
it’s done
it's
What makes x -‐ 2 equal
What makes 0? x =
equal to O? z 2.
Plug xx = 22 into
Plug the polynomial
into the 5x22 -‐ 4x
polynomial 5x 4x + 1.
1.

5(2)22 -‐ 4(2) +
5(2) +11 = 13
13

A the remainder
that’s the
n d that's
And remainder we
we obtained Example 3.
obtained in Example

165
CHAPTER 18
CHAPTER 18 SYNTHETIC DIVISION
SYNTHETIC DIVISION

What
What is the
the remainder - 2x 2 +
when ‐23(2
remainder when + 5x is divided + 1?
divided by x + l?
makes x +
what makes
Well, what + l1equal
equal to zero? x = -‐ 11.. Plugging
to zero? Plugging that the polynomial,
into the
that into polynomial,

‐2(‐1)2+5(‐1)
- 2(- 1) 2 + 5(- 1) = -‐7
7

Boom. ‐7
Boom. - 7 is the remainder..
the remainder
What is the
What the remainder 4x 4 +
when 4x4
remainder when 3x22 -‐ 4 is divided
+ 3x divided by 2x
2x -‐ 1?
l?
1
What makes 2x -‐ 1 equal
What makes equal to zero? z 2'.
zero? x =
2
Plugging
Plugging that
that into the polynomial,
into the polynomial,

2 - 4 = 41+ 43- 4 = - 3
(21) + 3 (1)
4 2
4

Boom . -‐33 is the


Boom. the remainder.
remainder.

A
,,.: .
.· 2x2‐5x+1. .· m
2x2x- _Sx + 1 IS· wntten ·. th e equiv
.· alent form 2x + 1 + -x-R
EXAMPLEdzlftheexpreesmn‐‐;‐:§‐‐1swnttenmtheeqmvalentform2x+l+m,whatrsfire
uv"' 11.•PLE If th e expression
R .
__- , what is the
J;,.IUUT-1,
3 3
valueofR?
value of R?

R represents the remainder


represents the after dividing
remainder after 2x22 -‐ 5x + 11by
dividing 2x by x -‐ 3. Using the remainder
Using the theorem, we
remainder theorem, can plug
we can plug in
x = 3 into 2x -‐ 5x +
2
into 2x2 + 1 to get
get the remainder.
the remainder.

2(3)2‐5(3)+1=18‐15+1=.
2(3) 2 - 5(3) + 1 = 18 - 15 + 1 = [±J

No need for synthetic


No need synthetic division.
division .

last thing
One last
One the remainder
about the
thing about remainder theorem. that we
say that
Let’s say
theorem . Let's we divide x 2 ‐- 3x + 2 by x ‐- 2.
divide x2 2. Plugging 2, we
Plugging in 2, we
the remainder
that the
see that
see remainder is
(2) (2) +
(2)22 -‐ 33(2) +2 2z
=0
Since the
Since the remainder x 2 -‐ 3x + 2, just
remainder is 0, x ‐- 2 is a factor of x2 just like factor of 18. And
a factor
like 3 is a indeed, if we
And indeed, factor
we factor
x2
2 ‐ 3x + 2,
x - 3x + 2,
(x -‐ 2)(X
(x 1)
2)( x ‐- 1)
see that
we see that x -‐ 2 is in fact a factor.
you just
Don't you
Don’t just love how everything
love how everything in math
math is connected?
connected?
Some questions
Some become much
o w become
questions nnow easier . For example,
much easier. + 1a
example, is x + 1 a factor x 3 + 1?
factor of x3 1?
Well, plugging that the remainder
plugging in -‐ 11,, we find that = 0. Therefore,
- 1) 3 + 1 =
remainder is ((‐1)3 3+
factor of xx3
Therefore, x + 1 is a factor 1.
Do note
note that the remainder
that the theorem only
remainder theorem only works we’re dividing
when we're
works when like x + 1.
monomials like
dividing by monomials If we
1. If were
we were
dividing
dividing x3 + 11by
x3 + by something x2 + 2, we
like x2
something like we would use synthetic
have to use
would have synthetic division.
division .

166
THE COLLEGE PANDA
THE COLLEGE PANDA

EXAMPLE 5:
EXAMPLES:
f(x) = 3x3
f(x) = kx2 +
3x 3 ‐- kx2 +5Sxx++ 2
the polynomialf
In ·the (x) defined
polynomial f(x) above, kit is
defined above, is a
a constant. (x) is divisible
constant. If ff(x) divisible by
by x ‐- 2,
2, what value of k
the value
what is the k??

A) 12
12 B) 9
B)9 C)
C)66 D)
0)33

If f (x) is divisible
If f(x) divisible by x -‐ 2,
2, then
then the remainder is 0
the remainder (x) is divided
when ff(x)
O when divided by x ‐- 2.
2. In other words, x ‐- 2 is a
other words, a
factor of f ((x).
factor r ) The
The remainder theorem tells us
remainder theorem us that we plug
when we
that when plug 2 (the
(the value makes x ‐- 2 equal
that makes
va lue that equal to zero)
zero)
into (x),
into fJ(x ), we
we should get 0.
should get

2 ) = 00
f/ ((2)
3(2)33
3(2) ‐k(2)2+5(2)
- k(2) 2 + 5(2) +2+ 2 := o
0
2 4- ‐ 4
24 1 0++22=: 0
4kk++ 10
36 -‐ 4k =
36 =0
‐- 44kk := ‐- 336
6
k= 9

Answer ~(B) .
Answer

EXAMPLE 6:
EXAMPLE
X p(x)
-3 1
- 1 0
0 5
2 -3
4 4

The table
The above gives
table above the value
gives the polynomial p(x)
value of polynomial p(x) for some values of x. Which
some values Which of the following must
the following be
must be
a factor of p(x)
p(x)??

A ) x ++ l1
A)x B ) x- ‐ 1
B)x ) x-‐ 4
C)x
C D ) x ‐- ‐ 5
D)x

The remainder
The theorem makes
remainder theorem question easy.
this question
make s this easy. Because p((-‐ 1)
Because p = 0, x ++ 1 must
l) = must be
be a
a factor p(x). Answer
factor of p(x). Answer
~-(A) . Becareful‐the answer is
Be careful-the answer NOT x -‐‐ 1.
is NOT 1.

167
18 SYNTHETIC
CHAPTER 18
CHAPTER SYNTHETIC DNISION
DIVISION

CHAPTER EXERCISE:Answers for this chapter start on page 317.

A calculator should
should NOT be used
N O T be the
used on the
following questions.
following questions.
X g(x)
-3 2
4 -2 3
The expression
The expression x4_x equall to which
x 2 is equa the
which of the
x- 2
0 -4
following?
following?
1 -3
A ) -‐ 2
A)
3 6
8
B) ---+ 4
x- 2 function g is
The function
The defined by
is defined by aa polynomial.
polynomial. The
The
8 table
table above
above shows some values
shows some and g(x).
va lu es of x and g(x).
C ‐‐
C) --+ 4 What is the
What the remainder when g(x)
remainder when g(x) is divided by
divided by
) x -‐ 2 +
xX + 33??
D) 44 ‐‐
- 2x
2x
A
A)) -‐ 2
B) 1
C) 2
2
. 6x + 5x + 2 . . . th D) 6
IIff the expression M x +
th e expression is wntten
1s in thee
written m
22x + 11
__1
form _2 xl_+ 1 +
form
2x + 1
' '
+Q,whatisQ1ntermsofx.
Q, what is Q in terms of x ? 7
A ) 33x
A) x -- 1 223‐kxzz+5xz+2x‐2
2z3 - kxz2 + 5xz + 2x - 2
B) 3x + 1
the polynomial
In the above, kk is
polynomial above, is aa constant.
constan t. If 2
z -‐ 11
C) 6x2+3x+1
C) 6x 2 + 3x + 1
is
is a factor of the
a factor the polynomial above, what
polynomial above, what isis the
the
D)
D) 6x2+5x+1
6x 2 + 5x + 1 value of kk??
value

expression 4x
The expression
The 4x22 + 55 can be written
can be as
written as
A(2x ‐
A(2x - 1) + R, where A anexpression in terms
R, where A is an expression terms
and R
of x and R is aa constant.
constant. What
What is the
the va lu e of R
value R ??

What th e remainder
What is the remainder when x 2 + 2x
when x2 2x + 1 is
is
divided by xx ++44 ??
divided by

168
THE COLLEGE
THE COLLEGE PANDA
PAN DA

allowed on the following


calculatorr is allowed
A calculato following
questions.
question s. p(x) =
If p(x) 3 + x2
= xx3 x2 ‐- 5x + 3, then p(x)
3, then p(x) is divisible
is divisible
by which of the
by which the following?
following?
I. xX -‐ 2
3x22 -‐ 8x
When 3x
When Bx -‐ 4 is
is divided
divided by
by 3x -‐ 2,
2, the
the II. x ‐- l1
II.
I]. x +
x ~ . What
IIII. + 33
can be
result can
result expressed as
be expressed asAA -‐ £133
3 2
A
What is A
A) I and 11only
and II only
intermsofx?
in terms of x ?
B) I and III only
and 11] only
A ) xx-‐ 4
A) C)
C) [I
II and l l only
and IIII only
B)) xx-‐ 2
B D)
D) I, II, and
I, II, III
and III
C ) xx ++ 2
C)
D ) xx ++ 4
D)

If the polynomia l p(x)


the polynomial p(x) is divisible by
is divisible by x ‐- 2,
2,
which of the
which following could
the following could bebe p(x)
p(x)??
expression 2x
The expression
The 2x22 -‐ 4x
4x -‐ 33 can be written
can be as
written as A) p(x))= - x 2 + 5x - 14
A (x +
A(x + l1)) +
+ B, where
where Bis constant. What
B is a constant. What is A
B) p(x)=x2~6x‐2
B) p(x) = x 2 - 6x - 2
in terms
terms of x ?
C) p(x)=zx2+x‐8
C) p(x) = 2x 2 + x - 8
A) 2x
A) +6
2x +
D) p(x)=3x2‐2x‐-8
D) p(x)= 3x2 - 2x - 8
B) 2x
2x + 2
C ) 22xx -‐ 2
C)
D ) 22xx -‐ 66
D)

If and x +
If x ‐- 1 and are both
+ 1 are both factors
factors of the
the
polynomial
po lynomial axax4 + bx3
4 3 3x2 + 5x
bx -‐ 3x 2
5x and
and a and b are
and bare
constants, what
constants, the value
what is the value of a ?7
The expression xx22 + 4x
The expression 4x -‐ 9 can be written
can be as
written as A ) -‐ 3
A)
(ax+ b)(x -‐ 2) +
(ax + b)(x + c, where
where a, b, and
a,b, c are
and care
B) 1
constants. What is the
constants. What value of a +
the value + b+
+ c?
C) 3
A ) -‐ 2
A)
D
D)) 5
B) 3
C) 7
D
D)) 1100

For a
For a polynomial
polynomial p(x),
p(x), p (D= 0.0. Which
(5) of the
Which of the

following must be
following must be a factor of p(x)
a factor p(x)??
For a
For a polynomial
polynomial p(x) (2) = 0.
p(x),, pp(2) 0. Which of the
Which of the A) 3x ‐- 1
l
following m u s t be
folJowing must true about
be true p(x) ??
about p(x) B 3xx +
B)) 3 + 1l
A) 2x is a
a factor p(x).
factor of p(x). C) x - 3
B) 2x -‐ 2 is aa factor p(x).
factor of p(x). D) x + 3
C) x -‐ 2 is a
a factor p(x).
factor of p(x).
D) x +
+ 2 is aa factor p(x).
factor of p(x).

169
ComplexNumbers
Complex
2 =
Numbers
19 the imaginary
What value of x satisfies
What value satisfies xx2 : -v1? There were
1? There values until
no values
w e r e no mathematicians invented
until mathematicians invented the imaginary
number i, which
number which represents ;=1. They
represents \/‐1. defined i2
They defined equal -‐ 11,, and
i2 to equal and from
from there, any other power of i can
there, any other power can
be derived.
be derived.

1 “ =: -7 1]
;2
;3 =4-‐1i
1K
14=
;4 z 11
1'5 L i

1"= :- fl1
;6
{7 =z -‐1i
;7
i” =
;B z 11

The results
The repeat in cycles
results repeat cycles of 4.
4. You can use the
can use the fact that i4 =
that i4 : 1l to simplify
simplify higher powers of i. For
higher powers For
example,
example,
1-511: (ml: X 1'2 ; 1 x i 2 : _]

When i is used
When an expression
used in an like 3 +
expression like + 2i, the expression is called
the expression a complex
called a number. We
complex number. We add, subtract,
add, subtract,
multiply,
multiply , and divide complex numbers much like we would algebraic expressions.
and divide complex numbers much like we would algebraic expressions.

EXAMPLE 1: If i = J=I,
EXAMPLE \/ ‐ 1 , which
which of the is equivalent
following is
the following (3 +
equivalent to (3 + Si) ‐- (2
(2 -‐- 3i)
3i) ??

A)9i
A)9i B )11+
B) +22ii C)) 1 + 8Bii
C D ) 55 ++ 8Bii
D)

Just expand
Just and combine
expand and combine like
like terms.
terms .

( 3+
(3 + Si) ( 2 -‐ 33ii ) =
5 1 )- ‐ (2 + 5Sii -- 2 + 3
2 .3 + : 11 ++8Bii
3i i =

Answer
Answer ~(C ) .

170
THE COLLEGE
THE COLLEGE PANDA
PANDA

EXAMPLEZ:
EXAMPLE that i =
Given thati
2: Given A,
= \/‐1,whatisthe
what is the product (4 + ii))(5
product (4+ ( 5- ‐ 2i) ?

A)18‐-3i
A) 18 - 3i B)22‐-3i
B) 22 - 3i 1 8+
C ) 18
C) + 33ii -. D) 2 2+
D ) 22 + 3i
3i

Expanding,
Expanding,
((4+i)(5‐2i):20‐8i+5i‐2i2=20‐3i+2=22‐3i
4 + i) (5 - 2i) = 20 - Bi + 5i - 2i2 = 20 - 3i + 2 = 22 - 3i

Answer ~( B ) .
Answer

. . . 22 + 3 i
EXAMPLE 3: Which
EXAMPLE Winch of the followmgisequal
the following to 1+
is equal to ~i ??
+1.
1 +i
1 1. 1 11.. 5 1. 55 1.
1.
A)§‐§l B)§+'il
B) -+- 1
2 2
(Di‐El D)§+El
D) -+ -t
2 2

When you’re faced with


When you're fraction containing
with aa fraction denominator, multiply
the denominator,
containing i in the multiply both top and
both the top and the bottom of
the bottom
the fraction
the fraction by the conjugate of the
the conjugate the denominator
denominator.. What the conjugate,
What is the you ask? Well, the
conjugate, you conjugate of 1 + i
the conjugate
conjugate of 5 -‐ 4i is 5 + 4i.
is 1 -‐ i. The conjugate get the
41". To get conjugate, simply
the conjugate, sign in between.
reverse the sign
simply reverse between.
this example,
In this multiply the top
example, we multiply and the
top and the bottom conjugate 1 -~ i.
the conjugate
bottom by the

(2+3i).(1‐i)_2‐2i+3i‐3i2_2+i‐3i2_5+i
(2 + 3i) (1 - i) _ 2 - 2i + 3i - 3i2
2 + i - 3i2 5 +i 5 +1 i.
( 1+
(1 + ii)) . (1
( 1-‐ ii)) - 11-‐ i + i ‐- i ;2
2
1
_ 11‐12
- i2 = - 22- = 2 + 2

The whole
The point of this process
whole point process is to eliminate
eliminate i from the denominator absence of i in the
denominator.. The absence denominator is
the denominator
good indicator
aa good things were
that things
indicator that done correctly. The
were done answer is ~(D) -.
The answer

171
CHAPTER 19 COMPLEX
CHAPTER 19 COMPLEX NUMBERS
NUMBERS

CHAPTER EXERCISE: Answers for this chapter star t on page 319.

A calculator should NOT


calculator should N O T be on the
used on
be used the
following
following questions.
questions .
((66++2i)(
fl fl 22+
+ Si)
5)
If
If the expression above
the expression is equivalent
above is equivalent toto aa +
+ bi,
bi,
For z \/‐_1,
For ii = ;=T , which of the
whic h of following is
the following is where a and b are constants, what is the
where and bare consta nt s, what is the va lue value
equivalent to (5 -‐ 3i)
equivalent 3i) -‐ (( -‐ 22 + Si)??
+ Si) of aa??
A
A)) 33 -‐ 8
Bii A) 2
B) 3 +
+ 2i B)
B) 12
12
C)
C) 7 -‐ Bi
81' C
C)) 2222
D) 77 +
D) + 2i
2i D
D)) 3344

that i = \/‐_1,
Given that
Given ;=T, which of the
which of following is
the following is Which of the
Which following is
the following equall to
is equa to
( i + 1)
equal to ii(i 1) ?? 3
3(i0++2)
m-‐ 2(5
2 w- - 4i)
4 0?u(Note:
w m ei =i =H¢ f fl)
A) i -‐ 2 A) 16
16 -‐ Si
B) i -‐ 1 B) -~44 + 7i

+ 1l
C) i + C) -‐ 44 ++ 11i
Hi
D) 0
D) 16 ++ 11i
D) 16 11i

fi+3fl+2
i4 + 3;2 + 2 For = \/R‐ l ,, which
For i = which of the
the following is
following is
equivalent 3i(i + 2) - i(i - 1) ?
equ iva lent to 3i(i + 2) ‐ i ( i ‐ 1) ?
Which the following
Which of the following is equal
equa l to the
the expression
expression
above? (Note:: i = H
above? (Note \/‐_1)) A
A) ) ‐- 4 ++ 7ifi

A) i
A) B
B) ) -‐ 22 ++ 7ifi

B)) -‐ 1
B C
C)) -‐ 4 ++ 5ia
C
C) ) 0 D)) ‐- 2 ++ Si
D &

D) 1

R‐ 1 , which
For i = \/ which of the following
of the following is equal to
is equal to
?
,-93 ?
193
22+m+43+5fi+6fl
2
+ 3i + 4i + 5{'- + 6i 4

A)) -‐ 1
A
If the expression
If the above is equivalent
expression above to aa +
equiva lent to + bi,
bi,
where a and constants, what value of B
B) ) 1
where and b areare constants, is the
what is the value of
a+ +bb?? ((Note
N o t ei i= H )
= \/‐_1) C
C)) -‐ ii
A
A)) 2 D)) i
D
B) 6
C
C)) 110
0
D
D)) 112
2

172
THE COLLEG
THE COLLEGEE PANDA
PANDA

Which of the
Which following complex
the following complex numbers is
numbers is Which of the following is equal to -1 -- 3i. ??
equivalent
equivalent to (3 ‐ i )2? (Note:
to (3 - i)2? (Note: i i = \/-‐‐1)
J=T) 3+ 1
= \/‐‐1)
(Note: i =
(Note: R )
A)
A) 88 -‐ 61"
6i
8) 88 + 61'
B) 6i + A
A)) -‐ i ,-

C)
C) 10
10 -‐ 6i
6i B)
8) i

0)
D) 10 +
10 + 61’
6i C)‐§i
C) - ~i

D)
0) ‐‐§i
~ - ~i
4 4

(-i)2
(‐i)2 -‐ ((‐i)4
- i)4
the complex
In the number system,
complex number system, what is the
what is the
value
value of the expression above?
the expression = \/‐_1)
above? (Note: ii =
(Note: J=T) Which of
Which the following
of the following complex numbers is
complex numbers is
A
A)) -‐ 2
- ~ ?? (Note:
2
equivalent to 3‐1;
equivalent : ‘/_1)
(Note: ii = J=T)
B)
8) 0 2+ 1
C) 1 3 4,
D) 2
0)
A)
A) 2‐31“
5 5
-
- - l

B)1‐§i 4,
8) 1 - - I
5

C)g‐gi
5 4.
C) - - - l
3 3
(5 -‐ 2i)(
(5 2i)(44 -‐ 3i)
31')

Which of the
Which following is
the following equal to
is equal the expression
to the D)l‐§i 4,
0) 1 - - /

_I__________
expression 3
above?
A)) 114
B)
8) 14
4 -‐ 7
7ii
14 -‐ 23i
23i
= \/‐_1)
(Note: ii =
above? (Note:
A
R )
__,
C)
C) 26
26 + 7i 4 + i + 2 ‐1
- i
0) 26 ‐- 231"
D) 26 23i 1 ‐1
- i+ l ++1i
Which the following
Which of the is equal
following is to the
equal to the expression
expression
above? = \/‐‐1)
(Note: ii =
above? (Note: J=T)
A) -‐22 -‐ i
1 11 1 B) 2 + i
Which of the following is equal to -:- i‐2+i_4'
+~ +~ ?
I
8)
I I
C) 4 + i
(Note: i = J=T)
D)
0) 4 ‐- i
A) - i
8)
C) 0
0) 1

173
The absolute
The denoted by lxl,
value of x, denoted
absolute value
Absolute Value
Absolute
|x|, is the
the distance
20
distance xisx is from O. In other
from 0. other words,
words, absolute value makes
absolute value makes
everything positive.
everything positive. If it's positive, it stays
it’s positive, positive. If it's
stays positive. negative, it becomes
it’s negative, positive.
becomes positive.

EXAMPLE 1: How
EXAMPLE integer values
many integer
H o w many values of x satisfy |x| << 44??
satisfy lxl

Think of the
Think possible numbers
the possible numbers that work and
that work and don't forget the
don’t forget the negative possibilities. Every
negative possibilities. Every integer between -‐33
integer between
and 33 works,
and total of [zJ
works, aa total 7 integer
integer values
values..
We could've
We also solved
could’ve also solved this
this problem algebraically. Any
problem algebraically. Any absolute value equation
absolute value like the
equation like the one above can
one above be
can be
written as
written as
-‐44 <
< Xx <
< 4

and since
and an integer,
since x is an integer,
- 3 :S:X '.S3

EXAMPLE.2:
EXAMPLE H o w many
2: How values of x satisfy
integer values
many integer satisfy Ix + 11!I < 5?
ix + 5?

Here we
Here through the
we go through the same process. The
same process. largest possible
The largest possible integer
integer for xx is 3 and
and the smallest is -‐ 55.. So
the smallest So
-‐55 :S:
S xx :S:
S 3,
3, aa total
total of []9 possibilities.
possibilities.
algebraically,
Solving algebraically,
Solving
-‐55 < Xx +
+ l1 <
<5
Subtracting 1,
Subtracting 1,
-‐66 < Xx <
< 4
-7 55 :S:
§ Xx :S:
§ 3

3: For
EXAMPLE 3:
EXAMPLE of the
which of
For which following values
the following values of xis |2x -‐ SI
x is l2x 5| < 0?
O?

A)O
A) 0 B) 2.5
B)2.5 C) 5
C)5 There is no
D) There no such value of x.
such value

question.. The
Trick question The absolute something can
value of something
absolute value n e v e r be
can never negative. There
be negative. no solution,
There is no answer I (D)
solution, answer (D) I·.

174
THE PANDA
COLLEGE PANDA
THE COLLEGE

manufacturer of cookies
EXAMPLE 4: A manufacturer
EXAMPLE cookies tests the weight
tests the weight of its
its cookie packages to
cookie packages ensure consistency
to ensure consistency
product. An acceptable
the product.
in the package of cookies
acceptable package cookies must weigh between
must weigh between 16 ounces and
16 ounces 18 ounces
and 18 ounces as
as it
comes out
comes production. If w
out of production. tois
is the
the weight an acceptable
weight of an acceptable cookie package, then
cookie package, which of the
then which following
the following
inequalities correctly
inequalities correctly expresses
expresses all
all possible values of w
possible values w??

A ) Jw-171
A) [ w ‐ 1 7 |>> 1
1 B )Jw-
B) | w ‐ 116J
6 |<< 22 C )Jw+
C) | w +17J
1 7>[ >1l D )Jw-1
D) | w ‐ l 71
7 |<< 11

In these
1n of absolute
types of
these types value word
absolute value problems, start
word problems, start with the midpoint
with the of the
midpoint of the desired
desired interval, 17in
interval , 17 this case,
in this case,
and subtract
and subtract it from
from w: Jw 17|. Think
|w -‐ 171. this as
Think of this the ”distance,”
as the "distance," or ”error,” away from
"error," away from the midpoint of the
the midpoint the
interval. We
interval. We don
don’t't want this "error"
want this to be
”error" to greater than
be greater since w would
than 1 since would then outside the
be outside
then be the desired interval. So
desired interval. So
ouranswer
our answer is §} |w‐
lw 17| <
- 171 1.
1.

We can
We confirm this
can confirm this answer by solving
answer by solving the inequality. Remember
the inequality. Remember that the end
that the result should
end result be 16
should be 16 < w
w < 18.
Let’s see
Let's if our
see if gives us
answer gives
o u r answer us that result when
that result we isolate
when we isolate w.

|w‐l7|<1
lw - 111 < 1

-‐ 1l < w
w ~- 1177 << 1
Adding 17,
Adding
16 << w
16 w < 18
We have
We have confirmed that ~
confirmed that is the answer.
correct answer.
the correct

This is the
This graph of y = x:
the graph
y

Now this is the


N o w this graph of y =
the graph |x|:
!xi:
y

See how graph changed?


the graph
how the Taking the
changed? Taking value of any
absolute value
the absolute function makes
an y function makes all the
the negative y‐values become
negative y-values become
positive y-values
positive y‐values (points
(points in the quadrants III
the quadrants and IV are
I I I and are reflected across the
reflected across x-axis). All the
the x-axis). positive y-values
the positive y-values
stay where they
stay where they are. V-shape is the
This V-shape
are. This classic absolute
the classic absolute value graph that
value graph that you
you should be able
should be able to recognize.
recognize .

175
CHAPTER 20 ABSOLUTE
CHAPTER VALUE
ABSOLUTE VALUE

A table values is another


table of values way to see this
another way absolute value
this absolute value transformation. If fJ (x) = 2x, then
transformation . If compare fJ (x)
then compare
with | f ( x
with IJ (x)J.) |

X -3 - 2 - 1 0 1 2 3
J (x) -6 -4 - 2 0 2 4 6
IJ (x)I 6 4 2 0 2 4 6

values of Jf (x) become


negative values
The negative positive and
become positive the positive
and the values of Jf (x) stay
positive values stay positive.
positive .

Which of the following


EXAMPLE 5: Which
EXAMPLE following could be
be the graph of y = j2x
graph ofy IZX ‐- 1|?
1j ?

A)
A) B)
B) C)
C)
yy yy yy

D)
D)
y31

The entire
The entire function enclosed in an
function is enclosed an absolute value and
absolute value and since the absolute
since the value of something
absolute value something can
can never be
never be
negative, y must
negative, always be
must always be greater
greater than equal to 0.
than or equal 0. In other graph m
words, the graph
other words, u s t lie on
must on or above the x-axis.
above the
That eliminates
That and (8).
(A) and
eliminates (A) (B). In fact, (A)
(A) is the graph of 2x -‐ 1 without
the graph without the absolute value. To get
absolute value. get the
the answer, we
answer, we
take all the
take points with
the points with negative y-values in the
negative y-values graph of (A)
the graph and reflect them
(A) and them across
across the so that
the x-axis so they’re
that they're
positive. graph we
positive. The graph we end with is (C) .
up with
end up [19].
One great tactic that's
One great worth mentioning
that’s worth mentioning here narrowing down
here is narrowing down thethe answer choices by obtaining
answer choices obtaining points
points that
that
are easy to calculate.
are easy calculate. For example, ifif we let
For example, then y =: 1|2(O)
let xx =z 0, then 2(0) -‐ 11|I = The point
= 1. The point (0,1)
(0, 1) must then be
must then be on
the graph, eliminating
the graph, eliminating (A)
(A) and Letting y =
and (B). Letting z 0,
0, we
we now
n o w find that (0.5,0)
that (0.5, 0 ) must be on
also be
must also the graph.
on the This
graph . This
eliminates (D)
eliminates because (D)
(D) because has two
(D) has x‐intercepts wherea
t w o x-intercepts the graph
whereass the should only
graph should have one
only have one..

176
THE
THE COLLEGE PANDA
COLLEGE PANDA

CHAPTER EXERCISE:Answers for this chapter start on page 320.

should N
A calculator should used on the
O T be used
NOT
following
following questions.
questions.
yy

llff fJ(x)
(x) == -‐2x2
2x 2 -‐ 3 + 11, ,what
3xx+ what iissthe value o
the value off
|f(1)|?
1/(1) 1?
A) 3 x

B) 4
C) 5
D) 6
Which of
Which the following
of the could be
following could be the equation of
the equation of
the function graphed
the function graphed inin the xy-plane above?
the xy-plane above?
A) ) yy =
A = -l‐ Mx l -‐ 2

If
If 1|2
2 -‐ x]
x i > 5 and positive integer,
x is aa positive
and xis integer, what is
what is B) y =
B ) y = hl h- 4
lx 2
the minimum
the possible value
minimum possible value of
of x
x?? q y =
C ) y = h H 42
lx l +
D) ) yy=: Ix
D M - ‐ 21 fl

If |x
Ix ‐- 3] which of
10, which
3 1 > 10, the following
of the could be
following could be
the value of |xl
the value lx l ?
?
A
A) ) 2
m4
B) 4
Which the following
Which of the following expressions is equal
expressions is equa l to
to C
C) ) 6
‐5
- 5 for some value
for some value of x ?? D
D)) 8
A
A)) | x-‐ 661
Ix | + 22
B
B)) | x- ‐ 221-| ‐ 6
Ix 6
C
q) | x++ 2
Ix 21| +
+ 66 A calculator is allowed on the following
D
D)) [ x++61
Ix 6 |-‐ 2
2 questions .

H o w many
How many different integer values
different integer of x satisfy
values of satisfy
|x+ 6I< 3?
Ix + 61< 3?

177
CHAPTER 20 ABSOLUTE
CHAPTER ABSOLUTE VALUE
VALUE

y If In - 21 = 10, what is the sum of the two


possible values of n ?
A) 4
B) 6
C) 12
D) 20
graph of the
The graph function f is shown
the function shown in the
the
xy-plane Which of the
above. Which
xy-plane above. the following be
could be
following could
the graph
the graph of the function y := |f(x)|
the function IJ(x) ?
I ?
A)
A) If |x 101=
Ix ‐- 10| where x < 10, then
= b, where which of the
then which
y following is equivalent
following equivalent to bb -‐ x ??
A
A)) ‐- 110
0
B) 10
10
C)
C) 217‐10
2b - 10
D
D)) 1
100-‐ 22b
b
B)
y

hot dog
A hot dog factory must ensure that
must ensure hot dogs
that its hot dogs
are between 6};
are between inches and
6~ inches and 62 inches in length.
6~ inches length .
the length
lfIf h is the length of a hot dog from this
hot dog this factory,
which of the
then which
then following inequalities
the following inequalities
C) correctly expresses the accepted
correctly expresses values of h
accepted values h7?
y
A) lh- 6~I< ~
B) \h- 6~\< 1
D)
C) \h- 61\< i
y
D) 11,- 611> i

178
THE COLLEGE PANDA
THE COLLEGE PANDA

In - 21< 5 A bakery muffins to weigh


standardizes muffins
bakery standardizes weigh
How integers n satisfy
many integers
How many satisfy the inequality
the inequality between
between 11Z~3 and 1
2 ! ounces.
and 21 [f m
ounces. If m is the weight of
the weight
above?
above? 4 4
a
a muffin
muffin from this bakery,
from this which of the
bakery, which following
the following
A) Six
inequalities
inequalities expresses the possible
expresses the values of m
possible values m ??
B) Seven
Seven
3 1
C) Eight
D
Eight
D)) N ine
Nine
A) lm-1~1<~
A) m‐11‘<Z

B) Im- 21<-
B)|m‐2|<1 1
4

C) Im- 21<-
C)|m‐2|<1 1
22
Rolls of tape
tape must be made
must be made to a certain length.
certain length. 3 1
They must contain enough
must contain tape to cover
enough tape cover
D) lm-}~1<1
between
between 400 feet and and 410 feet. If the length
If/I is the length of
a roll of tape
a roll tape that meets this requirement,
that meets requirement, whichwhich
of the following inequalities
the following inequalities expresses
expresses thethe
possible values of I ?
possible values ?
A) I/
|I -‐ 400| 10
4001< 10
B) I/
B) |I -‐ 405|
4051> 5
C) II
U+ + 405|
4051 < 5
I/ -‐ 4051
D) H 4051<<5

HIlllllllllllllilllllllll
41= 8 and
|4x -‐ 4|
If l4x |5y +
and l5y 101=
+10] = 15, what
what is
is the
smallest possible
smallest value of xy ??
possible value
A ) ‐- 220
A) 0
B
B)) -‐ 115
5
C
C)) -‐ 5
D)) -‐ 1
D

If lal 1,then
la} < 1, which of the
then which the following must be
following must be
true?
true?

L1>1
1
I. - > 1
a
IL
ll . a¥
2
<1
1
H
III .Laa>> -‐ 1l
A) [ I ] only
III only
B
B)) 1l aand 11only
n d 11only
C) II and I I only
and III1
D
D)) II,, JI, and IIII
I I , and II

179
Angle Theorem
Exterior Angle Theorem
21 Angles
Ang les
An exterior angle is
exterior angle is formed when any
formed when side of
any side a triangle
of a is extended.
triangle is extended . In below, xx°0 designates
triangle below,
the triangle
ln the designates an
an
exterior angle.
exterior angle.

An exterior angle is always


exterior angle always equal
equal to the s u m of
the sum the two
of the angles in
t w o angles the triangle
in the furthest from
triangle furthest from it. In this
it. In this case,
case,

xI z= aa ++ b

EXAMPLE 1:
D

100° 3x°0
3x
A C E
E B
What is the
What value of
the value of x in the figure above?
the figure above?

A
LD C E must
DC£ be 80°.
must be 80°. N o w there
Now lot of
are aa lot
there are ways to
of ways do this,
to do but using
this, but using the exterior angle
the exterior angle theorem is the
theorem is the
fastest:
fastest:

80 ++ Xx =: 3x
80 3X

80 =: 2x
80

r=E
180
THE
THE COLLEGE PANDA
COLLEGE PANDA

Parallel
Parallel Lines

711

When
When two lines are parallel,
two lines following are
the following
parallel, the true:
are true:
0 angles are
• Vertical angles are equal
equal (e.g. L'.l =
(e.g. £1 = 44)
L 4)
0
• Alternate
Alternate interior angles are equal
interior angles equal (e.g. L4 =
(e.g. 44 : £5 and 43
L S and = 46)
L3 = L 6)
0
• Corresponding
Corresponding angles are equal
angles are equal (e.g. 41l =
(e.g. L : L
45)
S)
0
• Same interior angles
side interior
Same side angles are supplementary (e.g.
are supplementary (e.g. 43
L3 + LS := 180°)
+ 45 180°)
No need memorize these
need to memorize just need
terms . You just
these terms. need to know
know that
that when w o parallel
when ttwo parallel lines are cut
lines are another line,
by another
cut by line,
there
there are sets of equal
t w o sets
are two equal angles:
angles:
L 1 = L'.'.
4 = LS = LB
L2 = L'.'.
3 = L6 = L7

EXAMPLE2:
EXAMPLE 2:
C

A G F E
In the figure above‐If
Inthefigure II"CfiandE‐F'
above, AC || II CE. IfACAE
GD and BF HE. = 70°
If LCAE = 70° and ACE=
and [LACE =40°,whatisthevalueofx?
400, what is the value of x?

Here is the fastest


Here fastest way: L ACE
[ A AABF =
C E = .LABF : 40° because
because they are corresponding
corresponding angles
angles (A‐C cuts parallel
(AC cuts lines
parallel lines
W and fiCE).
BF and ) . Since angle
angle x is an
an exterior angle to 6A AABF
exterior angle z 70 +
B F,, x = + 40 =
= I‑ 110° 1-

181
CHAPTER
CHAPTER 21 ANGLES
ANGLES

Polygons
Polyg ons
Triangle
Triangle Quadrilateral
Quadrila tera 1 Pentagon
Pentagon Hexagon
Hexagon

180°
180° □ 360°
00 540° 720°

As you can see from


you can the polygons
from the polygons above,
above, each additional side
each additional side increases
increases the sum interior angles
sum of the interior angles by 180°
180°..
any polygon,
For any u m of the
polygon, the ssum interior angles
the interior angles is

180(n -‐ 2)
lBO(n 2) where
where n is the number
number of sides
sides

50
So for an
an octagon,
octagon, which has 8 sides,
which has sides, the
the sum interior angles
the interior
sum of the angles is 180(8 = 180 x 6 := 1080°.
180(8 ‐- 2) = 1080° .
A regular polygon is one
regular polygon one in which
which all sides angles are
and angles
sides and equal. The polygons
are equal. shown above
polygons shown are regular.
above are regular . If
our octagon were regular, each interior angle would have a measure
our octagon were regular, each interior angle would have a measure of 1080° -;-8 =
+ 8 : 135°.
135° .
The 180(n
lBO(n ‐- 2) formula comes from
formula comes from the fact that any polygon
that any be split
polygon can be split up several triangles
into several
up into triangles by drawing
drawing
lines from
from any vertex to the
n e vertex
any oone the others.
others .

number of triangles
The number triangles that
that results from this process
results from always ttwo
process is always w o less than number of sides.
than the number Count for
sides. Count
yourself! each triangle
Because each
yourself! Because contains 180°,
triangle contains s u m of
the sum
180°, the the angles within a polygon m u s t be 180°(n
angles within a polygon must be 180° - 2),
(n ‐~
where n is the number
where number of sides.
sides.

EXAMPLE3:
EXAMPLE3;

sides of a regular pentagon


Two sides pentagon are extended
extended as shown in the figure
as shown value of x '?
above. What is the value
figure above. ?

The total number of degrees


total number degrees in a
a pentagon 180(5
pentagon is 180( 5 -‐ 2) =
2 540°.
540° . So interior angle
each interior
So each angle must
m u s t be + 5 = 108°
be 540° -;-5 108°..
The angles
angles within the triangle
within the formed by the
triangle formed intersecting Jines
the intersecting must be
lines must be 180 - 108 =
‐ 108 = 72°.
72°.

108°

108°

So,x=180‐72‐72:-.
So, x = IBO - 72 - 72 = j 36° 1-

182
THE COLLEGE PANDA

CHAPTER EXERCISE:Answers for this chapter start on page 322.

A calculator should
should NNOT be used
O T be used on the
following
following questions.
questions.

c.....----~-----------1

kO

Note: Figure
Note: Figure n drawn to scale
o t drawn
not scale..

the figure
1n the
In i=
above, 1”
figure above, = 50 and k =
50 and = 140. What
What is In the figure
1n the lines I and
above, lines
figure above, m are
and m parallel.
are parallel.
the value of j?
the value j ? Whatisthevalueofa+b+c+d?
What is the value of a + b + c + d ?

A
A)) 660
0 A) 270
B)
B) 70
70 B) 360
C)
C) 80
80 C) 720
D
D)) 9900 D) It cannot
cannot be
be determined
determined from
from the
the
information given.
information given.

Note:: Figure
Note drawn to scale
not drawn
Figure not scale.. Note: Figure not drawn to scale.

ln the
In figure above
the figure above,, what the value
what is the value of y ?? In the figure
1n the above , ifif x =
figure above, = 40, what
what is the
the value
value
of yy??
A
A)) 330
0
A ) 440
A) 0
B
B)) 4400
B)
B) 50
C
C)) 550
0
C)
C) 80
80
D)) 7700
D
D
D)) 9900

183
183
CHAPTER 21 ANGLES
CHAPTER 21 ANGLES

A calculator is allowed following


allowed on the following
questions.
questions.

In the figure above, lines I and m are parallel.


What is x in terms of a and b ?
Note: Figure
Note: Figure n drawn to scale.
o t drawn
not scale.
A) a +
+b
B) a -‐ b
B) the figure
In the
In figure above,
above , what
what is the va lue of x +
the value + yy??
C)) b -‐ a
C A
A)) 125
125
D
D)) 180
1 8 0- ‐aa-‐ b B
B)) 180
180
C) 235
D) 280
_ r» 280

70°
30°

11° 17°

Note: Figure not


Note: Figure not drawn to scale
drawn to scale..

the figure
In the above, what
figure above, what is the value of a +
the value + b ??
A ) 880
A) 0
B) 100
C) 110
C) Note : Figure
m Figure nnot
o t drawn
drawn to scale
scale..
D) 120
ln the figure
In the what is the
above, what
figure above, value of zz ??
the value
A
A)) 335
5
B) 45
45
C)
C) 55
55
D
D)) 880
0

184
THE COLLEGE
THE COLLEGE PANDA
PANDA

(x + 40)° x°

the figure
In the figure above, what is
above, what value of
the value
is the of x
x??
A)
A) 60
60
B) 70
70
C)
C) 75
75
D)
D) 80
80 In the figure
In the above, a
figure above, a rectangle and a
rectangle and a
quadrilateral overlap.
quadrilateral What is
overlap . What is the u m of
the ssum of the
the
degree the shaded
measures of the
degree measures shaded angles?
ang les?
I_ A)
A) 360
360
B) 540
C) 720
D) 900

I -'1 Ar ~•
4
" :.·•. • • \ "'v,)
', ~ ~ .,

A regular hexagon is shown


regular hexagon shown in
in the figure above
the figure above..
What is the
What the value
value of x ?
?
A) 15
m
Note: Figure
Figure n drawn to scale
o t drawn
not scale.. A) 15
B) 20
20
the figure
In the above, what
figure above, what is the value of y?
the value y? C) 25
C) 25
A)
A) 100 D) 30
B) 130
C) 140
D) 150

185
CHAPTER 21 ANGLES

Note: Figure
Note: Figure not drawn to scale.
not drawn scale .

In the
the figure lines I and
above, lines
figure above, and m are parallel.
m are parallel.
Which of the
Which the following
following m u s t be
must be true?
true? Note: Figure not drawn to scale.
I. a =
I, = 3b
3b
the figure
In the lines I, m,
above , lines
figure above, and n are
m, and are parallel.
parallel.
II . a +
11. = b+
+ b= c
+ C What is the
What the value + b ??
value of a +
III. bb == 45
45
A) IIllonly
A) I I only
B) and [I
8) Il and II only
only
C)
C) II
ll and lil only
and 111 only
D
D)) I,
1 ,[II,
L aand
n d III
111

Note: Figure not drawn to scale .

the figure
In the above, what
figure above, what is the value of xx +
the value + yy??
A
A)) 110
0
B ) 220
8) 0
C
C)) 330
0
D
D)) 550
0

186
22 Triangles
Triangles
Isosceles 8.EquilateralTriangles
Isosceles & Equilateral Triangles
isosceles triangle
An isosceles triangle is one
one that has ttwo
that has sides of equal
w o sides equal length.
length. The angles opposite
The angles opposite those
those sides are equal.
sides are equal.

Because AB =
Because : AC, 4C =
AC, L.C : AB.
L.B.
In an
an equilateral
equilateral triangle, sides have
triangle, all sides have the same length.
the same Because equal
length. Because sides imply
equal sides equal angles,
imply equal the angles
angles, the angles
are all 60“.
are 60°.

EXAMPLE 1: In an an isosceles triangle, one


isosceles triangle, one of the angles has
the angles measure of 50°. What
has a measure the degree
What is the degree measure
measure
the greatest
of the possible angle
greatest possible the triangle?
angle in the triangle?

isosceles triangle
An isosceles has nnot
triangle has only ttwo
o t only equal sides
w o equal but also
sides but w o equal
also ttwo equal angles. There are
angles. There possibilities for
w o possibilities
are ttwo
an isosceles triangle
an isosceles triangle w i t h an
with angle of 50°.
an angle Another angle
503. Another angle could
could be 50C, making
be 50°, making a 50‐50‐80 triangle,
a 50-50--80 triangle, or the
the
other w o angles
other ttwo could be
angles could be equal, making aa 50-6.5-<>5
equal, making 50‐65‐65 triangle. Given these
triangle . Given t w o possibilities,
these two 80° Iis the
possibilities, j 800 greatest
the greatest
possible
possible angle
angle in the
the triangle.
triangle.

187
CHAPTER
CHAPTER 22 TRIANGLES
TRIANGLES

w a s
C

A , B
B
maxe‘i‘fggm‘meghemieucmequflatem. is the value of j + k + l + m + n + o ?
Whatisthevalueofj+k+l+m+n+o?
· lS eq ·.. teral. What
,ih1~} i;:~ I

Solution
Soluti on 1: There smaller triangles
are 3 smaller
There are triangles within
within the equilateral one.
the equilateral Each of
one. Each these triangles
of these has aa total
triangles has degree
total degree
measure of 180°,
measure 180°, for aacombined total of 180° x 33 = 540°. We
combined total subtract out L A CBto get what we want.
need to subtract
We need out L ACB to get what we want.
Because
Because triangle ABC is equilateral,
triangle ABC equilateral, L C Bis
ACB
[ A is 60°. So 60° = I-.480°
80 540° -‐ 60° I.

Solution
Soluti AA BC
Because .0-A
on 2: Because BCis both j and
equilateral, both
is equilateral, and o0 are Because k
are 60°. Because and II form
k and straight line,
a straight
form a they add
line, they add
up
up to 180°. Because
Because m and n also
m and form aa straight
also form line, they
straight line, also add
they also up to
add up to 180°. Adding up
180°. Adding all oour
up all values, we
u r values, we
get 60° + 180° + 180° + 60° =
get : I-.480° 1-

Right Triangles
Triangles
Right triangles
Right are made
triangles are up of two
made up legs and
t w o legs the hypotenuse
and the (the side
hypotenuse (the side opposite the right
opposite the angle).
right angle),

C
a

Every right
Every triangle obeys
right triangle pythagorean theorem:
the pythagorean
obeys the b2 = C2,
theorem: a£122 + b2= c2 , where a and
where a bare
and b are the of the
lengths of
the lengths legs and
the legs and
the length
c is the length of the
the hypotenuse.
hypotenuse.

188
THE
THE COLLEGE PANDA
COLLEGE PANDA

EXAMPLE 3:
EXAMPLE

The rectangle above


The rectangle has a
above has diagonal of
a diagonal 20. If
length 20.
of length ba se of
the base
If the the rectangle
of the is twice
rectangle is twice as long as
as long as the height,
the height,
what is the
what height?
the height?

The diagonal of
The diagonal any rectangle
of any rectangle forms
forms ttwo right triangles.
w o right the height
Let the
triangles . Let be xx and
height be the base
and the be 2x.
base be Using the
2x. Using the
pythagorean
pythagorean theorem,
theorem ,

x2 + (2x)
x2+ (2x)22 = 202
202
xx2+4x2=400
2
+ 4x 2 = 400
5x2=400
5x 2
= 400
xx2=80
2
= 80

If you take
If you take the enough times,
the SAT enough what you’ll
times, what find is that
you'll find that certain right triangles
certain right come up
triangles come repeatedly. For
up repeatedly. For
example, 3‐4‐5 triangle:
the 3-4-5
example, the triangle:

A set of three
three whole that satisfy
numbers that
whole numbers the pythagorean
satisfy the theore m is
pythagorean theorem called a
is called a pythagorean triple.. Though
pythagorean triple Though not
not
necessary,
necessary, it’ll you quite
save you
it' ll save quite a bit of time
a bit improve your
and improve
time and your accuracy
accuracy ifif you learn to
you learn recognize the common
to recognize the common
triples show up:
that show
triple s that up:

3, 4, 5
3,4,5
6, 8, 10
6,8,10
5,12,
5, 13
12,13
7, 24, 25
7,24,25
8, 15, 17
8,15,
Note that
Note that the 6-8-10 triangle
the 6‐8‐10 just aa multiple
triangle is just of the
multiple of 3-4-5 triangle.
the 3‐4‐5 triangle .

189
CHAPTER
CHAPTER 22 TRIANGLES
TRIANGLES

Special
Special Right Triangles
Right Triangles
have to memorize
will have
You will w o special
memorize ttwo special right triangle relationships.
right triangle relationships. The first is 45°‐45°‐90°:
is the 45°-45 °-90 ° :

45°

45°
X

The best
best way
way to think
think about this triangle
about this that it's
triangle is that isosceles‐the ttwo
it's isosceles-the w o legs are equal.
legs are We let
equal. We their lengths
let their lengths be
be x.
x.
The hypotenuse, which is always
hypotenuse, which biggest side
always the biggest side in right triangle,
in aa right u t to
turns oout
triangle, turns to be /2 times
be fl times x.x.
We can prove
We can relationship using
prove this relationship the pythagorean
using the where h
theorem, where
pythagorean theorem, 11is the hypotenuse.
is the hypotenuse .

x2+x2
x2 zh2
+ x2 = 112

2x2=h2
2x 2
= 112
x/27:\/h‘2
v'2li = ~
xx/Ezh
xV2= 11
1
l show
show you these proofs
you these n o t because
proofs not be tested
because they will be on the
tested on SAT,but
the SAT, because they
but because they illustrate problem-solving
illustrate problem-solving
concepts that
concepts you may
that you have to use
may have on certain
use on questions.
certain SAT questions.
second is the
The second 30°‐60°‐90°:
the 30° -60 ° -90 °:

60°

Xfi
Because 30° is the
Because smallest angle,
the smallest side opposite
angle, the side from it is the shortest.
opposite from shortest. Let that be x.
side be
that side x. The hypotenuse,
hypotenuse,
the largest
the largest side,
side, turns out to be
turns out be twice x, and side opposite
and the side opposite 60° turns out
60° turns to be
out to \/§ times
be ,/3 x.
times x.
One common mistake
One common mistake students make is to think
students make that because
think that because 60° is 30°, the
twice 30°,
is twice side opposite
the side must be
600 must
opposite 60° be
twice as
twice as big as the
big as the side opposite 30°. That
side opposite relationship is
That relationship is N
NOT true . You cannot extrapolate the ratio of
O T true. You cannot extrapolate the ratio of the
the
sides from the
sides ratio of the
the ratio angles. Yes,
the angles. Yes, the opposite 60°
side opposite
the side is bigger
60° is side opposite
than the side
bigger than 30°, but
opposite 30°, it isn't
but it isn’t
twice as long.
twice as long.

190
THE COLLEGE PANDA
COLLEGE PANDA

We can prove
We can 30‐60‐90 relationship
the 30-60-90
prove the using an
relationship by using an equilateral
equilateral triangle. Let each
triangle . Let be 2x
side be
each side 2x (we
(we could use xx
could use
you’ll see
but you'll
but see why 2x makes
why 2x makes things easier in aa bit)
things easier bit)::

A 2x
2x C

Drawing aa line
Drawing down the
line down from B
middle from
the middle B to A_C creates ttwo
AC creates 30‐60‐90 triangles.
w o 30--60-90 Because an
triangles. Because an equilateral triangle
equilateral triangle
symmetrical,
is symme AD is half
trical , AD just xx.. That's
half of 2x, or just why 2x
That’s why i t avoids
u s e d- ‐ it
was used
2x was any fractions.
avoids any fractions.

A X D C

To find BO, we use


BD, we the pythagorean
use the theorem:
pythagorean theorem:

3022 = A82
AD22 + B0
AD AB 2
2
xx2 BD22
+ 80 (2x)2
= (2x)2
31322 =
80 x2
: (2x)2 -‐ x2
3022
80 4x 2 -‐ xx2
: 4x2
= 2

3022
80 3x 2
= 3x2
M 2: = ~
✓ao @
BD =
BO xV3
= X\/§

Triangle ABO
Triangle proof of the
ABD is proof 30‐60‐90
the 30--60- relationship.
90 relation ship .

191
1 9]
CHAPTER
CHAPTER 22 TRIANGLES
TRIANGLES

Wu 4:
A

C B
WintistheareaofAACBshownabwe?
Wbatis the area of ..6.ACBshown above?

AN?
A)-y2. ism/i
'8)2-/2 04
C) 4 D) 8 ·
D18

the 45‐45‐90
Using the
USing AC =
relationShiPr AC
triangle relationship,
45--45-90 triangle BC‐= 7~ (the
= BC hypotenuse 1s
(the hypotenuse times greater
v'2Z times
is \/‐ than each
greater than leg).
each leg).

22 (1/5
‐ )(x/i‐ ) 22 (_>
1 4 4 ) ( 4 1 16
The area
area is
1s then ‐1 ( v'2
. then v'2 z _1 ( 16) =
4 ) = z 4.
22
Answer ~(C) .
Answer

EXAMPLE5:
EXAMPLES:
A

10

I
D C B

hi, the £igur-e


In the figure above,
above, AD =
AD = DC,
DC, 48 =
LB = 30°,and AB =
30° apd A8 = 10 What is the
10... What ratio of AC to CB ?
the ratio

V2 1/5 «5 2
“W7 ”7th 0m ”5
Because
Because AADD ‐=‐ DC,
DC, A A D C15
6 ADC not only
is not only isosceles but also
isosceles but 45-45-90 triangle.
also a 45‐45‐90 triangle . A A D BISa
6 ADB 30-60-90 triangle
is a 30‐60‐90 triangle
with a
with hypotenuse of 10. Using
a hypotenuse 30-60-90 relationship,
the 30‐60‐90
Using the relationship, AD
AD 15
is half hypotenuse, 5, and
half the hypotenuse, and D DBB‐=‐ 5\/§.
3Using
5v'3. Using
45--45-90 relationship,
the 45-45‐90
the relationship, A
ACC= ‐‐ 5\/§, DCc _
5v'2, D =‐ 5,
5, and
and CCBB‐=‐ D
DBB- ‐ D
DCC= 5f3‐- 5.
‐ ‐ 5y'3 5

A_c_
AC 5\/§
5y'2 _ W2
y'2
= =
C
CBB 5v'3
s fi ‐- s5 fiv'J-‐ 1i
Answer ~(A) .
Answer

192
THE COLLEGE
THE PANDA
COLLEGE PANDA

Similar Triangles
Similar Triangles
When w o triangles
When ttwo triangles have
have the angle measures
same angle
the same their sides
measures,, their are proportional:
sides are proportional :

A C

In the
the figure
figure above,
above, 6 ADBE
DBE andand A6 AABC
B C both
both have right angles
have right and share
angles and share AB. Because W
L.B. Because DE is parallel
parallel to A‐C,
AC,
[L.BED equal in measure
B E D is equal measure to ABCA. ADBE
Therefore, 6
L.BCA. Therefore, DBE and
and A6 AABC have congruent
B C have angles and
sets of angles
congruent sets and are
are
similar to each
similar each other. word s, 6ADBE
other words,
other. In other DBE is just a smaller
just a version of 6
smalJer version AABC.
ABC. It has the same
has the shape but
same shape but not
not
the
the same If we
size. If
same size. we draw
draw the w o triangles
the ttwo separately and
triangles separately and give sides some
the sides
give the arbitrary lengths,
some arbitrary we can
lengths, we can see
see
this more clearly.
this more clearl y.

B
B

4 5
10
8

D E
3

A 6
6 C

that m
Notice that
Notice ”ma‐tches
AB "ma up”" with
tches up with E,
BD, AE_matches
AC matches up with
with W, and Ematches
DE, and BC matches up with E.
up with BE. Using math
Using math
terms,
term say that
s, we say that AB correspo nds with BO,
corresponds BD, AC corresponds with DE,
corresponds and
DE, and BC corresponds
corresponds with BE.
BE.
illustrative purposes,
For illustrative we made
purposes, we sides of the
the sides
made the big triangle
the big triangle twice as long
twice as long as
as the sides of the
the sides smaller one.
the smaller one. But
regardless
regardless of what the actual
what the actual numbers the important
are, the
numbers are, thing to
important thing remember is
to remember is that
that the corresponding
the cor of
sides of
responding sides
similar triangles
similar are proportional,
triangle s are proportional, so
so the of their
ratios of
the ratios their lengths are equivalent.
lengths are In oour
equivalent. In example,
u r example,

Q _ E _ E _
BD_DE_BE_
Forming of equations
types of
these types
Forming these is your
equations is your goal in every
goal in question dealing
every SAT question with similar
dealing with triangles.
similar triangles.

193
CHAPTER 22 TRIANGLES
CHAPTER TRIANGLES

EXAMPLEG:
EXAMPLE6:
B

A C
Figure not
Note: Figure
Note: drawn to scale.
not drawn

lnAABCabove,fiE‐isparalleltoza
In .ti.ABC above, DE is parallel to AC, AD DB =
AD = 99,, DB = 33,, aand DE= 22..
n d DE

Pmnwmetheimgmofzc?
PART1: What is the length of AC1

rakrziwhamthe
PART 2: What. the ratiooftheareaofABDE-Ito area of l:i.BAC?
the areaofABAC
ratio of the ai:ea of ABD E to the ?
.1 1 1 1
Al‘s.
A)!
3 3
B) !
)
4 ;
• C);
C) 9 Dlfi
D) 16

Part 1 Solution:
Solution: The
The answer
answer is NOTNOT 6.6. A common mistake that
common mistake students make
that students whenever they
make whenever similar
see similar
they see
triangles
triangles is to assume correspond with
segments correspond
certain segments
that certain
assume that other when they do not. Of course,
with each other when they do not. Of course, many
each many
questions
questions are designed so
are designed so that mistake is easy
this mistake
that this easy to make. particular case, it's easy to assume that
this particular
In this
make . In case, it's easy to assume that
because AD is 3 times
because BO, AC must
times BD, but this
DE, but
times DE,
be 3 times
must be would be
this would incorrect. We
be incorrect. have to
We have look at
to look at the ratios
the ratios
between sidesof the
between the sides the triangles, not portions
triangles, not those sides
portions of those that A‐D
(Note that
sides (Note AD isis not a side
not a of any
side of triangle).
any triangle).
So
So let’s start from
let's start the beginning
from the make sure
and make
beginning and sure we up the correct
we set up ratios . Because
correct ratios. DE and
Because D‐E AC are
and KC parallel,
are parallel,
ABDE is equal
L.BDE ABAC and
equal to L.BAC and L.BED is equal
[ B E D equal to L.BCA. Therefore , .6.BDE and .6.BAC are similar. Equating
ABCA. Therefore, ABDE and A B A C are similar. Equating the the
ratios of the
ratios relevant corresponding
the relevant corresponding sides,
sides,
BD _ DE
BO DE
E
BAX _AC
R
33 __2 2
= R
35 +? 9) _ AC
3 2
fi12 == AC
E
Cross multiplying,
Cross multiplying ,
3AC =
3AC = 24
24

AC:‑
AC =~

As a word of warning,
a word are valid
there are
warning, there shortcuts that
valid shortcuts some students
that some students take working w
when working
take when i t h similar
with triangles .
similar triangles.
For example,
For 80:0A is equal
example, BDzDA equal to BEzEC,
BE:EC, even though we
even though just showed
we just showed that 80:0A is not equal to DEzAC. II
that BD:DA is not equal to DE:AC.
recommend that you
recommend that avoid these
you avoid shortcuts altogether
types of shortcuts
these types altogether because they are
because they misused . Just
easily misused.
are easily Just focus
focus
on setting
on setting up
up the right corresponding sides
between corresponding
ratios between
right ratios sides and you'll be
and you'll able to
be able handle any
to handle any similar triangles
similar triangles
question the SAT might
question might throw
throw atat you.
you.

Solution: When
Part 2 Solution: When ttwow o triangles are similar,
triangles are the ratio
similar, the ratio of their areas is equal
their areas the square
equal to the square of the ratio
of the of
ratio of
their sides.
their The ratio
sides. The sides is 1:4. Squaring
the sides
ratio of the Squaring that ratio, we get the
that ratio, ratio of the
the ratio areas, 1
the areas, 2 :4 =
12:42
2
: 1:16. Answer
1:16. Answer
/-.(D) /. Note
Note that the figure
that the figure is nnot drawn to scale.
o t drawn scale.

194
..- .··

THE COLLEGE PANDA


THE COLLEGE PANDA

In this example,
1n this it was
example, it easy to
was easy to see the triangle
see the similarity and
triangle similarity and determine
determine which corresponded with
sides corresponded
which sides with each
each
other. In
other. 1n tougher
tougher questions, similar triangles
questions, similar their corresponding
and their
triangles and sides are
corresponding sides are more difficult to
more difficult spot and keep
to spot and keep
track
track of. The
The ratios are n
ratios are o t always
not always obvious.
obvious .
these tougher
For these
For questions , II recommend
tougher questions, labeling equivalent
recommend labeling equivalent angles with tick
angles with Sides opposite
marks. Sides
tick marks. from
opposite from
angles with the
angles with the same number of
same number of tick marks will
tick marks correspond with
w i l l correspond with each other. Here’s
each other. an example
Here 's an example to
to illustrate:
illustrate:

EXAMPLE 7:
EXAMPLE
G

hithefigtmeabove,whatisthelengthofffi?
F
Lh
In the figureabove, what is the length of KH ?
K H

At first glance,
At glance, this
this does n o t look
does not like aa similar
look like triangles question.
similar triangles But ifif we
question. But we label the angles
label the of the
angles of outside
the outside
triangle something interesting
FGH, something
triangle FGH, interesting happens.
happens. Let’s label [F
Let's first label L F with
with one tick mark
one tick mark and [HH with
and L tick
t w o tick
with two
marks.
marks.

G G
G

N o w whenever
Now
F
A
/n
whenever ttwo w o angles
K
K

one triangle
angles of one
H

triangle are
‐>

are equal
equal in measure
F

measure to ttwo
m&
w o angles
K H

another, the
angles of another, the third angles must
third angles must
also be equal.
also be outside triangle
Because outside
equal. Because FGH and
triangle FGH and triangle
triangle FGK on the
FGK on the left both have right
both have angles and
right angles share
and share
AF,
L F, AFGK m u s t have
L FGK must have the
the same measure as
same measure as LAH (two tick marks).
H (two marks). Likewise, outside triangle
Likewise, outside F G H and
triangle FGH triangle
and triangle
GKH the right
GKH on the both have
right both right angles
have right and share
angles and share AH, ZKGH must
so L.KGH
L.H, so have the
must have the same
same measure
measure as (one
AF (one
as L.F
tick mark).
mark).
The result
The that the
result is that the outside triangle, the
outside triangle, triangle on
the triangle on the left, and
the left, the triangle
and the on the
triangle on right all
the right all have
have the
the same
same
angle measures
angle They’re all
measures.. They're similar to one
all similar another!
one another!
AFFGK
Since 6.
Since 3‐4‐5 triangle
GK is aa 3-4-5 FK =
with FK
triangle with = 4,
4, we
we can
can n set up
o w set
now up an equation of
an equation ratios using
of ratios triangles
similar triangles
using similar
AF GKon
6. FGK the left
on the and A
left and 6.G
GKH on the
K H on right to find the
the right the length
length of m.
KH.

KH
KH (opposite l-tick angle
(opposite I-tick angle in right side triangle)
right side triangle) __ GK (opposite l-tick angle
(opposite I-tick side triangle)
angle in left side triangle)
GK
GK (opposite angle in right
(opposite 2-tick angle side triangle)
right side triangle) - FK (opposite
(opposite 2-tick angle
angle in left side triangle)
side triangle)
EA
KH 3
-
33 ‘4 4
4 ( K H )=: 9
4(KH)

KH =rn

195
CHAPTER 22
CHAPTER 22 TRIANGLES
TRIANGLES

Parallel
Parallel Lines
Lines and Proportionality
Proportionality
Lines
Lines that
that cut
cut through three or
through three or more
more parallel
parallel lines
lines are
are separated
separated into proportional parts.
into proportional parts.

n
m

To illustrate, lines m
illustrate, lines and n
771 and it above are transversals
above are transversals that cut through
that cut three parallel
through three parallel lines.
lines. Therefore,
Therefore, the
the three
three
parallel lines
parallel lines divide m and
lines m
divide lines and nn proportionally:
proportionally:
a
‘ 1 5C
bbd d

concept can
This concept can be proved using
be easily proved using similar triangles,, but
similar triangles but you need to know
don 't need
you don’t know the underlying proof.
the underlying proof .
Just memorize
Just memorize the rule . It's
the rule. It’s not tested very
n o t tested but it has
often but
very often past exams,
shown up on past
has shown usually in problems
exams, usually problems
involving aa trapezoid.
involving trapezoid.

EXAMPLE8:
· EXt\MPLE8:
A B

~ llt~ ► if;,
•~•=-t J~

~~ ~.
.-,/' :t,n.;'11,• ""trr ,..,. l
.......
'~ ,. "

... ,.
,,.;-..
• > I

P \ Q

- ... ~ I D C

lnflrefigumabave,fi,P§,and‐D'Careparakl.
In PointPliesonEandpointhiesonFC.
the figure above, AB, PQ, and DC are parallel. Point lfB
P lies on AD and point Q lies on BC. If Q = 44,,
BQ
= 2, and AD = 7.5, whatis the length of AP?
QC=2,andAD=7.5,whatisthelengthofAP?
-QC

Because E
Because and W
AD and through three
cut through
BC cut parallel lines,
three parallel they are
lines, they are divided proportionally.
divided proportionally.

£_@-L2
PD_QC_2_

: ~g
2
twice QC, AP is twice
since BQ is twice
can see, since
As we can ratio of 2 to 1). A 2:1 ratio
twice PD (a ratio ratio means
means that - =
that AP is -2‐3‐1‐
2+1 3
then ~5 x 7.5 =
‐ 2
length of AD
the length
the AD is then
AD.. AD : [fil.
I. If you prefer to do
you prefer do things
things algebraically, length PD
then let length
algebraically, then be x.
PD be
Then length
Then AP is 2x, and
length AP and since AP and
since AP PD sum
and PD 2x + x == 7.5. This equation
A D , 2x
sum to AD gives x =
equation gives z 2.5 and we get
and we
AP =
AP : 2x = : @J
2(2.5) =
= 2(2.5) I. .

196
THE COLLEGE PANDA
THE COLLEGE PANDA

Radians
A radian simply another
radian is simply another unit measure angles.
used to measure
unit used angles. Just
Just as
as we have feet
we have feet and meters, pounds
and meters, and
pounds and
kilograms, we
kilograms, we have degrees and
have degrees radians.
and radians.
radians = 180°
7Tradians
Tr 180°
If you've
If you’ve never radians before,
used radians
never used don't
before, don be put
' t be off by
put off by the Tr. After
the 7T. After all, just aa number.
it’s just
all, it's We could’ve
number. We could've
written
written
radians :::::;
3.14 radians z 180°
180C

but everything
instead, but
instead, typically expressed
everything is typically expressed in terms
terms of 7r when we're
Tr when working with
we're working radians.. Furthermore,
with radians Furthermore,
3.14 is only an
is only anapproximation. given the
approximation. So, given factor above,
conversion factor
the conversion above, how would we
how would convert 45° to
we convert radians?
to radians?

7Tradians __ .::
71 d"
45° xx Tr radians
450 180°
1800 - Z
4 radians
ra ians

Notice that the


Notice that degree units
the degree (represented by the
units (represented little circles) cancel
the little cancel oout just as
u t just they should
as they should in any conversion
any conversion
3
problem.. Now
problem would we convert
how would
N o w how convert 7”; to degrees? Flip the
degrees? Flip conversion factor.
the conversion

3rr d.1ans x ----


radians
-3‐H ra & 1800 : 2700
= 0°
2 7Tradians
Tr radians

You might
might be wondering why
be wondering why we
we even need radians
even need radians.. Why n just stick
o t just
not with degrees?
stick with another difference
this another
degrees? Is this difference
between the
between the U.S. and the
U S . and the rest the world,
rest of the world, like with feet and
like it is with and meters? Nope.. As we'll
meters? Nope we’ll see the chapter
see in the chapter on
on
circles, some calculations
circles, some are much
calculations are much easier angles are
when angles
easier when are expressed
expressed in radians
radians..

EXAMPLE9:
EXAMPLE 9:
y
m

xy-plane above,
the xy-plane
In the m passes
line m
above, line through the
passes through the origin and has a
origin and a slope Vi. IfIf point
slope of y/j. A lies
point A on line
lies on m
line m
and point B lies
and point lies on
on the
the x-axis as shown,
x-axis as the measure,
what is the
shown, what measure, in radians, angle AOB?
radians, of angle AOB ?

A); 71:
A).::
6
B)?n
B) n:
5
C).::
C);
4
7T
D)?
D).::
3
7T

197
CHAPTER
CHAPTER 22 TRLANGLES
TRIANGLES

We
We can draw aa line
can draw line down from A to the
down from the x-axis to make
x-axis to make aa right
right triangle.
triangle . Because slope is v'3,
the slope
Because the V3, the ratio of the
the ratio the
height this triangle
height of this triangle to its base always \/5
base is always v'3to
to 11 (rise
(rise over
over run).
run).

y
m

This right triangle


This right triangle should
should look familiar to you.
look familiar you. lt's 30‐60‐90 triangle.
the 30-60-90
It’s the Angle AOB
triangle. Angle A 0 3 is opposite
opposite the v'3,so
the fi, so its
its
measure is
measure is 60°. Converting
Converting that
that to radians,
radians,

7T
60 x 180° 2 3

Answer ~(D) .
Answer

198
THE COLLEGE PANDA
THE COLLEGE PANDA

CHAPTER EXERCISE:Answers for this chapter start on page 324.

V.
A calculator
calculator should
should N O T be
NOT be used on the
used on
following questions.
following questions.

X
The lengths
The length s of the sides of a
the sides right triangle
a right are x,
triangle are 6
x -‐ 2, and x + 5.
2, and Which of the
5. Which the following
following
equations
equations could be used
could be used to find x ??
A) X +X - 2 = X +5
B) x2 + (x + 5)2 = (x - 2)2 A square side length
square of side length 6
6 is shown the figure
shown in the figure
above. What is the
above. What value of x ??
the value
C) x2 + (x - 2)2 = (x + 5)2
D) (x - 2) 2 + (x + 5)2 = x2 A) WE
A) 3v'2
B) 6
C) WE
C) 6v'2
D) 6v'3
6\/.’§
B

I
A D C
Note: Figure
Note: Figure not drawn to scale.
n o t drawn

ln ABDC
In .6 above, what
BDC above, the length
what is the length of W
DC??
C
A) 3
B) 5 the figure
In the figure above, AB II
above, E || CB.
CD. What the length
What is the length
C) 5\/§
C) 5v'3 of AB
AB??

D) 8

199
CHAPTER 22 TRIANGLES
CHAPTER TRIANGLES

angles of a
Two angles a triangle
triangle have
have the same measure.
same measure.
IfIf two have lengths
sides have
two sides 15and
lengths 15 what is
and 20, what the
is the
greatest possible
greatest value of the
possible value the perimeter of the
perimeter of the
triangle?
triangle?

Note: Figure
Note: Figure not drawn to
not drawn scale..
to scale

In the
the figure
figure above, base of
the base
above, the of a has a
cone has
a cone a
radius of 6. The
radius cone is
The cone sliced horizontally so
is sliced horizontally so
that the top
that piece
top piece is a smaller cone with a height
smaller cone with height
N and a base
of 1 and radius of 2. What
base radius the height
What is the height of
of
N .
the bottom piece?
the bottom piece?

~
5 5 A) 1
B) 2
C) 3
M 8 O
0
D) 4
What is the area
What isosceles triangle
area of isosceles M NO
triangle MNO
above?
above?

B H 10 C
Note : Figure not drawn to scale .

In the
the figure above, E
figure above, AB is parallel
parallel to
to flGH and
and 57
OF
is parallel
parallel to 'BBC.
‐C". If
If DE = 1,
DE = 1,EH
EH = = 3,EC
3, EG =2 2,
2, and
and
In the
the figure
figure above, equilateral triangle
anequilateral
above , an on
triangle sits on HC =
HC 10, what
2 10’ what is
is the length OfAD
the length of AD??
top
top of a
a square. If the
square. If the square
square has an area
has an area of 4,
what is the
what area of the
the area the equilateral
equilateral triangle?
triangle?
A)) \ v'3
A /§
\/§
B
B) )v'3
T
2
3
OZ
C) -
4
D
D) ) 1

200
THE COLLEGE PANDA
THE COLLEGE PANDA

following
allowed on the following
A calculator is allowed
questions..
questions
cC

A)

B) -

0)
Sn
C) -
many radians
o w many
How
37r
37T
-
T4
77r
7rr
?6
5rr
74
337r
-
72
rr
are in 225° ??
radians are

In 6
AB
AA
AB =
DE
D...,______

B C above,
ABC
: 9and
length
9 and AC
length of CE
A
A)) 6
CDE =
ACDE
above, L
E

= 90° and
AC = 12. If DE
CE??
and AA
DE = 6,
B

90°..
L A = 90°
what is the
6,whatis the

B) 8
C) 9
D
0)) 110
0
A

B
~ 9 C E F
w
W
Triangle ABC
Triangle above is similar
ABC above similar to triangle DEF.
to triangle DEF.
What is the perimeter
What the perimeter of triangle DEF
triang le DEF? ?
A ) 220
A) 0 zZ
B) 26.8 20
20
C
C)) 3300
12
12
D) 36.2
0)

________________ J X 15
15 yY

Two poles
Two poles represented
represented byby W and W
XW and above are
YZ above are
isosceles triangle
In isosceles ABC,, E
triangle ABC BC is the shortest side
the shortest side.. 15 feet
15 feet apart. 20 feet
One is 20
apart. One feet tall and the
tall and other is
the other
If the degree measure
the degree measure of LA is a multiple
L A is a multiple of 10,
10, 12 feet
12 feet tall.
tall. A rope joins the
rope joins the top one pole
top of one pole to the
the
what is the
what smallest
the sma possible measure
llest possible measure of L [BB ?? top
top of the other. What is the length
the other . What the leng th of the rope?
the rope?
A) 75° A
A)) 112
2
B) 70°
B) B) 17
B) 17
C) 65° C)
C) 18
18
D) 60°
0) D)
0) 19
19

201
201
CHAPTER 22 TRIANGLES
CHAPTER 22 TRIANGLES

y
A( - 2,4)
24

35

What is the
What the perimeter the trapezoid
perimeter of the trapezoid above?
above?
A)
A) 100
B) 108
C) 112 B(- 2, - 3) C(5, - 3)
D) 116

Points A, B, and C form a triangle in the


xy-plane shown above. What is the measure, in
radians, of angle BAC?

A);6
A)
7[

8
B)?
B) 4
7[

X
C)?
C) 3
7[

What is the
What value of x in the
the value the triangle
triangle above?
above? D);
D) 2
7[

A z D
Two parallel
parallel lines shown in the
are shown
lines are xy-plane
the xy-plane
the figure
ln the
In figure above, square of side
ABCDD is aa square
above, ABC side above. If
above. AB := 15
If AB and point
15 and point B has
has coordinates
coordinates
length 3. If AW
length 3. AW =: AZ
AZ = CX = CY : 1,
CY = 1 , what is
w h a t is (m, n), what is the
n ), what value of n?
the value n?
the perimeter of rectangle
the perimeter rectangle WXYZ
WXYZ? ? A)) -‐ 66
A
A
A) ) 3v2
s fi B
B)) -‐ 8
B)
B) 4\/§
4v2 C
C)) -‐ 9
C) ) 6v2
C e fi D
D)) -‐ 112
2
D) 8

202
THE COLLEGE
THE PANDA
COLLEGE PANDA

B A B

D C

In the figure above, circle O is inscribed in the


square ABCD. If BO = 2, what is the area of the
circle?
In the figure above, equilateral triangle A BC is 7T
inscribed in circle D. What is the measure, in A) 4
radians , of angle ADB? 7T
8) 2
27r
27T
A)
A) ?3 C) 7T

37T
37r 37T
B) -
8)
T4 D) 2

47T
471
C) -
C)
“5‑
5
57T
Sn
D)
D)
76 y

X z
Z
2
[n the figure
In the the value
above, the
figure above, ~~ is k, where
value of % where

. 45° constant. Which


k is a constant. the following
Which of the ratios has
following ratios has
B 1
C a value of ik??
a value
What the length
What is the DB in the
length of DB the figure above?
figure above? YZ
A)
A)
flxz
A) M5
2v'3
3
3 XY
xy
8)
B)
W
xw
8)
2./6
216
B)
3
YZ
yz
C)
C)
W
XY
C)
C)
sfi
4./6
3
3 D)
YW
YW
D)
xw
XW
\/5
D) v'3

203
CHAPTER
CHAPTER 22 TRIANGLES
TRIANGLES

B y
B(2,8 )

+---+--------'~--x
0 0 (8,0 )
A D C
In the xy-plane above, points A and C lie on OB
Equilateral
Equilateral triangle DEF is inscribed
triangle DEF inscribed in __ and BO, respectively . If AC is parallel to the
equilateral
equilateral triangle
triangle ABC
ABC such that E
such that 1 AC.
ED 1- AC. x-axis and has a length of 3, what is the length of
What is the
What the ratio
ratio of the area of ADEF
the area 6. 0EF to the
the area
area BC?
of ABC
ABC??
A
A)) 11 ::4
4
B
B)) 1 ::33
C) 1 :2
1:2
D
0)) 5 ::88

v
A
B C

A‘
A D B
In the
the figure above, equilateral
figure above, equilateral triangle AED is
triangle AED In the
the figure above, a
figure above, a semicircle sits on
semicircle sits top of a
on top a
contained
contained within
within square ABC D.. What
square ABCD What is the
the square of side
square side 6. Point
Point A is at the
the3ptop of the
the
degree
degree measure
measure of [LBBEC?
BC?
semicircle. the length
What is the
semicircle . What length of AB?
AB ?
A) 60° A)
A) 3\/§
3 v15
B) 100°
100° B) 7
C) 120°
120° C) 9
D) 150° D
D)) 3v'IO
3m

204
THE COLLEGE
THE PANDA
COLLEGE PANDA

In AABC,
6 ABC, AB
AB = BCBC := 6and
6 and L ABC =
ZABC 120°..
= 120° A
What
What is the area of AABC
the area 6 ABC??
A
A)) 2v'3
N5
B)
B) 4\/§
4v'3
C ) s fi
C) 6v'3
D) W3
D) 9v'3 3

O
0 4 E C

y
3! the figure
In the figure above, square DBCE
above, square DBCE hashas aa side
side
length
length of 3. If OE
OE = 4,
4, what
what the length of
is the length AD ?
of AD?

e
x

N3,
( v'3, -‐ 11))

A B
In the xy-plane above,
the xy-plane
x-axis and
and the
measure,
measure, in radians,

A)
571
57T
angle 9
above, angle
line segment
the line
0 is formed by
is formed by the
shown.. What
seg ment shown
radians, of angle
angle 0 ?
?
the
What is the
-‘ 4 4

F
A) 12
?3 12
77T
77r
B)
B)
T4
97r
9n
C) -
C)
?5
1171
lln
D C
D)
D)
T6 Square ABC D above
Square ABCD side length
above has aa side length of 12. If
If
BF : 4,
BF = 4, what is the
what is length of BE?
the length of BE?
A
A)) 3
B
B)) 2,/2
zfz
C)
C) 3\/§
3,/2
D
D) ) 4,/2
Ni

205
CHAPTER22
CHAPTER 22 TRIANGLES
TRIANGLES

A R

D
Q 15
15 T

the figure
In the figure above, RT =
above, RT = 17
17 and
and gQS is
perpendicular
perpendicular to RT. the length
What is the
RT . What length of B7"
ST to
B C the nearest tenth
the nearest tenth of a unit?
unit?
the figure
In the above, AB =
figure above, and
= 12, AC = 13, and A)
A) 12.6
DE = 3. What
DE is the
What is the length
length of AE
AE?? B) 12.8
C) 13.2
D) 13.4

C D
X
2
B~---------JE
4 X

A C
A F
Note: Figure
Note: Figure nnot drawn to scale.
o t drawn scale.
Note: Figure
Note: drawn to scale.
o t drawn
Figure nnot
In the
the figure DE is parallel
above, 51?
figure above, parallel to K. AC. The
The
the figure
In the above, points
figure above, points B amuz" lie on A_C
and E lie AC
perimeter of triangle
perimeter BOE is
triangle BDE is at
at least 12but
least 12 no
but no
and fi,
and respectively,, such
OF, respectively such that BE is parallel
that BEis to
parallel to
than 16. If
~ore _than
more the perimeter,
If the perimeter, p, triangle ABC
p, of triangle
5. What is the
CD . What the value
value of x?
x?
is
1san
an integer, one possible
what is one
mteger, what value of p ??
possible value
A) ,/6
A) Jé
B ) z fi
B) 2./2
C)
C) 2\/§
2)3
D) 3

206
23 Circles
Circles
You Should
Facts Yoµ
Circle ~acts Know:
Should Know:

Area a circle: rrr2


Area of a m2

Circumference of a
Circumference 2m
a circle: 2nr

Arc Length: ~
Arc,,Length: % ·x 2m
2,rr OR
OR Or if 09 is in
Or in radians
radians

,Ar.ea ! 7tr22
Sector: % x nr
Area of a3 Sector: OR érze if 60is
½r20 radians
is in radians

Central angles have


have the
<;'.fil\tra.Langles measure as
same measure
the same as the a r t s that they
the arcs-that ”carve out.”
they "carve out."
A

‘ B

Many students
Many students confuse length with
oonfuse arc length with arc measure. The
arc measure. The arc length is
arc length is the actual distance
the actual one would
distance one travel
would travel
along the circle from
alotrg the from A to B. Arc measure
Ate measure is the number
the number of degrees one turns through from A to
degrees one turns through from A to B. You B. You
think of it as
can think
can asa along the
rotation along
a rotation circle from
the circle from A to B. A full rotation
A full rotation is 360°.
is 360°.

Inscribed angles are


Inscribed angles are half measure of the
the measure
half the that they
arcs that
the arcs they ”carve
"carve out."
out."
60°

I
.. .

207
CHAPTER CIRCLES
CHAPTER 23 CIRCLES

Angles
· inscribed
• . · P) semicircle arei
in aa sewi~ are always is just
90°. This is
.Oways 90°. an extension
just an extension of the previous fact. An angle
the previous angle
inscribed in aa semicircle carves out half ~,circle,
"circle~a,tves a circle, or 180°, which means
180°,which means the angle itself
the angle half that,
itself is half 90°..
that, or 90°

. is ./
Aradiusdrawntoalinetangenttothecimleisperpendicular
.;,
@
to that line:
E}"eirele perpendicular to that line:

Gen.etaleq:uation1)£a .circlemthe__
Germlequationofacircleinthexy-plane:
xy-plane:
® (x‐h)2+(y‐k)2=r2

‘x
',.'.

y31
'," II

where~{h,
wh~ fllsisthecenterofthecimleandrisits
(h,k) radius.
-the center.of the-circleand r is i~ ;radius.

EXAMPLE1:

~ i' ... ~.\,


~/' + ... ,.

In ,the
ln figure above,
the .fi~ above, the outer circle’s
outer citcle' radius is twice
s radius twice as long as
as long as the inner circle’s.
the inner What is the
cltcle' s. What ratio of the
the ratio the
~a o~ the $haded region-to the area
areaoftheshadedregiontotheareaoftheunshaded region?
of the unshaded region?

A); B)1
B)l C)2
C)2 D)3
0)3

208
THE COLLEGE
THE PANDA
COLLEGE PANDA

radius of
the radius
Let the of the inner circle be
inner circle Then the
be r. Then radius of the
the radius outer circle is
the outer is 2r.
Area of inner
Area m22
inner circle: n:r
Area of outer
outer circle: n:(2r )2
7T(2r)2
Area shaded region: 7r(2r)2
Area shaded region: n:(2r) 2 -‐ n:r
of : 4n:r
7rr22 = 47'tr22 -‐ n:r
m22 =
: 3n:r2
37rr2

Shaded
Shaded 37tr2
3n:r2
-U 3
-n-sh
_a_d_e_
d =
Unshaded : 7rr22 =
_n:_r_

The answer is J ((D)


The answer D) J..

mm 2;
EXAMFLE2:

C 2 B

Whatistheareaofmeshadedregioninfltefigureabove?
Whatis the areaof the shaded region in the figureabove?

Aug‐Vi wig‐Ni 0125‐2 m g - fi


get the
To get shaded area,
the shaded we must
area, we subtract the
must subtract the area the triangle
area of the triangle from the
the area of the
the sector.
45° 1
sector.. --
45° n:r _ -81 n( 22z) =
_
_ -N
7f
Area of sector· 360° 7Tr2 =
‐- 7r(2 2
. '360 ° 8 2
triangle: Draw
Area of triangle: height from
the height
Draw the point A to base
from point makes a3 45-15-90
base CB. This makes triangle. Because
45‐45‐90 triangle. Because AC also
AC is also

aa radius
radius,, its 2. Using
length is 2.
its length 45‐45-90 triangle
the 45-15-90
Using the the height
relationship, the
triangle relationship, height is then ~ = fl.
then % ,/2..

1
: ébh
1 1
Area =
2bh == 2(2)(fi)
(2)( v'2)=
= x6
v'2

shaded region
Area of shaded region = i-
: g ‐ flV2

answer is ~-(D) .
The answer
The

209
CHAPTER
CHAPTER 23 CIRCLES
CIRCLES

EXAMPLE3:
EXAMPLE3:
A

120°
0

Adielewithadiame’wrof’misshowninfitefigureabova IfziAOB =-120°,whatisthelengthofminor


mXE“? , ,
$2955 0 ·'1107:
011.
,, 3
t.

120° 1 1071’
w(2flr) ‐
1200 (2m ) =
360°
§(27T x5)‐‑
~(2n
3
x 5) = 110n
3
I

The answer is ~(C) .-


The answer

“W134:

In ,the
ln figureabove,
the ngure .,above, [LACB is inscribed
A C E is in 0ircle
inscribed in 0. What
circle O:_What is the measure of
the measure angle ACB ?
of angle ?

A) 15°
15° 30°
B) 30°· C) 45°
C)45° D) 60°°
D)60

The measure
The measure of minor arc AB
minor arc XE is the
the same as the
same as measure of central
the measure angle LAAOB,
central angle Inscribed angle
AOB, 90°. Inscribed AC B is
angle ACB
half of that,
half that, 45°. Answer §j.
Answer (C) .

210
THE COLLEGE PANDA
THE COLLEGE PANDA

EXAMPLE5:
BXAMPLH5:
fi‐a+f+@=m
x2- 4x + y 2 + 2y = 31
"f .:i".i
' Theeqnafiom::of ·.aniflemfm
~uati:on the,~
'i:ltd e&:1,
the“ xy‘plane
~ is given
· is above. What
gh'ren.above. the ,coordihates · of the
are thecoordinatesof
What are eentet of the'circle?
the center the ·eirde?

A}( - 2,-‐ 11))


A )(‐2: ij))((-42,1)
8 :1) C - 1,2)
C)()(412) D )(2, ‐- 11))
0)(2,

To get
get the equation of the
the equation circle in the
the circle the standard h)2+
form (x ‐- h)2
standard form + (y ‐- k)2 = r2,
k) 2 = r 2 , we have to complete
we have the square
complete the square
twice, once for the
twice, once x 's and
the x’s once for the
and once the y's. If you
y’s. If you don’t
don't know how to complete
know how complete the the square,
square, you
you should review
should review
the quadratics
the quadratics chapter,
chapter, which
which contains examples of how
many examples
contains many how to do
do it. Starting with x,
Starting with

((xx-‐ n
2)2
2
-‐ 4
4 + fy2
++e2y
y ==3311
Then
Theny,y,
u‐2P‐4+@+1F‐1=m
( X - 2 )2- 4 + (y + 1)2- 1 = 31

(x‐n2+@+1V=36
(x - 2)2 + (y + 1) 2 = 36

From the standard


From the form, we
standard form, we can that the
see that
can see center is at (2, ‐- 11)) and
the center and the
the radius Answer ~(D) .-
radius is 6. Answer

211
CHAPTER 23 CIRCLES
CHAYTER 23 CIRCLES

CHAPTER EXERCISE:Answers for this chapter start on page 330.

allowed on the following


A calculator is allowed following
questions.
questions.

A B

pP The circle
circle above 36rr and
area 367t and is divided
The above has
has area divided into
into
congruent regions.
8 congruent the perimeter
What is the
regions . What perimeter of
one of these
one these regions?
regions?
D C
A) 6 + l.Srr
A) 6+1.57r
[n the figure
In the above, the
figure above, square ABCD
the square ABC D is B) 6 + 2n
B) 6+27T
inscribed a circle. If the radius
inscribed in a circle . If the radius of the
the circle is r,
r, 12 + l.Srr
C) 12+1.57r
what is the
what the length
length of arc APD APO in terms
terms of r?r?
D) 12 + 2rr
D) 12+27r
nr
A) 4

B rr;
B)) - ‐

C) nr Which the following


Which of the following is
is an equation of a
an equation a circle
circle
C)
in the xy‐plane with
the xy-plane center ( -‐ 22,, O)
with center and an
0) and area
an area
nr 2
D)
D) of 4971
49rr?7
4
A) ) ((xx- ‐2) 22+) y 2 = 7
A
B)
B) (x+2)2
(x + 2) 2 ++y2=7
y2 = 7
C)
C) (x ( -‐2)2
2)2+ +y2=49
y2 = 49
D) (x + 2)2 +
D) (x+2)2 +y2=49
y2= 49

the figure
In the figure above,
above, three congruent circles
three congruent circles are
are
tangent
tangent to each other and
each other and have on
that lie on
centers that
have centers
the diameter of a larger
the diameter circle. If
larger circle. [f the each
area of each
the area
of these small circles
these small circles is 97t, what is the
9n, what the area
area of
the large
the large circle?
A)
A) 3671
36n
B) 497r
49n
C) 64n
647t
D) 8171
81rr

212
THE COLLEGE PANDA
THE COLLEGE PANDA

Note: Figure not


Note: Figure drawn to scale.
not drawn scale.
In the figure
ln the above, equilateral
figure above, equilateral triangle
triangle ABC is
A BC is
the figure
In the figure above, CB is inscribed
inscribed in aa
circle.
above, [L AACB
The length
circle. The length of minor arc AB
minor arc AB is what
what
inscribed
inscribed in circle
36m
36n, what
D. If
circle D.
what is the
If the
the length
the area
area of
length of minor -
of circ/lxe
arc AB
minor arc AB ??
D is
circle Dis

fraction of the
fraction the circumference
circumference of the
the circle?
circle? A) 27r
2n

A);1
A) 3
B)
B) 37r
3n
C) 47r
47T
a);
B) -
4
1 D) 67r
D) 6n

C);61
C)

mg 1
D) 12

In the
the figure
figure above,
above, circle C has
circle C has aa radius of 6.
radius of If
6. If
the area of the
the area shaded sector
the shaded what is the
1071, what
sector is 10n, the
measure, radians, of angle
measure, in radians, AC B ?
angle ACB?
27r
27T
A)
A)
the figure
In the above, AC is a
figure above, a diameter
diameter of the circle
the circle ?5
and the length
and the length of AB
AB is 1.
1. If the radius of
the radius of the
the 471
47T
degrees, of B)
B)
circle
circle is 1, what
what is the
the measure,
measure, in degrees, 79
LBAC
L BAC?? 57r
57T
C)
79
Sn
57T
D)
D)
T8

213
CHAPTER 23 CIRCLES
CHAPTER CIRCLES

In the figure
1n the above, a circle has
figure above, and
center C and
has center base of a right
The base
The circular cylinder
right circular cylinder shown
shown
radius If the measure
radius 5. If central angle
measure of central angle AC
ACBB is
above has radius
above has a radius of 4. The height
4. The 5. What
height is 5. What is
7T 7T
between g and
between and g radians, what is one
radians, what possible
one possible surface area
the surface the cylinder?
area of the cylinder?
4 2
A) 4
40rr
integer
integer value length of minor
value of the length arc AB?
minor arc XE ? A) 0”
B) 607r
60rr
C) 7271
72rr
D) 81rr
D) 8171'

H
_...;..._--------~------1

In the figure above, circle P and


figure above, and circle U each
each
figure above,
1n the figure four circles, each have a radius
have radius of 3 and
and are tangent to each
are tangent each other.
other.
In above, four with radius
each with radius
are tangent
4, are tangent to each
each other. What is the
other. What area of
the area
ifIf API-I
.6.P HUU is equilateral, what is the area
equilateral, what area of the
shaded region? shaded region?
shaded region?
the
the shaded region?
A) lOrr
A
A)) 16
1 6-- 44rr
7r A) 1 0 "
B
B) 64 - 4rr1
) 6 4 ‐ 4 7 B
B)) 1271
12rr
C
C)) 6 C)) 1471
644 -‐ 88rr
7r C 14rr

D) 64
D) 64‐167r D) 1
D) 15rr
5”
- 16rr

214
THE COLLEGE
THE PANDA
COLLEGE PANDA

(x + 2)2 + (y + 4)2 =4
The equation
The equation of a
a circle the xy-plane
circle in the given
xy-plane is given
above. Which of the
above . Which following must
the following must be true?
be true?
I. The center of the
The center the circle (2,4).
circle is at (2,4 ).
II.
I I , The circle is tangent
The circle tangent to the x-axis.
the x-axis.
III. circle is tangent
The circle
I I I , The tangent to the y‐axis..
the y-axis
A)
A) I]II only
only
AA I I I only
B) III only
C) I andand II 11only
only
Note Figure not
Note:: Figure scale.
drawn to scale.
n o t drawn
D
D)) II,, III, and 111
I , and Ill
If the
If the area the shaded
area of the region in the
shaded region figure
the figure
above 247r and the radius circle 0 is 6,
above is 24rr and the radius of circle O 6, what
what
the value
is the value of x ?
A ) 115
A) 5
B) 30
B)
C) 45
C) 45
D
D)) 6600

A
AA D B

figure above,
the figure
In the circle A is tangent
above, circle circle B
tangent to circle 3
at point D. If
at point If the circles each
the circles have aa radius
each have radius of 4
and A‐C
and tangent to circle
AC is tangent point C, what
at point
circle B at what is
the area of triangle
the area triangle A BC ?7
ABC
A) 8
B ) 8y'2
B) s fi
C) 8v'3
est/5
D ) 116
D) 6

215
Trigonometry
Trigon ometry
24

We'll use a 5‐12‐13
use a right triangle
5-12-13 right triangle to illustrate trigonometric functions
three trigonometric
illustrate the three need to know:
you need
functions you know:

13
5
5~
u
12
12

sinxzmzi
. opposite
sm x = -~---
hypotenuse
hypotenuse
= -
5
13
13
c o s x z n g
adjacent
cosx = -~--
hypotenuse
hypotenuse
12
= -
13 ·
opposite
tanxzopPOSltezi
adjacent
adjacent
= -5
tan X = ----'---''---
12
12

It’s important
It's important to see these
these trigonometric functions as
trigonometric functions as ifif they were ordinary numbers.
were just ordinary numbers. After they ' re just
After all, they’re
1 . . . . .
ratios . For example,
ratios. example, sin 30° is always equal to 5.. Why? Because
always equal right triangles
Because all right triangles With a 300
with a 30° angle similar.
angle are Similar.
2
The ratios sides stay the same.
ratios of the sides same.

4
2

. I
1

\f3
v'3 2 v'3
2\/§
Many students over-complicate
Many students over-complicate trigonometry
trigonometry because
because they treat tan x differently
and tan
treat sin x, cos x, and than regular
differently than regular
Perhaps because
numbers. Perhaps
numbers. because of the notation make mistakes
notation, students sometimes make mistakes
, students sometimes like the following:
following:

sin 2x .
--= : ssm
i n2
X

above is not
The above possible because
n o t possible because sin 2x o n e "entity."
2x is one cannot separate
"entity.” You cannot separate sin and 2x
sin and 2x and
and treat them
treat them
independently just like you
independently separate Jfl (x)
can’t separate
you can't into f and
x ) into and x.

216
THE COLLEGE PANDA
THE COLLEGE PANDA

The definitions
The definitions of sine, and tangent
cosine, and
sine, cosine, are best
tangent are best memorized through the
memorized through c t m y m SOH-CAH-TOA,
the aacronym SOH-CAH-T OA,
S
S formomite
for sine (opposite over hypotenuse),C
over hypotenuse), for cosm'
C for e (adyacenft over
cosine_(adjacent ovet hypotenuse), and T
hypotenuse),and for tangen.
T for t (opposite
tangent(opposite
ov~r
w t)..
e t adjacent)
'

Aside
Aside from the definitions,
from the you should
definitions, you also memorize
should also the following
memorize the very important
following very identity:
important identity:

sinx = cos(90°
sinx cos{90° - x)
Themeverseisalsotrue.
The reverse is also true.
cos x = sin(90 ° - x)
cosx=sin(90°‐x)
Expressedin
Expressed radians,
in radians,

sinx = cos ( - x)
1
sinxzcos(‐72£‐-x) and
and . (1f ) .
cosx=sin(g‐x)
COS X = SlI\ Z - X .

Now, sign of each


Now, the sign each of the trig functions
the trig depends on the
functions depends the quadrant
quadrant in which the angle
which the angle terminates.
terminates.

II I

-----0 -+------ X

m IV

o
• Sine, cosine, and
Sine, cosine, tangent are
and tangent are all in the
positive in
all positive the first quadrant.
quadrant.
0 Only sine
• Only is positive
sine is in the
podfive in quadrant.
second quadrant.
the second
0
• Only is positive
tangent is
Only tangent positive in third quadrant.
the third
in the quadrant.
0
• Only
Only cosine is positive
cosine is positive in the fourth quadrant.
the fourth quadrant.
These are
Those are best
best memorized
memorized through
through the acronym ASTC
the acronym Students Take Calculus).
ASTC (All Students A l l the
Calculus). All the functions
functions am
are
positivein
positive in the first quadrant,
the first quadrant, only is positive
sine is
only sine positive in the
the second,
second, and so on.
and so on.

217
CHAPTER24
CHAPTER 24 TRIGONOMETRY
TRIGONOMETRY

Tofin'd tb:e~,µ~e
find the
' 'I;Q. ofa’ttig
value of foran angle
Extraction'for-an
a qijg p;mdion angle without
:without a'oalculator,
a ·ca1culator, .
' >1.:
la.mewhat&iesigxioffiiemlt
.
shouldbebe (positive ornegative).
J)et ~tiiune w:hatth~cSignof the .results'he.tild or negative).
.2( Find flievzefiexenoemgle
Z Fin,d.the (the acute angle you
referen~~gle (the get by drawing
you get drawing aa straight line to the
straight line the x‐axis). If the
X'-axis). If the
angleis225°,f01'example,tchenel‘erenceanglei5225o
angle = 45°.:
·is 225°, for example, th reference angle is 225° -‐ 180° =45°:
·,
y
'+
l • •
"'-. ' ,,.
·"' .. ,·
.. ,\1,.'' ',· ..
'
J-1•'
... ...
,

''
~

. .
'I
"' :

t l : l~ , l
t ., ,t ,1 ••~ ~ •
1
~--.i..~• :. } ,c,- .. 2250
/'
•/
,1,,,t
:t;,1 Don’t tQemorize
Dontt memorize any
arty formulas for finding the
formulas £bi; the reference angle. Inst
reference angle. draw aa line
Just draw line to -the x-axis and
the x-axis and
fig:u:.re
figmeitoutyomselfl
if out yow:se:lf!
3. Use
3. Useyourworm‐Mspecialfighttinglestogetthetfigvaluefortherefemceangle.
your 45-45--90.or 30-60-,90.special right ttiangles to get the trig value for the refe:r;enceangle.
TheSAT you tocalculate
ask you
won't as~
l'he SAT won't trigvalues
to calculate trig for angles
values for aren't in thesespecial
that aren'tin
angles that these special right triangles
right triangles
unless-ymflmabletouseymcalculator.
~ess ryou' re able t-0use your calculator,.
4.
4. Makesumyourresnlthasthecomectsignfimstepme.
M:akE}sw:e
your result has t:l,;le
c;o,rect sign from step one.

Let’s do aa couple
Let's do simple examples
couple simple examples..

1. What
l. What is the value of sin
the value sin 330°?
quadrant and
fourth quadrant
Since 330° is in the fourth and sine negative in the
sine is negative the fourth quadrant, the
fourth quadrant, the result be
should be
result should
negative.. Now
negative let’ss find
N o w let' find the angle with
reference angle
the reference the help
w i t h the a diagram:
help of a diagram:

330°

The reference angle is 360° - 330° = 30°. Using the 30-60-90 triangle ,

v'3
. 300 opp 1
Stn = - = -
hyp 2

result should
Since the result be negative,
should be negative, sin
sin 330° = ‐12 . I-~/.
218
218
THE COLLEGE
THE PANDA
COLLEGE PANDA

What is the
2. What value of cos 135°7
the value 135°?
Since the second
Since 135° is in the quadrant and
second quadrant cosine is negative
and cosine negative in the second quadrant,
the second quadrant, the should be
result should
the result be
negative. we find
Next, we
negative . Next, find the angle::
reference angle
the reference

y
135°

reference angle
The reference 180° -‐ 135° = 45°. Using
angle is 180° Using the 90 triangle,
45‐45‐90
the 45-45- triangle,

cos 45° = adj = - 1- = v'2


hyp ,/2 2

result should
the result
Since the be negative,
should be negative, cos 135° = 1-1;1.-
: ‐ ‐‐

3. What value of tan


What is the value 210°?
tan 210°7
Since 210° is in the
the third quadrant and
third quadrant tangent is positive
and tangent positive in the third quadrant,
the third the result
quadrant, the be
should be
result should
positive. we find
Next, we
positive . Next, find the reference angle::
reference angle

210°

The reference angle is 210° -‐ 180° = 30°. Using


reference angle Using the - 90 triangle
30‐60‐90
the 30--60 triangle shown
shown earlier,
earlier,

0 11 \/§
)3
t a n 3=0 -° = =‐ -= ‑
tan30
\/5
)3 3
3

result should
Since the result be positive,
should be positive, tan J;
\/3
= I ‐3‐ 1.-
tan 210° =

219
CHAPTER
CHAPTER 24 TRIGONOMETRY
TRIGONOMETRY

• j•
Myywshmldnmoflzeflxefofloufingvahzesformmdw:
Finall~ you should memorize the following values for 0° and 90°: ' I,

~ • t l .,•.¢
sin0 ° =0
sin0°=0 sin90°=lV
sin90 °= 1 ~-, "'i ,,..., 1 .,.t •• - ;, ,I

cos0 ° = 1
cosO°=1 cos90°=0
cos90° = O
tan0°=0
tan.O° = 0 tan =
m190°=undefined
90° undefined

Expmsshagwmmdians,
fag,ressing 90° in radians ,

sin(i) =_1
cos (i) = 0

t¥t (;) = undefined

220
220
THE COLLEGE PANDA
THE COLLEGE PANDA

CHAPTER EXERCISE: Answers for this chapter start on page 332.

A calculator
calculator should
should N O T be
NOT used on the
be used the
following questions.
following questions.
In right
right triangle ABC, the
triangle ABC, measure of AC
the measure is 90°
L'.C is

and AB
and AB = If cosA
= 30. If cos A := ~,
2, what
what is the length
is the length of
of
If what is sin 50° in terms
If cos 40° = a, what terms of a
a??
AC
AC??
A) a
1
B) -
E
a
C) 9
900-‐ a
D) ) a../2
D a fi

a right
In a right triangle, one angle
triangle, one measures x°
angle measures such
x 0 such Iftanx
1f = m,
tan x = m , wha
w h att is
i s sin
s i nxx iinn tterms
e r m sof
o fm
m?
that
that tan x := 0.75. What
tan x°0
the value
What is the value of cos xx°0 ?? 1
A
A) *
) x/m2+1
Jm + 1 2

B
B) )
1 ;
J1 - m2
\/1‐m2
C m
rn
‐ _
C)
) Vm2+l
Jm2 + 1
D m ‐
D)
) x/l‐m2
J1 - m2
sin 0 + cos(90
sinG cos (90 -‐ 0)
6) + cosbl sin(90
cos 0 + sin (90 -‐ 0)
6)
any angle
For any
For angle 9, which of the
0, which the following
following is
equivalent
equivalent to the
the expression
expression above?
above?
B
A) 0
B) 22sin
sin 6
0
C) 2 e
cos 0
2cos 5
2(sin 0 +
D) 2(sin + cos 9)
0)

A C

Given that
Given : 5 and
that AB = tan B = ;g in the
and tan right
the right
triangle
triangle above, what is the
above, what value of
the value
sin B +
sin + cos B
B??

221
CHAPTER 24 TRIGONOMETRY
CHAPTER TRIGONOMETRY

allowed on the following


A calculator is allowed following
questions.
questions. cos32 = sin
cos32 sin(5m
(Sm ‐- 12)
- 12)
In the
the equation
equation above, the angle
above, the angle measures are in
measures are
degrees.
degrees. If
If 0° < m
m < 90°, what
what is the value
the value
B of m
m??

I
A C
C

If sin x°
If sin x 0 = 0.25, what the length
what is the length of BCin
BC in the
the
triangle
triangle above?
above?
y

A( - 3, 5)

N
B C(12, - 3)
B

Right triangle
Right ABC is shown
triangle ABC shown in the xy-plane
the xy-p lane
A C M
CM 0 above . What
above. the value
What is the va lue of cos C ?

In the
In the figure right triangle
above, right
figure above, ABC is similar
triangle ABC similar A _8
8
A)) 17
17
to right
right triangle
triangle M N O , with
MNO, vertices A,
with vertices A, B, and C
B, and
corresponding
corresponding to vertices
vertices M, N, and
and 0,
0,
i8
B)
respectively.
respectively . If B = 2.4, what
tan 8
If tan what is the value of
the value 15
13
cos
cosN?N ? C) E
15
D)1‐7

222
THE
THE COLLEGE PANDA
COLLEGE PANDA

Note: Figure
Note: Figure n drawn to scale.
o t drawn
not scale. A 6 C

the figure
In the abo:7e, cos(90°
figure above, x0 ) =
cos(90 ° -‐ x°) : it;
8
17
. What is
What is
In right triangle
In right ABC above,
triangle ABC cos x°
above, cos : ~-
x0 = g.

the value
the value of cos xx°0 ?? If BC := NE,
If 2 /k, what
what is the value of
the value of k
k?7

A) E8
A)
15
15
17
17
B) E
B) 15

C) 8
C) 8fi
17
15
D) fi
D) 15
17
B

In a right triangle,
a right sine of one
the sine
triangle, the one of the two
of the two

acute angles
acute angles is ?. J;. What is the
What sine of the
the sine other
the other
acute angle?
acute angle?
1
A)
A) 5
2 A C
j3 Note: Figure
Note: Figure n drawn to
o t drawn
not to scale.
scale.
B) 2
the figure
In the above, ABC and
figure above, and DBE are right
DBE are right
1
C)
C) /3 triangles.
triangles . If = 10
If DE = 10and tangent of
the tangent
and the of angle
angle
BAC is 1.25, what the length
what is the of segment
length of segment BE BE ??
D)
D) ,Ii
2

223
CHAPTER 24
CHAPTER 24 TRIGONOMETRY
TRIGONOMETRY

the figure
In the above, AC
figure above, AC is a
a diameter
diameter of the
the
AC := 1,
circle. If AC 1, which the following
which of the gives
following gives
the area of triangle
the area ABC in terms
triangle ABC terms of 00??
0
A) 5
A)
2
tan0
m m)
B
B) ) ‐ 2
2
C) ZsinG
2sin0
sin9c056
sin0cos0
D)
D)
2

Given that
Given that sin
sin 9 = 0, where
0 ‐- cos 0 = where 9
0 is the
the
radian measure of an angle,
radian measure angle, which
which of the
the
following could be
following could be true?
true?
rr
7T
I,0<9<'§
I.O < 0< 2
rr
II. g << e0 << 7r
11. rr
2
3rr
377
III.
Ill. H
rr < 9
0< ‑
22
A)
A) [only
I only
B) II
B) II only
only
C) I and
C) and IIII I I only
only
D
D)) I, I I , and
I , II, III
and III

224
Reading Data
Reading Data
25
The SAT loves to test your
SAT loves ability to read
your ability graphs and
read graphs charts . Fortunately,
and charts. Fortunately, these typically the easiest
are typically
these are easiest
questions because they never involve
questions because they never involve too much math. Most them just
much math. Most of them just test you
you on simple arithmetic with the
simple arithmetic with the
extra step
extra having to interpret
step of having a graph.
interpret a away!
Practice away!
graph. Practice

225
CHAPTER 25 READING DATA

CHAPTER EXERCISE:Answers for this chapter start on page 334.

allowed on the following


A calculator is allowed following
questions.
questions .
Voter Turnout
Turnout in
Congressional
Congressional and Presidential Elections
and Presidential Elections
80
90
Commute Times
75
75_ l ' _ -+-
. _ Congressional Election 1
c, ....
1/)
70
Cl) 70
E. ........
+ Presidential Election |
Presidential Election
..
-" 75 .....
0 65 ~ I

~ > 65
E 60
_g .....60
cii
0
.....
60 "'- ---,,.- -
<II ..... 55
55
E 45
--
0
.....50
~
~
::, 30
B• c::
Cl) 45
45 m.‘ __ ...
~
--- ...__ ---
E ~ -~~
0... 40
Cl)
E D• 40
(3 15 35
I
30
15 30 45 60 75 90 ~ ~
~ ~
8
N
~ ~
~ ~ ~
;::
Commute Time to Work
Commute Ttme
Year
For four
For four work days, Alex plotted
work days, plotted the commute
commute
time to work and
time and the time from work
commute time
the commute work in The graph
graph above shows the
above shows the voter turnout for
voter turnout
the grid
the grid above. which of the four days
above. For which was
days was year a
each year
each a congressional election or aa
congressional election
the
the total commute time to and
total commute and from work
work the presidential
presidential election
election was held. In which
was held. which ttwo
wo
greatest?
greatest? year period
year was the difference
period was difference in voter
voter turnout
turnout
between the congressional
between the congressional election
election and
and the
A
A)) A
presidential
presidential election
election the smallest?
smallest?
B) B
A} 1996 to 1998
A)
C
C)) C
B) 2000 to 2002
D
D)) D
D
C) 2004 to 2006
D) 2008 to 2010

226
THE COLLEGE
COLLEGE PANDA
PANDA

Cream Sales
Ice Cream Sales Average Precipitation in Kathmandu
1.75
,,...._ 1.70 ''f ‑
!_§ 400
350 -----------, f--"--- -------t
...
<J)

_§ 300
..!!l 1.65 '
0 .... 250
-0 ~
..... 1.60 ·5.. 200 f------- ---1'----- -+- -----,
0
1.55 '
<J) ·o 150 ----- --+---- ---- -------<
C ~
.9 1.50 p,.. 100 ---- - ----- -+-----,
ro 50
], 1.45
~ o=------------- --=...,
a n s - u » ~ ogf~u
o m ~ >
<J)

.s:!
(f)
(ll
1.40
1.35
§a
@ § 3 §.............
-~<~
...... 5 i 3<(f)ozo
j :a o..~§:i a5°8 5 ~
Month
Month
N"J x" )x " ,3
q? '\9'\9 line graph
The line graph above the monthly
shows the
above shows monthly
precipitation
precipitation in Kathmandu
Kathmandu last year . According
last year. According
Ky Y’Q \\>Oc Q\\0\'?f’§C} to the graph, the total
graph, the precipitation in September
total precipitation September
was what
was percentage of the
what percentage total precipitation
the total precipitation in
According to the
According line graph
the line graph above, cream
above, ice cream June?
June?
were highest
sales were
sales both in 2013 and
highest both and in 2014 A) 40%
during which three
during which month period?
three month period?
B)
8) 50%
A) January
January to March
March C) 60%
B)
B) April
April to
to June
June D) 75%
C) July to September
September
D) October
October to December
December

227
CHAPTE R 25 READING
CHAPTER DATA
READING DATA

Population
Population in
in 2010 Birth
Birth Rate
Rate

-- South Korea
40 1-E1--.;::--c------~ -+- Japan
San Diego
San Diego

Chicago
Chicago ......
~
1! 20 1--------- ~"-c- ---- ~H
Los Angeles
Angeles t::
co

1 10
10 2006
2006 2008 2010 2012 2014
Population (millions) Year

Researchers created
Researchers created the graph above
the graph compare
above to compare on the graph,
Based on graph, which the following
which of the following best
best
their population estimates with
their population estimates with the actual
actual describes
describes the general trend
general trend in birth rates in
birth rates
populations
populations of different
different cities in 2010. For which
which Korea and
South Korea
South Japan from
and Japan from 2006 to 2014?
of the
the cities did
did the researcher s underestimate
the researchers underestimate
A) Each year, birth
Each year, birth rates decreased in both
rates decreased both
the population?
the population?
South Korea
South Korea and Japan..
and Japan
1, San Diego
I. San Diego B) Each year, birth
Each year, birth rates increased
increased in both
both
II. Chicago
11, Chicago South and Japan.
South Korea and Japan .
Ill Angeles
111.. Los Angeles
C) Each year, birth rates increased
birth rates increased in South
South
A) I only Korea but decreased in Japan.
but decreased Japan .
B) lI and
and 1111only
onl y Each year, birth
D) Each birth rates decreased in South
rates decreased South
C) II and 111
lI and only
III only but increased
Korea but Japan..
increased in Japan
D)
D) I, 11, and
l, II, and IIII
II

228
228
THE COLLEGE
COLLEGE PANDA
PANDA

!
I 30 :
!

I\
28
U)
I \
g 26 -
~

• .It 24
~ I/
·- ~
u
;:l 22 I/ \
• ~
..... I \
,_
- ;


:
1•
0
....
20 >--
..c: 18
t)0
lL
/ 1! ' i..
~
I 16
6 ::
• ~ 14

5 12
10 20 30 40 50 60 70
Age (years) 5
0 2 3 4
Hours since 9:00 A.M.
In a certain study,
a certain researchers created
study, researchers the
created the
scatterplot above
scatterplot above to summarize
Summarize the ages of the
the ages the Starting
Starting at 9:00 A each day,
M . each
A.M. Musa picks
day, Musa picks upup
participants
participants andand the number of hours
the number sleep
hours of sleep packages
packages atat various locations until
various locations his trailer
until his trailer
they required each night.
required each night. Whichof the following
Which of the following truck
truck reaches capacity. He
maximum capacity.
its maximum
reaches its He then
then
is the the age,
closest to the
the closest years, of the
age, in years, the delivers all
delivers packages that he picked up that
the packages
all the that he picked up that
participant who required
participant who required the least amount of
least amount day.
day . The graph above
The graph shows the weight his
above shows the weight of his
sleep each night?
sleep each night? truck
truck at points during
different points
at different during the day. What
the day. What is
A ) 335
A) 5 maximum weight
the maximum
the Musa's truck
weight Musa's truck can hold, in
can hold,
tons?
B) 40
A
A)) 114
4
C)
C) 55
55
D B)
8) 16
16
D)) 6600
C
C)) 224
4
D
D)) 3300

229
CHAPTER READING DATA
CHAPTER 25 READING DATA

Annual Salt Production in the U.S. Video


Video Game
Game Console
Console Sales in 2015
ui'
§
.....
u 40
.E ui' 250
QJ "O
E @
CJ)
200
"O
@ g
CJ)
;:l
0 20
20
2 150 L
"d
2 ]
i::
0
·.uc 10
;:l
"d
0
M
p... 0
2009 2010 2011 2012 2013 2014 2015 A B C D E
Year Console
Based the graph
Based on the above , for which
graph above, the
which of the The graph
The graph above shows the
above shows the number units sold
number of units sold
following
following ttwo consecutive years
w o consecutive years was the percent
was the percent in 2015 for five different video game
different video consoles.
game consoles.
increase in U.S.
increase U S . annual salt production
annual salt production the
the same
same
The prices
The prices of consoles
consoles A, B,C,
B, C, D, and
and E are
are $100,
as percent decrease
the percent
as the from 2010 to 2011?
decrease from and $300, respectively.
$150, $200, $250, and respectively. Which
Which
A) 2009 to 2010 consoles generated
of the five consoles generated the total
most total
the most
B) 2012 to 2013
B) revenue?
revenue?
C) 2013 to 2014 A) A
A)
D)
D) 2014 to 2015 B) B
D
C) D
D) E

ui'
E ro
~ 400
.s
ro
M
..0 300
......
0
CJ)
CJ)
ro 200
E
QJ
00
<13
M
QJ

~
Cow Wolf Goat Cat
Cat Pig
Animal

According
According to the graph above,
the graph the average
above, the mass
average mass
of a wolf's brain is what
wolf's brain what fraction
fraction of the average
the average
mass of aa pig’s
mass brain??
pig's brain

230
THE COLLEGE
IBE PANDA
COLLEGE PANDA

State Health Care


State Health Spending in 2013
Care Spending
20 -e-CompanyX
,....,_ -+-- Company Y
...
Ill
_- Hospital Care I
~
0
-0 15
~ 15
~
I --Prescription Drug I
..... 0
0 "C
0
le
0 ~ 10
:5 10 ~
I..... e00
..;:: .s 5
e
ll..
5 -g
Q)

~
0
2 3 4
AL AK AZ AR
Quarter
Quarter State

The graph
The graph above
above shows profit of Company
the profit
shows the Company X graph above
The graph
The shows the
above shows health care
the health spending
care spending
and Company
and Company Y in each
each quarter last year.
quarter of last year. In of four
four different Alabama (AL),
states, Alabama
different states, (AL), AK
which quarter was
which quarter Company X's
was Company profit twice
X’s profit twice (Alaska),
(Alaska), AZ (Arizona), and AR (Arkansas)
(Arizona), and (Arkansas) in
Company Y's?
Company Y’s? Based on
2013. Based on the graph, which
the graph, state had
which state the
had the
A) 1 highest combined hospital
highest combined hospital care and prescription
care and prescription
drug spending in 2013?
drug spending
B) 2
Alabama
A) Alabama
C) 3
B) Alaska
Alaska
D) 4
C) Arizona
C) Arizona
D) Arkansas
Arkansas

231
CHAPTER READING DATA
CHAPTER 25 READING DATA

~
...
... 10 ~ 190
;:l ·c ,_ --
0
Q)
"O 180 --
:r:
...
..._,_ i\
8 "' 170
Q)
E
0.. J \
-0
...
Q)
6
j"' 160
l/ \ I I
i [\
Q) 150
], J ~ ._ ..
<
~
"'
i::

.!!?
4 ~
>
Q)
.....l
140
130
' .. ~

<O
u 2
Q)

"'
0
120
u 110
;:l
G '
0 1 2 3 4 5 6 7 0 1 2 3 4 5 6 7 8 9
Time (hours) Time (Hours after 8:00 A.M.)

Jeremy works
Jeremy call center.
works at aa call graph above
The graph
center. The above the day
On the day of a medical evaluation, Greg
medical evaluation, Greg ate
ate
the average
shows the
shows number of calls he
average number he answered
answered breakfast
breakfast at 8:00 A.M. and lunch at
and lunch at 12:00 PM.
P.M.
per during his
hour during
per hour his 7-hour shift. What
work shift.
7-hour work is
What is During
During each meal,, doctors
each meal recorded his
doctors recorded glucose
his glucose
the total
the number of calls he
total number he answered during his
answered during his levels the graph
levels in the until they
above until
graph above able to
were able
they were
shift?
shift? calculate the glucose
calculate the glucose recovery
recovery time, the time it took
the time took
for the body 's glucose
the body's level to return
glucose level return to its
its initial
initial
value at
value at the start of the
the start meal. According
the meal. According to the the
graph, by how
graph, how many
many hours
hours was Greg's glucose
w a s Greg's glucose
recovery time after
recover y time lunch greater
after lunch greater than his
than his
glucose recovery time after breakfast?
glucose recovery time after breakfast?
A) 1.5
B) 2
C) 3
D) 5.5

232
THE COLLEGE
THE PANDA
COLLEGE PANDA

Car X

§ 55
V -........_
=;;; 50
~
bO45 7
I/
i 40 ,
~ 35
7 I
] 30
/
'; 25
~ 20
I\. 7
17
?
~

t::
15
10 ' '
~ 5
!
l?
0 10 20 30 40 50 60 70
Speed (miles per hour)

The graph
The above shows
graph above shows the gas mileage
the gas mileage for Car
Car
X at different
at different speeds.
speeds . Based
Based on
on the
the graph,
graph, how
how
many gallons of gas
many gallons gas are needed to drive
are needed drive Car X
Car X
constant speed
hours at a constant
for 5 hours speed of 30 miles per
miles per
hour?
hour?

233
Generally probability can
speaking, probability
Generally speaking, be defined
can be as
defined as
26
Probability
Probability
number target outcomes
number of target outcomes
number of total
number possible outcomes
total possible outcomes

Nearly probability questions


Nearly all probability on the
questions on will involve
the SAT will tables of data.
involve tables 80 for the
data. So purposes of the
the purposes the SAT,

‐-‑
probability can
probability can more narrowly be
m o r e narrowly be defined as
defined as

number target group


number in target group
number in group
number under consideration
group under consideration

EXAMPLE 1:
EXAMPLE
Beef Oricken
First Class 18 27
Coach 62 138
table above
The table
The above summarizes the meat
summarizes the preferences of passengers
meat preferences passengers on aa particular
particular flight. If a
flight. If a first class
is chosen
passenger is
passenger at random
chosen .at from this
random from flight, what
this flight, is the
what is probability that
the probability that the passenger chosen
the passenger prefers
chosen prefers
hoofl
beef? ·
9 2 3
POE
A)~ B)§
B) ~ C)§~
C) D)§2
D)~
3
40 5 5

number of first class passengers


The number 18 + 27 =
passengers is 18 : 45. This the group
This is the under consideration.
group under number of
The number
consideration. The
passengers who
class passengers
first class prefer beef
who prefer This is the
beef is 18. This group.
target group.
the target

number in target
number group
target group 18 2
18
group under
number in group
number under consideration
consideration 45 =
45 T 5

Answer (B) .
Answerefm.

234
234
‐-‐‑
THE COLLEGE
THE PANDA
COLLEGE PANDA

m m . 2:- The
EXAMPLE Themanagerofalargeassemblylineusesthetablebelowtokeeptrackoffliemmmerof
manager of a large assembly line uses the table below to keep track of the number of
vehicles that
vehicles that are produced during
are produced different shifts
during different the clay.
shifts in the day.
. .:'
Cars Trucks Total
-
First,shift • 173 126 299
Second shift 182 143 325
“m
Third shift 165 109 274
“Total m
520 378 898m
If a vehicle is following is closest
is selected at random at the end of the day, which of the following to the probability
closestto probability
thatthevehiclewillbeeitheracarproduceddufingthefirstshiftoratruckproduced
that the vehicle will be either a car produced during the first shift or a truckproduced duringthethird
during the third
shift?

A) 0.193
A)0.193~ B) 0.314
•B) 0.314 C) 0.352
(30352 D) 0.421

question, the
this question,
In this group under
the group under consideration includes all
consideration includes vehicles, aa total
the vehicles,
all the at the
total of 898 at the day.
end of the
the end day.
target group
The target
The group includes
includes cars produced during
cars produced during the
the first
first shift and trucks
shift and produced during
trucks produced during the shift, aa
third shift,
the third
total + 109 =
total of 173 + = 282 vehicles
vehicles..

target group
number in target
number group = 282 ~
. . . ‐ x 00.314
314
number in
number group under
m group con51deration
under consideration 898 ·

Answer (B) .
Answer~.

235
235
CHAPTER 26 PROBABILITY
CHAPTER PROBABILITY


CHAPTER EXERCISE:
EXERCISE: Answers this chapter
Answers for this start on page
chapter start page 336.

A calculator is allowed
allowed on the following
following questions.
questions.

Violation Type
Violation
Speeding Stop sign Parking Total
Truck
Truck
Car
m 68
83
39
51
‐ 17
26
‐ 124
160

Total 151 90 43 284

district police
A district police department
department records driving violations
records driving violations by type
type and vehicle in
and vehicle in the
the table According
above.. According
table above
to the
the record,
record, which the following
which of the is closest
following is to the
closest to the proportion of stop
proportion of stop sign committed by
violations committed
sign violations by truck
truck
drivers?
drivers?
A) 0.137
B) 0.315
C) 0.433

----‑
D) 0.567

Color
Color Red
Red Blue Black White
White Silver
Percent 20% 33% 10% 14%

A car manufacturer produces


car manufacturer cars in red,
produces cars red, blue, black, white,
blue, black, and silver.
white, and The incomplete
silver . The table above
incomplete table above shows
shows
the
the percentage cars it produces
percentage of cars each color. If
produces in each If a car from the
car from the manufacturer chosen at
manufacturer is chosen random,, what
at random what is
the probability
the probability that
that the
the car’s color is red
car 's color red or silver?
or silver?
A) 23%
B) 33%
C) 37%
D) 43%

236
THE COLLEGE PANDA
THE COLLEGE PANDA

-------------------T-------------------
Questions
Questions 3-4 refer to the following
following information.
information.
The table below
The table shows the
below shows number of workers
the number with at least
California with
workers in California year of experience
one year
least one experience in five different
different
construction-related occupations.
construction-related occupations.

‐‐ ‐ - - -‐ ‐- ‐ ‐ | ‑|
Years of Experience
Experience
1 2 3 4 5+ Total
Painter 22,491 26,973 29,086 33,861 37,061 149,472

‐Roofer - 23,908 -27,634 30,932 - 34,146 ‐ 39,718- 156,338 |


‐Welder - 27,062 -29,812 -
32,784 -
36,902 42,680
- 169,240
|
‐Plumber - 28,637 - 28,806 - 34,867 ‐ 37,418 - 43,922 - 169,409 |
33,119 36,670 40,083 45,376 183,885

‐Carpenter - 24,396 - ‐ - - |
I‐Total -
1 126,494 1 -
146,344 1 -
164,339 1 ‐
1s2,410 1 -
2os,757 11s2s,344 |1

Based a plumber
table, ifif a
the table,
on the
Based on California is chosen
plumber in California at random,
chosen at following is
the following
which of the
random, which closest to
is closest the
to the
probability that the plumber has at least four years experience?
probability that the plumber has at least four years of experience?
A) 0.10
B) 0.22
C) 0.25
D) 0.46

IfIf a with at
worker with
a worker least four
at least experience is chosen
years of experience
four years random from
at random
chosen at those included
from those the table,
in the
included in which
table, which
of the following is
the following closest to
is closest the probability
to the probability that person is a
the person
that the a plumber?
plumber?
A) 0.10
B) 0.22
C) 0.25
D) 0.46

_ ‐ ‐ _ _ _ ‐ _ A _ ‐ ‐ ‑

237
CHAPTER
CHAPTER 26 PROBABILITY

The table
PROBABILITY

table above results of


shows the results
above shows
favorite (expected
favorite (expected to win) in
to win)
‐-‑
Underdog
Underdog
Favorite

Total

of a
a baseball
-
Won

baseball team,
game or the underdog
in the game
10
25
35 -
Lost
35
5
40
Total
45
30
m
75
categorized by whether
team, categorized whether the team
(expected to lose). What
underdog (expected
was considered
team was considered the
fraction of the
What fraction
the
games in
the games
which
which the team
team was considered the underdog
was considered underdog did
did the team win?
the team
2
A} -
5
2
B) 7
2
C) -
9
2
D) 15

‐ W e e Week
k 11 Week2 W
Week3e Week4
e k Total
s ‑
‐ Box springs
mMattresses
- 35
47
W

m 61

een
40
68
- 55
‑ 198

‐ Total ‐ 82
‐ 101 ‐ 88 m 77
m
348

store manager
A store summarizes the
manager summarizes the number spring and
number of box spring units sold
mattress units
and mattress sold over weeks at a bedding
four weeks
over four bedding
store in the incomplete
store incomplete table above . Weeks 2 and
table above. and 3 accounted
accounted for what fraction of all box spring
what fraction units sold?
spring units sold?
2
A
A) ) 15‐
4
B
B) ) 15‐

2
C) 5

4
D)
D) 5

238
THE COLLEGE PANDA
THE COLLEGE PANDA

Country Gold Silver Bronze Total


USA 46 29 29 104

m China
Russia
38
24
27
26
23
32
88
82
Great Britain 29 17 19 65
Germany 11 19 14 44
Total 148 118 117 383

The table
The table above
above shows the distribution
shows the medal s awarded
distribution of medals awarded at the 2012 London
at the Summer Olympics.
London Summer Olympics . If an
an
Olympic
Olympic medalist be chosen
medalist is to be chosen atat random from one
random from countries in the
the countries
one of the which country
table, which
the table, country gives the
gives the
highest probability
highest selecting a
probability of selecting medalist?
Bronze medalist?
a Bronze
A) USA
USA
B) Russia
Russia
C) Great
Great Britain
Britain
D) Germany
Germany

Number
Number of Fish
Fish Species
Species
Cartilaginous
Cartilaginous Bony
Bony
Philippines
Philippines 400 800
Caledonia
New Caledonia
New 300 1,200

AU
A be categorized
can be
l l fish can categorized as cartilaginous or
either cartilaginous
as either bony . The
or bony. data in
The data in the above were
table above
the table produced by
were produced by
biologists studying the fish species in the Philippines and New Caledonia. Assuming
biologists studying the fish species in the Philippines and N e w Caledonia. Assuming that each fishthat each species
fish species
has an
has an equal being caught, the
chance of beingcaught,
equal chance the probability catching acartilaginous
of catching
probability of fish in
a cartilaginous fish Philippines is
the Philippines
in the how
is how
much greater
much the probability
than the
greater than probability of catching one in N
catching one New Caledonia?
e w Caledonia?
2
A)
A) 15
1
B) 4
3
C) 10
1
D) 3

239
26 PROBABILITY
CHAPTER 26
CHAPTER PROBABILITY

‐ u g h m g -Lightning-caused
c a u s e d fi m s
fires Human-caused fires Total
East Africa
East Africa 65
65 ‑
South Africa
South Africa 30
30 ‑
Total 135 220

The incomplete
The table above
incomplete table above summarizes
summarizes thethe number wildfires that
number of wildfires occurred in ttwo
that occurred regions of Africa
w o regions Africa in
2014 by cause. Based on the
cause. Based what fraction
table, what
the table, fraction of all wildfires in East
all wildfires Africa in 2014 were
East Africa were human-caused?
human-caused?
11
A ) 24‑
A)

~~
B ) ‑
B)

~!
C ) ‑
C)
11
D ) 15‑
D)

Defective Not defective Total


Assembly Line
Assembly A m_ 300
L i n eA 5,700 6,000
Assembly Line
Assembly LineBB m_ 500 3,500 4,000
Total 800 9,200 10,000

A manufacturer
manufacturer uses
uses two assembly lines
t w o assembly produce refrigerators.
lines to produce refrigerators. The
The results
results of each
each assembly line's quality
assembly line’s quality
control are shown in the table above. If
control are shown the table above. a refrigerator from the manufacturer turns o u t be defective, what
a refrigerator from the manufacturer turns out to be defective, what
the probability
is the that the
probability that refrigerator was
the refrigerator produced by
was produced Assembly Line
by Assembly Line A?
A?
A) 5%
A) 5°/o
B) 37.5%
C) 60%
D)
D) 62.5°/o
62.5%

240
THE COLLEGE
THE COLLEGE PANDA
PANDA

‐ _ Type of Residence

Family members Apartment Duplex Single residence
_ ‐‐ -- ‐‐
Total

_
1
2
10
20
22
12
3
13
35
45
‑‑
_ 3 - 8 n 8 ‐
12 28 ‑
‐Total
4 or more - 8 n 46 4 _
18 30 ‑
‐ - 46 m 46 138
The table
The table above summarizes the
above summarizes distribution of living
the distribution residences in a
situations for residences
living situations a neighborhood.
neighborhood. If a a
duplex in the
duplex neighborhood is to be
the neighborhood inspected at
be inspected at random, what is the
random, what probability that
the probability the residence
that the residence is
occupied by no
occupied no more family members?
than 22 family
more than members?
2
A) 23

B)
3) 6
23
17
C)
C) 69
17
D )23‑
D)

samples
Percent of samples
Percent
Number of soil
Number samples
soil samples
with Chemical
with Chemical A
Area 1 450 8%
8%
Area 2 550 6%

The data
The data in the table above
the table above were produced by
were produced by ecologists
ecologists who
who collected
collected soil samples from
soil samples from two areas to
t w o areas
determine whether they
determine whether they were contaminated w
were contaminated i t h Chemical
with Based on
Chemical A. Based the table,
on the table, what proportion of the
what proportion the
soil samples were
soil samples contaminated with
were contaminated with Chemical
Chemical A?
A) 0.067
B) 0.069
C) 0.070
D) 0.072

241
CHAPTER 26
CHAPTER 26 PROBABILITY
PROBABILITY

Test negative Test positive Total


Has v.
Has
Does
virus
..
o t have
Does nnot virus
have virus
‐_ 30
550
‐_ 370
50
m
Total
400
m
600
Total 580 420 1,000
1,000
The table above
The table above shows results of
the results
shows the of a
a test is designed
that is
test that to give
designed to give a positive indicator
a positive patients are
when patients
indicator when are
infected with a certain
infected with certain virus and a negative
virus and negative indicator when they
indicator when they are infected. According
o t infected.
are nnot According to the results,
to the results,
what is the probability
what that the test gives
probability that gives the incorrect
incorrect indicator?
indicator?
A) 5°/o
A) 5%
B) 8%
B) 8°/o
C) 10%
D)
D) 12%

Cured
Cured Not cured
Not cured
Drug 90 25
Sugar Pill
The incomplete
incomplete table
table above the results
shows the
above shows study in
results of aa study which doctors
in which gave patients
doctors gave experiencing back
patients experiencing back
pain either aa drug
pain either drug or a sugar pill.
a sugar pill. Three
Three times as many
times as many patients were cured
patients were cured from
from the drug than from
the drug than from the the
every 2 patients
sugar pill. For every
sugar patients cured
cured byby the
the sugar pill, 5
sugar pill, patients by sugar pill.
5 patients were not cured by the sugar
were n o t cured the According
pill. According
to the results, if a
the results, patient is given
a patient sugar pill, what
given a sugar what is the
the probability
probability that
that the be cured
w i l l be
person will
the person of back
cured of back
pain?
pain?
1
A)?I
A) 4

B);2
B) 7

C) 3
C)13‐0
10

D);2
D) 5

242
242
‐‐‑
THE COLLEGE PANDA
THE COLLEGE PANDA

Gym equipment
Gym equipment Computers Total
-Juniors ‐240 " 300 m
540
Seniors
seniors 160
Total 460

The principal school is deciding


principal of aa school deciding whether
whether to spend a budget
spend a budget surplus
surplus on nnew gym equipment
e w gym computers .
equipment or computers.
The incomplete summarizes the preferences
above summarizes
incomplete table above preferences among
among junior
junior and
and senior students . If
senior class students. If a senior
a senior

school is chosen
from the school chosen at the probability
random, the
at random, probability that student prefers
the student
that the prefers gym equipment is 33.
gym equipment ~- HHow
ow
many
many seniors are at the
seniors are the school?
school?

243
243
Consider numbers:
list of numbers:
this list
Consider this
27
Statistics
Statistics II
5,6, 2, 2, 2, 7
5,6,2,2,2,7

The m e a n of the
The mean list is the
the list the average:
average :
5+ 6 + 2 + 2+ 2+ 7
‐ _ 6 _ 4
The
The median
median is the number in the
the number the middle list is in order.
the list
when the
middle when example, the
For example,
order. For median of
the median {1, 2,
of {1, 2,3,4,5} is 3.
3, 4, 5} is 3.
For o
For particular list,
u r particular
our list, which
which looks like
looks like
2,2, 2, 5, 6, 7
2,2,2,5,6,7

when
when ordered, there is no
ordered, there single middle
no single middle number
number we we can
can consider the median.
consider the because there’s
That's because
median . That’s even
an even
there's an
number
number of numbers
numbers in the list.. When
the list that’s the
When that's the median
case, the
the case, the average
median is the average of the t w o middle numbers, 2
the two middle numbers,
and
and 5:
22 +5
T‐+ 5 _ '357
3.5
2 -l..::'.::::J

Now what if the


Now what the list long? How
numbers long?
were 100 numbers
list were would we
How would we determine
determine the median? Well, if we’re
the median? looking for
we're looking for
the middle
the number, it would
middle number, would make sense to take
make sense half of 100 to get
take half designates the
which designates
get 50, which number. But is
50th number.
the 50th is
the 50th number
the 50th number the median? Probably
the median? Probably nnot since there’s
o t since an even
there's an even number numbers. Maybe
number of numbers. Maybe it’s the average
it's the average
of the 50th and
the 50th numbers . Or is it the
Slst numbers.
and 51st average of the
the average 49th and
the 49th 50th numbers?
and 50th See, with
numbers? See, large lists,
with large it’s hard
lists, it's hard
to tell.
Here's my technique
Here’s technique for getting the median:
getting the regardless of whether
median: regardless there's an odd
whether there’s number of numbers,
even number
odd or even numbers,
we always ”go up.”
always "go So for a list
up." So numbers, we
list of 100 numbers, divide by 2 to get
we divide and “go
get 50 and Since we
up" to 51. Since
"go up” have
we have
w o whole
ttwo whole numbers,
numbers, 50 and 51, the
50 and median is the
the median the average the 50th
average the and 5lst
50th and numbers. For
51st numbers. 101 numbers,
For a list of 101 numbers,
we divide
we get 50.5 and
divide by 2 to get and "go up” to 51. Since
”go up" we only
Since we have one
only have whole number,
one whole median is the
the median
number, 51, the 51st
the Slst
number.
number .
this technique
Yes, this weird but
technique is weird but it works. Just for reassurance,
works . Just let's test
reassurance, let's test it out (the median
numbers (the
out on a list of 3 numbers median
is obviously
obviously the 2nd number) and a list
the 2nd number) and a list of 4 numbers (the median is average of the 2nd
numbers (the median the average the 2nd and 3rd numbers).
the and 3rd numbers).
For a list
For list of 3 numbers,
numbers, we we take half of 3 to get
take half get 1.5, and "go up”
then ”go
and then up" to 2, which confirms that
which confirms the 2nd
that the number
2nd number
is the median. For
the median. For a list of 4 numbers,
numbers, we take half
take half of 4 to get and then "go up”
get 2, and then "go up " to 3. The whole numbers 2
The whole numbers
and
and 3 confirm
confirm that the average the 2nd and 3rd numbers the median.
that the average of the 2nd and 3rd numbers is the median.

The mode is the


The mode number that
the number shows up
that shows up the our particular
often. In our
most often.
the most it’s [I].
list, it's
particular list, ..

244
THE
THE COLLEGE PANDA
COLLEGE PANDA

range is the
The range
The difference between
the difference the biggest
between the biggest number the list and
number in the the smallest
and the number :
smallest number:

7‐2:‑
7 - 2 =~

The standard
The standard deviation a measure
deviation is a how spread
measure of how spread oout numbers is. In
u t a list of numbers In other words, how
other words, much the
how much the
numbers ”deviate”
numbers "deviate" from
from the mean. The standard
the mean. lower when
deviation is lower
standard deviation more numbers
when more are closer
numbers are the mean.
closer to the mean.
The standard deviation
The standard higher when
deviation is higher when more numbers are
more numbers are spread
spread out away from
out away mean. For
the mean.
from the example, oour
For example, ur
list
2, 2, 2, 5, 6, 7
would have a higher
would have higher standard deviation than
standard deviation the following
than the following list

5, 5, 5, 5, 6, 7
5,5,5,5,6,7

because
because the the second more tightly
second list is more tightly clustered around the
clustered around turns oout
mean. It turns
the mean. u t that the standard
that the deviation
standard deviation
u r list
of oour list is 2.28 and the standard
and the standard deviation
deviation of the
the second worry about
Don't worry
second list is 0.83. Don’t about how we got these
how we these
values‐you’ll
values-you'll never be asked
never be asked to
to calculate deviation on
standard deviation
the standard
calculate the on the Just know
SAT. Just
the SAT. how to
know how compare one
to compare one
list’s standard deviation
list's standard deviation with another's as
with another's aswe did.
just did.
we just

EXAMPLE1:
EUMPLB 1: . . .'
i Daily Hours
Daily Spent Playing
Hours Spent Playing Sports
Sports

1----
130
tl)

... o 20 ---

fz 10 --

0 1 2 3
I• '-·. 'L
Hours

histogram above
The histogµun above summarizes daily number
the daily
summarizes the hours spent playing
number of hours sports for 80 students
playing sports at aa
students at
school.
school.

PART 1: What
PART is the
What is mean daily
the mean number of hours
daily number hours spent playing sports
spent playing sports for the 80students?
80 students?

PART 2: What
What is the median daily
the median number of hours
daily number spent playing
hours spent playing sports students?
for the 80students?
sports forthe'80

Part 1 Solution: up the total


Sum up
Solution: Sum total number
number of hours student. Then
every student.
hours for every divide that
Then divide number of
the number
that by the
students..
students
Total hours
Total hours (0x5)+(1x35)+(2x15)+(3><25)
= (0 _ 140 =
x 5) + (1 x 35) + (2 x 15) + (3 x 25) = _ I1.75 I
Number
Number of students
students ‐ 80 _ 80

Solution: In a group
Part 2 Solution: group of 80 students, the
80 students, 40th and
the 40th 41st students
and 4lst are the ttwo
students are w o in the middle (the
the middle histogram
(the histogram
already orders the
already orders students by their
the students so we don’t
hours so
their hours students spend
have to). The first 5 students
don't have hours playing
spend 0 hours playing
sports each day. The next
sports each spend 1 hour.
students spend
next 35 students hour. This group
group includes student, so
40th student,
includes the 40th 40th student
so the 40th student
spends 1 hour.
spends hour. The next
next 15
15 students
students spend
spend 2 hours.
hours. N o
Now w this group
group includes
includes the
the 41st
4lst student,
student, so
so the
the 41st
4lst
student spends
student spends 2 hours.
hours. Taking
Taking the average,
average,

Daily
Daily hours spent by 40th
hours spent Daily hours
student + Daily
40th student hours spent
spent by 441st
l s t student
student _ 1 + 2 _ l1ci
2 -= 22 2 -~

245
CHAPTER
CHAPTER 27 STATISTICS
STATISTICS I

EXAMPLEZ:
"EXAMPL'E2: /
O
O
O O
0 0 Q
., . ~- •1 ~ I
0 O 0
':~f·•.,1 '••J, :/ +'! \ I
O O“ 0 Q
•6
·, ,: 11-1 ,,' I

.! o o o o o
0o 11 2 2 33 4 4 5 5 6
Flights'I‘akeninaYear
Flig,hts Taken ih a Year
, I\ ·.• I• I ., • ,

The dot plot above


dot P,lot above summarizes the number
sumtnarizes the of flights
number-of taken in a year by 19 college students.
flights taken students. If the student
If the student
who -took flights in
took 6 Bights a year
in a is remov-ed
year is removed from
from the data, which of the
datfl, which the following
following correctly
correctly describes
describes the
changes to the statistical measures of the data?
chc:Utges data?
·· I.I Tue mean1decreases.
The tneatt d~as~s
/"'
. ·
-\ t
n. 1-¥'.J. . : , >''1

median decreases.
I I . The median decreases.
114 i~ I' ~

,,.m. Therartge
III. The d~creas~ ~
range decreases.

1,»leme •
B)Iandllon1y
B.)I and.ll.ol'.d
.r •~
y
I ,I
C)IandIIIon1y
qr.andIDQnly D)I , ll,andW
D)I,II,andIlI

The took 6 flights


who took
student who
The student flights in a year
year is called an
is called outlier, an
an outlier, data point
extreme data
an extreme that is
point that where
outside where
is far outside
the data
most of the
most lies. Because
data lies. Because this outlier is greater
this outlier greater than
than the data, itit brings
rest of the data,
the rest the average (mean)
brings the average (mean) up.up . It
also
also increases the range
increases the range since larger gap
there is a larger
since there between the
gap between the minimum (0) and
mi:nimwn (0) and the maximum (6).
the maximum
When this outlier
When this removed, the
outlier is removed, the mean decreases and
mean decreases the range
and the decreases. The
range decreases. median, however,
The median, unaffected..
however, is unaffected
To confirm let's calculate
this, let’s
confirm this, outlier is removed,
the outlier
Before the
calculate it. Before removed, there are 19
there are 19 students, the median
and the
students, and median isis
represented by the
represented 10th student,
the 10th who took one flight. After the outlier removed, there are 18 students, and
student, who took one flight. After the outlier is removed, there are 18 students, and
the
the median represented by the
median is represented the 9th
9th and 10th students,
and 10th both of whom
students, both whom took one flight. 50
one flight. median of 1 does
the median
So the does
n o t change.
not change. A n d in fact, outliers
And outliers typically the mean
typically affect the but
mean b n o t the
u t not median. Answer
the median. (C) .
A n s w e r [@]

EXAMPLE 3:
~PLE 3: The average weight of a
avetage weight group of p'illt~~
a group pandas is 200· pounds.
ts 200 pounds. Another panda, weighing
Anqther panda, w.eighing 230
pounds,jjoins
pQ~ds the group,
oins:the raising the
grqup., :rais-mgthe average weight of the
average wej,ght the entire group to 205 pounds.
entire ..group ..How
poun.ds H o w many pandas
matty pandas
wei;e·in
were fl),eoriginal
ht the group? .
Qti~a.l \~oup'? ,

while, you
Once in a while,
Once will get a
you will word problem
a word problem that involves averages.
that involves These questions
averages . These questions have less to do w
have less with
ith
statistics and
statistics o r e to do with
more
and m algebra, but
with algebra, but because cover averages
because we cover this chapter,
averages in this decided
chapter, we decided to cover
cover
types of word
these types
these word problems here as
problems here well.
as well.
When dealing
When dealing w average questions
i t h average
with questions on the SAT, think
think in terms of sums
sums or totals. always find the
totals. You can always
s u m by multiplying
sum multiplying the average
average with subjects.
number of subjects.
with the number
the number
Let the pandas in the
number of pandas original group
the original group be r. The
be x. total weight
The total the original
weight of the original group then 200x.
group is then When
200x . When
another panda
another panda joins the group, the
the group, the number
number of pandas + 1 and
pandas is x + total weight
the total
and the + 1).
weight is 205(x + 1).
Since
Since that panda weighs
that panda weighs 230 pounds,
pounds ,

200x + 230 = : 205(x + 11))


200x +
+ 230 = 205x ‐l‐ + 205
-‐ 55xx =
= -‐ 225
5
: 5
x= S
There were [fil pandas
There were the original
panda s in the original group.
group .

246
246
THE COLLEGE
THE PANDA
COLLEGE PANDA

EXAMPLE4:
Neighborhood A Neighborhood B
r,._•
...
.,, ,·
j......30
!
cl 20
t
l 10

'0‘123456
I "'

0 1 2 3 4 5 6
Numberofcars
Number of cars owned
awned Number of cam
Nw;nber cars owned
owned
bar charts
The bar summarize the
above summarize
charts above number of cars that
the number residents from ttwo
that residents neighborhoods, A
w o neighborhoods, and B,
A and B,
own. Which of the
own. Which the following correctlycompares
following correctly the standard deviation of the m1mbei of cars
compares the standard deviation the number of cars ownedb o
- wned by
the neighborhoods?
residents in each of the
tesidents neighborhoods? · • .
fhe standard
A) The standard deviation
deviation of the ca:rsowned
number of cars
the number residents in
owned by residents A is larger.
eighborhood A is larger.
in Neighborhood
B) The
B) deviation' of
standard deviation
The standard the
o.fi number cars owned residents Neighborhood"B-
number of cars oWhed by residents in Neighborhood Bisis larger.
larger.
C) The standard
standard deviation owned by residents
number of cars owned
deviation of the number Neighborhood A
in Neighborhood
·residents .in and
A and
Neighborhood B
Neighborhood Bis
is the same.
same. .
relationship cannot
D) The relationship cannot bebe determined
determined from information given.
from the information given.

Most of the
Most data for Neighborhood
the data Neighborhood B are at the
are at ends and
the ends are much
and are more spread
much more out from
spread out the mean,
from the which,
mean, which,
the bar
because the
because graph is symmetrical,
bar graph we can
symmetrical, we estimate to be
can estimate be 3 cars. The data
cars. The data for Neighborhood A, on
Neighborhood A, on the
the
other
other hand, are more
hand, are clustered towards
more clustered low end,
the low
towards the end, where mean
the m
where the is. Therefore
e a n is. , the standard deviation
Therefore, the standard deviation forfor
Neighborhood
Neighborhood Bis
B is larger. Answer ~-(B) .
larger. Answer

Boxplots
Boxplots
Every nnow
Every o w and boxplot question
then , aa boxplot
and then, shows up
question shows up on SAT. Like histograms
the SAT.
on the dotplots, boxplots
and dotplots,
histograms and are just
boxplots are just
another way
another way of visualizing numerical data. There are 5 statistical metrics that you need to construct one: the
visualizing numerical data . There are 5 statistical metrics that you need to construct one: the
m
minimum, quartile,, the
the first quartile
i n i m u m , the the third
median, the
the median, and the
quartile, and
third quartile, maximum .
the maximum.
an example
's an
Here’s
Here boxplot that
example of a boxplot that summarizes
summarizes the weights of 30 tortoises:
the weights tortoises:

Min
Min Q1
Ql Median
Median Q3
Q3 Max
t ‐ ‐ ( : : l ‐ ‐ i
q
r + ‐+‐‐ + + 1 l
10
10 20
20 30
30 40
40 50
50 60 70
70 80
Weight (in pounds)
Weight pounds)

The left and


The sides of the "box"
right sides
and right represent the
”box” represent third quartiles,
and third
the first and and the
respectively, and
quartiles, respectively, line segment
the line segment
the box
inside the
inside indicates the
box indicates segments are
median. Line segments
the median. drawn from
are drawn the left and
from the right sides
and right of the
sides of box to
the box the
to the
minimum and
minimum and the maximum.. These
the maximum segments are
These segments sometimes called
are sometimes whiskers.
called whiskers.
From the
From boxplot, we can
the boxplot, can see that the
see that weight among
minimum weight
the minimum among thethe 30 tortoises pounds, the
tortoises is 15 pounds, quartile
the first quartile
is 30
is 30 pounds, median is 40
the median
pounds, the 40 pounds,
pounds, the quartile is
third quartile
the third is 60 pounds, and the maximum is 70 pounds..
60 pounds, and the maximum is 70 pounds

247
CHAPTER
CHAPTER 27 STATISTICS
STATISTICS I

N o w before
Now we get
before we ahead of
too far ahead
get too of ourselves, let’s go
ourselves, let's go over
over what first quartile
the first
what the quartile and third quartile
and third quartile mean.
mean.
The first quartile,
The first also known
quartile, also as the
known as quartile or
lower quartile
the lower or 25th
25th percentile,
percentile, is the value
is the value for
for which 25% of
which 25% the data
of the data is
is
less than. If your score on an exam is equal to the first quartile, then you scored better than 25% of the people
less than. If your score on an exam is equal to the first quartile, then you scored better than 25% of the people
who took
who took the exam.
exam .
third quartile,
The third
The quartile, also known as
also known as the upper quartile
the upper quartile or
or 75th
75th percentile,
percentile, is
is the value for
the value for which 75% of
which 75% of the is
data is
the data
less
less than. If your
than. If your score on an
score on exam is
an exam equal to
is equal to the
the third quartile, then you scored better than 75% of the people
third quartile, then you scored better than 75% of the people
who took
who the exam
took the exam..
To calculate the quartiles,
calculate the quartiles, follow these steps::
these steps
1. Make sure the
Make sure the data set is ordered.
data set ordered .
2. Find
Find where
where the
the median
median is.
3. Use median to split
Use the median split the data set
the data into two
set into t w o halves o t include
halves.. Do nnot the median
include the median in
in either
either half. 50 when
half . So when
the data
the set contains
data set numbers, the
contains 7 numbers, the two halves are
two halves the first 3 numbers
are the last 3 numbers
and the last
numbers and numbers since the
since the
4th element
4th the median
element is the median and and needs
needs to bebe excluded.
excluded. When
When the data
data set contains 8 numbers, the
contains 8 numbers, the two
two
halves
halves are the first 4 numbers
are the numbers and last 4 numbers.
and the last median is already
numbers. The median ”excluded” since
already "excluded" it's the
since it's the
average
average of the
the 4th and 5th
4th and numbers.
5th numbers.
4. The quartile is the
The first quartile median of the
the median lower half
the lower the data.
half of the third quartile
data . The third is the
quartile is the median of the
median of upper
the upper
half of the
half the data.
data.
Note that
Note that there
there are actually several
are actually several ways use to calculate
mathematicians use
ways mathematicians calculate the quartiles, and
the quartiles, the resulting
and the resulting values
values
differ depending
can differ
can depending on the method.
on the Fortunately, this
method. Fortunately, isn’t something
this isn't something you need to worry
you need worry about. For the
about. For the
purposes
purposes of the just use
the SAT,just use the method described
the method described above and you’ll
above and you'll be
be fine.
To illustrate,
illustrate, let’s
let's do an example.
do an example. Let's construct a boxplot
Let's construct from the following
boxplot from prices for 15
following set of prices 15 different
different
textbooks:
textbooks:
{ 24,28, 30, 30, 30, 72, 75, 82, 88, 90,
{24,28,30,30,30,72,75,82,88,90 100,100, 100, 100, 130}
,100,100,100,100,130}
o The m
• The i n i m u m iiss 24.
minimum
o
• The
The median
median is the 15+
the 15 2 = 7.5 ‐>
-:-2 -+ 8th number: 82.
8th number:
0 Because the median
• Because number, we can
median is the 8th number, use itit to split
can use split the data w o halves:
into ttwo
data set into the first 7
halves: the
numbers and
numbers and the last 7 numbers.
the last numbers. We do not include the
not include the median
median in either half..
either half
0
• The first quartile
quartile is the median of the
the median the first 77 numbers,
numbers, so it's the 7
so it’s + 2 z= 3.5 ‐>
7-:- 4th number:
-+ 4th number: 30.
o third quartile
The third
• The quartile is the median of the
the median the last numbers, so
last 7 numbers, so it’s +2=
it's the 77 -:-2 = 3.5 -+
‐> 4th
4th number
number from the
from the
median the 8 +
median or the + 4 = 12th number in the
12th number overall set: 100.
the overall
•o The maximum
maximum is
is 130.
Using
Using these
these values,
values, we can construct the
o w construct
now
can n boxplot:
the boxplot:

H _ _ _ J _ l ‐ H
t ‐ t ‐ i ‐ t ‐ H ‐ t ‐ t ‐ t ‐ t ‐ f ‐ t ‐ F ‐ ‘ l
2
200 3300 440
0 550
0 660
0 70
70 8 0809 0901 0100
0 11110
0 1 2120
0 1 130
3 0 1140
40
Price
Price

248
THE COLLEGE PANDA
THE COLLEGE PANDA

CHAPTER EXERCISE:Answers for this chapter start on page 338.

A calculator is allowed following


allowed on the following
questions.
questions.

The average
The height of 14
average height 14students
students in oneone class is
class is
63 inches
63 The average
inches.. The height of 21
average height 21 students
students in
another classs is 68. If
another clas the two
If the t w o classes are
classes are .....
0
combined, what is
combined , what is the
the average height, in inches,
average height, inches, ....
QJ

the students
of the students in the combined class?
the combined class? "E
:l
2
A) 64.5 z
B)) 665
5
0
C)) 666
6 J, "'-~ ~ ~ {' ?:,~
6.5
D)) 666.5 ~ ~ ...,_lo'
lo' ...... 'I,~ "v'd"

Books read

The histogram
The histogram above shows the
above shows number of
the number
read last
books read
books by 20
year by
last year 20 editors at a
editors at a publishing
publishing
Kristie has taken
Kristie has taken five tests class.. The
science class
tests in science The company. Which of the
company. Which the following could be
following could be the
the
average all five of Kristie's
average of all Kristie’s test
test scores
scores is 94. median
median number books read
number of books the 20
read by the editors?
20 editors?
The average of her
The average last three
her last three test
test scores
scores is 92.
A
A)) 110
0
What is the
What average of her
the average her first
first ttwo
w o test scores?
test score s?
B
B)) 1122
A ) 995
A) 5
C
C)) 116
6
B) 96
B)
D ) 223
D) 3
C ) 9977
C)
D ) 9988
D)

Final Exam
Final Exam Scores
Scores
(out of 100 points)
(out point s)
company hires
food company
A food an independent
hires an independent research
research
agency to determine
agency determine its product’s shelf
its product's shelf life,
life, the
the
length of time
length may be
time it may be stored
stored before expires.
before it expires. l~
‐ i : l : l ‐ ‐ l
Using aa random
Using sample of 40
random sample 40 units the
units of the t ‐ t ‐ ‐ ‐ + ‐ t ‐ ‐ l ‐ ‐ ‐ l ‐ ‐ l

product,
product, the
the research agency finds
research agency finds that
that the
the 70 75 80 85 90 95 100
product’s shelf
product's has aa range
life has
shelf life range of 33 day
days. Which
s. Which
The box plot
The box plot above
above summarizes
summarize s thethe final
final exam
exam
of the following mu
the following must be true
st be about the
true about units in
the units scores of 26
scores studentss in a
26 student math class.
a math Based on
class . Based on
the sample?
the sample? box plot,
the box
the which of the
plot, which following best
the following best
All the
A) All the units expired within
units expired within 33 days
days.. estimates
estimates the
the number points by which
number of points which thethe
B) with the
unit with
The unit longest shelf
the longest shelf life took 33
life took median score of the
median score the 26 exceeds the
students exceeds
26 students the
B) The
longer to expire
days longer
days expire than
than the
the unit with the
unit with the individual score?
lowest individual
lowest score?
shortest shelf life
shortest shelf life.. A) 3
The mean
C) The shelf life
mean shelf the units
life of the units is 3 more
more B) 5
than the
than median.
the median.
C
C)) 8
The median
D) The shelf life
median shelf life of the units is 3 more
the units more
D ) 11
D) 11
than the
than the mean.
mean.

249
CHAPTER 27 STATISTICS
CHAPTER STATISTICS I

Miss World
Miss Titleholders
World Titleholders Locks are sections of canals
are sections canals in which
which the
the water
water
level can be
level can be mechanically
mechanically changed to raise
changed to raise and
and

••• •• •
0 lower
lower boats.
boats. The
The table
table below shows the number
below shows the number
0 0
0 0 0 of locks
locks for 10 canals
canals in France.
France .

••• ••• ••• ••• •• ••• •


0 O O 0 0
0 O 0 O O 0 Name I # Locks I
Q 0 0 0 O O 0 Aisne 27
18
18 19
19 20
20 21
21 2222 23
23 24
24 Alsace 25
Age (years)
Age (years) Rhone 5
Centre 30
The dotplot
The dotplot above the distribution
shows the
above shows distribution of ages
ages
Garonne 23
for 24
24 winners the Miss
winners of the World beauty
Miss World beauty pageant
pageant
at the
at time they
the time were crowned.
they were Based on
crowned. Based on the
the Lalinde 27
data, which
data, the following
which of the following isis closest
closest to the
to the Midi 32
average mean) age
(arithmetic mean)
average (arithmetic the winning
age of the winning Oise 27
Miss World
Miss World pageant
pageant contestant?
contestant?
Vosges 93
A
A)) 119
9 Sambre 29
B)
B) 20
20
Removing which of the
Removing which the following w o canals
following ttwo canals
C
C)) 2211
result in the
from the
from the data would result
data would greatest
the greatest
D
D)) 2222 decrease in the
decrease standard deviation
the standard the
deviation of the
number of locks
number canal?
each canal?
locks in each
A) Aisne and Lalinde
Aisne and Lalinde
8) Alsace
B) Alsace and
and Garonne
Garonne
Temperature (°F) Frequency Centre and
C) Centre and M idi
Midi
60 3 D) Rhone and Vosges
Rhone and
61 4
63 4
67 10
70 7 A shoe store surveyed
shoe store surveyed a
a random sample of 50
random sample 50
customers to better
customers estimate which
better estimate which shoe sizes
shoe sizes
The table above
The table gives the
above gives distribution of low
the distribution low should
should bebe kept
kept in stock.
stock. The store found
The store found that
that the
the
temperatures
temperatures for a over 28
a city over 28 days. What is the
days . What the median shoe
median shoe size
size of the
the customers sample
the sample
customers in the
median
median low temperature, in degrees
low temperature, degrees Fahrenheit
Fahrenheit inches. Which
is 10 inches. Which of thethe following
following statements
statements
(°F), of the
(°F), 28 days?
these 28
the city for these days? must
must be true?
true?
A) The sum of all the
The sum the shoe sizes in the
shoe sizes the sample
sample
is 500 inches.
inches .
B) The average
average of the the smallest size and
shoe size
smallest shoe and
the largest
the shoe size
largest shoe the sample
size in the sample is 1010
inches.
inches.
C The difference
C) The difference between
between the the smallest shoe
smallest shoe
size and
size the largest
and the size in the
shoe size
largest shoe sample
the sample
is 10
10inches.
inches.
D At least
D) At least half
half of the
the customers
customers in the sample
the sample
have sizes greater
shoe sizes
have shoe than or equal
greater than equal to 10
10
inches.
inches.

250
__
THE COLLEGE
THE COLLEGE PANDA
PANDA

The tables
The give the
below give
tables below the distribution travel
distribution of travel 10 ~~---_-_-_-_-_-_-_-
_-_-_-__:;--
-- ---~
times between
times between ttwo
w o towns
towns for Bus and Bus
Bus A and Bus B 9 -Company A 1---------i
over the
over same 40days.
the same 40 days . A28 Company B ,_______
8 _ i::::::::::J _.
Bus A
Bus “5 7
7
Travel time
Travel (minutes)
time (minutes) Frequency an 6

_‑ 44 5 --
g
... 5
0
Q)
.o
5
4
45
47
10
10
15 153
zZ 2 2
48 10
1
0
Bus B 45 46 47 48 49
Travel time
Travel time (minutes)
(minutes) Frequency Weight (in
Weight pounds)
(in pounds)
5
25 5 The bar
The chart above
bar chart shows the
above shows distribution of
the distribution
30 10 weights
weights (to
(to the nearest pound)
the nearest pound) for kayaks
for 19 kayaks
35 15 made by Company
made by Company A and and 19 kayaks
kayaks made
made byby
40 10
Company B.
Company B. Which of the
Which of the following
following correctly
correctly
compares the
compares the median weight of the
median weight kayaks made
the kayaks made
Which of the
Which following statements
the following statements is true about
true about by each company?
each company?
data shown
the data
the shown for these days?
these 40 days? The median
A) The weight of the
median weight kayaks made
the kayaks made by
by
standard deviation
The standard
A) The deviation of travel
travel times for
times for Company
Company A is smaller.
smaller.
Bus smaller.
Bus A is smaller. The median
B) The weight of the
median weight kayaks made
the kayaks by
made by
B)
B) The standard deviation
The standard deviation of travel
travel times for
times for Company B is smaller.
Company smaller.
B is smaller
Bus Bis smaller.. C) The
The median
median weight the kayaks
weight of the is the
kayaks is the
C) The
C) The standard travel times
deviation of travel
standard deviation times is same both companies.
same for both companies .
the same
the Bus A
same for Bus and Bus
A and Bus B. D) The relationship cannot be determined
D)
D) The
D) standard deviation
The standard deviation of travel times for
travel times from the information given.
Bus A and
Bus Bus B cannot
and Bus be compared
cannot be compared with
with
the data provided.
the data provided .

Quiz 1 2 3 4 5 6 7
Score 87 75 90 83 98 87 91
The table
The table above shows the
above shows the scores for Jay’s
scores for first
Jay's first
math quizzes.
seven math
seven quizzes . Which of the
Which of the following
following are
are
true about his
true about his scores?
scores?
1, The
I. The mode greater than
mode is greater than the
the median.
median .
n. The
II. median is greater
The median greater than
than the
the mean.
mean .
III. The
The range greater than
range is greater than 20.
A) llII only
only
B) IIII only
I I only
C) II and
JI and III
III
D) I, H,
11,and
and IIII
II

251
CHAPTER 27 STATISTICS
CHAPTER STATISTICS I

-School A
5 SchoolB
Vl
Q)
Vl
Vl
>. ro 4
~ 3 1---------- u
......
Q)
0
g. ....
Q)
3
~ 2 .0
1-L. 6 2
:l
1 z
1
0
0
5 6 7 8 910
1 2 3 4 5
Integers
Integers
Number of films shown
The graph
The graph above shows the
above shows the frequency
frequency The bar
The chart above
bar chart the number
shows the
above shows number of films
distribution
distribution of aa list randomly generated
list of randomly generated shown in class
shown over the
class over year for 19 classes
past year
the past classes
integers
integers between
between 5 and Which of the
and 10. Which the in School
School A andand 15classes
15 classes in School
School B. Which
Which of
following
following correctly
correctly gives the mean
gives the mean and the range
and the range the following
the following correctly
correctly compares
compares the mean and
the mean and
of the list of integers?
the list integers? median number of films shown
median number each class
shown in each class for
A) = 7.6, Range
Mean =
Mean Range ==4 the w o schools?
the ttwo schools?
B) Mean :
Mean = 7.6, Range := 5
Range A) The e a n and
mean
The m and median number of films
median number
C) Mean = 8.2, Range
Mean = Range == 4 shown each class
shown in each are both
class are greater in
both greater
School A.
School A.
D) Mean = 8.2, Range
Mean : 5
Range =
B) The mean and
The mean and median number of films
median number
shown each class
shown in each are both
class are both greater
greater in
School
School B.
C) The mean number
The mean number of films shown in each
films shown each
Calories
Calories in Meals
Meals class is greater
class greater in School
School A, but the median
but the median
520 550 550
is the same in both
the same both schools.
schools.
500 500
600 600 900 D) The
The mean
mean number shown in each
number of films shown each
550 550
class
class is greater School B, but
greater in School but the median
the median
The table
The above lists
table above number of calories
the number
lists the calories in is the same in both
the same both schools.
schools.
each Mary's
each of Mary 's last 10 meals.
last 10 meals. If If a
a 900-calorie
900-calorie
meal
meal that
that she had today
she had today is addedadded to the values
the values
listed,
listed, which the following
which of the following statistical
statistical
measures of the
measures the data
data will not change?
w i l l n_o_t change?
1, Median
I. Median
II.
1], Mode
Mode
Range
III. Range
A
A)) lIaandn d 11II only
only
B))
B [I aand III only
n d 111 only
C)
C) II
11and
and IIII I I only
only
D
D)) I I , and
II,, II, and IIIIII

252
THE
THE COLLEGE PANDA
COLLEGE PANDA

2 1 222
21

The dotplot
The
miles per
miles
••
•• ••• ••• ••• •
2 23
23 2

per gallon)
representing the
representing
244 25
Gas mileage
Gas

dotplot above
2 5 26
26 2
mileage (miles

gives the
above gives
gallon) of 15
the car
mileage is removed
mileage
277 28
(miles per

the gas
15different
with the
car with
2 8 29
2 9 30
gallon)
per gallon)

gas mileage
30

mileage (in

greatest gas
the greatest gas
dot-plot, what
from the dotplot,
removed from

(in
the dot
cars. If the
different cars. dot

what will
will
The
The table
“‑
table above
‐‑ Number of
lectures
lectures

summarizess the
above summarize
12
15
21
25
28
32
40
distribution of
the distribution
Number of
professors
professors
15
12

5
6
20
17
15

the number of lectures


the number each of the
lectures each the 90
90 professors
professors
happen to the
happen the mean, and standard
median, and
mean, median, standard
at
at acollege
a college gave last year.
gave last year. Which
Which of the
the
deviation of the
deviation the new data set?
n e w data
following box plots
following box plots correctly
correctly represents
represents the data
the data
A) Only the mean
Only the will decrease
mean will decrease.. shown in the
shown table?
the table?
Only the
B) Only the mean
mean and
and standard deviation will
standard deviation will A)
A)
decrease..
decrease
Only the
C) Only mean and
the mean will decrease
median will
and median decrease.. H'-_---'-_ _:------1
The mean,
D) The median, and
mean, median, and standard deviation
standard deviation l ‐ ‐ 0 ‐ ‐ f ‐ 0 ‐ f ‐ + ‐ ‐ f ‐ l * l ‐ ‐ l

will decrease..
will decrease 5 1 10
0 1 15
5 20
2 02
25 5 3
30 0 35
3 5 40
4 04
45 5 50
5 0
Number of lectures
Number lectures
B)
B)

Snowfall (in
Snowfall ( i n inches)
inches) l ‐ E I Z l ‐ l
mmmmmm
45 48 49 50 52 54
i ‐
5 1 10
0 1 15
i
5 20
2 0 25

2 5 30
3 03
f
35 5 40
4 04

45 5 50
5 0
O ‐ l ‐ l ‐ M ‐ l ‐ l

mmnmmm
55 57 57 57 58 59 Number of lectures
Number lectures
m m m m
60 60 61 61 65 90
C)
The table
The table above lists the
above lists the amounts snowfall, to
amounts of snowfall,
the nearest
the inch, experienced
nearest inch, experienced by 18 18 different
different F E D ‐ i
H
the past
cities in the
cities year.. The
past year The outlier measurement
measurement t ‐ t ‐ t ‐ t ‐ t ‐ t ‐ t ‐ + ‐ + ‐ ~ I

of 90 inches is an
90 inches error.. Of the
an error the mean, median,
mean, median,
5 1 10
0 1 15
5 2 20
0 2 25
5 3 30
0 3 35
5 440
0 445
5 550
0
and
and range the values
range of the listed, which
values listed, which will change
will change Number of lectures
Number lectures
the most if
the most if the 90-inch measurement
the 90-inch replaced
measurement is replaced D)
D)
the correct
by the correct measurement inches?
measurement of 20 inches?
A) Mean
Mean H
B) Median
Median r o + o +‐ i‐ r +‐ o ‐4
5 10 15
10 15 20 25
25 3030 3535 40 45 50
40 45 50
Range
C) Range
Number lectures
Number of lectures
None of them
D) None them will change.
will change.

253
e

Statistics
Statisti II
cs II
goal of statistics
The goal statistics is to be able to make
be able predictions and
make predictions estimations based
and estimations based onon limited time and
limited time information..
and information
For example,
example , aa statistician might want
statistician might want to estimate
estimate the mean weight
the mean weight of of all female raccoons
all female raccoons in in the United
the United
States. problem is that
States . The problem it's impossible
that it’s impossible to to survey
survey thethe entire population.. In
raccoon population
entire female raccoon In fact, by the
fact, by the
time that could
time that be accomplished,
could be accomplished, not only would
n o t only would the data be
the data u t of
be oout date but
of date but there would be
there would be nnewe w females in
female s in
the population. Instead,
the population. Instead, aa statistician
statistician takes
takes aa random sample of female
random sample raccoons to
female raccoons make an
to make an estimation
estimation ofof
what the
what actual mean
the actual might be
mean might be.. In other words, the
other words, sample mean
the sample mean is used to
is used to estimate
estimate the population mean
the population mean..
Using a
Using sample to predict
a sample predict something
something about entire population
about the entire population isis aa ccommon theme in
o m m o n theme in stati
statistics and in
stics and in SAT
SAT
questions.
question s.

EXAMPLE 1: A pet
EXAMPLE pet food store chose 1,000 customers
store chose customers at _random and asked
random and asked each customer how
each customer many pets
how many pets
he she has.
he or she has. The results
results are table below.
the table
shown in the
are shown
Number of pets Number of customers
I f t •
1 600
2 200
3 100
4ormore 100
There are
There total of 18,000customers
are a total customers in the database. Based
store’s database.
the store's the survey
Based on the survey data,
data, what the expected
what is the expected
total number
total number of customers who own
customers who pets?
o w n 2 pets?

Using
Using the sample data,
the sample data, we can estimate the
can estimate total number
the total number who
who o w n 2 pets
own pets to be
be

200 _ ~
18, 000 X
18,000 3, 600
x ] ,()()Q = ~
1,000 _

254
THE COLLEGE
THE COLLEGE PANDA
PANDA

w a s
Oxygtm Uptakeversus -Hea.rt Racte
..,..

.
i.., i....•

:..,,.
l..--'-'
.....
.
, I
I t.,..V •
l.;-v

• ~
1..... •
!..-

;~•v""' •
••
..:,.),- . 090 ..100 110 120 130
~~I~~: ,. I •~,.: '.

1' ' " • ' Heart rate (beatsperi‘rdnute)


Heartrate (beats per :i;ninute)

M
~ W W above
. sca~tot W shows
W the
Wrelatiqosltjp
W M bet;ween
M W M and;
heart-rate O oxyg~
W uupt~eM at '.a E Mpoiµts
;t.6diffeten; W
d m x yI~:
during< kyte'S'
' s eex,ercise
xmmu fi n e .The
~outil;t~. fineotbest fit is a1s6showi;i.
minnofbestfltisalsoshown. , ·· ; ·

M 1 : Medonthelumetbestfitwhathyle’spredmdoxygmuptdteataheertrateofllobeats
PARTl: _ d on the ,line of best-nt, -;what.isKyle's predicted o~gen ~e at a."be.,rrr!lfeof11Pbeats
perndnute?
permin,ute?

M12:
PAR'l' Whatistheoxygenuptake,
2: ''what lS the oxygen1,1ptake,in lliers per Jll.UlUte,ofthemeasurementiepresentedbythedaw
mittetsperminute, of the me-asurementreP.~~ted the data by
pomtfimtisfarfiiestfiomthelineofbestfit?
: tnat
,oinf .
is farthest,fromthe line of best fit?
; .,,

Part 1 Solution:
Part Using the
Solution: Using the line best fit, we
line of best we can see that
can see that at heart rate
at a heart beats per
rate of 110 beats minute (along
per minute (along the
the
x-axis), the
x-axis), the oxygen uptake is ~
oxygen uptake liters per minute
liters per minute..
Using the
Using line of best
the line to make
best fit to make a prediction can
a prediction be dangerous,
can be dangerous, especially when
especially when
•0 we
we are making aa prediction
are making prediction outside
outside the scope of oour
the scope data set (predicting
u r data (predicting the
the oxygen uptake at
oxygen uptake at aa heart
heart rate
rate
of 250 beats per minute,
beats per for examp
minute, for example‐you’d probably be
le-you'd probably be dead).
dead).
•0 there
there are outliers that
are outliers that may heavily
may hea vily influence the line
influence the line of best
best fit (see Part 2).
(see Part
•o the data is
the data is better
better modeled
modeled by by aa quadratic or exponential
quadratic or exponential curve than aa linear
rather than
curve rather one . In this
linear one. case, a
this case, a
linear model
linear be the
looks to be
model looks right one,
the right but something
one, but like compound
something like compound interest look linear
may look
interest may at first even
linear at even
though it's
though exponential growth
it’s exponential growth..

Part 22 Solution:
Solution: From
From the scatterplot, we
the scatterplot, we can
can see that the
see that data point
the data point farthest away from
farthest away line of best
the line
from the best fit is at
at
along the
118 along the x-axis.
x-axis. The point represents
The point represents an oxygen uptake
an oxygen uptake of 2.s liters per minute.
liters per minute . I I
that this
Note that
Note this data point is likely an
data point an outlier,
outlier, which can heavily
which can influence the
hea vily influence best fit and
line of best
the line and throw
throw off o
our
ur
predictions.
predictions . Outliers should be
Outliers should be removed
removed from
from the data if
the data if they
they represent
represent specia cases or exceptions.
speciall cases exceptions.
only will
N o t only
Not will you be asked
you be asked to make predictions using
make predictions using the line of best
the line best fit, but you’ll
but you also be
' ll also to interpret
asked to
be asked interpret
slope and
its slope
its y-intercept. We’ll
and y-intercept. We'll use data from
use the data this example
from this example in the
the next one to show
next one show you how these
you how concepts
these concepts
are tested.
are tested .

255
CHAPTER 28 STATISTICS
CHAPTER STATISTICS II
II

Exam 3:
EXAMPLE
3:
Oxygen Uptake versus Heart Rate

' • _.Al
I--'
I--''
' ~I--'

.
.,...i.-- I
I,, ...
:
'
·/ . .,,,i'...-;•
• •
~.,.. I;' ....


'.
,
! .. ~ J~
90 100 110
110 120 130
• 'I' ' II. ........ , I' - ~
~ \ ,- II Heart rate
H~ rate(beats per minute)
(beats per minute)

Themmwowdwwsflerdafimhipbetweenheanfitemdoxygenuptake
The saatterplot above shows the relationship between heart rate and oxygen uptake at 16dil‘ferentpoints
at 16 different points
vdminglfiyle‘sexemisemtine. Thelineothestfitisalsoshown.
dllring Kyle's exerciseroutine. The line ofhest fit is al.so shown.

“M1:Winchofthefollowmgisthebestintapretahonotflieslopeoffllehneofbestfitinthecontext
PART'l:Which, of th following is the best-interpretation of the slope .of the line of best fit in the context
ofthispmblem?
of this problell\'?
A) ‘I'hepredrctedmcreasemele’soxygenuptake inlitersperminute,
The predicted increase in Kyle's oxygen uptake, in foreveryonebeatperminute
liters per minute, for every one beat per min~te
' increase
incrmeinhishemme
in his heart,:rate
B) The
B) predicted increase
The predicted in Kyle's
increase'in Kyle's heart
heart rate, per minute,
beats per
rate, in beats for every
minute, for every one
one liter per minute
liter per minute
increasem
jnctease his oxygen
in his uptake
oxygen uptake
‘ C) Kyle's
Kyle'spredicted oxygen uptake
predicted oxygen uptake in
in liters minute at
per minute
lifers per at a rate of
heart rate
a heart of 0 beats per
Obeats minute
per minute
D) Kyle’s predicted heart
Kyle's predicted heart rate in beats
rate in beats per at an
minute at
per minute an oxygen uptake of
oxygen uptake of 0 liters per
Oliters per minute
minute

PART 2: Which
PART Which of the following is the
the following interpretation of
best interpretation
the best of the of the
y-intercept of
they-intercept line of
the line of best fit in
best fit in the
the
commit of this
context-of problem?
this problem?
A) The
A) predicted increase
The predicted increase in in Kyle’s uptake, in
oxygen uptake,
Kyle's oxygen in liters
liters per minute, for every
minute, for one beat
every one beat per minute
minute
increase his heart
increase in his rate
heart rate
B) the
13) predictedi n c r e a s e in
The _pre,djcted.increase in Kyle's
Kyle’s heart
heart rate, in beats
rate, in beats per
per minute,
minute, for every
every one liter per
pne liter per minute
minute
:m,his
increasein
increase oxygen uptake
his oxygen ptake
C)
C) Kyle's predicted oxygen
Kyle's predicted oxygen uptakeuptake in
in liters minute at
per minute
liters per at a rate of 0
heart rate
a heart beats per
0 beats minute
per minute
Kyle’s predicted
D) Kyle's predicted heart
heart rate in beats
rate in beats per
per minute at an
minute at oxygen uptake
an oxygen uptake ofof 0 liters per
0 liters minute
per minute

Solution: As we
Part 1 Solution: we learned
learned in the
the linear model questions
linear model questions in the
the interpretation chapter, the
interpretation chapter, slope is
the slope is the
the
increase in y31(oxygen
increase uptake)
(oxygen uptake) for each increase
each increase in x (heart rate).
(heart rate) . The only difference n o w is that it’s a predicted
only difference now that it's predicted
increase.
increase . The answer is (A) .
The answer [0]
Part 2 Solution: The y‐intercept
Solution: They value of y (oxygen
the value
-intercept is the when xx (the
uptake) when
(oxygen uptake) (the heart beat) is
heart beat) is 0.
0. The answer is
The answer is
[@.
-. Note that
Note value would
that this value have no
would have significance in
no significance in real
real life since
since you would be
you would dead at
be dead at a heart rate
a heart of 0.
rate of 0.
again illustrates
This again
This danger of predicting
illustrates the danger values outside
predicting values scope of
outside the scope of the sample data.
the sample data .

256
VGNVcI 5 1 9 3 7 7 0 2 ) EIHl
THE COLLEGE PANDA

j
$ . W ° W W W 3 ° W, W V “ W W F m e ' m m q u w w m
EXAMPl.E4: M~d~~ a town; th~stateoi.M~chuse~
“ W VW ‘ S M W W P W M ‘ M W m e m p m w m m s m w e d e . A'rea;t\i
stateagefifjandbnµy surve
yed°~
'j apartm~ts for .s~e m¥a:tden
WWPWW°PSWMWPWMWWmWwSaWW ~d foung that the a~f~ gepri ~eof ~j cl! ~~~ t .wa_s f 150A~<1Q J A:,;i
~ fhe:r
sttrVey and ~ att~.mpt-to ~et ~ spiall~ marg:tn ofl~r ,,..._ ,
real estate age.ttt mtends to replicate th_r

l of the following samples will most .likely result in a sinallet margin of error for the mean pnce o.
1° “13mm teams a U; 1111931 513111 new 111M 9111va M0110; 311310 11519331111 u m : 911: 1 ° } W
ammmvw‘mmwunmmwde ”
apartment in Malden, Ma$sachuse.tts? I\;
“ a n e w us fimumdv wastes 5111109118109 (v
A) 30 randomly selected apartments in Malden ,
:,,_ , -
_s1133m139mw10119111s1u31111179d9p313313$&11u0p119109 (a ' • • ....~ ., I\ '·"'j,,.;.;
\ '.,.-t·l';I',' ,
11 B) 30 randomly selected, apartments in allofMassa,cltusetts · , ..:!, ., •• , , , .., \•. :.-· .,~ _:,:-;~i.,11,-a:,:
I m p m m fl m m d e m p s x n n o p m o g ( 3
C) 80 randomly selected aparbnents in Malden , .... . , · 1,· < ,1 ""'' ~•,· , , .,,
mmmwwnemwmmmdvapsfinmpwos (a
D) 80 randomly selected apartments in <lll.o{Massachusetts ) ,,.

(3) S113MS119 3111


The answer is IT9J
.The margin of error refers to the room for error we give to an estimate. For example, we
10113 101 111001 311101 s13131 1011310 u1819u1 3111 ' 3M ’31du19x3 10:1 ‘319u11153 119 01 31118 am
5311du11 51111 0001113; 10 1011310 u1819u19 1111M 000119104; 51u 3 p 1 9 w 1111113111119d9 me10 3311d u93111 3111 A95 p1n03
could say the mean price of an apartment in Malden is $150,000 with a margin of error of $10,000 . This implies
s119A131u1 $1111 ‘000’091$ p u 9 000'017[$ u33M13q A13>111 S1u 3 p 1 9 w 111 s1u3u1119d9 119 10 3311d 1193111 3n11 3111 19111
that the true mean price of all apartments in Malden is likely between $140,000 and $160,000 . This interval is
I '(9 31du19xg1 33s) 19A131u1 aauapguoa 9 p31193
called a confidence interval (see Example 6).
8111133135 0313191111 111 s1113u1119d9 111011 133135 A1u01s111 p1n0115 3M ’17 31du19x3 111 1011310 u1819111 13119u1s 9 138 01
To get a smaller margin of error in Example 4, we should first only select from apartments in Malden. Selecting
3111 111011 s69115 0519 1nq 919p 3111 01 A1111q9119A 3 1 0 u 1 saanp011u1 A1u01011 s1133n1139ssew 10 119 u1011 s1u3111119d9
apartments from all of Massachusetts not only introduces more variability to the data but also strays from the

\
am p1n0115 3M 'A1pu0335 ‘s1u3u1119d9 u 3 p 1 9 w 10 3311d 38913119 3111 pay 01 51 113111M 'A3A1ns 31111011131111 191118110
original intent of the survey, which is to find the average price of Malden apartments . Secondly, we should use
p u 9 919p 1 n 0 319111339 3 1 0 u 1 3111 ‘A3A1ns 3M s1u3u1119d9 3 1 0 m 3111 “ a s u a s u o u 1 u 1 0 3 $1$1111 3215 31du195 138191 9
a larger sample size. This is common sense. The more apartments we survey, the more accurate our data and

] 's11011310 11181910 1 n 0 13M013111 p u 9 319 su0119u111s3 1 n 0


our estimations are and the lower our margin of error is.
3101391 0M1 uo spu3d3p 1u3u1113dx3 u9 u1011 319u111s3 A119 1011011310 u18191u 3111 ‘1391 u1
1n fact, the margin of error for any estimate from an experiment depends on two factors:
0 321g 31du195
,I • Sample size
(u01191A3p p19pu91s Aq p31n593u1 u3110) 919p 3111 111 A1111q9119A 0
• Variability in the data (often measured by standard deviation)
1,u93 A119a1d1<1 3M 1 0 11 3 10 111819u1 3111 13M01 3111 ’s1 919p 3111 31q9119A $531 3111 p u 9 321g 31d1119$ 3111 138191 3111
The larger the sample size and the less variable the data is, the lower the margin of error. We typically can't
A11M 03 ~3z1s 31d1u9s 3111 1011u03 1193 3M 1nq ’(s1 11mo p931ds M011) 919p 3111 10 uo1191A3p p19pu91s 3111 1011u03
control the standard deviation of the data (how spread out it is), but we can control the sample size. So why
919p 1311198 01 8111u1nsu03-3u111 p u 9 £11503 001 5,11 asneaag gsaz1s 31d1119s 38n11 asn sA9M19 s13113193s31 1,u0p
don't researchers always use huge sample sizes? Because it's too costly and time-consuming to gather data
'31311~1A13A3 pue 3qu13113 111011
from everyone and everywhere .

as am mama 112 9843091109 amtuswmflm m a m a a i m s - m a p mm


EXAMPtE5: :Researchers conducted/ an experimentio determine whether ex-e:rcise:im_pl'oves
Sim-90751?“ M a n - 0 1 m m m a m sumpms ooz 933338 51mm 69111 mm ma , stuc(ent
exam scores. 'Fhey.z:a,:1domly .sfildents who exercise atrleastonce.'a week..ano
'S.elected.2()(1 200 students
11335 9 1 0 1 W 31111313339 W M W W qaameammqseqw 3310131131011011 011m
who do .not exercise at1east once-a week. Af:tertraciking ,t'he.
students' acacleinicpetform~-ees ~lor a .year1
m W W w v w r m m m m m w m pane; n e w a m
the researchers found tnat-the students ·who exercise at1east'Ohee a week pedo.rfned s1gnifitant1y better
101991114913831: m u m p m - fl m p a u no 13353810101) 01m warms 3111 1mm m e m e s an m
on .fh:e,same exams thari the students who do not. l3ased.on the design.and.resrilts-..ofth-e stud.y, which of
guo1snpu03 3191.1d01dd9 119 $1811111101101 3111
the following is an appropriate conclusion?
031003 11191133de01 £13311] $1>133M 9 33110 1593119 8111s1313x3 (v
A) Exercising at least once a week is lik~y toimprove exam scores..
-
·1.·
(g 3 d e 1133,“ 9 3311111 331111811181313xg 3133111933110 1911!811181313113 119111310111amass 1119713 9
B) Exercising three times a week improves exam-scores more .than exen:isingjustonceaw.eek.
1113pn1s Auv (3
C) Any student who starts exercising at least once a week will improve his or her exam scores,
‘ s a m a s unaxa 131110 $11131101111111 111m 1133119 33110 man 19 811131313113 sums qu
w o o s 1119x3 1113pn1s pun 351313x3 1133M1aq uogeposse amped 9 $13.13111 (0 1
D) There is a positive association between exercise and student exam scores.
/r

u0119130ss9 10 3 5 9 3 31ss913 9 1111111 S193p 11011s3nb $1111


asn933q 1sn[ 'u0119sn93 ' S A (u0119131103 1331193 0s19)
This question deals with a classic case of association (also called correlation) vs. causation . Just because
students who exercise got better exam scores doesn't mean that exercise causes an improvement in exam
u 1 9 x 3 u1 1u31113Ao1d1111 u9 sasnvp 351313x3 19111 11931u 1,us30p 53103s 1 u 9 x 3 13113q 108 351313x3 011M s1113pn1s

scores. It's just associatedwith an improvement in exam scores. Perhaps students who exercise just have more
310111 311911 1sn13s1313x3 011m s1u3pn1s sd9q1ad 'sa103s u 1 9 x 3 111 1u3u13A01du11 119 1111M 1131171305517 1sn1s,11 3 3 1 0 3 5
1113111113dx3 3111 A9M 3111 01 3 n q '13p1911 Apn1s u131113>19u1011m s1u319d 8111pu9u13p 310111 311911 £3111 10 31111d1as1p
discipline or they have more demanding parents who make them study harder. Due to the way the experiment
31 1013918111A113p1m 3111 1mm 1131 1lu93 3M ‘p3u8153p 9 9 M
was designed, we can't tell what the underlying factor is.
S311du11 A1u0 10u 113sn933q 8uo1M 51(g) JaMSUV 'u0119sn93 sa11du1111 asn933q 8u01M 51( v ) 1 3 M S U 9 ‘3101313111
Therefore, answer (A) is wrong because it implies causation. Answer (8) is wrong because it not only implies
‘1u3u1113dx3 3111 111 p3>139111,us9M 191118u111131110$ ’s13119111351313113 10 buanb3113111 19111$311du110s19 1 n q u0119sn93
causation but also implies that the frequency of exercise matters, something that wasn't tracked in the experiment.

1.92
257
28 STATISTICS
CHAPTER 28
CHAPTER STATISTICS IT
[1

Answer (C) is wrong


Answer (C) wrong because suggests aa completely
because it suggests completely certain outcome. Even
certain outcome. I D improve
DID
Even if exercise D improve exam
exam
scores,
scores, not single student
every single
n o t every student who
who starts
starts exercising will improve
exercising will There might
improve their scores. There be students
might be students for
exercising makes
whom exercising
whom makes their
their scores worse. conclusion drawn
worse . Any conclusion drawn from sample data is a
sample data generalization and
a generalization and
should not
should n o t be regarded as
be regarded asa truth for every
a truth every individual
individual..

answer is ~-(D) . There


The answer positive association
There is a positive between exercise and
association between and student
student exam
exam scores.

One of the
One things the researchers
the things correctly was
did correctly
researchers did was to take random samples
take random group. The key word
each group.
samples from each word is
random.. If
random samples weren't
[f the samples random, we
weren’t random, we wouldn't
wouldn’t even have been
even have able to conclude
been able conclude that is aa positive
there is
that there positive
association between
association between exercise and and exam
exam scores. Why? Let's Let’s say the researchers picked 30 students
researchers picked students fromfrom the
the
tennis team for the
tennis team group and
the exercise group and 30 students just play
who just
students who video games
play video day for the non-exercise
games all day non-exercise
group Definitely not
group.. Definitely random.. Now,
n o t random did the
Now, did group do
the exercise group better on
do better their exams
on their because they
exams because they exercise
or because
because they playplay tennis?
tennis? Or was the video
was it the games that
video games that made group perform
non-exercise group
made the non-exercise worse?
perform worse?
selection wasn't
Because the selection random, we
wasn’t random, we can't
can’t tell how each factor influences
how each result.. When
influences the result the selection
When the selection is
random, all the
random, except the
factors except
the factors the one we're testing are
we’re testing “averaged out.”
are "averaged out."
N o w what
Now what if the
the researchers whether exercise does
wanted to see whether
researchers wanted does indeed an improvement
causee an
indeed caus improvement in exam
exam scores.
should they have
What should
What have done differently? The answer
done differently? answer is random assignment. Instead of randomly
assignment. Instead selecting 200
randomly selecting
students from
students from one group that
one group that already regularly and
already exercises regularly and 200 students
students from another group that
another group that does not,
does not,
should have
they should randomly selected
have just randomly selected 400 students next step
The next
students.. The step would
would be randomly assign
be to randomly assign each student
each student
to exercise or not. Everyone in the exercise group
not. Everyone group is forced to exercise at least
least once a week and everyone
week and the
everyone in the
non-exercise group is not
non‐exercise group allowed to exercise. If
n o t allowed If the group performs
the exercise group better on the
performs better the exams, then we can
exams, then can
conclude that exercise causes
conclude that causes an an improvement
improvement in exam course, conducting
exam scores. Of course, conducting this type of experiment
this type experiment
extremely difficult, which
be extremely
can be proving causation
which is why proving such a monumental
be such
causation can be monumental task.
task .
The following the conclusions
summarizes the
following list summarizes you can
conclusions you can draw different experimental
draw from different experimental designs involving
designs involving
two variables
two (e.g. exercise and
variables (e.g. and exam
exam scores).
1. Subjects not
1. not selected random & Subjects not
selected at random randomly assigned
n o t randomly assigned
0 Results cannot
• Results be generalized
cannot be generalized to the population
population..
•0 Cause and effect cannot
Cause and be proven.
cannot be proven.
Example: Researchers
•o Example: Researchers want whether medication
want to see whether treating the flu. People
medication X is effective in treating People
with the
with medication X. People
receive medication
the flu from Town A receive with the flu from Town B receive
People with receive a placebo
a placebo
(sugar pill). More
(sugar people in the
More people the medication group experience
medication X group experience a reduction symptoms. The
reduction in flu symptoms.
generalization that
generalization that medication
medication X is associated with a
associated with reduction in flu symptoms
a reduction symptoms cannot be made
cannot be made
since it was
was only tested Town and
tested in Town A and Town B (sample was not randomly selected
(sample was not randomly selected from the general
the general
population). There
population). may be
There may something special
be something special about
about Town A and and Town 8. B. No cause and effect
cause and
relationship
relationship can bebe established
established because
because the medication was not
the medication n o t randomly assigned. Perhaps
randomly assigned. Perhaps Town
experienced aa less severe
A experienced epidemic.
severe flu epidemic.
2. Subjects not selected at random
not selected randomly assigned
& Subjects randomly
random & assigned
•0 Results
Results cannot be generalized
cannot be generalized to
to the population.
population.
I
•0 Cause and effect can
Cause and can be proven.
be proven.
Example: Researchers
•0 Example: Researchers want
want to to see whether medication X
whether medication is effective
X is in treating
effective in treating the People
flu. People
the flu.
I'
I

with the
with flu from
the flu from Town A A and
and Town
To w n B B are randomly assigned
are randomly assigned to
to either medication X
either medication X or placebo
or aa placebo
(sugar pill).
(sugar pill). More people in
More people in the medication X group
the medication experience aa reduction
group experience reduction inin flu symptoms.
symptoms. The
generalization that
generalization medication X
that medication X isis effective
effective forfor everyone
everyone cannot
cannot bebe made since it
made since was only
it was tested
only tested
in Town A
in Town A and
and Town
Town B B (sample
(sample waswas n randomly selected
o t randomly
not selected from
from the
the general population). Perhaps
general population). Perhaps
only particular strain
one pa~cular
only one of the
strain of flu exi~ts
the flu_ exists inin Town
Town AA and Town B.A
and Town cause and
B. A cause and effect relationship
effect relationship
can be established because the medication was randomly assigned. For
can be established because the medication was randomly assigned. For the people in Town A the people in Town A and
and
Town B,
T~w~ B, we
we can conclude
can c_ medication X
that medication
onc_lude that X causes
causes a
a reduction
reduction inin flu symptoms.. Note
flu symptoms this is
that this
Note that is
still just a generalization‐as with any other medication, medication X does
s~ J~St a generalization-as with any other medication, medication X does not guarantee you willnot guarantee you will
definitely get
definitely better, even
get better, if you
even if live in
you live in Town
Town A A or
or Town
Town B.B.

258
THE COLLEGE
THE PANDA
COLLEGE PANDA

Subjects selected
3. Subjects selected at random Subjects not
& Subjects
random & randomly assigned
not randomly assigned
a Results can
• Results be generalized
can be generalized to
to the population.
the population.
•0 Cause and effect cannot
Cause and be proven.
cannot be proven.
Example:: Researchers
•0 Example Researchers want
w a n t to see whether medication
see whether medication X X is effective
effective in treating
treating the People
the flu. People
with the from the general population are randomly
with the flu from the general population are randomly selected . They are given the choice of
flu selected. They are given the choice of aa
n e w medication (medication
new medication (medication X) X) or a traditional medication (really a sugar pill). More
a traditional medication (really sugar pill). More people people in the
the
medication
medication X group experience a reduction
group experience a reduction in flu symptoms. We can generalize that people
symptoms . We can generalize that people who who
choose to receive
choose medication X fare
receive medication better than
fare better than those who don’t.
those who However, no
don't. However, cause and
no cause and effect
relationship can
relationship be established
can be established because
because the
the medication
medication was o t randomly
was nnot assigned . We
randomly assigned. don’t know
We don't know
whether the
whether the reduction symptoms is due
reduction in symptoms due to the medication or a
the medication a difference between those
difference between those who
who
volunteered and
volunteered those who
and those didn’t.
who didn't.
4. Subjects
4. Subjects selected
selected at random
random &
& Subjects randomly assigned
Subjects randomly assigned
•0 Results
Results can be generalized
can be generalized to the population.
the population.
•0 Cause and effect can
Cause and be proven
can be proven..
Example: Researchers
•0 Example: Researchers wantw a n t to see whether medication
see whether medication X is effective treating the
effective in treating People
the flu. People
with the flu from
with from the
the general population are
general population randomly selected
are randomly Using aa coin
selected.. Using toss (heads
coin toss (heads oror tails),
randomly assign
researchers randomly
researchers person to either
each person
assign each either medication
medication X or a a placebo
placebo (sugar pill). More
(sugar pill). More
people in the
people the medication
medication X group experience aa reduction
group experience symptoms.
reduction in flu symptom We can
s. We can conclude that
conclude that
medication X
medication X causes
causes aa reduction symptoms.. This conclusion
reduction in flu symptoms conclusion can be generalized
can be generalized to the entire
the entire
population of people
population people with the flu.
with the

EXAMPLE 6:
EXAMPLE 6:Environmentalists
Environmentalistsaretestingpfl levelsinaforestthatisbeinghamedbyacid
are testing pH levels rain. They
in a forest that is being hallned by acid rain. They
analyzed water
analy.z.ed samples from
water samples 40rainfallsin
from 40 past year
the past
rainfalls in the year and found that
and found the mean.pH
thatthe the water samples
mean pH of the-water samples
has a
has confidence interval
a 95% confidence interval of 3.2 to 3.8. Which
Which of the following ~onclusions
the following conclusions is :the most appropriate
the most appropriate
based on
based on the interval?
confidence interval?
the confiaence
all the
A) 95% of all rainfalls in
forest rainfalls
the forest in the past year
the past have a
year have pH between
a pH and 3.8.
between 3.2 and
B) 95% of all
all the forest rainfalls
the forest rainfalls in the
the past decade have
past decade have a between 3.2 and
pH between
a pH ~d 3.8.
C) It is plausible that the true mean pH all the forest rainfalls the past year
C) !tis plausible that the true mean pH of all the forest rainfalls in the past yearisbetween 3.2 and
is between 3.2 and 3.8.
D) ltisplausiblethatthetmemeaan
D) It is plausible that the true mean pH of ofallthefomstrainfallsinthepastdecadeisbetweenB.2and
all the forest rainfalls in the past decade is between 3.2 and
3.8.

you don’t
If you
If know what
don't know what aa confidence interval is, don
confidence interval don't worry . You’ll
't worry. You'll never need to calculate
never need one and
calculate one and the
the SAT
makes these questions very
makes these questions very easy. A l l a confidence interval does
All a confidence interval does is tell you where the true mean (or some other
you where the true mean (or some other
statistical measure)
statistical measure) for the population is likely
the population likely to be
be (e.g. between
between 3.2 and and 3.8). Even though the
Even though only
the SAT only
brings up 95%
brings confidence intervals,
95°/o confidence intervals, there
there are
are 97% and 99% (any
and 99°/o percentage) confidence
(any percentage) confidence intervals. The higher
intervals. The higher
the confidence,
the confidence, the more likely the
more likely the true
true mean
mean falls within the interval.
within the interval. 50 the example
So in the example above, can be
above, we can be
quite confident
quite that the
confident that the true
true mean
mean pH of all the forest rainfalls
the forest the past
rainfalls in the year is between
past year and 3.8. Answer
between 3.2 and Answer
~(C) -. The
The answer
answer is not
n o t (D) because we cannot
because we draw conclusions
cannot draw conclusions about past decade
the past
about the when all the
decade when samples
the samples
were
w gathered from
e r e gathered the past
from the year.
past year.
confidence interval
A confidence interval does
does NOT
N O T say anything about
say anything the rainfalls
about the rainfalls themselves.
themselves. You cannot say that
cannot say any one
that any one
rainfall has
rainfall has aa 95%
95"/o chance having aa pH between
chance of having between 3.2 and and you
and 3.8, and you cannot say that
cannot say that 95% of all
all the forest
the forest
rainfalls in the
rainfalls past year
the past had a pH between
year had between 3.2 and Always remember
and 3.8. Always that a confidence
remember that interval applies
confidence interval applies
only to the
only the mean, statistical measurement,
which is a statistical
mean, which measurement, NOTNOT an an individual data point
individual data point or a group
group of datadata
points.
points.
Secondly, aa 95% confidence
Secondly, confidence interval
interval does
does not imply that
n o t imply there is a
that there chance it contains
a 95% chance the true
contains the true mean. Even
mean. Even
though confidence intervals
though confidence intervals are computed for the
are computed mean, you
the mean, cannot say
you cannot say that interval of 3.2 to 3.8 has
the interval
that the a 95%
has a
chance of containing
chance containing the
the true
true mean pH..
m e a n pH

259
CHAPTER
CHAYTER 28 STATISTICS
STATISTICS II
II

Sowhat
So what does mean in statistics
does itit mean be 95% confident
statistics to be something? If
confident in something? were repeated
experiment were
If the experiment repeated again and
again and
again, each with 40 water samples,
again, each with 40 water samples, 95% of those experiments would
those experiments would give us a confidence interval that contains
us a confidence interval that contains
the true mean. In other
true mean. words, the confidence
other words, confidence interval given in the example
interval given example is thethe result one experiment.
result of just one experiment.
Another u n of the same
Another rrun same experiment (another 40
experiment (another 40 samples) would produce
samples) would produce a different confidence
a different interval. Keep
confidence interval. Keep
on getting confidence intervals
these confidence
getting these intervals and
and 95% of them them will contain the true
will contain mean. 50
true mean. the 95% pertains
So the pertains to all
intervals generated
confidence intervals
the confidence generated by repeated
repeated experiments, NOT the chance
experiments, NOT any one
that any
chance that interval
confidence interval
one confidence
contains the true
contains Again, don’t
mean . Again,
true mean. worry about
don't worry how confidence
about how intervals are
confidence intervals calculated, but
are calculated, but be that
aware that
be aware
this is how ”confidence” is defined
how "confidence" defined in statistics.
statistics.

260
260
THE COLLEGE
THE PANDA
COLLEGE PANDA

CHAPTER EXERCISE:Answers for this chapter start on page 340.

A calculator following
allowed on the following
calculator is allowed
questions.
questions.
"'
§
·.c
ro Light Violations
Traffic Light Violations in Various
Various Towns
Towns
0
Male Shoe Size versus Age ·;;: 100
.....
13 I
Tu 90
:..::i
u
12 ::E 80
(ll
,,,, .t: 70
11 ..,,,-v .....
0 .........
/ 60
•- @

--
_i..--_
►-
/
V
I s ;:I
50
•.---- -
---,--- - :

8
.....,v-
,,,,.v !
/ C:
>-,
40
::i2 30
<lJ
--- -

7 / <lJ
s: -
<lJ
20
20
I i bO 30
30 4 0 50
40 50 6600 7070 8 800 9 900 1 100
00
6 (ll
....
10 11
10 11 12
12 13
13 14
14 15
15 16
16 17
17 18
18 19
19 20
20 <lJ Number lights
Number of traffic lights
Age (years)
Age (years) ~
The scatterplot above
The scatterplot above shows
shows the number of
the number
scatterplot above
The scatterplot shows the
above shows relationship
the relationship
traffic lights
lights in 15
15 towns
towns and
and the average weekly
the average weekly
between age,
between years, and
age, in years, and shoe
shoe size maless
24 male
size for 24
number light violations
number of traffic light violations that
that occur
occur in
between 10
between 10and 20 years
and 20 old. The
years old. line of best
The line best fit
each
each town. The line
town . The line of best fit is also
best fit shown.
also shown.
also shown.
is also shown. Based the data,
Based on the how many
data, how many 19 19
Based on thethe line best fit, which
line of best which of thethe
year old
year old males had aa shoe
males had greater than
size greater
shoe size than the
the
following
following is the
the predicted
predicted average weekly
average weekly
one predicted
one by the
predicted by the line best fit?
line of best
number of traffic light
number violations in a
light violations with
town with
a town
A ) 1
A) 75 lights?
75 traffic lights?
B
B)) 2 A
A)) 440
0
C)3
C) B) 50
50
D ) 44
D) C
C)) 555
5
D)) 6600
D

survey of 400 senior


In a survey s, xatpercent
seniors, said that
percent said that
they plan
they plan on majoring in physics.
on majoring university
One university
physics. One
has used
has used this data to estimate
this data estimate thethe number
number of
physics majors
physics expects for its
major s it expects entering class
its entering class of
If the
students.. If
3,300 students university expects
the university 66
expects 66
physics majors, what
physics majors, what is the value of x ??
the value

261
CHAPTER 28 STATISTICS II

university wants
A university dietary
determinethe dietary
wants to determine Consumer Behavior
Consumer during Store
Behavior during Store Sales
preferences students in its
the students
preferences of the its freshman
freshman class. 'vJ 60
the following
Which of the
Which survey methods
following survey methods isis most
most I
likely to provide
likely the most
provide the
Selecting aa random
A) Selecting
valid results?
most valid results?
sample of 600 students
random sample students
l.E:! 55

- 45
50
!

from the
from university
the university .§ 40
..... 35
=

-
~
B) Selecting
Selecting aa random sample of 300 students
random sample students ~ 30 -----
.::l _,,..
from the
from university’s freshman
the university's class
freshman class 0.. 25
0.. _, 1...---"
...c: 20
0
Selecting aa random
C) Selecting sample of 600 students
random sample students ~
1...---"
the university’s
from the university's freshman class
freshman class
tJ)
15 i.,...---'
~ 10 ~

Selecting aa random
D) Selecting sample of 600 students
random sample students ~
Q) 5
from one
one of the university’s freshman
the university's freshman ~ 0
dining halls
dining halls 0 55 110
0 0 115
5 220
0 225
5 330
0 335
5 440
0 445
5 550
0
Store Discount (%)
Store Discount (%}

Shopping
Shopping time refers to the
time refers time a
the time customer
a customer
spends one store. The scatterplot above
spends in one store. The scatterplot above shows
shows
candidates are
Two candidates running for governor
are running governor of aa the average shopping
the average shopping time, minutes, of
time, in minutes,
state.
state. A recent poll reports
recent poll reports that out of a
that out random
a random customers stores offering
customers at 26 different stores offering various
various
sample of 250 voters,
sample support Candidate
voters, 110 support Candidate A discounts. The line
discounts. The best fit is also
line of best shown..
also shown
and support Candidate
and 140 support estimated
Candidate B. An estimated Which of the following
Which following is the best interpretation
the best interpretation
500,000 state are expected
residents are
state residents expected to vote
vote on of the meaning
meaning of the y-intercept of the
the y-intercept line of
the line
According to the
election day. According
election poll, Candidate
the poll, Candidate B best fit?
best
is expected
expected to receive
receive how many more
how many votes
more votes A) The predicted
predicted average
average shopping time, in
shopping time,
than Candidate
than Candidate A? minutes, of customers
minutes, store offering
customers at a store offering no
A) 60,000 discount
discount
B) 130,000 B) The predicted
predicted average
average shopping time, in
shopping time,
C) 220,000 minutes, of customers
minutes, store offering
customers at a store offering a
50% discount
discount
D) 280,000
C) The predicted
C) The increase in the
predicted increase average
the average
shopping time,
shopping minutes, for each
time, in minutes, each one
one
percent increase in the
percent increase store discount
the store discount
D) The predicted
predicted average number of
average number
customers store offering
customers at a store discount
offering no discount

262
COLLEGE PANDA
THE COLLEGE PANDA

Advertising for 16 Companies ,....._ Movie Length versus Box Office Sales
~
'"§' 500 200 ~~-~--~----~-~~
~ 200
--
ro
~ 450
.g 180 t---t---+---t--t-- - --+--+--.-- · --;

~ 400
• •- -- ·- ""'
0
r.r,
160 t---t---+---t--t--+---+--+--f---,

~ 350
"d
~ 300
r.r,
;::l 250 ~
--
,.
I
......

./"'
.......-
./"'
,,,,--- '
7 j
·s
140 ......_
120 r---+--t---+---+--
-.=
...........................
1
,.

_-+-- - -f- - --= b.--i==_...


:.§_100 1---..- -+ r---l - = -+-''ll->-+--+---+- . -+----l
.-

0
,,,,---
./"'
.... 200
..c: .,,,,---,. r.r, 80 f------<q.....
-t---J- - l---l--l---l--1----l
:.§. 150 :.----
__.,,,,,,..
• ~
Qi

60 l--+--+----+-+---+-.....j.......--4--+-~ •
• •
----- ,.
Cf)
100 Qi
u
40 t---+--+--+--+---+--+- --l- -+- ~
50 $ 20 -1-
_____ _*__.a... {www
• ·+-------t---+-
- . .,_
.............-+---+---+----t
_ , . . . . . . . . ~ _

0
0 >< o ~~--+--~---~~-~-~~
0 101 0 20
2 0 30
3 0 4400 550
0 606 0 70
7 0 80
8 0 90
90100
100 0
c:o 60 70 80 90 100 110 120 130 140 150
Advertising Expenses
Advertising Expenses (in(in thousands dollars)
thousands of dollars) Length (minutes)
Movie Length
Movie (minutes)

The scatterplot above shows


scatterplot above the relationship
shows the relationship The scatterplot above plots
scatterplot above the lengths
plots the lengths of 1515
between
between revenue advertising expenses
and advertising
revenue and expenses for movies against
movies against their
their box sales. The
box office sales. line of
The line
16companies.
16 line of best
companies . The line best fit
fit is also shown.
also shown. best fit is
best is also shown. Which
also shown. Which of the following is
the following is
Which the following
Which of the following is the
the best interpretation
best interpretation the
the best interpretation of the
best interpretation meaning of the
the meaning the
of the
the meaning the slope
meaning of the slope of the line of best
the line best fit? slope of the line
slope best fit?
line of best
A) The expected
expected increase
increase in revenue every
revenue for every A) The expected
expected decrease box office sales
decrease in box sales
one dollar
one advertising expenses
increase in advertising
dollar increase expenses per
per minute movie length
increase in movie
minute increase length
B) The expected increase in revenue
expected increase every
revenue for every B) The
The expected
expected increase
increase in box sales per
box office sales per
one thousand
one thousand dollar increase in advertising
dollar increase advertising minute movie length
increase in movie
minute increase length
expenses
expenses C) The expected
C decrease in box
expected decrease sales
box office sales
C) The
C) expected increase
The expected increase in advertising
advertising per
per 10‐minute
10-minute increase length
movie length
increase in movie
expenses
expenses for every
every one thousand dollar
one thousand dollar D) The expected increase
The expected sales per
box office sales
increase in box per
increase
increase in revenue
revenue 10‐minute increase
10-minute movie length
increase in movie length
The expected
D) The
D revenue of a
expected revenue company that
a company that
has no
has no advertising
advertising expenses
expenses

263
CHAPTER 28 STATISTICS
CHAPTER STATTSTTCS II
II

Mistakes
Mistakes Made
Made in Incentive-based
Incentive-based Task Fat and Calories of Ice Cream
60
60 ,, 440 ~

(I)
"O
(ll 50 ··-
,,, - 420 I/
E
<fl
(I)
40
• ~ ........... <fl 400 ~ /
~
.....
(ll "'-.
(I)
·c !/•
- ~
0 380
<fl

·e 30 cc •/
-~,
u
...... ~ ..... 360
~
0
.... 20
(I) '- ~ 340 - V
.0
§ 10 • 1
z 320 -
0 ‘- l
0 300 I

0 100 200 300 400 500 600 15 20


20 25 30
30 35
Prize i n dollars)
Prize ((in dollars) Total fat (grams)
(grams)

In a
a psychological
psychological study, researchers asked
study, researchers asked The scatterplot
The above shows
scatterplot above shows thethe fat content
content and
and
participants to each
participants complete aa difficult
each complete difficult task
task for calorie counts of 8 different
calorie counts cups of ice cream.
different cups cream .
a cash prize,
a cash prize, the amount of which
the amount varied from
which varied from Based on the line
Based line of best the data
best fit to the data shown,
shown,
participant to participant.
participant participant. The results of the
The results the what is the
what expected increase
the expected increase inin the number of
the number of
study,
study, as well as
as well line of best
the line
as the best fit, are
are shown
shown inin calories each additional
calories for each additional gram
gram of fat in a cup
in a cup
the scatterplot above.
the scatterplot Which of the
above. Which following is
the following of ice cream?
cream?
best interpretation
the best
the interpretation of the meaning of the
the meaning the
A) 5
y-intercept
y-intercept of the line of best
the line best fit?
B) 8
A) The
The expected decrease in the
expected decrease the number of
number of
C) 20
20
mistakes made per
mistakes made per dollar
dollar increase the
increase in the
cash prize
cash prize D ) 4
D) 40 0
B) The expected increase
The expected increase in the number of
the number of
mistakes per dollar
made per
mistakes made increase in the
dollar increase the
cash prize
cash prize
C) The
C expected dollar
The expected amount of
dollar amount of the cash
the cash record of driving
A record driving Violations
violations by type
type and
and
prize required
prize required for a person to complete
a person complete thethe vehicle is shown
vehicle below.
shown below.
task with 0 mistakes
task with mistakes
D) The
The expected number of mistakes
Violation
Violation Type
expected number mistakes aa person
person
makes in completing ‐ Speeding
speeding Stop Sign Parking
stop Sign Total
makes the task
completing the when no
task when no cash
prize
prize is offered
offered
cash
m Truck
Car
68
83
m 39
51
17
26
- 124
124
160
” Total 151 m 90 43 284 u
If the data
If estimate driving
used to estimate
data is used violation
driving violation
information about 2,000 total
information about violations in
total violations in a
a
certain
certain state, which of
state, which the following
of the following isis the best
the best
estimate of the number
estimate speeding violations
number of speeding violations
committed
committed by cars in the state?
state?
A)
A) 479
B) 585
C) 1063
D) 1099

264
THE COLLEGE
THE PANDA
COLLEGE PANDA

Nitrogen Fertilizer and Oats


~::i
Food Courts in Various Malls
,,...._80 100
~
~ 70 •
I
ro
......
en
~
I
.... /
V ... 80
~ 60
v-
QJ

en • 0...
·r--, ~
~ 50
v .l!l
~

..c: ro
QJ
60
en

-----
en
6
::i 40
L// •
......
0
.... 40
r---- r--.._ ,i
!

~ 30 QJ
..0 r---
-
0
0 20 ~~ 20 --
"Cl
~ 10 ~
>= 0
0
, ...
ro
QJ 0
0 100 200
200 300
300 400
400 500 ~ 4 5 6 7 8 9 10
Amount of nitrogen
Amount applied (pounds
nitrogen applied (pounds per
per acre)
acre) Number
Number of restaurants
restaurants

The scatterplot above


The scatterplot above shows
shows the the amount
amount of The scatterplot
The scatterplot above
above shows the distribution
shows the distribution of
nitrogen fertilizer
nitrogen applied to
fertilizer applied to 8 oat and
fields and
oat fields seats the restaurants
seats for the restaurants in 7 different mall food
different mall food
yields.. The
their yields
their line of best
The line is also
best fit is also shown
shown.. courts. The line
courts. The best fit is also
line of best shown.
also shown.
Which of the
Which following is
the following is closest to the
closest to the amount
amount According to the
According the data,
data , what is the
what is the total number
total number
applied, in pounds
nitrogen applied,
of nitrogen pounds per acre, to the
per acre, the of seats
seats at
at the food court
the food represented by
court represented by the
the data
data
oat field whose
oat field is best
yield is
whose yield predicted by
best predicted by the line
the line point that
point that is farthest from the
farthest from line of best
the line best fit?
of best
best fit? A) 200
A) 200
A) B)
B) 240
240
B) 350 C)
C) 320
320
C) 400
C) 400 D) 560
D)
D) 450

Researchers must
Researchers conduct an
must conduct experiment to see
an experiment see
whether a
whether annew effective in relieving
vaccine is effective
e w vaccine relieving
certain allergies . They
certain allergies. selected a
have selected
They have random
a random
sample
sample of 100100 allergy
allergy patients. Some of the
patients. Some the
patients are assigned
patients are assigned to to the vaccine while
n e w vaccine
the new while
the are assigned
rest are
the rest assigned to to the traditional treatment.
the traditional treatment.
Which of the
Which the following methods of assigning
following methods assigning
each patient’s
each treatment is
patient's treatment most likely
is most likely to lead to
lead to
a reliable conclusion
a reliable about the
conclusion about effectiveness of
the effectiveness
the
the n e w vaccine?
new vaccine?
A) Females are assigned
Females are the n
assigned to the vaccine.
e w vaccine.
new
B) Those who have
Those who more than
have more one allergy
than one are
allergy are
assigned
assigned to the n e w vaccine.
the new vaccine .
The patients
C) The divide themselves
patients divide evenly into
themselves evenly into
ttwo
w o groups. coin is tossed
groups. A coin tossed to decide
decide
which group receives
which group receives the e w vaccine.
the nnew vaccine.
D) Each
D patient is assigned
Each patient assigned aa random
random number.
number.
Those with an
Those with even number
an even number are assigned to
are assigned
the
the n vaccine.
e w vaccine.
new

265
CHAPTER 28 STATISTICS
CHAPTER 28 II
STATISTICS II

A basketball manufacturer selects


basketball manufacturer selects aa random
random The length of a
The length blue-spotted salamander's
a blue-spotted tail
salamander’s tail
sample
sample of its
its basketballs each week
basketballs each ensure aa
week to ensure can be
can used to estimate
be used its age.
estimate its A biologist
age. A biologist
air pressure
consistent air
consistent within them
pressure within them is selects 80blue-spotted
selects salamanders at
80 blue-spotted salamanders random
at random
maintained.
maintained. ln In Week 1, 1, the sample had
the sample had aa mean
mean and finds
and finds that
that the average length
the average length of their
their tails
tails
air pressure
air pressure of 8.2 psi (pounds per
psi (pounds per square
square inch)
inch) has a
has 95% confidence
a 95°/o interval of 5 to 6 inches.
confidence interval inches.
and a
and a margin
margin of error
error of 0.1 psi. ln
0.1 psi. In Week 2, the
2, the the following
Which of the
Which following conclusions
conclusions is the
the most
most
sample had
sample had aa mean pressure of 7.7 psi
mean air pressure and a
psi and a appropriate based on
appropriate based the confidence
on the confidence interval?
interval?
margin error of 0.3 psi.
margin of error psi . Based
Based onon these
these results,
results, A) 95°/o
95% of all
all blue-spotted salamanders have
blue-spotted salamanders have aa
which
which of the following is aa reasonable
the following reasonable
tail that between 5 and
that is between length..
inches in length
and 6 inches
conclusion?
conclusion?
B) 95% of all
all salamanders have aa tail
salamanders have that is
tail that
A) Most
Most of the basketballs produced
the basketballs produced in Week 11 between
between 5 andand 6 inches length.
inches in length.
had an
had air pressure
an air pressure under psi, whereas
under 8.2 psi, whereas
C) The
The true average length
true average tails of all
the tails
length of the all
most of the
most produced in Week 2
basketballs produced
the basketballs
blue-spotted
blue-spotted salamanders
salamanders isis likely between
likely between
had an
had pressure under
air pressure
an air psi.
under 7.7 psi.
5 and
and 6 inches.
inches.
The mean
B) The mean air air pressure all the
pressure of all the basketballs
basketballs
D) The
The true average length
true average length of the tails of all
the tails all
produced
produced in Week 11 was psi more
was 0.5 psi than
more than
salamanders is likely
salamanders likely between
between 5 and
and 6
·tthe
h e mean
mean air air pressure
pressure of all
all the basketballs
the basketballs
inches.
inches .
produced in Week 2.
produced 2.
The number
C) The basketballs in the
number of basketballs the Week 1
sample was
sample was more than the
more than number of
the number
basketballs
basketballs in the the Week 2 sample.
sample.
D) It is very likely that
very likely that the
the mean
mean air pressure
air pressure economist conducted
An economist research to determine
conducted research determine
all the
of all basketballs produced
the basketballs produced in Week 11 whether there is a
whether there a relationship
relationship between
between the the price
price
was less
was than the
less than the mean
mean airair pressure all
pressure of all of food and population
food and density. He
population density. He collected
collected
basketballs produced
the basketballs
the produced in Week 2. 2. data from a
data from random sample
a random sample of 100 U U.S. cities and
S . cities and
found significant
found significant evidence
evidence thatthat the price of food
the price food
lower in places
is lower with a
places with a high population
high population
density.
density. Which
Which of thethe following conclusions is
following conclusions
best supported
best supported by by these
these results?
results?
student is assigned
A student assigned to conduct
conduct a survey to
a survey ln US.
A) in U.S. cities, there is a
cities, there positive association
a positive association
determine
determine the mean number
the mean servings of
number of servings between
between the price of food
the price and population
food and population
vegetables eaten by aa certain
vegetables eaten group of people
certain group people density.
density .
each day.. The
each day student has
The student n o t yet
has not decided which
yet decided which B) In US. there is aa negative
cities, there
U.S. cities, negative association
association
group people will
group of people will be
be the focus of this
the focus survey.
this survey. between
between the price of food
the price and population
food and population
Selecting aa random
Selecting sample from
random sample from which the
which of the density.
density.
following groups would
following groups would m likely give
o s t likely
most give the
the
margin of error?
smallest margin
smallest error? C) In UU.S. cities, aa decrease
S . cities, the price
decrease in the price of food
food
is caused
caused by by anan increase in the
increase in the population
population
A) Residents
Residents of the same city
the same density.
density.
B) Customers
Customers of a certain
certain restaurant
restaurant D) In U an increase
cities, an
S . cities,
U.S. increase in the
the population
population
C) Viewers
Viewers of the same television
the same show
television show density
density is caused decrease in the
caused by a decrease price
the price
D) Students are following
who are
Students who the same
following the daily
same daily of food.
food.
diet plan
diet plan

266
266
volume of all
The volume
The regular solids
all regular solids can be found
can be using the
found using following formula:
the following formula:
29 Volume
Volume
Volume = Area of base x height

why the
That’s why
That's the volume cube is V =
volume of aa cube 3 (the
: s53 area of the
(the area 2 and
base is s52
the base and the height is s)
the height s)

sS

volume of aa rectangular
The volume
The rectangular box/prism
box / prism is V=
is V : lwh
Izuh (the area of the
(the area Iw and
base is lw
the base and the
the height h)
height is h)

w
I

A n d the
And volume of aa cylinder
the volume V=
cylinder is V : m 2 h (the
rrrzlz area of the
(the area base is
the base 7rr22 and
is m and the
the height is 11)
height is h)

Even though the


Even though gives you
the SAT gives you these formulas at
these formulas beginning of each
the beginning
at the math section,
each math they should
section, they should be
be memorized,
memorized,
addition to
in addition the volume
to the volume of aa cone
cone
V : ‐3 rrrzlI

267
CHAPTER 29 VOLUME
CHAPTER VOLUME

and the volume


and the a sphere
volume of a sphere
4 3
V=
V : -§nr3
rrr
3
But what we have
what if we have a hollowed-out cylinder?
a hollowed-out What’s the
cylinder? What's volume of
the volume of that?
that?

I
'

Well, ifif we look at


we look at the base, it's
the base, just a
it’s just ring.
a ring.

A
The area the ring
area of the outer circle minus
the outer
ring is the minus the inner circle.
the inner

7tR2 ‐ 7tr2 = 7r(R2 ‐ r2)

To get vo lume, we
the volume,
get the we multiply
multiply this
this area by the
area by the height.
height.

v =
V 7r(R22 -‐ r2)h
: rr(R r 2 )h

addition to finding
In addition an object’s
finding an you'll also
volume, you'll
object's volume, also need
need to know how to find its density.
know how you’ll
Sometimes you'
density. Sometimes ll
given the
be given density formula
the density and sometimes
formula and you won't,
sometimes you won’t, so it's important
so it’s important to memorize
memorize it.

e ·ty i =
DeI\S;l =Mass‐ ‐
ns ty Volume
Volume

Denser objects are heavier


Denser heavier relative
relative to their size.
their size.

268
THE COLLEGE
THE PANDA
COLLEGE PANDA

CHAPTER EXERCISE: Answers for this chapter start on page 342.

calculator is allowed
A calculator following
allowed on the following
questions.
questions.
What is the
What the vo lume of a
volume a cube with surface
cube with area
surface area
24a 2 ?
24a2
A) 4a2
A) 4a2
8a2
B) 8a2

, , I C) 8a3
C) 8a3
,,
I

D)
D) 16a 3
16113
,
,
,, 5cm
I

A cylindrical water tank


cylindrical water with a
tank with a base radius of 4
base radius 4
and a
feet and
feet height of 6 feet can
a height be filled
can be filled in 33 hours
hours..
4cm
At that
that rate, how many
rate, how hours will
many hours will it take
take to fill a
In the above, aa cylindrical
figure above,
the figure cylindrical block of wood
block of wood cylindrical water tank
cylindrical water with a
tank with a base radius of 6
base radius
sliced into
is sliced into two pieces as
two pieces as shown
shown by the dashed
by the dashed and a
feet and a height
height of 8 feet?
8 feet?
What is the
curve. What
curve. volume of the
the volume the top piece in
top piece A) 4.5
cubic centimeters?
cubic centimeters?
B) 6
107i
A) lOrr C) 7.5
157r
B) 15rr D) 9
2071
C) 207T
D) 407r
40n

A clay brick
brick in the shape of aa right
the shape right rectangular
rectangular
prism has
prism length of 6 inches,
has aa length inches, a width
width that
that is
James wants
James wants toto cover
cover aa rectangular
rectangular box
box with
with 25% greater than its length, and a height
greater than its length, and a height that that is 2
wrapping paper.. The
wrapping paper The box
box has square base
has aa square base with
with inches shorter
inches shorter than length. The
its length.
than its brick has
The brick has aa
an area
an square inches.
area of 25 square inches. The volume of the
The volume the mass of 5.85 kilograms.
mass kilograms. WhatWhat isis the density, in
the density,
cubic inches.
box is 100 cubic
box inches. How
H o w many square
many square grams per cubic
grams per inch, of the
cubic inch, brick?
the brick?
wrapping paper
inches of wrapping
inches will James
paper will James need to
need to
exactly cover all faces the box, including the
exactly cover all faces of the box, including the
top and
top and the
the bottom?
bottom?
A) 120
B) 130
C) 150
D) 160

269
CHAPTER VOLUME
CHAPTER 29 VOLUME

A cube with a
cube with length of 5 inches
side length
a side inches is painted
painted
black on all
black all six faces.
faces. The cube is then
entire cube
The entire then cut
cut
into smaller
into smaller cubes with sides
cubes with sides of 1 inch. How
inch. How
many small
many cubes do
small cubes not have
do n_ot have any black paint
any black paint
on them?
them?
A)
A ) 227
7
B
B)) 3
311
C
C)) 336
6
D
D)) 4488

Yuna finds
finds aa box with an
box with an open
open top.
top. Each side is 8
Each side
inches long.
inches long . If she
she fills this with identical
box with
this box identical 2
A container
container in the shape of a right
the shape right circular
circular in by 2 in by 2 in cubes, how many
cubes, how many of these
these
cylinder shown above is just large enough to
cylinder shown above is just large enough to fit cubes will be
cubes will be touching
touching thethe box?
box?
exactly
exactly 33 tennis
tennis balls with aa radius
each with
balls each radius of 22
A
A)) 440
0
inches.. If
inches If the
the container were emptied
container were emptied o out and
u t and
filled to
filled to the
the top with water,
top with would be
what would
water, what the
be the B)
B) 48
48
volume of water,
volume cubic inches
water, in cubic held by the
inches,, held the C
C)) 552
2
container?
container? D
D)) 556
6
A) 16n
A) 1671
247r
B) 24n
IN
C) 32n
327:
D) 48n
D) 487r A 3 x 4 x><5Ssolidblockismadeupofl
A3><4 solid block is made up of 1 xx11 x1 x1
unit cubes.
unit outside surface
The outside
cubes. The surface of the block is
the block
painted black.
painted black. How many unit
How many cubes have
u n i t cubes have
exactly one face painted
exactly one painted black?
black?
A ) 116
A) 6
An aquarium
aquarium has an 80
has an 80 inch
inch by 25
25 inch
inch
rectangular base
rectangular and aa height
base and height of 30
30 inches The
inches.. The B
B)) 118
8
aquarium
aquarium is filled withwith water
water to depth of 20
to aa depth 20 C
C)) 220
0
inches.. If
inches solid block
If aa solid volume of 5,000 in
with a volume
block with in33 D
D)) 222
2
is completely
completely submerged
submerged in the
the aquarium,
aquarium, by by
how many
how inches does
many inches the water
does the level rise?
water level rise?

A right
right circular has aa volume
cone has
circular cone 6rra 4 cubic
volume of 67ra4 cubic
centimeters, where a is a positive
centimeters, where positive constant. If the
constant. If the
2
height
height of the
the cone
cone is 2a centimeters, which
2112 centimeters, which of
the following
the following gives the radius,
give s the radius, in centimeters,
centimeter s, of
the
the base
base of the cone in terms
the cone terms of a ??
A) a./3
A) tat/3
3a
B) 311
B)
C) 3a 2
C) 3a2
D)
D) 9a
941

270
THE COLLEGE
THE PANDA
COLLEGE PANDA

15 cm 10 8

crate that
A crate that is 10 inches long,
10inches long, 8 inches
inches wide,
wide, and
and
inches high
3 inches shown above.
high is shown The floor
above. The floor and
and the
the
four
four walls are all one
walls are one inch
inch thick.
thick. H o w many
How many
manufacturer produces
food manufacturer
A food packages of
produces packages one-inch cubical
one-inch blocks can
cubical blocks can fit inside
inside the
the crate?
crate?
frozen ice
frozen ice cream
cream cones. ice cream
Each ice
cones. Each cream cone
cone A
A)) 884
4
right circular
consists of aa right
consists circular cone
cone that with
filled with
that is filled
B)
B) 96
ice cream
ice until aahemisphere
cream until formed above
hemisphere is formed above the
the
as shown
cone as
cone shown in the figure above.
the figure right
The right
above. The C) 120
circular cone
circular has aa base
cone has base radius cm and
radius of 9 cm and aa D) 144
slant height 15cm. What is the volume of ice
slant height of 15 cm . What is the volume ice
cream, cubic centimeters,
cream, in cubic centimeters, the manufacturer
the manufacturer
uses each ice cream
uses for each cone?
cream cone?
A) 729n
A) 7297r
8107r
B) 810n
C) 8917:
891 rr
96071
D) 960n

right circular
A right cylinder has
circular cylinder has aa base radius rr that
base radius that
is 22 inches longer than
inches longer than its height. Which
its height. Which of thethe
following expressions gives
following expressions gives the
the volume, cubic
volume, in cubic
5 6
inches, cylinder in terms
the cylinder
inches, of the terms of rr ??
A) 271'r33
A) 2nr Note: Figure
Note: Figure n o t drawn
not to scale
drawn to scale..
r3 +
7rr3
B) n 2m2
+ 2n r2
r 3 -‐ 2n
C) nm3 r2
27rr2 The concrete
The concrete staircase shown above
staircase shown built from
above is built from
a
a rectangular
rectangular base that is 55 meters
base that and 6
long and
meters long 6
D) 2n
D) r3 +
27rr3 r2
+ nm2
meters wide . The
meters wide. three steps
The three equal
have equal
steps have
dimensions and
dimensions and each
each one has a
one has rise of 0.2 meters.
a rise meters.
If the density
lf the density of concrete
concrete is is 130 kilograms
kilograms per
per
cubic
cubic meter, what is the
meter, what mass of the
the mass concrete
the concrete
staircase in kilograms?
staircase (Density is mass
kilograms? (Density mass divided
divided
by volume)
by volume)
A) 1,420
B) 1,560
C) 1,820
D) 2,040

271
271
Answers to.the
Answers to the
Exercmes
Exercises
30
l: Exponents & Radicals
Chapter 1:Exponents
Chapter Radicals
EXERCISE
EXERCISE1:

1. 1 36
11. -~36 9
20. 9

2. -711 12. 64 1l
21. 9
21. 9
3. 1 72
13. -7‐72
22. 125
4. -A11 14. 108
1
23. ‑
23
5. 11 648
15. -7648 · 125
125

16. 11 24. 49
6. -211 24- 49
1
7. -‐1 1
7.
17.. !6g
17 1 25_ *
25. 49
49

8. 727
- 27
11 26. 1,000
26
18. 4
18. 7
9. -727
27 4 1
1
27
27
10. 27 19. 1 · 1,000
1,000
10. 27

272
THE COLLEGE
THE PANDA
COLLEGE PAN DA

EXERCISE
EXE RCISE 2:
2:

1. 6x55
1 6x 10 x33
10. x
19 36m8
19. 36m8
88 6 1
2. k2
2. ‐ X
x6 ~1
k2 11. y‐3
11. 3 20. ”6
20. 6
y a
15x22
3. 15x 3112 21. 1,12
3u 2 21. b12
12. T
4
4. -‐ 2211 4 22 m4
1114
22.' -n
11 13. -‐8u3v3
13. 8113 v 3
5.
5. 7
8x 6
n

8x 14 15 23.1’2
14. x 5 23. x 2
9b55
6. -‐&a3 15. 3x
15. 3x88 24‘ L 1
113 24. -mn2
-2
mn
16. Xx
7 n4
114 25. k
7. 25. k
. 22 17. xx99
17. 6
m6
8. a4b6
8 crib6 22
26. ‑m9
"9
18. 3‐3 n
X
y22 x 27 5 7 9
9. 2
9. y_2 27.. x5yy7zZ9
x
X

EXERCISE
EXE 3:
RCISE 3:

1· l.1. 2/3
Ni 7. 4\/'2
7. Ni 13. x = 2
13. X = 2
2. 4\/6
2. 4/6 8, 10\/'
8. 1o\/§ 2 14. x = 8
14. X = 8
3. 3v'S
3. 3\/5 9. 2\/'2
9. zfi 15- x
15. = 21
X = 21

4.3f2
4. 3\/'2 10. sfi
10. 8\/'2 x= 5
16. 11
X =
16.
5. 6\/§
5 . 6/3 11 .xx =
= 5500 17x26 2
11.
17. X = 6
6.15\/§
6. 15/3 12. Xx =
12. =55 18. x : 6
18. X = 6

I
I
II
\
I

•'
j

273
CHAPTER 30 ANSWERS
CHAPTER THE EXERCISES
ANSWERS TO THE EXERCISES

EXERCISE:
CHAPTER EXERCISE:

10. ‑
l. 7. [fil avoid any
E To avoid any trickiness,
trickiness, it’s
it's best plug in
best to plug
numbers. Let a =
numbers . Leta = 2 and Going through
and b = 2. Going through
each
each choice,
choice,
a =3
1
A) (‐4)2
A) 16
(- 4) 2 = 16
7
fl : 3 B)
B) (‐4)4
(- 4)4 = 256
C)
C) (2-2)2
(2 · 2)2 = 1616
11=3\/E
= 3y'a
2 · 24 =
D)) 2-24
D =3322
= 22-·1166 =

w;
~ = y'a (B) is the

[Q]
the largest.
largest.

-91‐a
1 8. Cube both
[__D_] Cube sides
both sides the first equation,
of the equation,
=a
(x2)3 = (y3)3
2-I x6 = y9
P = 233
2x 9 6
N y9
ow y
Now can be
can be replaced
replaced by xx6,,
2.1/
2x‐y=23
2x-y = 23 X32=y9
x3z =y9
x ‐ yy =: 3
x- x3z=x6
x3z = x6
x = yy +
x= +3 3 2=: 6
3z
z=
2 : 2
3. [E] Raise
[Q] to the
side to
each side
Raise each the 4th power:
4th power:

yy5
5 = 10
= 10
9 .‑
9. 0

(y5)4 = 104
(y5)4 = 104 Ir' f! ti
Vn/Esz-xé
v x = y x • x ==\/x_%=(x%)"l’
X \f y x = (x 2 ) 2 ==xx
2 1
31 3
4

3/20 = 10,000
y20
3
Therefore , a = 3
Therefore,
4
4. 0
10.
4G-::i, 2 4 ! 2· 1 4.! ! 1 r=
v x- y ·=(x y )4=x 4y 4= x "y =y v x
In ' xbc = x30

xac+bc ___ x30

m+m=w
ac + be= 30
(a + b)c=
b)c : 30
5c : 30
Sc= 30
cc== 6
6 [g
6..‑

274
I
I
THE COLLEGE PANDA
1HE COLLEGE PANDA

11.@]
22(2n+3) =
22(2n+3} = 23(n+5)

3 (n + 5)
+ 33) )=: 3(n
2(2n +
4n + 66 =
4n + : 3n
3n ++15
15
n=9

12. 0( - 2); = n =

v'l- 2 . - 2 . - 2 1- - 2 . - 2 = - 2 . 1 - 2 . - 2 =
- 2-¼
13. [£]
2x+ 3 - 2x = k(2x)
(2x) (23 ) - 2x = k (2x)
2x(23 - 1) = k (2x)
2x (7) = k(2x)
7= k

14. [filMultiply the exponents.


Multiply the exponents.

(53)4k
(53)4k== (5§)24
(5! )24
= 5s
512k =

Since the bases


Since the are the
bases are same, we can
the same, can equate
equate
the exponents: 12k = so k =
and so
= 8 and = g..
= % =
8 2
the exponents:
12 3
15.
15. [filThe means raised
2a means
The 211 raised to the
the 2a power and
2a power and
the b
the b on the bottom
on the the bth
means the
bottom means bth root.
root.
16. [Q]
16. Multiply both
@ Multiply equations together.
both equations left
The left
together . The
hand
hand side gives x5y5.
side gives The right
x y5. The
5
right hand side
hand side
gives 80.
gives

275
CHAPTER
CHAPTER 30 ANSWERS TO
30 ANSWERS TO THE EXERCISES
THE EXERCISES

Percent
Chapter 2: Percent
CHAPTER
CHAPTEREXERCISE:
EXERCISE:

f"o""i:l12.75
1. ~ 501‐‐ 150 =‐0.085 _ 8.5
0.085 = 8.5%A, 9. [D]
9. ~ Let xx be the sales
be the (as a
sales tax (as a decimal for
decimal for
now). We'll
now). convert it to a percent
We'll convert at the
percent at the end.
end.
2.
2. @32,m0(1.15) :
@]3 2,000 (1.15) = 36,800
36,800
105.82 (.90)( 1 + x) =
105.82(.90)(1+ = 100
3...O‐5 = 0.03125 ~ 3.1
[]J ~:6‐0.03125~ 31% % l1 +x=
+x=
100
100
(105.82)(.90)
(105.82) (.90)
4.. .Letz
[I] Then x
Let z = 100. Then =1.50(100)
x = 1.50(100) = 150 _ 100
and y = 1.20(100) := 120. x is
and is X = (105.82)( .90) -‐1 l
_ (105.32)(.90)
150‐120
150 - 120 30
30 0 x
X z 0.05 = 5%
=
120
120 ‘ fi120‘ =z25M
= %

larger than y.
larger than 10. I140 ILet xx be
be the number of
the number dishes served
of dishes served
during
during lunch. Then
lunch . Then
5. 0 Each year,
Each Veronica keeps
year, Veronica whatever she
keeps whatever she
has
has in her account plus
her account the interest
plus the interest on that
on that 1.175x = 940
l.175x
Because m
amount. Because
amount. misis a
a percentage,
percentage, we we can
can xX = 800
convert it to
convert to a decimal by
a decimal by dividing
dividing it by 100,
by 100,
giving
giving us 0.0lm . Therefore,
us 0.01m. Therefore, x = = 11 +
+ 0.0lm.
0.01m. Therefore, 940 -‐ 800 = 140 more
Therefore, dishes were
more dishes were
served during
served during dinner.
dinner .
fol new
El
6-~
new value old va
value -‐ old lue x 100%
value O
_2
Id
old
o va I
value
ue x 100 1/o- 11.
11. ~
2,690
2,690 -‐ 2, 140
2,140
2,140 x 1100%
X 0 0 / o~z 25.7%
21140 A := ((1.25)
A 1 2 5( B)
70: =
7 (1.25)(B)
0 (1.25) ( B)
7. []2JLet the original price of the book be $100.
James bought 6= 8
56
Then James
Then bought the book at
the book at
100(1‐- 0.20)(1
100(1 0.20) (1 -‐ 0.30)
0.30) = 100(0.80)(0.70)
= 100(0.80) (0.70) =
=
5
$56, which is ; = 56% of the original price . 12. [I] Kyle ate 20(1.20) =
Kyle ate 24 pounds
= 24 of chicken
pounds of chicken
1 0 wings and 15(1.40)
wings and 15(1.40) == 21 pounds of
21 pounds dogs..
hot dogs
of hot
21 =
8. 0 Let x be
be the
the number
number ofof pistachios at the
pistachios at the That’s
John
a total
That 's a total of 24
had 20+
John had 20 + 15
24 + 21
15 =
= 45
: 35pounds
pounds of food.
45 pounds
35 pounds of
food.
food.. The
of food The
start. At the
start. the end each day,
end of each what’s left
day, what's left is
percent
percent increase John to Kyle
from John
increase from Kyle is
the day’s
1 -‐ 0.40 = 0.60 of the day's starting
starting amount.
amount .
Over w o days,
Over ttwo days,
45
45 -‐35
35
35
35 29 = 2
z. .29
~ 9°/o
29%
x(0.60)(0.60)
x(0.60) : 27
(0.60) = 27
0.36x =
0.36x : 27
x
X = 75
= 75 13. [I] Let her starting
Let her starting card bex.
count be
card count x. A
A loss of
loss of
18 percent reduces
18 percent total to
her total
reduces her (0.82)x. From
to (0.82)x. From
there, an
there, increase of 36 percent
an increase gets the
percent gets the total
total
to (1.36)(0.82)x.
(1.36)( 0.82)x. Now,
Now,

(1.36)(0.82)x
(1.36) (0.82)x = n
n
x=~---
x : (1.36)
(1.36)(0.82)
(0.82)

276
THE COLLEGE
THE COLLEGE PANDA
PANDA

14. [Ij 12,000


12,000(0.94)10 ~ 6,460.
(0.94) 10 m 6,460 . 19. [g The
The total savings account
the savings
amount in the
total amount account
after 55 years
after will be
years will 3,000(1.06)5,
be 3, 000 (1.06 ) 5, but
but the
the
15. I100 ISince
Since scarves and ties make
scarves and make up 80% of interest earned will
interest earned will be 3,000(1.06)5
be 3,000 (1.06) 5 -‐ 3, 000..
3,000
the
the accessories, the 40 belts
accessories, the m u s t account
belts must account for The total
The amount in the
total amount the checking
checking account
account
20%. Letting
20°/o. the total
Letting the number of accessories
total number accessories after 55 years
after will be
years will 000(1.01 ) 5 , but
1,000(1.01)5,
be 1, but the
the
be x,
be x, interest earned
interest earned willwill be 1,000(1.01)5
be 1, 000 (1.01 ) 5 -‐ 1,000
1,000..
20% of Xx =
20°/o of = 40 With
With a a larger
larger initial deposit and
initial deposit and a a higher
higher
1 interest
interest rate, it's obvious
rate, it's obvious the savings account
the savings account
-5x:40
x = 40 will
will have
have eamed more interest.
earned more interest. The difference
The difference
5
in earned
earned interest will be
interest will (3,000(1.06)5
be (3, 000 (1.06) 5 -‑
xX :=2 200
00 3,000 ) -‐ (1,000
3,000) (1,000(1.01)5
(1.01) 5 -‐ 1,000
1,000). ).
There accessories in the
are 200 accessories
There are store.
the store.
Hopefully
Hopefully you're able to get
you’re able this without
get this without 20. [g The
The percent change is the
percent change the new minus the
n e w minus the
having to make
having make an equation, but
an equation, there’s no
but there's no old over the
old over the old times 100. Notice
old times Notice that the P's
that the P’s
harm in aa little
harm algebra! Now
little algebra! we can
N o w we can cancel out.
cancel out.

determine
determine that
that there are ~%x
there are x 200 =
z 40
40 scarves
scarves P(1 +- r)5 - P
lOO X 100 =
and ~g x 200 =
and ties . Half
= 120 ties. the 120
Half of the ties
120 ties
p

(60 ties)
ties) are replaced with
are replaced so the
scarves, so
with scarves, the store
store
w i l l end
will with 40 +
end up with + 60 =
= 100 scarves.
scarves.

I
After 3 years,
16. 1.728 After the market
years, the value of the
market value the
bond is
bond
9000.2)3 = 900(1.728)
900(1.2) 3 = 900(1.728) == 900 (1 + 0.728).
900(1+ 0.728).
Therefore, p =
Therefore, z .728

17. [g To get
get the value after
final value
the final after aa percent
percent
increase, you
increase, have to multiply
you have initial
the initial
multiply the
value by 1 plus
value the percentage
plus the (as aa decimal).
percentage (as decimal) .
So in 2016, Sims must have
Sims must spent l.34x
have spent 1.34x
dollars on
dollars on groceries.
grocerie s . In 2017, she must have
she must have
spent (1 +
spent +1.45)(1.34x) : (2.45)(1.34x)
l.45 )( 1.34x) = (2.45)( 1.34x)
dollars on groceries.
dollars groceries .

18. [Q] x be
|__D_] Let x be the
the amount
amount of taxes, millions
taxes, in millions
of dollars, County A in 2016.
collected by County
dollars, collected
the taxes
Since the
Since taxes decreased from 2016 to
decreased by 25% from
2017,

(1 -‐ 0.25)x =
: 60
60
0.75x = 60
60
= 80
xX = 80

Because County
Because County B Bcollected the same
collected the amount
same amount
County A in 2016, County
as County
as County B also
also collected
collected
80million
80 taxes in 2016. In
dollars of taxes
million dollars In 2017,
County
County B collected
collected 20% more
more than so
than in 2016, so
County B must
County have collected
must have collected 80 (1.20) =
80(1.20) : 96
96
million dollars in 2017.
million dollars

277
CHAPTER 30 ANSWERS TO lliE EXERCISES

Chapter 3: Exponential vs. Linear Growth


CHAPTER EXERCISE:

1..
1 0 situation presented
The situation presented in the question is a
the question a case of exponential value of the home
exponential decay. The value home
decreases
decreases m o r e significantly
more significantly in the
the beginning
beginning and then by smaller
and then and smaller
smaller and smaller increments
increments over time .
over time.
Only answer shows a graph that models exponential
Only answer A shows a graph that models exponential decay.

2.. lg the employees


[Q] Since the shelves at a
employees stock shelves a constant number of shelves
rate, the number
constant rate, shelves left to be decreases
stocked decreases
be stocked
at a constant time. Therefore,
over time.
rate over
constant rate function p is a decreasing
Therefore, the function decreasing linear function .
linear function.
25 20
3.. []] With exponential growth, we
exponential growth, need to calculate
we need calculate the percent increase, which
percent increase, out to be
turns out
which turns be 252‐020
; =
growth factor is 1.25, and
Therefore, the growth
0.25. Therefore, and the exponential growth
the exponential can be
growth can modeled by P := 20(1.25)',
be modeled 20(1.25 ) 1,
where 20is
where 20 initial population.
is the initial population.

4.. [I] The constant increase is 125 -‐


constant increase 100 = Therefore, the slope
: 25. Therefore, slope is 25 and the y-intercept
25 and initial
(the initial
y-intercept (the
population)
population) is 100.

[QJThe given
5.. @ definition of f is in the form
given definition form of an
an exponential
exponential equation, where 20
equation, where metal alloy’s
20 is the metal alloy's
temperature at the beginning
temperature at the experiment
beginning of the and 15is
experiment and percent by which
15 is the percent which itit increased second.
each second.
increased each

We know
know the temperature each second
increased each
temperature increased because the growth
second because + 1%
growth factor, 1 +
1
/!= 1.15, is greater
z 1.15, greater
15
than answer C to be
than 1. For answer the growth
correct, the
be correct, factor would
growth factor have to be 1 -‐ % =
would have
100
= 0.85.
f
6.. 0 Since the growth and ~5 is the
growth factor is 2 and the exponent, the predicted
exponent, the predicted number infected cells doubled
number of infected doubled
every 5
every days . So
5 days. after 5 days,
So after days, C(5) =: 80 (2) 1 =: 160. After
80(2)1 After 10
10 days,
days , C(10) 80(2) 2 := 320. And
= 80(2)2
C(lO ) = And so
so oonn .

7.. II] Based on


Based on the which we
model, which
the model, can express
we can as N =
express asN : 1,
1,000(0.97)4" 1, 000 (0.97 )/r/( l / 4 ), 97°/o
000 (0.97)4h = 1,000(0.97)"/“/4), 97% of the
the
1 1 .
bacteria after every
remain after
bacteria remain ~ hour.
every 5 That's a
hour. That’s a 3%
3% decrease every ~3 x 60 =
decrease every = 15
15 minutes.
minutes .

8.. II] Since the growth


growth factor in the equation is 1.002, the
the equation number of cars registered
the number increases by 1.002 -‐ 11 =
registered increases :
1
0.002 =
: 0.2% every year (6 months).
every %year Therefore, n =
months). Therefore, z 0.2
2
[QJSince we
9.. [E have a
we have a case of exponential
exponential decay, we can use
decay, we equation y = 1abk
use the equation 11115 as
as a reference. The
a reference.
population of trees
population starts at
trees starts at 14,000 and decays exponentially
and decays exponentially by a
a factor
factor of 1‐
1 - 0.06 = 0.94 every 4
every years .
4 years.
t
Therefore, a = 14000,b
Therefore, 14000,b := 0.94,k
0.94,k z= 4,
4, and
and the equation is P
the equation P== 14,000(0.94)i
14,000 (0.94)4

10. II] Scatterplot


Scatterplot C is the forming a
closest to forming
the closest a straight
straight line.

11. II] Keep


Keep track of the total amount she
total amount has received:
she has Because the total
received: 3, 9, 27, 81. Because total amount she has
amount she received
has received
triples relationship is exponential
each day, the relationship
triples each exponential growth.
growth .

12. []] Each month, Albert


Each month, loses a
Albert loses a book. this is a
Because this
book . Because decrease, the relationship
constant decrease,
a constant relationship is linear
linear decay
decay
(decreasing linear).
(decreasing linear) .

13.. [El
[QJThe cell count
count doubles
doubles every
every hour growth factor, r,r, is 2. The initial
so the growth
hour so count is 80 so
initial count c=
so 6 = 80.

278
THE
THE COLLEGE PANDA
COLLEGE PANDA

14. CgFive
Five percent the original
percent of the square footage
original square constant.
footage is a constant. doesn't change,
It doesn't which would
change, which make it
would make
linear growth..
linear growth

number of items
15. []] Since the number items gets half every
cut in half
gets cut year, the model
every year, one of exponential
model is one exponential decay.

equation is the
second equation
The second
16. [[] The model: V
linear model:
the linear V = 1, 500 (4) := 6,000.
1, 500(4) 6, 000. The first equation exponential
equation is the exponential
model:
model : V =
: 200 (24
200(24) ) =
= 3,200.
3,200. The
The difference
difference is 6,000
6,000 ~
- 3, 200
3,200 =
= 2,800.
2,800.

17. CgThe given equation is an


given equation an exponential
exponential growth equation . Since P = 50
growth equation. = 0 (i.e. at
when t =
50 when m
start), m
at the start),
m equal 50
u s t equal
must 50 (you can confirm
(you can plugging in t =
confirm this by plugging and P =
= 0 and = 50 into the equation).
50 into equation) . 50 we know
So we the
know the
either choice
answer is either
answer choice C or choice
choice D. N
Now each of the two
have to check each
o w we have answer choices
t w o answer which
choices to see which
better approximates
one better
one values in the
the values
approximates the table. If
the table. If we use a
we use we'll quickly
calculator, we’ll
a calculator, that the
quickly see that equation
the equation
with n =
with = 54.38 models
models the given values of P for t =
given values = 15,30,45 the equation
than the
better than
30, 45 better equation with = 86.12.
with nn =

18. [QJ
@ Let's standard exponential
the standard
use the
Let's use equation y := abf
exponential equation abt as From the given
reference. From
as a reference. information,
given information,
a = 16
a = (the initial
16 (the and bb := 1.02 (the
amount) and
initial amount) (the growth standard exponential
growth factor). Since the standard equation
exponential equation
15 5
requires tt and
requires and k to be
be in the same we have
units, we
same units, have to convert
convert 15 into days,
hours into
15 hours gives k =
which gives
days, which ~
= fl~~ := 5
days. Putting everything
days . Putting together, we
everything together, get g
we get (t ) = 16(1.02)
g(t) 1
/(SIB) = 16(1.02)
16(1.O2)'/‘5/8) 160.02)? .
W-

279
CHAPTER 30 ANSWERS TO THE EXERCISES

Chapter 4: Rates
CHAPTEREXERCISE:

1. [I!]For
For one week, Trm's
one week, Tim’s diet plan would
diet plan require aa protein
would require 60 =
intake of 7 x 60
protein intake = 420 grams.
grams. Since each
Since each
protein bar
protein provides 30
bar provides 30 grams he would
protein, he
grams of protein, need to buy
would need buy 420 + 30 = 14
7 30 14protein
protein bars
bars..

2. 0 Over 6 years, the screen size increased by a total of 18.5 - 15.5 = 3 inches. That's 3 7 6 = 0.5 inches
year.
each year.
each

3. [I] The pressure increases by 70 - 50 = 20 atm while the submarine descends - 900 - (- 700) = - 200
That’s 20 7+ 200 =
meters. That's
meters. atrn per
= 0.1 atm meter, or 1
per meter, a i m per
I atm meters.
per 10 meters.

4. WThe pool has a capacity of 5 x 300 = 1,500 gallons. At an increased rate of 500 gallons per hour, it
only take
would only
would 1,500 7+ 500 =
take 1, hours to
: 3 hours to fill the pool.
the pool.

5. []]

2051i"'"f''""
a,,nDkrt, dollars
p p l e Xs dd‐ollars d II
z?-20d dollars
20
a ~,
,,nDkrt,
""
= a
0 ars

6. [f]The racecar burned 22 - 18 = 4 gallons of fuel in 7 - 4 = 3 laps. To get to 6 gallons left, the racecar
have to consume
will have
will 18‐- 66 =
consume 18 z 12
12more gallons. That's
more gallons. That’s

12gallon§x
12
lapss
43
3 laps
_galk,nsx _galk,ns= more laps
‐ 9 9 more laps
4

which is Lap
which +9=
Lap 7 + = 16.

7. I100 IIt took 2.5 hours for 65 - 40 = 25 boxes to be unloaded. There are 3.5 hours from 3:30PM to 7:00PM.
25 boxes .
3 . 5 h o u r § x M .het:rrs =
hours, 3.5.het:rrs
In 3.5 hours, : 35
35 more will be
boxes will
more boxes unloaded. That’s
be unloaded. That's a
a total 65 +
total of 65 + 35
35 = 100
2.5mm
25
boxes.
boxes.
8. I120 IAverage
Average speed total distance
just total
speed is just distance over total time.
over total The total
time. The total distance,
distance, in inches, 12 =
was 2400 x 12
inches, was :
The total
28,800.. The
28,800 total time,
time, in seconds, w a s 4 x 60 =
seconds, was 28,800 7+ 240 z= 120 inches
: 240. 28,800 inches per second.
per second.

432 I
9. 1432
. 90 words
90 words
l m e u t é § xx fi
12.mi:m:rtes .
2 .5 .mi:m:rtes
=
‐ 432
432 words
words

l ~m,SJOn
:_w- --~ m 100 ~
xX ‐OO‐‐‐1’rl 1 jar
180
l :_ s- --~ m xX _2O10_ _m) amr ==60jars
15 ..!c!
.l-l...l-'H'tTJll:,SJOn
2
;n60 jars
nr~
~ ......s

11. [[]
6 0.mim:rtes
m m ? 32 32 kilometers
kilometers .,
22m
60
.het:rrs xX f l.lwtrf m ‐ s
xX _ .mi:fttrtes ~
14 5
~ 265 kilometers
265 kilometers

1n7 3 liters 8 dollars


12..@
~ 1111 hours
hours x 3hliters x
2 hours
ours
xm I'
1 liter
1ter
=
: 132 dollars
dollars

280
THE COLLEGE PANDA
COLLEGE PANDA

3
lye
@3cu
13. [Qj 3~
s r o i l xX 2 2 ‐ 2 ‐=
-é cups
cups of lye
11.25 cups
= 11.25 cups of lye
of lye
gw
2
5~
14. [I]An
An 88 inch
inch by 10 inch
by 10 of cardboard
piece of
inch piece cardboard has
has an area of
an area of 8 10 =
8 x 10 z 80
80 square A 16
inches.. A
square inches 16inch by 20
inch by 20 inch
inch
cardboard has
piece of cardboard
piece an area
has an area of 16 20 =
16 x 20 = 320 square
square inches.
inches.

£ ~
320 o. nx 2 dollars
‐2d‐°"a.rS‐‐
~= =8dollars
8
80 0 W 8 dollars

l1ci 29 ¢k6f
29 ' 2 large bahar
-1,000kulacl€x
15. ~
15. 1,000.hmrct< x fl
400144th
400
xx M _Fiko[ ~
.kalaci< 9M z16largebahar
16 large bahar
9

16. 0 The first 150 miles


The first miles took + 30 == 55 hours
took 150 -;-30 hours.. The next 200 miles
The next miles took 200 + 50
took 200-;- : 44 hours
50 = His average
hours.. His average
speed, total
speed, total distance over total
distance over was (150 +
time, was
total time, + 200) / (5 +
+ 4) m miles per
~ 38.89 miles per hour.
hour .

17. CgThe behind by 8 minutes


clock falls behind
The clock every hour.
minutes every hour. There are 6.5 hours
There are between 4:00 AM and
hours between 10:30 AM
and 10:30 A M,
the clock
so the
so behind by 8 x 6.5 =
clock falls behind : 52
52 minutes. The correct
minutes. The correct time then 52
time is then 52 minutes past 10:30
minutes past 10:30 A M,
AM,
which is 11:22 AM
which AM..
2240
18. 0 Jared's rate is E~ =
Jared’s rate
1
= 16 pages per
16 pages per hour Robert’s rate
hour.. Robert's rate must
must then 16 x 2 =
be 16
then be = 32 pages per
32 pages per hour.
hour . It

would take
would Robert %
take Robert hours to review
~; = 3.75 hours the 120-page
review the 120-page report. 60 =
That's 3.75 x 60
report. That’s = 225 minutes
minutes..

19.. ~
19 A
'A77.1
7.1 x 1015 h
1015 ~ ’ ns 0.8 grams
0.8 grams 1000m‘lf ~
1000.mt"
‑ 1L z 1.2x
_ grams
0 5 - L-
2 10-51‘3‐ra‐’ms
1
lmJHL
lf 4.8 X 1023 ~ X
x4.8><1023hy,dmgerr‘iofi§x
X 1L ~ 1. X l L

20. [I] What


What makes
makes this question aa little
this question tricky is
little tricky is that we don't
that we know the
don't know Brett travels
distance Brett
the distance travels each
each month
month
the number
or the gallons he
number of gallons he uses
uses each month.. But
each month But we need
need to start
start somewhere,
somewhere, so let’s say
so let's he needs
say he needs 2
gallons of gas
gallons gas each
each month (you can
month (you make up
can make up any
any number
number you want).. That
you want) means he
That means he travels 30 x 2 = 60
travels 30 60
miles each
miles and each
month and
each month gallon costs
each gallon + 2 =
costs 160 -;-2 = 80 dollars
dollars (ridiculous,
(ridiculous, I1 know) N o w if he
know).. Now switches
he switches
the new
to the n e w car, he'll only need
he’ll only + 40 =
need 60 -;-40 gallons of gas
= 1.5 gallons gas each
each month
month (distance divided
miles divided
(distance of 60 miles
by the 40 miles
the 40 per gallon).
miles per gallon). Because price of gas
the price
Because the stays the
gas stays same, that
the same, amount of gas
that amount gas w i l l cost
will him
cost him
80 =
1.5 x 80 z 120 dollars
dollars each
each month. answer ends
The answer
month. The up being
ends up being 120 nono matter number we
what number
matter what we make up
make up
for the
the number gallons of gas
number of gallons gas Brett
Brett uses each month.
uses each month.
. 160
an alternative
Here’s an
Here's solution.. If we
alternative solution we let
let x be the price
be the per gallon
pnce per gallon of gas, Brett currently
then Brett
gas, then uses -7
currently uses
X

gallons of gas
gallons gas each
each month Using that
month.. Using amount of gas
that amount gas means that he
means that 160 ((30)
drives -152‐0
he drives 30 ) = 4’
4,800 m iles each
--800 miles eac h
X X
44, 800 12
120
month. With
month. n e w car, he
the new
With the he will need ' BOO-;-
w i l l need ‐Z‐ 40 gallons of gas
40 = 70 gallons gas each
each month drive that
month to drive that distance.
distance.
x
X X
120
Since each
Since gallon of gas
each gallon gas costs atdollars,
costs x he will
dollars, he spend -% x x =
need to spend
will need : 120 dollars on gas
dollars on gas each
each month.
month .
X

21. I48 j Each


Each jar of honey +4=
costs 9 -;-4
honey costs = 2.25 dollars
dollars.. She sell each
can sell
She can 15+
each jar for 15 -;-33 = 5 dollars.
dollars. That's
That’s aa profit
profit
of 5 -‐ 2.25 = per jar. To make
dollars per
: 2.75 dollars profit of 132 dollars,
make aa profit dollars, she would have
she would have to sell + 2.75 =
sell 132 -;-2.75 = 48
48
jars.
jars .

281
281
CHAPTER
CHAPTER 30 ANSWERS
ANSWERS TO THE EXERCISES

Chapter 5: Ratio & Proportion


CHAPTER EXERCISE:
EXERCISE:

1. ~ Since a z= 28, the value


value of b is 6.@
6. @J

; x 28 = 24. The value


g value of c is then
then 1 2
5
void 2
Vold =
3mzh
nr h
5x24‐15.
BX24 = 15. 1 2
=
Vnew =
Vm’w 7r(0.80r)2(1.10h)
n( 0.80r) (1.10h)
2. [lJRemember
Remember that
that ratios are essentially
ratios are essentially just
3
fractions. So the given
fractions. 50 ratio is equivalent
given ratio equivalent to = (0.80)2(l.10) (~m
2h) = 0.704V
(0.80)2(1.10) (értrzh) 0.704v0m
0 1d

119392 9
zl+1l‐_+ = -4 XX -3 =‐ §3-2‐=33:. 22
2- ...!... } - = - ...!... -
4
4· 2 _44·2 The volume
volume of the cone decreases
decreases by
1 ‐- 0.704 := 0.296 = 29.6%.
Therefore, n =
Therefore, = 3. 7. [[]
3. [El be the price
@J Let y be price of Product
Product Y. Then the
Y. Then the
price
price of Product
Product X 1.25y and
Xisis l.25y the price
and the price of 1
Product Z is 0.75y.
0.75y. Simplifying ratio of
Anld = §(bl + b2)h
Product Simplifying the
the ratio
their prices,
their prices, we
we get l 1 l
Anmu : E (Ebl + 2172) (2h)
l.25y = 1.25 = 5 : 3 = 5 X 4 _ 5 _ 5 :33
1-25.v_@_§-§_§xe_§_5 1 1
0.75y‘0.75‘4
0.75y 0.75 4 ' 4
4 ‐4
4 33 ‐ 33 ‑ = (§)(2> [5071+ w]
1
4.@] = 5071+ bzlh = Aold
4. @
vv2
2
Pold R
Paid== F .
Notice how
Notice
1
how %was
was factored u t from
factored oout from In and
b1 and
2
P
P -_ (0.SV)2 - 0.25v2
(o.sv )2 _ o.25v 2 _
- 0o225P
51) b.
b2. The area stays the same.
area stays same.
new -‐ R ‐- R -‐ -· old
be the radius
8. [[] Let r be radius of Kevin’s sphere, and
Kevin's sphere, and
power drops
The electric power drops to a fourth
fourth of what
what let x be
be the factor the radius
radius of Calvin’s
Calvin's
it was.
was. sphere greater by.
sphere is greater
.The
5. [[] The area square is A =
area of aa square 2, where
= s52, where ss is
the length
length of each side.
each side. 4
VKc’vin : 57"
A
A new= (l.10 s) 2 = (1.10)2s2
m , = (1.105)2 (1.10)2s2 =
: 1.21s
1.2152 2
4
= 1.2mm,
= l.21A old Vca lvin = 43n(xr) 3 = x 3 ( 37Tr3 ) = X 3 V Krvin
The new area is 21°/o
new area 21% greater.
greater .
=4x3
X = v'4~ 1.59

282
THE COLLEGE
THE PANDA
COLLEGE PANDA

9. @
9. E TheThe area
area of the original triangle
the original triangle is
is 13. ~M
l~ :a~erback x 50 hardcover =
50hardcover = 125
11 _ 11 22 4 hardcover
ar cover
(s)(s) =
-§(5)(5) _ -s paperback copies
225
paperback copies
2

1 2((552)
1 ( XS )2 = Xx2
§(xs)2
1
1 2) xZAOM
2
14. @
[QJ
Amy
An ew =2 2s = X Aold
= 2.7
1__EZ
)!_
2.4 _ 3.6
xx2=0.64
2
= 0.64
2.7 ( )
2.7 99
=
x = . 8.80
X 0 y = 3.6 2.4 = 1.8 = 5
s5 must been decreased
have been
must have decreased by
11 -‐ .80 = 0.20 = 20%. 15. II] The ratio of the
The ratio the weight
weight of Box A to
to the
the
10. [Q]
E] weight reduces to 7:5.
weight of Box B reduces 7: 5. Since the
Since the
weights
weights of Boxes
Boxes C and D follow
C and follow the same
the same
7 7
Lotherslar : 47rd2b ratio, must weigh
ratio, Box C must weigh m:
7 5
=
‐ fi; of
2
Ofthe
the
Lstar =
Lstar Z 4rr(3d)2
47T(3d)2(2b)
(2b) total weight.: Therefore,
total weight weighs
Therefore, Box C weighs
2 7
= (3)2(2)(4rrd
(3)2(2)(47Td2b)b) % x 180 = 105 pounds.
pounds.
12
=
: lBL
18Lother
otherstar
star

11. II] Let


Let x be the fraction
be the that Star
fraction that Star A's distance
A’s distance
B’s.
Star B's.
is of Star

1
Lsmm =
LstarA = 5 LstarB
LSmrB
9
2
47r(xd)2b =
4n(xd)2b : ~(4rrd
%(47rd2b)b)

2
(4rrd2 b) =
xx2(47td2b) game»)
: i(4rrd 2
b)

11
2 z _
X =-
x 9
1
xx =-1
“ 33

12. I20 or 65 IThe given ratio


The given ratio means
means thatthat there
there are
are
17 sixth
17 graders and
sixth graders 28eighth
and 28 graders for
eighth graders
batch of 17
every batch
every 17+ 28 =
+ 28 = 45
45 sixth
sixth and eighth
and eighth
graders. This
graders. means that
This means total number
the total
that the number of
and eighth
sixth and
sixth graders must
eighth graders must bebea multiple
a multiple
the case
of 45. In the that this
case that total is
this total is 45,
45, the
the
remaining 110 -‐ 45
remaining 45 = 6565 students
students must
must bebe
seventh graders.. In the
seventh graders the case that this
case that this total is
total is
45 x 22 = 90, the
45 remaining 110 -‐ 90
the remaining 90 = 2020
students must
students be seventh
m u s t be graders. Therefore,
seventh graders. Therefore,
possible values
the possible
the values of nn are 20 and
are 20 and 65. Notice
Notice
we don’t
that we
that have to
don't have to consider multiples of 45
consider multiples 45
higher than 90
higher than 90 since multiples would
those multiples
since those would
exceed the
exceed the total number of students.
total number students.

283
CHAPTER 30 ANSWERS TO THE EXERCISES

Chapter6: Expressions
CHAPTER EXERCISE:

1.
1.0 We factor
factor out 6xy from
out 6xy from both
both terms
terms to get 6xy(x +
get 6xy(x y).
+ y).
1l 4 + 3a
.The
IDl
2. ~
2. least common
The least
. .
is 4a. So, -‐ +
denominator 1s
common denominator
a 4
3 4
+ -g = -i +
4a 4a
3a
+ - = --
4 + 3a
4a
3_a

3.. [fil Expanding, (x22 + y)(y


Expanding, (x 2
y)(y + z) = xxzy 2
y + xx22 2
z + yy2 + yz
11 +21:
4.. [g Divide the
Divide top and
the top and bottom get
bottom by 4 to get ;}x.
3x
Another way
. Another get the
way to get the same
same answer split the
answer is to split the
fractions and
fractions reduce.
and reduce.
5..@3x4
@]3x 4 -‐ 33 = 3(x
3(x44 -‐ 1) 3(x22 +
1) = 3(x +1)(x2 1) =
1)(x 2 -‐ 1) 3(x22 + l)(x
: 3(x 1)(x + l)(x ‐- 1)
+1)(x

.The
6.. [IjThe expression
expression follows (a + b)
the (a+
follows the Zab + b2
b)22 = a1122 + 2ab b2 pattern, where a = x + 1
pattern, where 1 and + 1.
and b = y + 1. Therefore,
Therefore,
the expression
the ((x + 1) + (y + 1))2 =
equivalent to ((x
expression is equivalent = (x + y + + 2)2.

7
'Elxy‐x
[Q]xy - x2Z= Jx (yW
- x) ‐= X
- x (L
x -M ~
y) = _x
· x y ‐ y22 _ yy(x
xy-y ( x -y)
‐y) y(x-y)
y(x‐y) yy
x-1 1 x + 5 _ 33(x
x+S ( x ‐- 11)) ++22(x
( x ++55)) _5x+7
rr7 .
Adding the
8.. L.::JAdding the two
.
t w o fractions
. .
the denommator,
fractions in the
2
+ - 3- =
denominator, -x g - +
3
6
6
= -5x 66+- 7 .
6
Now, 11 over
Now, result means
‘ result
over thls
this we can
means we fl'1p1
can flip ‐. ~
5x
't: Sx
it: + 7.
7
2+ 1 2x + 11
2x 2x + 11
2x
2 + -x
-x + -x _ -x- x 2x +
_ 2x +1 x x 2x +
2x +1
' --f
9 · CE]
2- -
=
1 -2_x_1
2x
- -
f= 2x
2x‐1- 1 = -x- x x 2x -
2x‐1 1 = 2x -
2x‐1 1
x
X x
X x
X x
X

10. [g First
First factor out an 8 from
o u t an both terms.
from both Then use
term s. Then 2
formula aa2‐
use the formula - b2 b)(a +
b2 = (a -‐ b)(a + b).
1 1 1 1
53/2 2 802 ‘ E312)
8x2 “ 2 16 :8("‘1M”+ 411>
Bx2 - 1y 2 = 8 ( x 2 - 1 y 2) = 8 ( x - 1 y ) ( x
4
1y )

1
Therefore, c =
Therefore,
4.
11. ~ First,
First, expand: 2
(x + 2)(x
expand: xx2(x 2)(x -‐ 2) + 4 = 2 (x 2 - 4) + = x 4 - 4x 2 + 4. Now we can apply the formula
z xx2(x2 ‐ 4 z x4 ‐ 41x2 N o w we can apply the formula
aa -‐2ab+l72
2ab + b = (a
2 - b)2, where a = xx2and
(a‐b)2,wherea 2
andbb= 2, to get x 4 -‐4x2
=2,togetx4 4x 2 + (x2‐2)2.
+44 = (x 2 - 2) 2 .

12. [fil Combining


Combining like
like terms, we get
terms, we 3x33 + (8x
get 3x (8x22 + 7x 2 ) + (- 4x -
7x2) + (‐4x ‐ 11x) 15x22 -‐ 15x -‐ 7.
3x33 + 15x
11x) ‐- 7 = 3x 7.

13. .Combining
[g like terms,
Combining like 5a-‐ 2a
terms, Sa 2a= 3aand
= 3a 3\/_ -‐ 5/a
and 3y'a = -‐2\/E.
5\/E = 2/a .

137
9(2y)2 +
14· 3 9(2y)2 36y2 +
2(6y) 2 __ 36y2
+2(6y)2 ny 2 _ 1 1_§
+72y2 _ 3 _1+
2 w8(3y)2
8(3y) 2 -‐ 72y
72y22
1
-‐ 2 + -_ 2
15. II] To get
get aa common
c o m m o n denominator 2(x -‐
denominator of 2(x both fractions,
2) for both we first factor
fractions, we negative from the
u t aa negative
factor oout the
second fraction
second and then
fraction and multiply the
then we multiply the top and bottom
top and the first fraction
bottom of the fraction by 2:
x +
X x
X = Xx + x
X : x _
X x
X = 2x
2X _ xX = xX
x ---2 + -22(_(22
-m ___x
_)
‐x) = -
x ---2
x ‐ 2 + ---2-(
x---2-)
‐2(x‐2) = -
x ---2 - -
2 (-
x ‐ 2 2(x‐2)x -- 2) = - -
2 (-
x -_ =
2 ( x ‐ 2 ) 2 ( x ‐ 2 ) 2 ( x ‐ 22~)
2 (
_ x___2_) - -
2 ) -
2 (
. ,......
x--- )

284
THE
THE COLLEGE PANDA
COLLEGE PANDA

Chapter 7: Constructing Models


CHAPTER EXERCISE:

1.
1. 0 the first
In the first y hours,
hours, the carpenter lays
the carpenter (x)(y) =
lays (x)(y) bricks. In the
= xy bricks. 2}; hours
the 2y hours thereafter,
thereafter, he lays
he lays
(g) (Zy) == xy
( ~) (2y) xy bricks. that’s xy + xy =
Altogether, that's
bricks. Altogether, = 2xy bricks.
bricks.

2.. [I] For dollars worth


For d dollars mozzarella to have
worth of mozzarella have been sold, _!!__
been sold, 861‐75 pounds
8.75
must have
pounds must been sold.
have been That leaves
sold. That leaves
d
pounds still
175 -‐ ‐ _~ pounds still available sale.
available for sale.
88.75
5
3.. @ The monthly total
store’s monthly
The store's total cost is 3,000 + 2,
is 3,000 an entire
2, 500x. For an we multiply
year, we
entire year, multiply by
by 12
12 months:
months:
c=
C 12(3,000 +
=12(3,000 + 2,500x
2,500x).
).

4.. @ The setup fees amount


The setup 100C, $100 for each
amount to 100c, each customer.
customer. The monthly cost
The monthly cost for all
all the customers
the customers
amounts to 50c,
amounts each customer
50C, $50 for each Over m months,
customer.. Over months, the monthly charges
the monthly charges add
add up to 50c
50C x m, or
50cm. The total charge
The total 100C +
therefore 100c
charge is therefore 50cm.
+ 50cm.

5.. @ For mn students,


For mn students, the total
total number slices must
number of slices be 2mn.
must be 2mn. Since there
there are 8 slices in each
are 8 pizza, the
each pizza, the
2mn mn .
school must
school order -T - = T pizzas
m u s t order pizzas..
8 4
. d . .
6.. [ElThe compound's
[Q] compound’s temperature
temperature increases !!_degrees
i n c r e a s e s by 5
m
per m
degrees per i n u t e . 50
minute. after x minutes,
So after nunutes, the temperature
the temperature

._
i n c r e a s e s by
increases
d
b y -Ex,
d dx Th final
dx
x, or -.a.
.
e final temperature
The
.· th
15
temperature 1s en tt +
then
dx
dx .
+ -E'
m m m
7.. [Q]
IE The
The bakers cupcakes each
make 3xy cupcakes
bakers make each day. Over 44 days,
day. Over they will
days, they make a
w i l l make total of 44 x 3xy =
a total z 12xy
cupcakes. The
cupcakes. number of boxes
The number boxes needed the total
needed is the cupcakes divided
number of cupcakes
total number divided by the number of
the number
12
cupcakes that
cupcakes that can fit in each
can fit each box: xy =
box: icy = 12y.
X

8.. @ The reduced price of each


reduced price each souvenir after the
souvenir after the first is 0.6a. So
So the
the first souvenir
souvenir costs dollars and
costs a dollars and the
the
remaining ‐ 1 souvenirs each cost 0.611 dollars. Therefore, the total
remaining n - l souvenirs each cost 0.6a dollars. Therefore, the total cost is a + (n ‐ 1)(0.6a).
- l)(0.6a ).

9.. [fJDuring
During the biking portion
the biking portion of the
the commute, graph should
the graph
commute, the goup
should go up and berelatively
and be steep since
relatively steep since Kaiba
Kaiba
covers the
covers initial 4 miles
the initial miles at
at a faster pace than
faster pace he walks.
than he walks . When stops at
Kaiba stops
When Kaiba at the
the rest area for 15
rest area minutes ,
15minutes,
the graph shou
the graph should be flat since
ld be he does
since he does not
n o t cover any distance
cover any during this
distance during this time.
time. After Kaiba leaves
After Kaiba leaves the
the rest
rest
graph should
area, the graph
area, go up
should go up and be at
and be at aa gradual
gradual incline since he
incline since he covers
covers the mile of his
last mile
the last his commute
commute atat aa
walking pace.
walking Only the
pace. Only graph in answer
the graph answer C fulfills all of the above
above criteria.
criteria.

10.. [E]Since Mike's


[Q] Mike's distance home increases
from home
distance from during his
increases during his commute, looking for a graph
we're looking
commute, we're graph that goes
that goes
and to the
up and right. Since his
the right. from home
distance from
his distance increases slowly
home increases slowly at first and
and then more quickly
then more quickly later,
later,
we’re also
we're looking for the graph
also looking to go
graph to go from
from aa low
low slope high slope
steep) to aa high
slope (less steep) (more steep).
slope (more Only the
steep). Only the
graph in answer
graph answer D meets conditions.
these conditions.
meets these
d
11.. [f] Since
Since p tokens
tokens can be used
can be play E_
used to play games, the
5 games, the cost per game is _!!.__p=
per game = dw dollars.
dollars .
£1710
w -E p
w

285
CHAPTER 30 ANSWERS TO THE EXERCISES

rr7 24, 500 -‐ 17,900


24,500 17,900 6,
6,600
12. ~ The plane
plane descends
descends at a rate
rate of T =
= 16200 := 550 feet per
550 feet per minute.
minute . Since the plane
Since the plane
12 12
started descent at an
started its descent altitude of
an altitude of 24,500 feet, its altitude
feet, its after tt minutes
altitude after can be
minutes can be represented
represented by A =
by A =
24, 500 ‐- 550t.
24,500
13. [fil The
The passenger spent 24
passenger spent 24 ‐- a dollars on
a dollars on additional miles after
additional miles the first,
after the first, which the passenger
means the
which means passenger
24
24 -‐ aa
traveled additional miles
traveled - b- additional miles after the first.
after the Adding the
first. Adding the first then gives
mile then
first mile gives us total distance
the total
us the distance
b
traveled:
traveled :
24 2 4-‐ aa +é_
2 4-‐ a + 1 _ 24 b 244-‐ a + bb
2
- bb -+ l =- b -+ b‑
b = b
_ b b

14. [El
[Q]Mark’s annual salary
Mark's annual salary does
does not change during
not change during the year except
the year for June
except for when itit ”jumps”
l s t , when
June 1st, "jumps " by
by $15,000.
$15,000.
Therefore, correct graph
Therefore, the correct should be
graph should be flat throughout
throughout each year except
each year except for jump at
for aa jump at the
the end each year
of each
end of year
(since we
we are starting from June
are starting June 1st, the end
l s t , the end of each refers to
each year refers the next June lst). Only the graph in
to the next June 1st). Only the graph in
answer D fits this description.
answer description .

[Q]Initially,
15. @ Initially, the members are each
members are responsible for paying
each responsible paying 5 dollars . But ifif k
.!_ dollars. k members fail to
members fail to pay,
pay, then
then
m
r
m ‐ k members remain
m - k members and each
remain and each one
one becomes responsible for _!__ k dollars.
becomes responsible amount is
This amount
dollars. This greater than
is greater than
m -‐ k
the original
the amount by
original amount

r r _ mr
mr _ rr(m
( m-‐ kk ) _ m mrr + kkrr _
mr r- ‐ m kr dollars
m
m -‐ k ‐- E = m(m
‐ m ( m-‐ kk)) m ( m-‐ kk)) ‐
m (m m
mm (( m-‐ kk)) = _ m(( m-‐ kk)) dollars
mm

Here is an
an alternative
alternative solution. . ..
member is initially
. rr
Here solution . Since each
each member initiall y responsnble
responsible for -a dollars,
dollars, the club loses
club loses
m
out
o u t on !_ (k) dollars
on L dollars when the k members
wh en the fail to pay. To make
members fail make up this lost each of
amount, each
lost amount, the remaining
of the remaining
m
m ‐- k members
m members must pay an
must pay an additional
additional

r kr
- (k ) kr
m
.!!1__ z _n
= m _i - =: ---- ll
dollars
m
m -‐ k m
m -‐ k m ( m-‐ kk)) d o ars
m (m

286
286
THE COLLEGE
THE PANDA
COLLEGE PANDA

Chapter 8: Manipulating & Solving Equations


EXERCISE
EXERCISE1:

1. rr =
= 19.
19.
1. ± ~ fl7r
X
X
_ xX++ 1l
=
2. r _ CE7T
= _ YY + +Z Z
_ 271
2
X ( Y++Z)Z =) : X
X(Y X ++ ll
2A
2A
3. b =
3. z h
‑ XY X Z- ‐XX=: 1
X Y++XZ
b h
V X ( Y++ZZ-‐ 11)) == 1
X(Y
4. w = lh
x-111
X :
1

5. hI =
5. z -‑
V
~
Y+Z‐1
I m22
?Tr
20.
20.
6. rr =±
6. z j: ~ 17th
x(y + 2) = y
7. bb :
7. = ±i J c2e2 -‐ a1122 xy + 2x = y
8. sz W
s= W 2x = y - xy
9. _ 27rr2 2x = y(l - x)
9. h
h=_ S
S - 2m2
27rr
2rrr
bc
be
10. ‐ ti
10. aa = 7

be First, cross‐multiply.
21. First, cross-multiply.
11. d =
11. = -E
a
y- b 2ac=ab+b
2ae = ab + b
m:=
12. m “ll‐b
-- 2 a c- ‐ ab
2ae a b=z b
x x
y2 =
13. 3/2 : m(x2 -‐ x1) + y1 =
+ Y1 m x 1+
mxz -‐ mx,
= mx2 + yY1
1 a ( 2 -c ‐b)b )=: b
a(2e
_ mx2 - y2 + y,
x 1- :
14.. X1
14 ‐ m e
" y z +3“
m
a‐
a-

b
2c‐ b
v2
02 -‐ 142
u2
15. a =
15. 25

16. y =
16. _ :i: ±/f;bx
25

7
22.
22.
I~I
3t

17.
17. g
g=z
4n 2 L
47r2L
t‐2
t2
Divide both
23. Divide
23. sides by 33 to
both sides get xx +
to get 2y =
+ 2}; = g[i]
72

A2
A2 Multiply both
24. Multiply sides by
both sides by 2 to 2x + 10
get 2x
to get 10== j 4b I
18. p = 22
7t2r2
7T r
- q
1
2t = %,
25. Since 2t a- , we
we can multiply both
can multiply both sides
sides
a

by2toget4t=
by 2 to get 4t =M@

287
CHAPTER 30 ANSWERS TO THE EXERCISES

26. Cross multiply.


Cross multiply. 1
31. n =
3(p - h) = 2(p + h)
3(p‐h>=2(p+h) x
Jx + 1
( Sx2 - 3- x
3)
3 h ==22p
3pp-‐ 33h p ++22h
h
p=
= 5Shh 32. a = 5(c+ l) 3-c

t=~
5-‑
P_
33.
b2 + 2

Cross multiply.
27. Cross multiply .
2 7x24‐3
7x 2+
k(x2 +
+ 44)) + kkyy = 2 3
2 ( 1++2r)
2(1 2 r=
) =11- ‐ tt 2
2 + 4r 2 + 4 + y) = 7x2 + 3
4 r== 11- ‐ tt k(x2+4+y)=7x
k(x +3
2
, = 1-1- 4, 1
k_ 7x2+3
2
k= 7x +3
_ 2(x2+4+y)
2(x + 4 + y)
2

28. Square both sides to get (xY)2 = x 2Y = [{]


34.
29. p:
p=
(x3 - x2)(x5 - .x4) a 3 a++xx++33 =
axx++ 3a =b
a ( x++3)
a(x 3 )++ (x
( x++ 3)
3 )== bb
2 x(X3-- 1)+- 12 ((x+
x + 33)(a
) ( a+ 1l )=
) =b
X x
30. m =
x2 +1 x + 3 = --
b
a ++ l1

x n-31
:
x =I
b
b l ‐3
a+

288
288
THE
THE COLLEGE PANDA
COLLEGE PANDA

CHAPTER EXERCISE:
CHAPTER EXERCISE:

1.@] 7.
7. 0 Cross multiply to
Cross multiply 12=
get 12
to get = kx +
+ 2x. Then,
Then,
(a + b)3 3
= (- 2) = - 8
k=---
k:
12 -‐ 2x
12
x
2x

2. @]
[El The
The answer
answer should
should be
be obvious just by
obvious just 8. ~ Note
8. E Note that
that
lookingat
looking Testing n
at it. Testing n=0 gives us
0 gives us::
(‐6)2 = 36
(- 6)2 = 36
2 2
( 0- ‐ 4)
(0 4)2 = ((00++ 4)
4)2 which happens
which when xx = -‐ 33.. Then
happens when Then
(- 4)2 = (4)2 xx2=
2
= ((‐3)2 9.
- 3)2 = 9.
<‐4)2 : (4)2
16 == 16 9
9.0. ‑
We
We could

(n
expand and
also expand
could also and solve like so:
solve like

4)(nn -‐ 44)) = (n
(n -‐ 4)(
50:

+4)(n
(n + 4)( n + 44))
J=
f‘ ( i
r-
_ p(( 1(1++i ) " ‐ 11)
)
fJi
1==pp((l
( ( 1 + 1i)"
) "-‐ 11))
n2‐8n+16=n2+8n+16
n2 - 8n + 16 = n 2 + 8n + 16
6 n=: 0
-‐ 116n Ji
fi _
n=
=O0 m i l- 1‐ =Pp
(1 + i)"

3.‑
3. []]
10. 0 Multiply both
Multiply sides by
both sides by 22 to get %
to get 4,
m = 4,
n
. n l
which means -1 =
which means = -1.. Then,
Then,
2 := 1l
~ n l n
m
m
l 1
4
1
ac " _
- · -=- l · -=-
b=
= aacc 2m '
271? 2
E' m
m _2
2 Z4 _ 8
8

If bb =
If : ac, then
then b -‐ ac must equal
ac must equal 0.
11. .x2
[fil x 2 + 5x
5x -‐ 24 = 00 can
24 = be factored
can be factored as
as
(x ++ 8)(x
8)( x ‐- 3) = 0. The
The two possible
t w o possible
@If
4.. [Q]u 3
4 3xx -‐ 8 z
= -‐ 223,
3 , then 3x =
then 3x : -‐15.
15.
solutions are
solutions
k== -‐ 88aand
then -‐88 and
are then
|k| =
n d lkl = 88..
Since k < 0,
and 3. Since
Multiplying both
Multiplying sides by
both sides 2, 6x = -‐ 330
by 2,6x and
0 and

5.
5.
6x
6x -
0
‐ 7 z
Cross
‐37.
= - 37.
multiply.
Cross multiply.
12 m
·w
-4 = -38 m
9 Gf = %2
12 = 72m y3 : J_1
1
1 8 32
-= m 2 8
8
6
yy 2==32‑

6.0 y=ti =1
3x +1= - 8
3x =-9
X =-3
(x+2)3
(x : (‐3+2)3
+ 2) 3 = (- 3 + 2) 3 := (‐1)3 z -‐11
(- 1)3 =

289
CHAPTER
CHAPTER 30 ANSWERS THE EXERCISES
ANSWERS TO THE EXERCISES

13.‑
13.1771 17. I30 ITo make this problem
make this easier to
problem easier to work
work

2\/x
2y'x+4 4=6
=6
with, let A =
with, let i.
= 2. Then,
Then,
3
2 \ / x++ 4 = 118
2Jx 8 (g>2_2<g>_15=0
(if - 2(i) - 15 = 0
V
Jxx ++ 44 =z 99 A2‐2A‐15=0
A2 - 2A - 15 = 0
xX + 44 ==8811 ( A -‐ 55)(A
(A ) ( A++ 3)
3 )== 0
0
xX ==777
7 A== 5, -‐33

xX xX .. ..
14.‑
14. (E] So 8 := 5 or E =
So
6 6 Solvmg these
= -‐ 33.. Solvmg these equations
equations
gives x = 30
gives and x = -‐ 118.
30 and the question
8 . Since the question
2
20‐fi=§fi+10
20 - ..rx
= Fx+10
3
specifies that x
specifies that x > 0, the answer
answer is 30.

18. [zJThere
There are
are ttwo ways to approach
w o ways approach this
this
102§fi
10=
5
3../x
3 problem. The faster
problem. The way is to factor
faster way factor the
the
numerator first.
numerator first.
e
6 =zv'xfi
366=
3 = Xx xx2_
2
- 4x+3z4
4x + 3 =
4
x -‐ 1l
15. 11]
Square
Square both sides of the
both sides the equation.
equation. (x‐3)jJ//1‘)':4
(X - =4
3 )__(,x.----lj
M
.x--T
2 x -‐ 3 =
: 4
(x +y)2 = («33
+ y)2 = y2 +16)
+y2
(Jx2 + + 16)2
x=
=7
= x 2 + y2+ 16
x2 + 2xy + y2=x2+y2+16
x2+2xy+y2
2xy The second
The second way
way is to get r i d of the
get rid the fraction
fraction
2xy = 16
first by
by multiplying sides by x
both sides
multiplying both x -‐ 1
1 and
and
xy =
xy = 88 then factor later.
then factor later.

16. [g Cross
Cross multiply and expand
multiply and sides.
both sides.
expand both
xx2‐4x+3
2 - 4x + 3 _
=4
x -‐ 1 4
4(2x‐
4(2x - 1) = ((xx ++ 22)(x
1 )z ) ( x-‐ 22)) x2 - 4x + 3 = 4 (x - 1)
x2‐4x+3:4(x‐1)
8
Bxx- ‐ 44=
: xx22 -‐ 44 x2‐4x+3=4x‐4
x 2 - 4x + 3 = 4x - 4

0=x2‐8x
0 = x 2 - Bx x2-8x+7=0
x 2 - Bx + 7 = 0
0
0== xx((xx -‐ 88)) (( Xx -‐ 77))((Xx- ‐ 1
1)) =
=0
xX ==00,8
,8 xX ==77,1
,1

Neither are
Neither are false solutions, so the
solutions, so the solution set
solution set Now, x =
Now, = 1 is a false solution
solution since
since it causes
causes
is
is {0,8}. division by 0. Therefore,
division solution is x =
the solution
Therefore, the = 7.

290
COLLEGE PANDA
THE COLLEGE PANDA

19. [}] Because this is a


Because this a no
no calculator question,
calculator question, 23. II]
guess and check
guess and check is a valid strategy.
a valid strategy. You can
can
also do
also the following:
do the following: m2g ‘- µm1g
= "128
a= l e g
m1+
m1 "12
+ m2
x2 (x4 ‐- 9)
x20:4 8x44
9) = 8x
a(m1 + m2) =
a(m1+ = ng
m2g ‐- µm1g
ymlg
x6‐9x2‐8x4
6 2
x - 9x - 8x = :00 4
a(m1 + m2) ‐- "128
a("11+ m2g == ‐- yµm1g
m1g
x 2 (x 4 ‐8x2
x2(x4 2
- 8x -‐ 9) =
z 00 ng a(m1 + m2)
m2g ‐- a(m1+
x2(x2‐9)(x2+1)=0
m1g
m lg = fl
x2 (x 2 - 9)(x 2 + 1) = 0
x2(x‐+‐3)(x 3)(x 2 + l)
x 2 (x + 3)(x ‐- 3)(x2 1) = 0
0
24. [JJBecause 2's cancel
the 2’s
Because the out, the
cancel out, the
Because x > 0,
Because 0, x must
must be
be 3 for the
the equation
equation acceleration
acceleration stays
stays the
the same.
same.
above
above to be true.
be true.
_ 2 n g ‐ y(2m1)g = 207123 ‐ umig)
20. II]
20‘ Plug iinn tthe
C Plug ‘venxand
h e 81
given y values.
x and J/values. anew ‑
2m1+2m2 20711 + 7712)

kx 2 + 5
y + 2kx = kx2 :=” oaold
ld

2(k) (3) =
23 + 2(k)(3)
23+ k(3)22 + 5
= k(3)
233+
2 6 k== 99kk+ 5
+ 6k
25. []J
‐- 33kk = ‐- 118
8 3 ( x-‐ 22y)
3(x y ) -‐ 33z2 =
: 00
k =: 6 3
3xx -‐ 66y
y --3 23z == 0
3x =: 6y
3x 6y+32
+ 3z
21. II] Cross
Cross multiply.
multiply . xX =
= 2y
2y+z
+z

E_x+12
X X + 12
-
6 42
26. [}QJ
First, expand
First, expand the left hand
the left hand side.
side. Then
Then
combine like
combine terms and
like terms and factor.
factor .
4 2 x=: 66xx ++7722
42x
3 6 x = 7722
36x ((xx+
+ ll ))((xx-‐ 22)) = 77xx -‐ 118
8
x= : 2 xxz‐x‐2=7x‐18
2
- x - 2 = 7x - 18

6 x2‐8x+16:0
x 2 - Bx+ 16 = 0
Now, x6
N o w , =‐ ‐ ‐ == 3.
2 3
(x‐4)2=0
(x - 4)2 = 0
22.‑
22. II]
Therefore,
Therefore, x = 4 and
and
7x ‐ 18=
7x - 18 = 7(4) ‐
- 18 = 10.
18=
c+1
d = a( c~ l)
d=“< 24)
=a(c~
i§=“(z_4>
a C+
l)
1 cC + 11
5
2 : 247
1 2=: cC +
12 + 11
l11l =
z cC

291
291
CHAPTER ANSWERS TO THE EXERCISES
CHAPTER 30 ANSWERS EXERCISES

IRJ
27. El We can either plug
can either plug in the answer choices
the answer choices is false.
false. We can
can eliminate
eliminate A A.. For choice B,
For choice B,
or solve algebraically. Plugging
solve algebraically. Plugging in the
the answer
answer 2\/2
2,/2 = Js, so
= \/§, so ifif that's
that's the value of Jx
the value \/x -‐ 10,
choices is more
choices more efficient here, but
efficient here, since that’s
but since that's then x would
then have to equal
would have we plug
equal 18. If we plug
self‐explanatory,
self-explanatory, let’s
let's solve algebraically.
solve algebraically. x = 1818 into
into the equation, the
the equation, hand side
the left hand side is
First, square
First, both sides.
square both sides. \/18
J 18 -‐ 10
10== 2\/2
2 ,/2 and right hand
the right
and the is
side is
hand side
m‐ ,/2 = 3 ,/2 - «
/18 - fz=3\f2‐ ,/25=: 2NE.
,/2 . Sinceboth
Since both
(2/x)2 = (x - 3)2
(2\/§)2:(x‐3)2 sides match, choice
sides match, choice B must the answer.
be the
must be answer .
4x=x2‐6x+9
4x = x 2 - 6x + 9 30. IT]
O=x2‐10x+9
0 = x 2 - l0 x + 9
xy2+x‐y2‐1=0
xy 2 + X - y2 - 1 = 0
0== ((xx ‐- 1l ))(x( x -‐ 99))
x = 11,9
X ,9= (y2+ 1) - (y2 + 1) = 0
X(y2+1)‐(y2+1)=0
X

(y2+1)(x‐1):0
(y2+ 1) (X - 1) = 0
Since 1 is a
Since solution, the
false solution,
a false only value
the only value of x
that satisfies
that satisfies the equation is 9.
the equation
Since y2+
Since y2 + 11is always positive,
is always positive , x must equal
must equal
28.
28.[Il 1.
31. [I] Divide
Divide both and take
sides by P and
both sides take the
the tth
tth
4
4 _
= 9 root of both
root both sides.
sides.
6xx++99 _
xx 22- ‐ 6
V =
: P r)’1
P((11- ‐ r)
44 =
=99
(x - 3) 2
(Y‐3)2 ~p = (1 -‐r)1
r) 1
g=(x‐3)2
; = (x - 3) 2

~=
±'9= / (x-
1- r
fig: (x‐3)2
3) 2
,IV
r = l - Vp
i§=x‐3
2
±- = x - 3
3
32. [I] the previous
From the
From question, we
previous question, we know
know
29. I]] We can use the
can use answer choices
the answer choices to
_ ,IV
i/V Because Vis- half P, p
zP =‐_12..
V 1
backsolve
backsolve or we can
can solve the equation
solve the equation h a rt r=‐ 11‐-
tthat V $.BecauseVishalfP,
p· 2
algebraically. solve it algebraically
algebraically. We'll first solve algebraically
by squaring
squaring both
both sides.
sides. Thus,r:1‐{‘/gz0.l3
= 1 - if"{:=::::
Thus , r 0.13

(✓x - 10 ) 2 = (vx - 12)2


(x/x‐10>2=(f‐\/§)2
1 0=
xx -‐ 10 (Jx)2
: fi f -‐ 2(
aw /x)(n /h5 ) + ( /2
x / )2
if
x‐ 10= :x -x‐zx/fln
x - 10 2../h + 2
-‐12=‐2\/fl
12 = - 2./ix
6=\/2_x
6 = ...fix,
3 6=: 22xx
36
1 8== Xx
18

Let’s say we
Let's we wanted
wanted to test
test the answer
the answer
choices instead.
choices instead . H
Howo w would
would we do that? that? For
For
choice A, if
choice if \/5
../6is the value of J
the value \/xx -‐ 10, then x
10, then
would have
would have to equal
equal 16. If we plug x =
we plug z 1616
the equation,
into the
into equation, we we get = 4 -‐ \/2,
get 44 = ,/2, which
which

292
292
THE COLLEGE PANDA
THE COLLEGE PANDA

Chapter 9: More Equation


Equation Solving Strategies
Solving Strategies
CHAPTER EXERCISE:
EXERCISE:

1. 30 ((x3
1. I55 I30 x3 +
+ ~éxz + ~§x>
x2 + x) = 30x33 +
= 30x 5x22 +
+ 5x + 20x.
20x. haven’t
haven't assumed that a and
assumed that are positive
and b are positive
here.
here . But of the
the answer choices, only
answer choices, only 6 is aa
Therefore, 30,bb = 5, and
Therefore,aa = 30, z 20.
and c = possible value of ab
possible value (when a =
ab (when = 3 and
and bb =
= 22 or
+ bb ++cc =
a+ : 555.
5. when
when a = and b =
z -‐33 and = -‐ 22).
).
2.. [Qj
E] For
For an
an equation to have
equation to have infinitely
infinitely many
many 5. 0 Expand both sides of the equation.
solutions,
solutions, both must be
sides must
both sides be equivalent.
equivalent.
ga = ~g
3 1
Comparing terms on
the terms
Comparing the on both sides, ~a
both sides, xX--‐ 5X
x -‐ 44 =
‐ 44 --
‐' 5xx
2 2
and 3 =
and Solving these
= 9b. Solving equations, we
these equations, we get
get 1 1
1 a 4 ‐ Ex‐ 4‐ ‐ 2x
-- x - 4 = 4 --
2 2
x
a= and b = 5.. Therefore,
= 4 and Therefore, b
5 = I= 12.
3 3
Let’s rearrange the
Let's rearrange right side
the right side so that the
so that the terms
terms
3.. [Qj
[E For an equation
For an equation to have solutions, the
no solutions,
have no the line with the
line up with the left side.
left side.
coefficients of the
coefficients the x terms
terms m be the
u s t be
must same
the same
on either side but
either side but the
the constants be
m u s t be
constants must 1 1
‐ ‐xx -‐ 4 =--
-- : ‐ ‐xx+4 4
different. we expand
different. If we the right
expand the side, we
right side, we get
get 2 2 +

a = 66xx + 3
axx-‐ b = Now
Now it’s easy to see
it's easy see that the coefficients
that the the
coefficients of the
x terms are the
terms are same but
the same the constants
but the constants are
are
Therefore, aa =
Therefore, and b f=
= 6 and 742 - Only answer
‐ 33.. Only answer different
different ( -‐ 4 vs. 4). Therefore, the equation
Therefore, the equation
choice these conditions.
satisfies these
choice D satisfies conditions. Choice
Choice C has no
has solutions.
no solutions.
would result in infinitely
would result solutions
many solutions
infinitely many
since both sides
since both would be
sides would beequivalent.
equivalent.
6. [QjFor an equation to have no solutions, the
coefficients of the
coefficients terms must
the x terms be the
m u s t be same
the same
0
4.. Remember,P2
Remember, p2 -‐ £72 q2 == (p + q)(P
(F7 + q)(p -‐ q).
Q)- We
We
on either
either side but the
side but
different. First,
different.
the constants
let’s expand
First, let's expand the
must be
constants must
side.
left side.
the left
can apply
can this factorization
apply this factorization here once we
here once take
we take
out aa 2:
3 x+
3x +3 a x=
3aa-‐ 22ax = 1122-‐ 77xx
18x22 -‐ 8 = 2(9x2
18x 2(9x 2 -‐ 4) = = 2(3x ++ 2)(3x
2)(3x -‐ 2),
2),
which 2(ax + b)(ax
equals 2(ax
which equals b)(ax -‐ b). Comparing
b). Comparing 3 a-‐ 22ax
3a a x = 112
2 -‐ 1lOx
0x
the coefficients,
the coefficients, a = 3 andand b = : 2. Therefore,
Therefore,
Comparing
Comparing the the coefficients
coefficients of the
the x terms,
terms,
ab =: (3)
(3)(2) : 6. Note that this factorization
(2) = Note that this factorization
method assumes
method assumes that the constants
that the constants are are
a=
-‐ 22a = -‐ 110,
0 , a = 5. Note that a is
Note that is N O T equal
NOT equal
to 4 since the goal
since the goal is n o t for the
not the constants
constants to
positive, but that's
positive, but okay since
that’s okay all the
since all the answer
answer
be the same..
the same
choices are positive.
choices are It’s possible
positive. It's that aa =
possible that z -‐33
and b =
and : 2 or a = = 3 and
and b = -‐ 22,, for instance,
instance, 7. [II Expand the right side.
but
but these
these are cases that
are cases you generally
that you generally don't
don't
need to worry
need worry about this factorization.
about for this factorization . (2x + 3)(ax =12x2
3)(ax -‐ 5) = 12x2 + bx -‐ 15
15
If you
you didn’t
didn't use
use this factorization and
this factorization and 2ax 2 + 3ax -‐ lOx
2ax2 12x2 + bx
10x -‐ 15 = 12x2 bx -‐ 15
instead expanded the
instead expanded right side,
the right side, you
you
would’ve gotten
would've gotten 2ax 2 + (311
2ax2 (3a -‐ 1lO)x
0 ) x -‐ 15
1 5= 12x2 + bx
= 1212 b x‐- 15
15

18x2 ‐ 8 = 2a2x2 ‐ 2172 Comparing


Comparing both sides, 2a
both sides, 2a = 12 12 and
and
b== 3a
3a ‐- 10, which
which yields
yields a =z 6 andand
Comparing the
Comparing the coefficients
coefficients on either
either side,
side, b = 33aa -‐ 1100 =: 33(6)
( 6 )-‐ 1100=: 88..
18 2a2 and
18 = 2a2 2b2 . Solving
and 8 = 2b2. these equations
Solving these equations
gives
gives a
a := ±
$33 and
and b =
= ± 2. As
:l:2. you can
As you we
see, we
can see,

293
CHAPTER
CHAPTER 30 ANSWERS THE EXERCISES
ANSWERS TO THE EXERCISES

8. j 49 IExpand
Expand the
the left side:
side: @JMultiply
13. E] sides by
both sides
Multiply both by xyp.
xyp.

x2+6xy+9y2=x2+9y2+42
x2 + 6xy + 9y 2 = x2 + 9y 2 + 42 _1 + 1_ : _1
-+-=-
xx yy p
p
/xl+6xy+9f=xz+9
+ 6xy + JI = / + JI +42
+ 42
6xy =
6xy = 42 w+w=w
YP + xp = xy

xy
xy = 77 yypp = xxyy ~
- xxpp
yp = x(y
y p = X ( l / -~ pp))
x2y2=
x2y2 = 49 yp =
__1E_ : X
yy -‐ p
p
9. [fil For an
9. equation to
an equation have infinitely
to have infinitely many
many
solutions, both
solutions, both sides must beequivalent.
sides must be equivalent. 14. @]Expand
LE] side of the
the left side
Expand the the equation.
equation.
First, let’s
First, expand the
let's expand right side
the right side of the
the
equation:
equation:
(x3+kx2‐3)(x‐2)
(x3 + kx2 - 3) (x - 2)
6xx =: xx-‐ 66nx
6 nx++33xx =x4+kx3‐3x‐2x3‐2kx2+6
= x 4 + kx3 - 3x - 2x3 - 2kx2 + 6
6x = 4x -‐ 6nx
6x 6nx
=x4+(k‐2)x3‐2kx2‐3x+6
= x 4 + (k - 2)x 3 - 2kx 2 - 3x + 6
2x == -‐6nx
2x 6nx

when we compare
N o w when
Now compare the the coefficients
coefficients on Comparing
Comparing this x 4 + 7x3
this to x4 18x22 ‐- 3x
7x3 -‐ 18x 3x + + 6,
sides, we get 2 =
we can see that k - 2 = 77and
wecanseethatk‐ and ‐- 22kk =
= -‐ 118.
8 . lInn
both sides,
both = -‐ 66n, which gives
n , which gives
both cases, k =
both cases, : 9.
2 1
n:_‐6=
n =- 6 =-3·3. 15. ~ Multiply sides by
both sides
Multiply both + 3)(x
by (x + 3)(x ‐- 2). We
We
get
10. [j Multiply
Multiply both sides by
both sides by b.
b.
5 ( x-- 22)) ‐- 22(x
5(x ( x + 33)) = aaxx -‐ b
a
abb++aa = aa++5Sb
b
5
5xx-‐ 1 100- ‐ 2x2 x- ‐ 66 =
z aaxx -‐ b
a b =
ab Sb 5 b
3x 16 = ax
3x -‐ 16 ax -‐ b
a=5 S
Comparing
Comparing thethe coefficients
coefficients on
on either
either side,
side,
a := 3andb
3 and b =
= 16. Therefore,
Therefore,
11. [IjMultiply
Multiply both
both sides by x(x
sides by x(x -‐ 4).
4). a + b ==33++116
a+b 6 ==119.
9.
((xx-‐ 4
4)) -‐ xx ==x(
x ( xx -‐ 44))
lNotice
1
16. j ; INotice that
that x22‐- 11==( x(x++1 l))(x
( x -‐ 1
1))o
onn
-‐44 z=x2
x ‐- 4x
2
4x
the right
right hand
hand side.
side. It’s easy to see
then easy
It's then see that
that
0
0=z x2
x 2 -‐ 4x
4x +
+ 44 we should multiply
we should multiply both sides by
both sides by
o
0== (x 2)22
(x -‐ 2) ((xx + l ))(x
( x -‐ l1).
).

We can that x = 2.
can see that
4 ( x++ l1)+2(x-
4(x ) + 2 ( x ‐ 1)
1 )=: 335
5
4 2xx-‐ 22 ==335
4xx++ 44 ++ 2 5
12. 0 Expanding
Expanding the right side,
right side,
6xx++ 22 z=335
6 5
4x2+mx+9 z4x2+4nx+n2
4x2 + mx + 9 = 4x2 + 4nx + n2 6
6xx = 3333

xV2
11
Comparing both
Comparing both sides, we see
sides, we see that
that =
_ 22
9:
9 = 1n122 and m =
and m 4n
= 4n

and m =
Therefore, n = -‐33 and
Therefore, = 4(‐3)
4( -3) = -‐ 112
2

m+
m +n = 2 ++((‐- 33)) = ‐- 115
: ‐ -1 12 5

294
THE COLLEGE PANDA
THE COLLEGE PANDA

Expand the
17. [[) Expand the left
left hand side.
hand side.

(2x -‐ b)(
b)(7x + b) =
7x + 14x 2 -‐ ex
=14x2 cx -‐ 16
16
14x22 +
14x b2 =
Zinc -‐ 7bx - b2
+ 2bx = 14x 2
143:2 -‐ ex
cx -‐ 16

14x22 -‐ Sbx
14x 5bx -‐ b2 14x22 -‐ ex
b2 = 14x cx -‐ 16

Comparing
Comparing bothboth sides, we can
sides, we can see that b =
see that =4
(b cannot
cannot be because b > O).
be -‐44 because 0).
ec = 5b = 5(4) =
2 Sb = 20.
18. lMultiply
@] sides by(n
both sides
Multiply both + 1).
by (n -‐ 1l))(( n + 1).

3(n +
3(n 1) +
+ 1) +2n(n
2n (n -‐ 1) (11+
1) = 33(n +1)(n
l )( n -‐ 1)
1)
3n+3+2n2‐2n
3n + 3 + 2n 2 - 2n =
=3(n2‐1)
3(112 - 1)

2n2+n+3=3n2‐3
2n 2
+ n + 3 = 3n 2 - 3
00 =
= nn2
2
- ‐11
n- ‐66
00 := ((n
n ‐- 33)( +22))
)(n+

e c a u s ne n>>0,
n = 33 oorr -‐ 22.. BBecause 0 ,nn== 33..

295
295
CHAPTER
CHAPTER 30 ANSWERS
ANSWERS TO THE EXERCISES
TO THE EXERCISES

Chapter
Chapter 10: Systemsof
Systems of Equations
Equations
CHAPTER
CHAPTER EXERCISE:

1. [fil Substituting
1. Substituting the second equation ; 6. [I] IfIf the intersect at
the second equation into the
into the 6. w o lines
the ttwo lines intersect the point
at the (2, 8),
point (2, 8),
first,
first, then (2,8)
then (2,8 ) is a solution to
a solution to the system.
the system.
Plugging the point
Plugging the into the
point into equation of
the equation of the
the
3(1~3y)‐5y=
3 (1 - 3y) - 5y =- ‐ 111 1 second line,
second can solve
line, we can for b,
solve for b,
3 -‐ 99yy -‐ 5Syy =
z ‐- 111
1
3
3 -‐ 114y
4 y== ‐- 1111 yy =z ‘- bbx"
‐- 114y
4 y == ‐- 1144 8 == ‐b(2)
8 - b(2)
‐- 4 = =b
yy =
=1l
Finally,, x =
Finally z 11 -‐ 3(1) =
= -‐ 22.. Plugging the point
Plugging the point into equation of
the equation
into the of the
the
first line,
line,
[QJFrom
2. E] the first equation,
From the equation, 31
y== 20
20 -‐ 2x.
2x.
Plugging
Plugging this into the
this into second equation,
the second equation, yy = ax +
: ax +b b
8 = a(2)
a (2) -‐ 4
6
6xx -‐ 55(20
( 2 0-‐ 22x)
x ) = 112
2 1 2=: 22a“
12
6 0 0+
6xx -‐ 1100 +1 0 x=: 112
lOx 2 6
6 =: aa
16x =
16x : 112
x =
z 7
7. 0 The two graphs do not intersect at all, so
Wealready
We know the
already know answer is
the answer is (D) at this
(D) at this there are no
there are no solutions.
solutions.
point, but just
point , but just in case,
case, y =
z 20 ‐ 2(7) =
20 - 2(7) = 6.6. 8. [1]From the first
From the first equation,
equation , we can isolate
we can isolate yy to
to
get y = -‐ 5Sxx ‐- 2. Substituting
get Substituting this into the
this into the
3. [1]Add
Add the w o equations
the ttwo equations to
to get
get second equation,
second equation ,
7x
7x -‐ 7y
7y = 35. Dividing both sides
Dividing both by 7,
sides by 7,
x ‐- y == 5.We can multiply
5. We can multiply both sides by
both sides by ~1
- 1 2(2x -‐ 1)
2(2x = 33 -‐ 3(‐5x
1) = 3(- Sx -‐ 2)
2)
to get y -
togety‐x=‐5.x = - 5. 4
4xx-‐ 22 :=
3 3+ +
15 x ++66
15x
4. ~ The fastest way
The fastest way to do this problem
do this problem is to
is to 4x ‐- 2 := 15x + +9
subtract
subtract the second equation
the second equation from the first,
from the first, -‐ 1ll1 = 11x
= ll x
which yields x +
which yields +y =z 9.
9. _1
- 1=X
z x

5. [I] In the
the first equation, we can
equation , we 3x to
m o v e 3x
can move to Finally,
Finally,yy z _5( _1) _ 2 = 3.
= - 5( - 1) - 2 = 3.
the right
the side to
hand side
right hand get y =
to get : ‐- 5Sxx +
+ 8.
8.
Substituting
Substituting this the second
into the
this into equation,
second equation, 9. [1]Divide
9_ first equation
the first
Divide the equation by by 22 to
to get
get
x ‐- 2y := 4. We can’t
can't get
get the
the coefficients
coefficients to
to
”- 33xx ++2 2((-‐ 55xx ++ 8)
8) =
= -‘ 110
0 match vs . 2
match ((-‐ 2 vs. the y’s).
for the
2 for y's). Therefore,
Therefore, the
the
-‐ 33xx ‐- 10x
lOx + + 16 z
16 = ‐- 1100 system has
system has one solution.. In
one solution fact, we
In fact, can even
we can even
-~13x :
13x = -‐ 226 6 solve
solve this system by
this system by adding
adding the w o equations
the ttwo equations
x Z 2 to get
get 2x = 8,8, x = 4, which makes
4, which makes y y== O.0.
x= 2

Then,y - 5 (2) + 8 z
Then, y = ‐5(2) = ‐- 22.. Finally,
Finally,
10. [I] To get
get the
the same coefficients, multiply
same coefficients, the
multiply the
first equation
equation by ‐2- 2 to get
get ‐- 44xx + 10y =
+ lOy = -‐2a.
2a.
xy = (2)( - 2)
xy = <2>(‐2) ‐ 4 .= - 4.
N
Nowo w we
we can see that
can see a = ‐- 88,, a
that ‐- 22a a== 4.
4.

296
THE COLLEGE
THE PANDA
COLLEGE PANDA

11. 0 Multiply the


Multiply first equation
the first by -3
equation by ‐3 toto get
get ~ Plugging
17. E] Plugging the first equation
the first equation into the
into the
3ax -‐ 6y =
-‐3ax = -‐15. constant a cannot
The constant
15. The cannot bebe second,
second,
-‐ 1.
1. Otherwise,
Otherwise, the second equation’s
the second equation 's
coefficients would then
coefficients would be equal
then be the first
equal to the v'4x
m ‐ -( fi(v1x
++
3 3)) == 33
equation’s
equation's coefficients, resulting in aa system
coefficients, resulting system z fi -‐ fiVX
2/x ‐ -33== 33
with no
with no solution
solution..
multiply the
First, multiply
12. @ First, equation by
first equation
the fust by 33 to
to fi ==6
/x 6
get rid the fraction:
get rid of the fraction: 12x - y =
‐ = ‐24. Next,
- 24. Next, x z=336
X 6
substitute the
substitute the second equation into
second equation first,
the fust,
into the
Therefore, y = v'36
Therefore, \/3_6 +
+3=
: 9.
12x -‐ (4x 16) =
(4x + 16) = -‐ 224
4
1 2 x- ‐4x
12x 1 6=: -‐ 224
4 x- ‐ 16 4 18. [II Lets, m, and
Lets, m, and I be
be the weights of sma
the weights small,
ll,
medium, and
medium, and large jars, respectively.
large jars, respectively. Based
Based
8 x = -‐ 8
Bx on the information,
on the information, wewe can the
create the
can create
x
X == -‐ 1 following ttwo
following equations::
w o equations

Finally,y
Finally, y = 4(- 1) +
4(‐1) 6=
+ 116 = 12. = 2m + I
16s=2m+1
16s
13. [IQ]We
Wecan isolate x in the
can isolate the second equation to
second equation to 4
4s+ m = 1I
s +m
get x =
get = y -‐ 18. Substituting this into
Substituting this into the first
the first
To get the weight
get the weight of the large jar
the large jar in terms
terms of
equation,
equation,
the weight the small
the weight of the small jar, we need to
we need to get rid
get rid
= O.S
y3]= (y - 18) +
0.5(y‐18) + 114
4 of m,
m, the
the weight
weight of the medium jar
the medium Wecould
jar.. We could
certainly use elimination,
certainly use but here,
elimination, but we’ll use
here, we'll use
. 5 y-‐ 99 ++114
yy ==00.Sy 4 substitution. Isolating m
substitution. Isolating m in the second
the second
0.Sy =5
O.5y=5 equation,
equation, m m = I -‐ 4s. Substituting
Substituting this
this into
into
y ==110
0 the first
the equation, we
first equation, we get
get

16s = 2(/ - 4s) + /


16s=2(1~4s)+l
14. Cf]To match the coefficients,
match the multiply the
coefficients, multiply the = 21- 8s + I
16S=21‐SS+1
16s
equation by
first equation by 18
18to 6x -‐ 3y =
get 6x
to get : 72. We
We
24s = 31
24s=3l
can then see
can then that aa =
see that = 33 if
if the system is to have
the system have
no solution
no solution.. 8s5=: 1I
8
15. [Q] Divide the
E’ Divide the fust equation by
first equation by 3 to
to get
get Eight jars are
small jars
Eight smaJl needed to match
are needed match the
the
2y = 5. Divide
x -‐ 2y Divide the equation by
second equation
the second by -‐22 weight
weight of one large jar.
one large
to get 2y =
get x -‐ 2y : 5. They're
They're the
the same, so there
same, so there
an infinite
are an
are infinite number solutions..
number of solutions 19. [Q]
E] Since
Since there 30 questions,
were 30
there were questions, James
James must
must
have had
have answers, x + y =
had 30 answers, The points
z 30. The points
16. Cf]For
For aa system to have
system to infinitely many
have infinitely many
he earned
he earned from
from correct answers total
correct answers 5x. The
total 5x. The
solutions, equations must
the equations
solutions, the essentially be
must essentially be
the same.
the Looking at the
same. Looking the constants, can
constants, we can
points lost from
he lost
points he from incorrect answers total
incorrect answers total 2y.
Therefore, 5x
Therefore, 231 = 59.
5x -‐ 2y
make them
make them match by multiplying
match by multiplying the second
the second
equation by 2. The
equation equations then
The equations look like
then look like Leta and
20. @] Leta and b be be the
the number points you
number points you get
get
this::
this hitting regions
for hitting regions AA and
and B,respectively.
B, respectively.
From
From the information, we
the information, we can form the
can form the
mx 6y =: 10
mx -‐ 6y 10
following
following twot w o equations:
equations :
4x -‐ 2ny = 10
4x 10
a++ 22bb = 118
8
N o w it's
Now easy to
it’s easy to see that m
see that m= and 2n
z 4 and Zn =
: 6,
6,
2a + b 211
2 a + b = 2
. m_ 4
n =‐ 3.
n 3. Finally,
Finally, ;: ‐ ;.5.
=
To solve b,multiply
solve for b, multiply the first equation
the first by 22
equation by
and subtract to get
and subtract 3b =
get 3b 15, b = 5.
= 15, 5.

297
CHAPTER 30 ANSWERS TO THE EXERCISES

21. [Q]Let r and c be the number of rectangular 24. II] To find the point(s) where two graphs
tables and
tables circular tables,
and circular respectively, at
tables, respectively, at the
the intersect, solve
intersect, the system
solve the consisting of their
system consisting their
restaurant. Based
restaurant. Based on the information,
on the information, wewe can
can this problem,
equations . In this
equations. problem, that system is
that system
make the
make following two
the following equations:
t w o equations:
y=x2‐7x+7
y = x 2 - 7x + 7
4r+
4r + 88cc ==1144
44 y=: 22xx -‐ 1
rr +cC ==3300
+
Substituting the
Substituting the first equation into the
equation into the
To solve multiply the
solve for r, multiply the second equation
second equation second,
second, we get
get
by 8 and subtract to get
and subtract : -‐ 996,
get -‐ 44rr = 6 , r = 24.
xx2‐7x+7=2x‐1
2 - 7x + 7 = 2x - 1
22. ~ The solution to the
The solution is the
system is
the system the
xx2~9x+8:0
2
- 9x + 8 = 0
intersection point the t w o lines. Each
intersection point of the two lines. Each
( x ‐ 1l )(x-
(x- ) ( x ‐ 8)=
8 ) = 00
horizontal step
horizontal along the
step along the grid represents ~2 of
grid represents xl e= o1 or
r 8
aa unit,
unit, and vertical step
each vertical
and each along the
step along grid
the grid
represents 1 unit.
represents unit. So, the intersection
So, the point is
intersection point So the x-coordinates
So the x-coordinates of the points of
the points
intersection are 1 and
intersection are Since the
and 8. Since question
the question
“(74> 3
at( - ~, - 3). already us the
gave us
already gave (1, 1), p must
point (1,1),
the point be
must be
equal
equal to 8.
23. [Q]From the second equation, x = 2y.
Plugging this
Plugging this into
into the equation, we
the first equation, we get
get I add 11 to both
First, add
25. 9 or 16 j First, both sides the
sides of the
second equation
second equation to get y =: x + 11. Then
to get Then
2 _ 2= : _1_
substitute this in for y in the
substitu te this equation:
the first equation:
(2y)2
(231) _ y2
y 12
12
x 2 - 2x = x + 11 - 1
x2‐2x:x+11‐1
4y22 _ 2 == -1_
4y - y2
y 12
12 xx2‐3x-1ozo
2 - 3x - 10 = 0

‐ _1
3y22 =
1_ ((xx ++ 2
2)(x 5 )=
) ( x- ‐ 5) =0
3 y ' i-12z x=
=-‐ 22or5
0r5
1
y2=l
y
2
= 36 W
When +1111 = 99.. W
h e nxx== -‐ 22,,yy == -‐ 22 + h e nxx == 55,,
When
1 y := 5 + 11 = 16. The The sosolutions
lutions to the system
the system
yy =
‐ i±-g then (‐2,9)
are then
are and (5,16).
(- 2, 9) and Therefore, the
(5, 16). Therefore, the
6
possible values
possible values of y are are 99 and
and 16.
1
Therefore, the
Therefore, values of y1 and
the values and y2
y; are and
are -‐2 and
6
1
6.
6

298
THE COLLEGE PANDA
THE COLLEGE PANDA

Chapter
Chapter11: Inequalities
11: Inequalities
CHAPTE R EXERCISE :

1.‑
-‐ xx ‐- 44>>44x
x ‐- 114
4
‐- 55xx > ‐-110
0
x< <2

Of the only -‐11 is


choices, only
answer choices,
the answer is aa solution.
solution.

2. [QJ
2. El Multiply sides by
both sides
Multiply both by 44 to
to get r i d of
get rid of the fractions..
the fractions

3 1
-Zx‐4>§x‐10
x - 4 > - x - 10
4 2
3
3xx-‐ 1166 > 22xx -‐ 4400
xX >> ‐-224
4

3.
3. [g The shaded region
The shaded below the
region falls below horizontal line
the horizontal y=
line 31: 3,
3, soy The shaded
3. The
soy < 3. shaded region also stays
region also above
stays above
= x,
y= soy
X, SO y > x.
X.

4. [fil Let's
Let’s say Jerry's estimate,
say Jerry's estimate, m, marbles.
m, is 100 marbles. If the
If actual number
the actual marbles
of marb
number of within 10
is within
les is of that
10 of that
estimate, the actual
then the
estimate, then number must
actual number be at
m u s t be least 90
at least 90 and
and at most 110. Using
at most Using variables, m ‐- 10
variables, m 5 n
10 ::; S
11 ::;
m+ + 10.
lO.
5.
5. [I] Setting
Setting up the inequality,
up the inequality,

M 2N
M ?. N
12P +
12P +100
100 2 + 970
?. ‐- 33PP -+- 970
15P 2
15P ?. 870
P ?.
2 58
58

6. [zJ
3 ( n-‐ 22)) > -‐ 44(11
3(11 ( n -‐ 99))
3n n ++336
3 n-‐ 66 >>‐ -44n 6
7
7nn > 442 2
n>6

Since n11is
Since is an integer, the
an integer, least possible
the least of 11
value of
possib le value n is 7.
is 7.

7. [fil The
The shaded region is
shaded region is below
below the
the horizontal line y3;=
horizontal line but above
= 33 but above the horizontal line
the horizontal = -3.
line y = ‐ 3 . Therefore,
Therefore,
y2
?. ‐3
- 3and
and y 5
::;3.
3.

is ~
16 hours. Since
8.
8. [I] The time Harry
The time spends on
Harry spends bus is
the bus
on the 2 hours
hours and
and the
the time he spends
time he spends on the train
on the is %
train is hours. Since
X y
8 16
16
the total number
the total number of hours
hours is never greater than
never greater than 1, z + y- S::;1.
- + 1.
X y

299
299
CHAPTER 30 ANSWERS TO THE EXERCISES

9. [I] If the distributor


U the distributor contracts u t to Company
contracts oout Company A for xx hours,
A for then it
hours, then it contracts
contracts oout to Company
u t to Company B for
B for
10
10 ‐- x hours. Company A then
hours. Company produces
then produces 80x cartons and Company B produces 140(10 ‐ x)
cartons and Company B produces 140(10 - x ) cartons.cartons.
Setting up
Setting the inequality,
up the inequality,
80x +
+140(10
140(10 ‐- x) > 1,100
1,100

10. [El
[QJPlug
Plug in x =
= l,1, y z= 20
20 into first inequality
the first
into the inequality toto get
get 20
20 >
> 15 15++ a,
a, 55 >> a.
a. Do
Do the same for
the same for the
the second
second
inequality
inequality to getget 2020 < 55 + b, 15 < b.
b, 15 So, a
b. 50, is less
a is than 5
less than and b
5 and bisis greater
greater than
than 15. The difference
15. The between
difference between
the
the ttwo
wo m u s t be
must be more than 15 ‐- 5 =
m o r e than 10. Among
= 10. Among the answer choices,
the answer choices, 12 is the
12 is only one
the only one that
that is greater than
is greater than
10.

[QJThe line
11. E, line going
going from the bottom-left
from the bottom-left to the top-right
to the must be
top -right must beyy =
= ~x
2x +
+2 line going
the line
and the
2 and from the
going from the
top-left the bottom-right
top-left to the bottom-right must beyy =
must be = -‐ 22xx +
+ 55 (based
(based on the slopes
on the and y‐intercepts).
slopes and y-intercepts). Answer
Answer (D)
(D)
correctly shades in the
correctly shades the region
region above : ~2xx +
above y = + 22 and below yy = ‐- 22xx +
and below + 5.
5.

12. [I] One


One manicure takes 1/3
manicure takes 1 /3 of an One pedicure
hour. One
an hour. pedicure takes
takes 11//22 an
an hour. The total
hour . The total number of hours
number of she
hours she
spends doing
spends and pedicures
manicures and
doing manicures be less
must be
pedicures must less than or equal
than or equal to 30, so
to 30, so ~m
3m +
1
+ 5pp ~
5 30.
30. She
She earns
earns1
25m for the and 40p for the
manicures and
the manicures the pedicures. Altogether, 25m
pedicures. Altogether, 25m + 40p 2
+ 40}: 2: 900.
900.
[Q]From
13. |E given inequality,
From the given ~ 3k
inequality, x 5 3k + Subtracting 12
-+- 12. Subtracting 12 from
from both gives x
sides gives
both sides x ‐- 12
12 5
~ 3k, which
3k, which
confirms that I is always
confirms that always true.
true.
the given
From the
From inequality, 3k + 12
given inequality, 2 k, which
12 2: means 2k 2
which means 2, k
2: -‐ 112, k22: ‐- 66,, so
so 11
II m also be
u s t also
must be true.
true.
given inequality,
the given
From the
From ~ x. Subtracting
inequality, k 5 Subtracting k from
from both gives 0
sides gives
both sides OS~ x
x -‐ k.
k. Therefore, I“ must
Therefore, III must also
also
be true..
be true

14.13~ < x < ?~ ILet’s


Let's solve these separately.
solve these First,
separately. First,
2
20
‐30 < -‐ 22xx ++ 4
--
3
0 < -‐ 66xx + 1
-‐ 220 122
‐- 3322 << -‐ 66xx
16
-L" >
>xx
3

N o w for the
Now second part,
the second part,
9
-‐2x
2x + 44 << -‐ ‐
2
-‐ 44xx + 88 < -‐ 9
-‐ 44xx < -‐ 117
7

>_ 17
x >-
x4 4

17
17 16
16 9
9 1
10
Putting
Putting the w o results
the ttwo together, I < x < 3. . Therefore,
results together, 2 < 30..
Therefore, Z < xx -‐ 2
4 3 4 3
[QJIf the
15. El area is at
the area at least then xy
least 300, then 2 300. The
xy 2: perimeter of
The perimeter the rectangular
of the garden is
rectangular garden is 2x + 2y,
2x + 2y, so
so
+ 2y
2x + 2 70, which
2y 2: which reduces
reduces to x ++ y ~2 35.
35.
16. ~ I is nnoto t always because of negative
true because
always true values. Take a
negative values. a z
= ‐5 and b
- 5 and = 2
b = for example.
2 for example. a a << b,
b, but
but
a b2 . II is
a > b2. is definitely It's the
true. It’s
definitely true. equivalent of
the equivalent of multiplying
multiplying both sides by
both sides by 2. I I is
2. IIII is also
also true. It’s the
true. It's the
equivalent of multiplying
equivalent multiplying both which necessitates
sides by -‐ 11,, which
both sides a sign
necessitates a sign change.
change .

300
THE COLLEGE PANDA
THE COLLEGE PANDA

Chapter 12: Word


Chapter Problems
Word Problems
CHAPTER EXERCISE:
CHAPTER EXERCISE:

1. [Q]
E] The square of the
The square and y is
sum of x and
the sum [Q]Susie
6. E]
6. pounds of
2x pounds
bought 2x
Susie bought of salmon
salmon and y
and y
(x + y)2. The
The product
product of x and is xy.
and y is xy. The
The pounds of trout.
pounds of The total
trout. The cost is
total cost then
is then
question asks
question asks for the difference:: (x +
the difference y)22 -‐ xy
+ y)
(3.50)(2x)
(3.50)(2x) + 5y
Sy== 77
77
213
h ++ Sy
7x w =z 77
w
9 X ==33(X
988-‐ X ( X ‐- 110)
0)
98 _ X = 3X _ 30 Since x and
Since and y must be integers,
must be plug
can plug
we can
integers, we
98 - X = 3X - 30 each answer choice
each answer choice into the equation
into the equation above
above to to
4X = -‐ 128
-~4X see ifif we
see we get
get an value for x. When
integer value
an integer When
X = 32
X = 32 y== 4, 4, for example,
example, x z ~ 8.14, which
which is n o t an
is not an
integer.
integer. TheThe answer out to be
turns out
answer turns be 7.
7. When
When
y == 7,x
7,x =: 6.
6.

7. ~ The nickel alloy


The 35% nickel alloy contains
contains
vx + 5s = 9
J; 2(0.35) = 0.7 grams
= 0.7 of nickel.
grams of The x%
nickel. The nickel
x°/o nickel
flvx =
: 4 alloy
alloy contains 6 = 0.06x grams
~ ·-6
contains £6 nickel.
grams of nickel.
1 0
x
X =
z 16
16 When combined, these
When combined, formed
alloys formed
these alloys
2+
+6= grams of a
= 8 grams a 20% nickel alloy, which
nickel alloy, which
xX +
+22= Z 18
18 must contain 8(0.20) = 1.6 grams of nickel.
must contain 8(0.20) = 1.6 grams of nickel.
4. 1 s or 10 )Based
Based onon the information, we
the information, can
we can ‘ Setting up an
Setting up equation, we
an equation, get
we get
form the equation
form the equation 4x4x ++ lOy =: 60. N o w it's
Now it's
0.7 + 0.06x = 1.6
just matter of guess
just aa matter guess and
and check. and
Since x and
check. Since 0-7 ‘l' 0-06" Z 1-6
y are integers,, it won't
are integers be long
won’t be before we
Jong before find
we find 0.06x =
0.06x 0.9
: 0.9
something
something that For example,
works. For
that works. example, 6x =2 90
90
x= 5,yy =
= 5, one possible
= 4 is one solution.
possible solution. xX =
Z 15
15

5.
5. G!J
The width
The the monitor
width of the is ~x.
monitor is Since the
331. Since the
perimeter
perimeter of a rectangle is
a rectangle the length
twice the
is twice length 8.
8' 0
plus twice the
plus tw1ce the Width,
width , 88 +
+ 5x : 2(x
Sx = 2(x -_ 5)
5)

+ 5 x=: 22xx ‐- 110


88 +sx
2x+2<§x>
2x i
1
+ 2 ( X) = 4488 3x =
3x = -‐18
18
0

2 X zw- 6
=
2x+gxz48
2x + x = 48
33
x
§x=48
~x = 48
3

= 48 · -38 = 18
xzw-§=w
X

301
301
CHAPTER ANSWERS TO THE
CHAPTER 30 ANSWERS THE EXERCISES

9. I60 IConverting
9. and 85% to fractions
Converting 75% and in
fractions in [QJMaking
14. [E Making an equation to figure
an equation u t x,
figure oout
the equation below,
the equation below,
3 =2211
3xx-‐ 3 z
3
~(68) = 17
17 n 3 x=
3x = 224
4
4 20
x=
=8
17
17
51=‐ -20
51 En n

20) _ 8+1(8)‐12
2 _
51(fi
51 (~~) ‐=nn
15. I90 IThree
Three times the price
times the price of a shirt
shirt is 120.
60 =
60 zn Since a tie, which less than
costs 30, is k less
which costs that, k
than that,
must be
must be 120 ‐- 30
30 = 90. As
As an
an equation,
equation,
10,
10. [z] : 3(shirt)
tie = 3(shirt) -‐ kk
4+
4 + NN _1
1
3 0=: 33(40)
30 ( 4 0 )-‐ k
15+
15 N _ 2
+ N
3 0=
30 : 1120
2 0-‐ k
Cross multiplying,
Cross multiplying,
k == 990
0
2 ( 4++N)
2(4 N )== 1
155++ N
N
8+ 2 2NN = 1155++ N
N 16. ~ Let the width
Let the width of the board be
the board be w. Then
Then its
N = 77 length
length is 2w. Since the
2w. Since area of a
the area rectangle is its
a rectangle
length times its width,
length times width,

11. I48 IThey start


start with
with the number x. Once
same number
the same Once (2w)(w) =: 128
(2w)(w)
Alice gives
gives 16to
16 to Julie,
Julie, Alice is left with
with xx -‐ 16
16
2w2=128
2w 2
= 128
and Julie then
and Julie has x + 16.
then has
w2=64
w2 = 64
xX ++ 116
6 = 22(x
( x ‐- 116)
6)
xX ++116
6==22xx ~- 3322 w=\/6_4=8
W = v64 = 8
-‐ x == ‐-4488 Finally, length is 2w
the length
Finally, the 2w =
= 2(8) =
: 16.
x
X z=448
8
17. Cf]Let xx be
be the
the number
number of seashells
seashells that
that Carl
Carl
1
12. Cf]The fraction of students
The fraction who take
students who math is
take math has. Bob then
has. then has
has éxx seashells
2 and Alex has
seashells and has
1 1 11 11
11_Z
- - -‐ 5_
4 6 8
- - -5 =
11
‐ -fl.Letxbethetotal
24
. Let x be the tota 1 ~gxx seashells.
seashells. Since and Bob together
Since Alex and together
number
number of students.
students. have 60 seashells,
have seashells,

11
fix
1l X ‐=33
24
33 1
-ixx +
3
+ 5x
- x ‐=60
60
2 2
x
X =
z 72
72 2x =
2x : 60
60
X = 30
= 30
13. [Q]Let x be
13. [E be the
the number
number of trades. trade,
Each trade,
trades . Each
Carl has 30 seashells.
Carl has seashells.
has a
[ a n has
Ian net gain
a net gain of 1 card
card while Jason has
while Jason has aa
n
net card.
e t loss of 1 card.

2
200++Xx =
= 44
4 4-‐ xX
2
2xx =: 224
4
x
X ==112
2

302
THE COLLEGE PANDA
THE COLLEGE PANDA

Let x be
18. J 45 Let
J total number
the total
be the number of books.
books. Mark
Mark J I
21. 40 Jessica runs at a rate of 4 yards per

then
then has ~x
books and
has ix books and Kevin has éx books
Kevin has books.. ;x second.
second . Let t be
overtake
be the
overtake Yoona. We
time it takes
the time takes for Jessica to
make an
can make
We can an equation
equation
Kevin owns
Kevin owns 9 more than Mark,
more than so
Mark, so with
with the side being
the left side Yoona's distance and
being Yoona’s distance and
the right side being
right side Jessica’s distance.
being Jessica's distance.
1 1
-3x
x -'‐ Ex
- x ‐-
= 99 =44tt
300++ tt =
3
3 4
300= 33tt
3
Multiplying sides by
both sides
Multiplying both by 12,
100=s tt
1
4xx‐- 33xx =
4 : 108
ItIt takes seconds for Jessica to catch
takes 10 seconds catch up to
xX ==1108
08 Yoona. In that = 40
runs 4(10) =
time, Jessica runs
that time,
yards.
yards.
The total
total number
number of books
books is 108. Mark ownss
Mark own
i~ x 108 =
= 27books and Kevin
27 books and owns
Kevin owns 22. 121 ILet the side length of the original patio
J

be
be s. renovated patio
Then the renovated
s. Then has a
patio has length of
a length
1 . s+4andawidthofs‐5.
s + 4 and a width of s - 5. Settingupan
Setting up an
%xx 108 =
z 36
36 books.
books. Lori
Lon must then own
must then own
3 equation
equation for the area of the
the area renovated patio,
the renovated patio,
108 ‐- 27 ‐- 36 =
z 45 books.
books . we get
[Q]Let
19. [E Let the
the number $5 coupons given
number of $5coupons out
given out
be Then the
be ixt . Then $3 coupons given
number of $3coupons
the number given ((ss++ 4)(s
4 ) ( s- ‐ 5
5)) = 990
0
out is 3x, and
out the number
and the coupons
number of $1 coupons 52‐5‐20=90
s2 - s - 20 = 90
given
given out out is 2(3x) =: 6x.
52‐5‐110=0
s2 - s - 110 = 0
+ 3(3x) ++ 1(6x) =
5(x) + = 360 ( s-‐ 11
(s 1 0 )== 0
) ( s++ 10)
ll )(s
5x + 9x +
5x+9x 6x =
+ 6x = 360
20x =
= 360 the side
Since the length 5
side length must be
s must be aa positive
positive
number, ss = 11, which
number, which means the original
means the original
x== 18
18
X
area of the patio
area 2
was s52=
patio was (11) 2 =
= (11)2 = 121 sq
sq ft.
The
The number coupons given
number of $3 coupons out is then
given out then 23. [II This problem follows the format of a
3x = 3(18) = 54. standard
standard W = = rtrt word
word problem, where W is
problem, where
the amount of work
the amount the overall
done, r is the
work done, rate
overall rate
20. j 144 J Pipe
Pipe A by itself
itself can
can fill ~ of the
41-1 tank
the tank at
at which work is being
which work done, and
being done, and t is the time
the time
1 spent. If Terry can
spent. finish the
can finish parking lot
the parking lot in xat
each
each hour, and Pipe
hour , and Pipe B by
by itself
itself can fill Z,of
can fill the
of the days then Andy
himself, then
6 days by himself, can finish it in 2x
Andy can
tank
tank each When used
hour.. When
each hour together, they
used together, they can
can days
days by himself.
himself . This that Terry paves
means that
This means paves

fill
fill 7}+i
. 1 1 _ 55
+ 8~ =
‐ 1‐2 of the tank
12
each hour
tank each hour.. Now
Now ~ of the
%of
X
the parking each day
parking lot each and Andy
day and Andy
we
we can use the
can use formula W =
the formula = rrt,
t , where
where W Wisis 1
paves of the
_.:!_
paves 51; parking lot
the parking each day.
lot each day .
5 2x
tank and r =
tank and : $5 of the tank per
the tank hour, to find t.
per hour, 1 1
12 Working
Working together, pave -%+
they pave
together, they - of the
+ 7x X 2X
5 parking each day.
parking lot each the overall
day . This is the rater r..
overall rate
1
1 == 12
‐ t
12t
12
12
The ~ is the
number 2
The number remaining fraction
the remaining fraction of the
the
g
5 ‐= tt parking
parking lot that has yet
that has be paved.
yet to be This is
paved . This
W. 50 the number
So the number 9 m u s t be
must bet.t. And
A n d given
given
Therefore, itit takes
Therefore, takes
2
12
both pipes
hours for both
hours pipes to what
what we have for W
we have and r, t m
Wand must be the
u s t be the
5 number of days
number take Terry and
days it will take Andy to
and Andy
fill the tank.
fill the tank. Converting
Converting this result to
this result pave
pave the remainder the parking
remainder of the parking lot
2 working
working together.
together .
minutes,
minutes, we getget m = ~ x 60 =: 144.
= 132

303
CHAPTER ANSWERS TO THE EXERCISES
CHAPTER 30 ANSWERS EXERCISES

Chapter
Chapter13: Minimum& Maximum
13:Minimum MaximumWord Problems
Problems
CHAPTER EXERCISE:
CHAPTEREXERCISE:

1. (m Katherine
Katherine w i l l need
will need a total
total of 6. [[] Working
Working at the the slowest pace, Jason
slowest pace, would
Jason would
28 x 4 =
28 z 112 batteries.
batteries. Given that there
Given that 6
are 6
there are take 100 +
take 6m
-;-6 :::::16.67 hours. Working
16.67 hours. Working at at the
batteries in aa pack,
batteries she will need
pack, she need fastest pace,
fastest he would
pace, he take 100
would take +8=
100 -;-8 : 12.5
112 -;-6 18.67 packs.
+ 6 x:::::18.67 it's implied
packs. Since it’s that
implied that The only
hours. The
hours. choice between
answer choice
only answer those
between those
packs
packs only come in whole
only come numbers, she
whole numbers, she w o numbers
ttwo numbers is 16.
needs round up to 19 packs
needs to round packs to ensure she
ensure she
has enough
has batteries..
enough batteries
7. [m
2. [I] Martha
Martha needs 16‐-
needs 16 2.5 = 13.5 more
more 1 6 m 29612115
16..ow=teeS" 29.6.mt:' student
1 student
6 r11D-<t- X ---- X --- X ---
6>-r_ |- x 9Jf1' xlgunce’
119.115 1 _o.i.mce- IOOmE
100 .mt:'
. 13.5
ounces glue, which
ounces of glue, which amounts
amounts to g 1. 7.7
z 7.7
:::::
75
a
:::::28.4 students
students
glue sticks.
glue sticks. Since glue
glue sticks can only
sticks can only be
be
purchased in whole
purchased whole amounts, Martha must
amounts, Martha must Since it wouldn't make sense
wouldn’t make sense to have
have
purchase
purchase a minimum of 8 glue
a minimum glue sticks.
sticks. four-tenths
four -tenths of aa student,
student, the
the most
most that can be
that can be
accommodated
accommodated is 28 28 students.
students.
3. 0 get the
To get number of greeting
minimum number
the minimum greeting
8. @] Giovanni made 0.15 (25)( 12) = $45
cards the shop could
cards the shop could have sold, we assume
have sold, assume
that the shop
that the shop sold many gift boxes
as many
sold as as
boxes as during lunch. If he
during lunch. he serves during
tables during
serves x tables
possible
possible (400 gift boxes). each gift box
boxes). Since each dinner, he
dinner, make an
he will make additional 0.15(45)x
an additional 0.15 (45)x
was sold
was sold for $7, the store sold
store sold dollars. the total
dollars. Since the the day
total for the should be
day should be
$7 =
400 x $7 = $2, 800 worth
$2,800 worth of gift boxes
boxes in this at least
least $180,
$180,
scenario, which
scenario, means the
which means store sold
the store least
sold at least
45 +
45 + 0.15(45)x 2 180
0.15(45 )x 2::
$8, 000 -‐ $2,
$8,000 800 =
$2,800 = $5, worth of greeting
200 worth
$5,200 greeting
cards. each greeting
cards . Since each greeting card was sold
card was sold for 0.15(45)x 2135
0.15 (45 )x ~ 135
$5, the store
store could have sold
could have sold a minimum
minimum of 135
$5,200-;- $5 = 1,040 greeting cards to meet its x
X ~ o.15(45)
0.15 (45)
goal.
goal.
x ~2
X 0
20
4. I27 J To get
get the required number
the required number of nail
nail files,
1,800
salon needs
the salon needs at least 1,550 =
least ~ : 22.5 ➔
‐> 23
toolkits. get the required
toolkits. To get number of nail
required number nail
buffers, the salon
buffers, the at least
needs at
salon needs least
4,000
m 26.7 ➔
4
, OOO::::: ‐‐> 27
27 toolkits. these
Based on these
toolkits . Based
150
numbers,
numbers, a minimum
minimum of 27 toolkits must
27 toolkits be
must be
purchased for the
purchased salon to receive
the salon receive both the
both the
required number of nail files and the required
number of nail
number nail buffers.
buffers.
5. 0 Two liters is equivalent to 2 x 33.8 = 67.6
ounces, which will fill 67.6 -;- 12 :::::5.63 plastic
cups. Soat
cups. most , 5 plastic
So at most, cups can
plastic cups can be be
completely
completely filled.

304
THE COLLEGE
TI-IE PANDA
COLLEGE PANDA

Let a be
9. [!!] Leta be the number of fish
the number Ashleigh
fish Ashleigh 11. []] Because we’re trying
Because we're maximize the
trying to maximize the
caught and
caught and nn be
be the
the number
number of fish Naomi
Naomi number of nighttime
number nighttime bottles, we assume
bottles, we assume that
that
caught. Using
caught. these variables,
Using these variables, we can
can set up aa
set up only
only 65daytime
65 daytime bottles
bottles were filled. The 65
were filled. 65
an equation
system of an
system equation and an inequality:
and an inequality: daytime bottles used
daytime bottles used up 65 x 2 = 130
up 65 130 ounces
ounces
of the active ingredient
the active ingredient and
and 6565 x 66 = 390
a = 33nn-‐ 9 ounces of flavored
ounces syrup, leaving
flavored syrup, leaving
aa + n z2:45
45 130 =
385 -‐ 130 ounces of the
= 255 ounces the active
active
ingredient and 850 -‐ 390 = 460
ingredient and 460 ounces
ounces of
The equation allows
The equation allows us
us to substitute for a in
substitute for flavored syrup.. The
flavored syrup remaining ounces
The remaining ounces of
the inequality:
the inequality: 255
. ingredient
active
active .
mgre d'1ent are
are enough or ¥ =
enoug hf for : 85
85
3
aa+r1245
+ n 2:45 nighttime
nighttime bottles, and the
bottles, and remaining ounces
the remaining ounces
(3n - 9) + n 2: 45
(3n‐9)‐+‐n245 460
4n254
of flavored are enough
syrup are
flavored syrup enough for g
5
=
= 92
92
4n 2: 54
n213.5
n 2: 13.5 nighttime
nighttime bottles.
bottles. Based on these
Based on numbers,,
these numbers
we're limited by the
we're limited the remaining amount of
remaining amount
Since implied that
it’s implied
Since it's that fish are caught in
are caught active ingredient,
active ingredient, soso the
the maximum
maximum number
number of
whole numbers,
whole numbers, the possible value
minimum possible
the minimum value nighttime bottles that
nighttime bottles that can be filled is
can be is 85.
14.
of n is 14.
12. [Q]
El Let’s set up a
Let's set system withs
a system with s as
as the
the
10. I125 IIf
If we let b be
we let be the black
number of black
the number number short tables
number of short and I as
tables and as the
the number
number of
pebbles, w
pebbles, w be
be the number of white
the number pebbles,
white pebbles, long
long tables.
tables.
be the
and j be
and number of jade
the number pebbles, then
jade pebbles, then
4s 8 l ==1168
4 S++ 8/ 68
j > ~ and w < 2b. Since j = 32,
g and the first
32, the
s + Il §::;3322
inequality becomes
inequality 32 > ~,
becomes 32 g, which simplifies
which simplifies
Divide both
Divide both sides the equation
sides of the get
equation by 4 to get
64 > b,
to 64 b,so the maximum
so the possible value
maximum possible value of bb
Using b = 63, the
is 63. Using the second inequality
second inequality
+ 21
s5+ 21:= 42. Isolating I then
42. Isolating then gives = 21
gives /1= 21 -‐ ~.
2.
becomes w
becomes w < 2(63), which simplifies to
which simplifies Substituting this
Substituting result into
this result into the inequality, we
the inequality, we
w < Based on this
< 126. Based this result, the maximum
result, the maximum get
get
possible value
possible value of ww is 125.
125. Now you might
N o w you be
might be
wondering why we used maximum
the maximum
used the s
wondering why we s + 21 - -2 <
-
32
possible value of bbin
possible value inequality.
second inequality.
in the second
Since w
Since w is less
less than maximizing w
than 2b, maximizing w means
means :. < 11
that we have
that we maximize b first.
have to maximize 2 -
s::; 22
Based on this
Based on result, the
this result, maximum number
the maximum number of
short tables
short tables that can be
that can used is 22.
be used
13. [!!] To get at least
To get $140 worth
least $140 worth of tacos,
tacos, a
customer would have
customer would receive at least
have to receive least
~-!~
m
140
~m 53.8
53.8 ➔‐> 54 tacos round up since
(we round
tacos (we since
it’s implied
it's implied that tacos are
that tacos given in whole
are given whole
numbers only). To receive
numbers only). at least
receive at 54 tacos,
least 54 tacos,
the
the customer would have
customer would buy at least
have to buy least
= 13.5 ➔
54
54‐4
4
= ‐> 14
14burritos (again, we
burritos (again, round up
we round up
since it’s
since implied that
it's implied that burritos sold in
are sold
burritos are
whole Therefore, 14
numbers). Therefore,
whole numbers). the
14 is the
minimum..
minimum

305
CHAPTER 30 ANSWERS TO THE EXERCISES

14. [I] Let a be the number of two-tier cakes and 16. I54 ILet s, m, and
and Il represent the numbers
represent the numbers of
b be number of three-tier
be the number cakes Ava
three-tier cakes small, and large
medium, and
small, medium, boxes shipped,
large boxes shipped,
decorates. Using these
decorates . Using these variables, we can
variables, we set
can set respectively. Based on
respectively. Based on the information given,
the information given,
up the following
up the system of inequalities:
following system inequalities:
ss + m
m + II==2 250
50
2 0 a+
20a +3 5 b~
35b § 3360
60 lI > s+m
>s+
a+b214
a + b 2'.14
Since m = the system
= 70, the system becomes
becomes
Note
Note that converted 6 hours
that we converted hours to 360 360
minutes to set
minutes set up
up the inequality.. The
the first inequality The first ss + II ==1 180
80
inequality simplifies to 4a
then simplifies
inequality then 4a++ 7b ~ 72. To
7b 5 Il >> ss ++770
0
solve this system, we have
solve this system, get the
have to get signs
the signs
pointing
pointing in the same direction
the same direction soso that we can
that we can IsolatingI
Isolating l in the equation gives
the equation = 180 ‐- 5.
gives Il = s.
add the inequalities.
add the Remember that
inequalities. Remember that Substituting this into
Substituting this into the inequality, we
the inequality, we get
get
inequalities can be
inequalities can be added only if
added only their signs
if their signs
point
point in the same direction
the same direction (do(do not subtract
not subtract 1 8 0- ‐ss > ss ++7700
180
inequalities;
inequalities; think only in terms
think only adding
terms of adding 110>25
110 > 2s
them) . Soif
them). So if we
we multiply
multiply thethe second
second 555 >
5 >s
inequality
inequality by -‐ 44,, we
we end
end upup with the
with the
following
following system (note the
system (note sign change):
the sign change) : Since s5is
Since implied to
is implied bea
to be whole number,
a whole number, the
the
greatest possible value
greatest possible value of s5 based on this
based on this
4a + 7b S
4a+7b ~ 72
72 result
result is 54.
-‐ 44aa -‐ 44b
bS~ -‐ 556
6
17. [zJBased
Based on the given
on the given information, 11=
information, n = 10
10
Adding the inequalities,
inequalities, we and w =
and = Bx
8x (8 ounces
ounces of water each of the
water in each the
Adding the get 3b
we get 3b 5 16,
~ 16,
which simplifies
which simplifies to b ~5 5.33. Since
Since it's x cups).
cups) . To use these values,
use these we first set
values, we set up
it's
the following
the inequality:
following inequality:
implied that
implied decorates cakes
that Ava decorates whole
cakes in whole
numbers, the
numbers, the maximum possible value
maximum possible value of b is
CS
~ 16%
16%
5. This question could’ve also
question could've been solved
also been solved
through guess and
through guess and check.
check. 100n <
100n < 116
6
nn + w -
15. [RJ For 11 pound
E] For pound of seasoning,
seasoning, Lianne will
Lianne will 100(10)
10 < 16
need 0.75 pounds
need pounds of sea salt
salt and pounds 100
and 0.25 pounds
10 + 8x s- 16
i 8.3
of black
black pepper. The sea salt
pepper. The salt will
will cost
cost
0.75 x $2$2 = $1.50 and
and the black pepper will
the black pepper will S 16
1000 ~
1000 (10 + Bx)
16(10 8x)
cost 0.25 x $8$8 = $2. Altogether,
Altogether, that'
that’ss 1000 ~
1000 160 +
S 160 + 128x
128x
$1.50 ++ $2 = each pound
= $3.50 for each pound of 840 S
~ 128x
seasoning.
seasoning . Since Lianne can spend
Lianne can spend no moremore
6.56 5
~ x
X
than
than $210, she limited to making
she is limited making
$210 . Since the question
Since indicates that
question indicates that x is aa whole
whole
$ = 60 pounds
: 60 pounds of seasoning. Therefore,
seasoning . Therefore,
ssfso
350 minimum possible
the minimum
number, the
number, value of
possible value of xx
60 is the
60 maximum.
the maximum. based on
based result is 7. It’s
this result
on this It's important
important to to note
note
thatwe able to
were able
that we were to multiply
multiply both sides by
both sides by
10 + 8x
10+ worrying about
without worrying
Bx without sign
about a sign
change because 10
change because 10++ Bx
8x is guaranteed
guaranteed to be
to be
positive
positive (x must
must be positive in the
be positive context of
the context
the problem).
the problem) .

306
THE COLLEGE PANDA
THE COLLEGE PANDA

14: Lines
Chapter 14:
CHAPTER EXERCISE
EXERCISE::

1.
1. [filA vertical
vertical line that intersects
line that intersects the
the x-axis at 5. [f] The graph of line/ goes up three units for
has an
3 has an equation
equation of xx =
z 3. every units to the
w o units
every ttwo the right,
right , which
which means its
means its
2.‑
2. [f] slope is ~.
slope 2 A parallel line must
parallel line have the
must have the same
same
slope.
slope. Only answer choice
Only answer choice (C) gives an
(C) gives an
yz‐y1
Y2- y1 z}1 equation a line
equation of a line with
with the
the same slope.
same slope.
xX2z -‐ xX)1 3

ng
7 1- ‐ 1
n
5 -(-
l
1)
1
=3 6. B From the graph, we can see that J
n -‐ 1l 1 goes up
up 11 unit every 2 units
unit for every to the
units to the right,
right,
--= -
6
6 =3
3 which means its slope
which means
1
slope is ~.. Since g is
n -‐ 11 = 2
E
perpendicular to J,
perpendicular f, the slope of gg must
the slope must be
be -‐22
n=3 (the negative reciprocal).
(the negative g passes
reciprocal). Since g passes
through
through the
the point ( 1, 2),
point (1, we can
~), we can use
use
3.
3. 0 Draw aa line
Draw line from the the x-intercept
x-intercept of -‐22 to
to
the y‐intercept
y-intercept of -‐ 44.. point-slope equation of
point -slope form to find the equation of g:

yll yy ‐- yy1
1 ==" m(x
1 ( x-‐ xxi)
i)
5
y‐§‐‐2(x‐1)
y --=-
2
2(x - 1)
5
yy == _- 2x+
2x+2+2
2 + -_
2
9
y‐‐2x+§
y = - 2x + -
2

.
‐4
Finally,g (- 1) = - 2(- 1) + 229 = 2 + 4~2l = l
6!

A quicker
quicker way would've been to work
would've been work
you can
As you see, it goes
can see, down for every
units down
goes 4 units every
2 units to the right.
units to slope is
The slope
right. The
4
~ = -‐ 22..
is _74 backwards
backwards from the point
from the ( 1, g),
point (1, ~), knowing
knowing
that
that the slope is -‐ 22.. So
the slope Soon
on the graph of
the graph g, 11
of g,
. . 8 ‐ 5 ‐_ 9 ‐-
_
( ) = ~ = !3..
4. 0 The slope of line / is 6‐‐(‐_3)
4.-Theslopeof1mells unit to the left brings
unit brings us us to
6- - 3 9 3
point‐slope form,
Using point-slope
Using form, ( 0, ~ +
(0,2. + 2) = (0,4%>,
2) = and 11 more
(0,41), and unit to
more unit to the
the

yy -‐ yY1 =m(x
l =m ( x - 11
x1)) brings usto
left brings us to ((‐1,4% +2)
- 1,41 + = ((‐1,6%).
2) = - 1,6D.
1 g(‐1)
Therefore, g( - 1) = 6.5.
- 8 = (x - 6)
yy‐8‐§(x~6) Therefore,
3
1 7. []]
yy = éxx +
+66 3 u = fl2 -=0 =i 2_ = _ 1l
xz‐xl 0 -‐ 4 -‐ 44 5
2
this point,
At this point, we
we test each choice by
answer choice
each answer
plugging
plugging in the x-coordinate
x-coordinate and verifying
and verifying
the y‐coordinate. Only
they-coordinate. ( A ) works.
answer (A)
Only answer works.
8. [filFrom the graph, slope mis positive and
y-intercept
y-intercept b is negative.
negative . Therefore, mb < 0.
Therefore, mb 0.

307
CHAPTER 30 ANSWERS TO THE EXERCISES

@The
9. [Q] The line = -‐ 22xx -‐ 2
line y = has a
2 has a slope
slope ofof -‐22 from the
from table . Answer
the table. Answer A A works
works for for Monday
Monday
and a
and a y‐intercept
y-intercept of -‐ 22.. Line
Line Il m
must have a
u s t have a (c(7.2) = 30(7.2) +
= 30(7.2) + 400 = = 616) butbut n o t for
not any
for any
slope that is the
slope that the negative reciprocal of
negative reciprocal of -‐ 22,, of the
the other days. Answer
other days. Answer B works
works for for
1 Saturday (c(8.5) := 60(8.5)
Saturday (c(8.5) + 210
60(8.5) -+- 210 = 720) but
= 720) but
which is 2'. Since
which Since they
they have
have the
the same
same
2 n o t for any
not any of the other days.
the other Answer D does
days. Answer does
y-intercept, the equation
y-intercept, the of line
equation of line Il must be
must be not give
not give the
the correct
correct value for any
value of c for any of thethe
given
given values
values of 5. s. Only answer C
Only answer C gives
gives the
the
= 1 x - 2.
yyzix‐Z.
2 value of c for each
correct value
correct the given
each of the given values
values
of 5.
s. These types of questions
These types questions require you to
require you
10. []] The line we're looking for must have a be
be thorough.
thorough . Don’t just test
Don't just one case
test one and
case and
1 choose the
choose the first thing that ”works.”
thing that "works." You have have
slope
slope that is the
that is negative reciprocal
the negative reciprocal of
of %,, 2 evaluate all
to evaluate all the answer choices.
the answer choices.
which is ‐- 22..
which
13. [E
[Q] A line with a
line with a positive y-intercept will
positive y‐intercept will nnot
ot
y = ‐2x +b the y-axis
cross the
cross negative point.
at a negative
y-axis at point. Therefore,
Therefore ,
- 2x+b
when x is 0, y cannot
when cannot be negative, which
be negative, which
Plugging in the
Plugging the point (1,5),
point (1,5 ), makes (E) the
makes (E) answer.
the answer.

5 =: ‐2(1)+
- 2(1) + bb 14. 1l.6 ~
1.6 or 2 IFirst,
First, plug point (2,6)
the point
plug the into the
(2, 6) into the

7=b equation
equation of the line so
the line that we
so that solve for a:
can solve
we can a:
Now
Now that
that we have b, the
we have line is y = -‐ 22xx +
the line + 7.
7.
1
a(2)‐§(6)=8
a(2) - (6) = 8
3
@
11. [Q]
2 a- ‐ 22 =z 8
2a
10‐4
10 -4 _2 2 2aa =110
2 0
= 3-
x ‐- l 1 _ 3
a=5
6
6 _z
x -1
X
2
‐ l ‐ 33 1
So the equation
So the equation of the line is 5x -‐
the line all z 8.
Cross multiplying,
Cross multiplying,
3y = 8.
x-intercept always
The x-intercept
The has aa y-coordinate
always has y-coordinate of
of
2 ( x-‐ 11)) = 118
2(x 8 0,soif
0, so if we
we plug
plug in Ofor y, we
we get
get 5x = 8,
5x = 8,

2 x-‐ 22 ==118
2x 8 which gives
which ~ == 1.6.
gives x = g
2xx = 220
2 0
x
X ==110
0
15. [1JOne
One easy way to approach
easy way approach this
this problem
problem is
is
to make
make up numbers for a and
up numbers and b. Let a = 1
b. Let

[SJWe can and


and b = 2 so that ~g = ;%.. Since
so that Since the second line
the second line
12. can use the values
use the from any
values from days
t w o days
any two
to find the line
find the that best
line that models the
best models the data. Let’s
data. Let's d
is perpendicular
perpendicular to the first, C;=
the first, - = -‐ 22,, which
which
use the
use values from
the values Monday and
from Monday and Thursday
Thursday to e
616 -‐ 584 . satisfies
satisfies the condition in answer
the condition answer choice
choice (A).
calculate the slope:
calculate the slope: ----
7.2 -‐ 6.8
=
= 80. At this
this
point, we can
point, we can tell
tell the answer is probably
the answer probably
going
going to be choice C, but
be choice but let’s find the
let's find the
y-intercept just to be
y-intercept just sure. Currently,
be sure. Currently, we we have
have
c= 80s +
z 805 -+- b. Plugging
Plugging in inss =
= 7.2 and
and cc= = 616
from Monday,
from Monday, we we get
get 616 = 80(7.2) +
: 80(7.2) ‐+‐ b,
gives 1)z
which gives
which 80(7.2) = 40.
b = 616 ‐- 80(7.2)
c(s) = 80s
Therefore, c(s)
Therefore, 805 ++ 40.
An alternative solution is to test
alternative solution each of the
test each the
choices by
answer choices
answer by plugging
plugging inin the values
the values

308
THE COLLEGE
THE PANDA
COLLEGE PANDA

Chapter15:Interpreting LinearModels
CHAPTEREXERCISE:

1. [I]
E The slope is -‐ 33,, which
The slope which means the water
means the level decreases
water level decreases by 33 feet each
each day.

E The value
2. []] 18refers
value 18 refers to the slope of -‐ 18, which
the slope which means number of loaves
means the number loaves remaining decreases by 18
remaining decreases 18
each hour.
each implies that
hour. This implies the bakery
that the bakery sells 18loaves
sells 18 loaves each
each hour.
hour .

3. [g They -intercept of 500 means that when n = 0 (when there were no videos on the site), there were 500
members.
members.
4. @ The number 2 refers to the slope of - 2, which means two fewer teaspoons of sugar should be added
every teaspoon
for every already in the
honey already
teaspoon of honey beverage. Don’t
the beverage. be fooled
Don 't be fooled by answers
answers (C) and (D), which
(C) and which
”reverse” the x and
"reverse" and they
the y (I: and s, in this
(hands, case). The slope
this case). slope is always the change
always the each unit
change in y for each increase
unit increase
in x, not
n o t the other way around.
other way around.

5. [I] The salesperson earns a commission, but on what? The amount of money he or she brings in. To get
we must
that,, we
that multiply the
m u s t multiply the number
number of cars by the average
sold by
cars sold average price of each
price of each car. Since xx and
and c6 already
already
represent the commission
represent commission rate and the
rate and the number
number of cars sold, respectively,
cars sold, the number
respectively, the number 2,000 must represent
must represent
average price
the average
the price of each
each car.

6. [) The number 2,000 refers to the slope, which means a town 's estimated population increases by 2,000
each additional
for each additional school the town
school in the town..
7. [I] The number 4 refers to the slope of - 4, which means an increase of 1° C decreases the number of
hours until
hours gallon of milk
until aa gallon goes sour
milk goes words, the
other words,
sour by 4. In other milk goes
the milk goes sour
sour 4 hours
hours faster.

8. [fil When t = 0, ther e is no time left in the auction. The auction has finished. Therefore, the 900 is the
price of the
auction price
final auction lamp.
the lamp.

9. [I] Because it's the slope, the 1.30 can be thought of as the exchange rate, converting U.S. dollars into
after the
euros. But after
euros. subtracted away,
conversion, 1.50 is subtracted
the conversion, which means
away, which means you euros less than
get 1.50 euros
you get you
than you
should Therefore, the
have.. Therefore,
should have interpretation of the
best interpretation
the best y-intercept is a
the 1.50 y-intercept e u r o fee the
a 1.50 euro the bank charges
bank charges
to do the conversion.
conversion.
. . 22 99
~
. To see the answer
10. L:.!JTo see answer more clearly, we
more clearly, we can
can put
put the equation mto
the equation = mx
mtoy = mx++ bb form:
form: tt == 5xx + 5 . The slope
The slope
5 5
2
is ~, which means
5, or 0.4, which the load
means the load time increases by 0.4 seconds
time increases image on the
each image
seconds for each the web page.
web page.

slope is
The slope
11. @ The is the change in y (daily
the change (daily profit)
profit) for each change in xx (cakes
unit change
each unit (cakes sold).
sold).

Notice that
12. [) Notice the y-intercept
that they negative. It is
-intercept is negative. the bakery’s
is the bakery 's profit when no
profit when no cakes
cakes areare sold. Therefore,
sold. Therefore,
anything that
anything that varies
varies with
with the number of cakes
the number cakes sold
sold is incorrect. example, answer
incorrect. For example, answer (D) is wrong
wrong because
because
cakes that
the cost of the cakes that didn
didn't depends on
' t sell depends on how many
how man the bakery
y the bakery did
did sell. It's
It’s n o t aa fixed number
not number
like the y-intercept
like they The best
-intercept is. The interpretation of the
best interpretation -intercept is the
theyy‐intercept the cost of running
running the bakery (rent
the bakery (rent,,
labor, machinery,, etc.), which
labor, machinery likely aa fixed number
which is likely number..

13. [J) The


The solution (5,0) means
solution (5,0) that the
means that bakery's daily
the bakery's daily profit
profit is zero when 5 cakes are
zero when sold.
are sold. Therefore,,
Therefore
selling five cakes
selling enough to break
cakes is enough break-even with daily
-even with daily expenses.
expenses.

14. I2.s j The slope


slope of the equation
equation is 5, which
which means
means the temperature goes
the temperature goes up degrees every
up by 5 degrees every hour 50
hour.. So
every half
every half hour the temperature
minutes), the
hour (30 minutes), temperature goes up by 0.5 x 5 =
goes up = 2.5 degrees.
degrees.

309
309
CHAPTER 30 ANSWERS TO THE EXERCISES

[[I Putting
15. [E
.
the equation
Putting the equation m
.
into
.
t o y = mx + mx
+ b form,
1
1x
+ 7. The
form, y = 5x + The slope 1
means that
slope of % means more turtle
one more
that one turtle
requires an
requires an additional
additional half gallon of water
half aa gallon SoIll
water.. So 11] is true.
true .
Getting terms of y, x = 2y
Getting x in terms The "slope”
2y ‐- 14. The "slope " of 2 means that
2 means more gallon
that 11 more gallon of water can
of water support ttwo
can support wo
more turtles.
more turtles. SoI
So I is true.
true.
16. CgBecause
Because this asking for the
question is asking
this question change in "x"
the change per change
” x ” per change in ”"y"
y ” (the reverse of
(the reverse slope), we
of slope), we need
need
to rearrange
rearrange the equation to get x in terms
the equation C.
terms of C.

+ 2.53:
C = 1.5 + 2.Sx

Dividing each element


Dividing each the equation
element in the by 2.5,
equation by

0.4C =
: 0.6 + x

x =
= 0.4C -‐ 0.6
The slope
The slope here which means
here is 0.4, which the weight
means the shipment increases
weight of aa shipment pounds per
increases by 0.4 pounds per dollar
dollar increase
increase
in the mailing cost.
the mailing cost. So50 a 10 dollar
a 10 increase in the
dollar increase mailing cost is equivalent
the mailing equivalent to a
a weight increase of
weight increase
10 x 0.4 =
10 : 4 pounds.
pounds.

310
THE
THE COLLEGE PANDA
COLLEGE PANDA

Chapter 16:
Chapter 16: Functions
CHAPTER
CHAPTER EXERCISE:
EXERCISE:

l.1. [Q]
FE] Check each answer
Check each choice to see
answer choice whether
see whether 10. @ The x‐intercepts
x-intercepts of ‐3 and 2 mean
- 3 and mean that
that
f(0)
J (O) == 20,J
20,f(1)
(l ) = 21, and
and /f(3)
(3) == 29. The only
The only fJ((x
x ) must have factors of (x + 3) and
must have 2).
and (x -‐ 2).
function that
function satisfies all three
that satisfies three is (D).
(D). That eliminates
That eliminates (C) and
and (D). AA y-intercept
y‐intercept of
@f(x)
2. [Q]J (x) := g(x)
g (x) when
when the two graphs
t w o graphs means that
12 means that when plug in x =
when we plug = 0,
intersect.
intersect. They intersect at 3 points,
intersect so there
points, so there Jf ((x)
x) == 12. Only answer
Only answer (B) meets all these
meets these
must be
must be 3 values where Jf ((x)
values of x where x ) = g(x). conditions.
conditions .
11. lg(2)
11- 22 -‐ 11=
[I)g( 2) =: 22 3.So,
= 3. So,
(3) = -‐ 22.. N
3. @ Jf(3) Now where else
o w where else is ffatat -‐ 22?
? ff(g(2))
(g(2)) = J (3) 3 2+1=
= f(3) = 32+1 2 110.
= 0.
When x =
When z ‐- 33.. Soa
So a must be -‐ 33..
must be
4. [I] Draw horizontal line at
Draw aa horizontal at y = 3.
3. This line
line 12. ~ The difference
B The difference between
between 2x2x22 -‐ 2 and
and
intersects J[ ((x)
intersects x ) four times, there are
times, so there four
are four 2x ++ 4 is a constant In other
constant of 6. 1n other words, 6
solutions (four
solutions values of x for which
(four values which needs added on to y =
be added
needs to be 2x22 -‐ 2 to get
= 2x
J (x ) = y= 2x +
: 2x22 + 4.
4. That entails a translation
That entails translation 6 units
units
f(X) = 33).
)‑
upward.
upward.
5. [g Plug
Plug in -‐33 and
and 3 into each of the answer
into each answer
choices to see whether you get
whether you get the same
the same
13. [I) Draw
Draw a
a horizontal at y =
line at
horizontal line C, passing
z c, passing
value. If
value. If you’re smart about
you're smart about it, you’ll realize
it, you'll realize through
through (0, c). This horizontal line intersects
horizontal line intersects
that answer (C) has
that answer has an 2 , which always
an xx2, always gives
with f three
with three times. That means
times. That there are
means there are 3
gives
a
a positive
positive value Testing (C)
value.. Testing (C) out,
out,
values x) =
which Jf ((x)
values of x for which = c.
Jf ((-‐ 33)) = 3(- 3)2 +
3(‐3)2 +11 = 28 and
28 and 14.
14.@
ff(3)(3) = 3(3)2 +
= 3(3) 2 +11 =
= 28. The answer indeed
answer is indeed
(C).
(C)- g(k)
g( = 8
k) = 8
6.-First,g(10) = f(20)
6. @] First, g(10) = f (20) -‐ l.1.N o w,
Now, 2f(k)
2f =8
(k) = 8
f (20) = 3(20) + 2 = 62. Finally,
f(20) Finally, J flJ (k)
k) =Z 44
g(10) = 62
62‐- 11 = 61.
iol 16 + (- 4) _ 22
32
Lookingat
Looking at the chart, = 44 only
chart, fJ (x) z only when
when
7. .~ f (J (-
7. ‐ 44))=: 16‐2%4)i
2(- 4) : 3‐1
= 2
- 8 = -4 4.. =3.3. So
x= Sokk = 3.
15. lg]
[Q]Since g(
g(x)
x) just adds constant of 7 to
adds a constant
8. [I] We plug values to solve
plug in values and b.
solve for a and every value of f(x),
every value the maximum
J (x), the maximum of g(x)
Plugging in (0,
Plugging ) , -‐22 =
(O, -‐ 22), : a(0)2 + b =
: b. So, occur at the same
must occur
must x-value as
same x-value as the
b = -‐ 22.. Plugging
Plugging in (1,3),(1, 3), maximum of f(x),
maximum J (x), namely
namely x = 3. So, the
maximum of g(x) is reached
maximum reached at point
the point
at the
3 = “(1)2
a(1)2+ +b (3,g(3)),
(3,g (3)), and g (x ) =
since g(x)
and since : Jf(x)
(x) + 7,
7, this
this
3=z a -‐ 2 point is
point (3,/ (3) +
is (3,f(3) + 7).
5== a
16. .0 f ( J1(18)
8 ) =: \/18
✓18 ‐- 2 =
= v'16 z 4.
\/1‐ = 4.
So a = 55 and
Soa x) =
and Jf ((x) 5x22 -‐ 2. Finally,
: 5x Finally, (11) =
/f(11) = \/11‐
/If=2 = J9 == 3.
= \/§ 3.
3 ) == 5(3)2 ‐ 22 == 43.
[J((3) 5(3)2 - ff(18) (11) =
(18) ‐- ff(11) = 1. Testing
= 4 -‐ 3 = Testingeach
each
choice, f (3) is the only one
answer choice,
answer one that also
that also
9. 0 Plug in the
Plug answer choices
the answer choices and check.
and check. equals
equals 1.
l.

f(§)
1
_ (5)
J (1)=
1 2 1
(1)
2
. . 1
~,which is less than 1
‐ z,whlchlslessthan
= 2'-
The answer (A).
answer is (A).

311
CHAPTER 30 ANSWERS TO IBE EXERCISES

17. ~
I IfIf we
we factor 3, we
factor g, get
we get 25. [[I
E Test each each of the the answer choices.. When
answer choices When
g(x) 2 + 4x + 4 = (x + 2) 2 . Since g(x) is x == -‐ 33,, Jf (x) = -‐22 according
according to its its graph,
g(x )= x + 4 x + 4 ‐_ (x+2)2. Sinceg(x) is
= x graph ,
f (x)) shifted
shifted k units the left,
units to the and g(x)
g(x) = ((-‐ 3 ++3)(‐‐3 3)( - 3 -‐ 1)
1) = 0. In this
0.Inthis
case, f ((x
case , x ) is notn o t greater than g(x).
greater than g(x ). When
When
g ( x )== J(x
g(x) f(x++kk)) x == -‐ 22,, Jf ((x)
x) ~
z 1.5 and and
g(x)
g(x) =: ((-‐ 2 + 3 3)( 1) =
) (-‐ 2 -‐ 1) z -‐ 33.. Ilnn this case,
this case,
(x+2)2=(x+k‐3)2
(x + 2)2 = (x + k - 3)2
x ) >> g(x) so
fJ((x) so we havehave oour answer..
u r answer
x++ 22 =: xx +
+ k -‐ 3
2= = k -‐ 3 26. [g For
For horizontal
horizontal shifts, the trick
shifts, the is to
trick is find
to find
the value
the value of x that
that makes
makes the substituted
the substituted
5=k
expression equal
expression equal to O. This value
0. This value tells
tells you
you
Alternatively, we
Alternatively, we could've
could’ve solved
solved this
this what the
what the horizontal shift is. For
horizontal shift choice A,
For choice
question
que stion by comparing
comparing J(x) flat) and g(x) to
and g(x) to 2
= 5 makes
x= makes 3x -‐ 2 equal
3
equal to 0, so the
so the
y= 2 . The
: xx2. graph o
The graph offfJ((x)
x ) == (x
( x~ 3)22 is
- 3) i s3
3
shift is ~g units
right of y =
the right
units to the
units 2, and
: xx2, graph of
the graph
and the horizontal
horizontal shift units to the right. For
the right. For
g( x) = (x +
g(x) 2)22 is
+ 2) is 2 unHs
units toto the
the left 2.
left of y = xx2.
g(x) is 5 unit
Therefore, g(x)
Therefore, the left
unitss to the f (x)
left of J( x ). 2 2
choice B, that
choice value is -‐ g,, so
that value so the shift is g
the shift
3 3
choice C, x = ~3 makes
E) (1,
18. [Q] (1,2) cannot be
2) cannot be on
on the graph of y since
the graph since an
an
units to the
units For choice
left. For
the left. makes
x-value
x-value of 1 would result division
would result in division by 0.
0.
the expression
the equal to 0, so
expres sion 2x -‐ 33 equal so the shift is
the shift
19. @
[Qj 3
~5 units
units to the right. This
the right. the answer.
This is the For
answer . For
g(aa) = 6
g(
choice D, the
choice shift is ~3 units
the shift the left.
units to the left.
v13a=6
3a = 36
36 [E The
27. [Q] key words
The key words are ”linear function
are "linear function": ": f is
is
12
a = 12 a
a straight line. So
straight line. what straight
So for what line can
straight line can
both f(2)
both / (2) 5 '.Sf[(3) and J(
(3) and f(4) 2 /f(5)
4) 2: be true?
(5) be true?
Only
Only a horizontal straight
a horizontal straight line. Take aa minute
line . Take minute
E] Using
20. [Q] the tabl
Using the table, g(‐1)
e, g( : 2. Then,
- 1) = Then, to think that through.
think that through . Now, since f is a
Now , since flat
a flat
/( 2) == 6-
f (2) 6. line and f/ (6)
line and (6) = 10, 10, then all values
then all values of
offf are 10,
are 10,
no matter
no matter what
what the value of x is. Therefore,
the value Therefore,
21
21. []Ju g(c) = 5, then
then c = since 1 is the
1 since only
the only f mO)) =
f( : 110.
a
input that
input gives an
that gives an output
output of 5.
5. Then,
Then,
f(c)
f(C) == fJ( (l1 )) == 33.‑ 28. 8JRemember
Remember that
that you
you can use your
can use your
calculator
calculator for graphing.. The
for graphing graph of
The graph
22.. []] From
From the
the second equation , f (a) =
second equation, = 20. So,
So,
Jf ((x)
x ) = xx33 is ”centered"
"centered" at (0,0).
at (0,0 The graph
). The graph of
f(a) + 55 g(x)
g(x) is is ”centered”
"centered" at (3, -‐2).
at (3, Comparing
2). Comparing
J( a) = -‐ 33a
a+
these points
these points of reference,
reference, we we can g(x)
that g(x)
see that
can see
20 == -‐3a
20 3a + 5 is shifted
shifted 3 units
units to the right
to the and 22 units
right and units
3a == -‐15
3a 15 downward from
downward from [(x).
J(x) . Therefore,
Therefore,
a =z -‐ 5 g(x) =
g(x) f ( x -‐ 3) -‐ 2,
: J(x 2,which means a =
which means = -‐33
and b = -‐ 22.. The
and The ssum +b
um a + bisis then
then
+ ((-‐ 22)) =
-‐ 33 + = -‐ 55..
.f-)g(
23. 8]g( (3) : 3=) /( 2(3)
2 ( : 3- )1) f(5)=2.Weget
‐ 1=) /( 5) = 2. We get
from the
f (5) =‐2 2 from the table.
table.

f@)f(8)
24. [Q] = 4(8) -‐ 33=
J (8) = = 29. Testing
Testing each
each
answer choice to see
answer choice which one
see which yields 29, we
one yields
see that
see 3(8) = 3(8) + 5 = 29.
that g(8)

312
THE COLLEGE PANDA
THE COLLEGE PANDA

When x =
29. @ When = 0, y =
= 9,
9, so
so the y-intercept is 9.
they-intercept 9. 31. [g The function g(x) is a
The function a line with aa slope
line with slope of
When y = O,x
When 0, x = 3, so the x-intercept
so the x-intercept is 3. l1 and
and a y-intercept of k. If
a y-intercept you draw
If you g(x) with
draw g(x) with
the different possibilities
the different possibilities for k from the
from the
y choices, you’ll
answer choices,
answer see that
you'll see that there’s an
there's an
A intersection of 3 points
intersection points with f (x) only
with J(x) only when
when
k = 1 as shown below.
as shown below .
y

6 AOB is a
AAOB right triangle
a right with a
triangle with base of 3
a base and
3 and
a height of 9. Using
a height the pythagorean
Using the pythagorean theorem,,
theorem

A
A0d2 +
+o0Bfi2z=AABW2
¢ + 3§2 == AB
92 + A §2
s =zABfi2
90

3 ¢ E =: AAB
3v'10 B 32. [I]Plugging
Plugging in (‐a,a),
( - a, a),

a=
= aa(-
(‐aa)) +
+ l12
2
30. [g The
The graph
graph of g is 4 units
units up
up from where J
from where f
is, but
is, because the
but because the slope of f is -‐ 22,, the
slope off the x and
and a:
a = ‐a2+12
- a 2 + 12
y intercepts
intercepts of g will will not increase by
not increase by the
the same
same a2+a‐12=0
a2 + a - 12 = 0
amount. They’ll
amount. increase in a
They'll increase ratio of 2:1. 50
a ratio So
((a+
a + 44)(a 3)) == 0
) ( a-~ 3
when the
when theyy-intercept
-in tercept gets shifted up
gets shifted up by 4, 4, the
the
x-intercept gets
x-intercept shifted to the
gets shifted right by
the right by 2. The
The a= = ‐-44,3
,3
n e w x-intercept
new therefore 1I + 2 = 3.
x-intencept is therefore
S i n c ea a>>0,
Since 0 ,aa=
= 33..
Another
Another way way to do this is to actually
do this actually solve
solve for
the x-in tercept. Using
the x-intercept. slope-intercept
Using slope -intercept form,
form,
we
we get
get f(x)
J(x) = ‐
- 22xx + 2.Adding
2. Adding 4
4 to
to get
get the
the
equation
equation of g, g(x)
g(x) = ‐
- 2 x
2x +
+ 6.
6. Setting
Setting
g(x) = 00 and
g(x) solving for x to get
and solving get the
the
x-intercept,
x-intercept, wewe getget x = z 3.

313
CHAPTER 30 ANSWERS
CHAPTER EXERCISES
ANSWERS TO THE EXERCISES

Chapter 17:Quadratics
Chapter17: Quadratics
CHAPTER EXERCISE:
CHAPTER

1. [g We factor
factor to find the
the x-intercepts.
x-intercepts. 6. []JThe minimum or maximum
The minimum maximum of a
a parabola
parabola
always vertex. Since m is
occurs at its vertex.
always occurs
y=x2‐3x‐10=(x‐5)(x+2)
y= x2 -
3x - 10 = (x - 5) (x + 2 ) positive, the parabola
positive, the opens upwards
parabola opens upwards in a a
U ” shape, which means we're dealing with
”"U" shape, which means we're dealing with
The x-intercepts
The are 5 and
x-intercepts are The distance
and ‐- 22.. The distance the parabola's
the minimum at
parabola 's minimum at its vertex . Since
its vertex. Since
between them is 5 ‐- ((-‐ 22)) =
between them = 7. expanding
expanding f gives
gives us
us aa vertex form of
vertex form
J (x) = m[(x‐m)2‐
f(x) m [(x - m)2 - 1] 1]==m m(x( x-‐ m)2 ‐- mm,,
2. [}] Using the quadratic
Using the quadratic formula,
formula, the vertex is at (m, ‐- m). Therefore,
the vertex Therefore, thethe
parabola’s
parabola 's minimum occurs at (m, ‐- m
minimum occurs ).
m).
-‐ 44 fl±: ./(4)?‐4(1)(2)
✓ (4 ) 2 - 4(1)( 2) _ v'8
-‐ 44±i \/§
X = 2(1)
2 (1) =
_ 22 7.
7. [}] Substituting the first equation
Substituting the into the
equation into the
second,
second,
_‐4:t2\/§
_#- 4 ± 2v12
2 ‐3
- 3==x2
x2 +
+cx
ex
:=-‐ 22i ±
\ / ..fi
§. 0
0 = x2
2
+ at + 33
x +ex+

The system of equations


The system equations will have ttwo
will have wo
3 .‑
3. []] solutions
solutions if the equation
if the equation above has t w o
above has two
solutions.
solutions. For the equation
For the above to have
equation above have ttwo
wo
2a2‐7a+3:0
2a2 - 7a + 3 = 0 solutions, the
solutions, the discriminant, b2 - 4ac,
discriminant, b2‐ must be
4ae, must be
((2a
2 a- ‐l1)(a
)(a- ‐3)3 )=: 0
0 positive.
positive.

If you
you had trouble factoring
had trouble this , remember
factoring this, remember e2 - 4(1)(3) > o
c2‐ 0
that you
that can always
you can always use the quadratic
use the quadratic
c2
e2 -‐ 12
12 > o
0
. 1
formula.
formula. Since < 1,a
Smee a < = g,, or 0.5.
1, a = 0.5.
2 2 > 12
cC2 > 12
4.
4. [iJExpandingeverything,
Expanding everything, Testing each of the
Testing each answer choices,
the answer only
choices, only
answer (A), ‐- 44,, gives
answer (A), value bigger
gives a value bigger than
than 12
(2x‐3)2
(2x - 3)2 = 44xx +
+5 when squared.
when squared.
4x2‐12x+9=4x+5
4x2 - 12x + 9 = 4x + 5 8. @ To find the intersection points,
the intersection treat the
points , treat
4x2-16x+4:o ttwo equations asa
w o equations system of equations.
as a system equations.
4x 2 - 16x + 4 = 0
Substituting the first equation
Substituting the equation into the second,
into the second ,
. . b b - 16
4:(x+2)2‐5
The the solutions
sum of the
The sum solutions IS_E
is - z - - 4- =
= ‐T : 4.a 4 = (x + 2) 2 - 5

99=(x+2)2
= (x + 2) 2
5.
5. lg!Move the left
the 8 to the
Move the side to get
left side get
i±S3 = x + 2
3x ++ 10x
l0x -‐ 8 = 0. N o w, we
Now, can either
we can use the
either use the
quadratic formula or factor
quadratic formula factor.. In this we’ll
case, we'll
this case, xX :=‐ -5 5,, 1
go w factoring: (x +
i t h factoring:
with + 4)(3x ‐- 2) = =0
The y‐coordinates
The y-coordinates of the intersection points
the intersection points
S orr xx == §.Smcea
o ,xx== -‐ 44 o
So, ~ . Since a > b,bmustbe
b, b must be must be
must be 4 (from the first equation),
(from the equation), so
so the
the ttwo
wo
points
points of intersection are (‐5,4)
intersection are and (1,4).
(- 5, 4) and (1, 4).
‐4
- 4 and b2 =
and 172 = (‐4)2
( - 4) 2 =
= 16.

314
THE COLLEGE PANDA
THE COLLEGE PANDA

9. [g Because
Because the vertex is at (3,
the vertex (3, -‐ 88),
) , the
the 14. 0 Substitute the first equation
Substitute the equation into
into the
the
must be either
answer must
answer (A) or (C).
either (A) Because the
(C). Because the second,
second,
parabola passes through
parabola passes (1,0 ), we
through (1,0), use
can use
we can
point to
that point
that to test
test oout w o potential
u r ttwo
our
ut o potential ‐3=ax2+4x‐4
- 3 = ax2 + 4x - 4
answers. When
answers. When we plug
plug in x = into (C),
= l1 into (C), we 0=ax2+4x‐1
0 = ax2 + 4x - 1
get y = 0,confirming
get 0, confirming that
that the answer is (C).
the answer (C).
the system
For the system to have one real
have one solution, the
real solution, the
the equation
From the
10. ) 2.5 ) From equation 0 v = St t2 =
5t -‐ t2 : t(5
t(S -‐ t),
equation above should have only one real
equation above should have only one real
we
we can see that
can see that the t-intercepts are
the t-intercepts are 0 and 5.
Oand 5.
solution.
solution . In other
other words, the discriminant,
words, the discriminant,
Because
Because the the maximum
maximum occurs the vertex,
occurs at the vertex,
b2 - 4ac, must
b2‐ must equal
equal 0.
whose t-coordinate is the
whose t-coordinate the average the two
average of the two
t-intercepts, t =
t-intercepts, t = 2.5 results the maximum
results in the maximum 2
4 (a)( - 1) =
(4)2 -‐4(a)(‐1)
(4) =0 0
value of 1).
value can confirm
v. You can this by
confirm this by graphing
graphing
4 a== 00
1 6++ 4a
16
the equation on
the equation your calculator.
on your calculator .
4aa== ‐- 116
4 6
I
11. ) 400 To find the minimum number
the minimum number of a=
= ‐- 4
mattresses the company
mattresses the company m sell so
u s t sell
must so that
that it
doesn’t lose money,
doesn't lose set P =
money, set = 0.
15. [ill We're
We're looking value of x that
the value
looking for the that
m2 -‐ 100m -‐ 120,000
m2 120,000 =
= 0o results
results in the minimum value
the minimum value of Jf (x). Since
Since
(m -‐400)(m + 300) = 00
400)( m +300) graph of
the graph
the offf is aparabola that opens
a parabola that opens
m= = ‐300,400
- 300,400 upwards
upwards in a U ” shape,
a ”"U" shape, the minimum of
the minimum offf
occurs at the
occurs the vertex, which is located
vertex, which located at
Since it doesn’t
Since doesn't make the number
sense for the
make sense number of xx =_-_baE=_- _- (24it) = 12. Therefore, the
12. Therefore, the
mattresses negative , m =
sold to be negative,
mattresses sold : 400. If
If 2a
2 2(l)
2 1
you trouble factoring
had trouble
you had the equation
factoring the equation above
above manufacturer should
manufacturer should produce
produce 1212 units each
units each
(it's tough),
(it’s graphing on
tough), graphing on your
your calculator
calculator and
and week minimize the
week to minimize cost per
the cost per unit.
unit.
the quadratic
the quadratic formula both good
are both
formula are good
alternatives..
alternatives 02]Since xx = O
16. El Since and x = b
0 and are x-intercepts,
bare x-intercepts ,
f (x), the
the data transfer speed,
data transfer speed, is 0 at x = 0 and
Oat and
12. [g The
The number 10,000 is they-intercept,
number 10,000 the y-intercept, the
the x= First, why
: b. First, why would data transfer
the data
would the transfer
total monthly
total monthly expenses when x, the
expenses when the number
number speed
speed bebe 0 at x =
Oat O? Well, the
: 0? the file transfer
transfer is
of tables,
tables, is 0. We can
can assume
assume these
these expenses
expenses just starting so no megabytes have
just starting so no megabytes have been been
to be rent, equipment,
be rent, equipment, worker
worker salaries,
salaries, etc.
etc. transferred yet. Now
transferred yet. why would
Now why would the speed
the speed
13. [[] We need to complete
We need the square.
complete the First
square. First bee 0Oat
b = bb?? The
a t xx = The best n S W e r iis
best aanswer that the
s that the file
transfer
transfer has just completed,
has just completed, so so there are no
there are no
divide everything
divide everything by
by -‐ 1,
megabytes of data
more megabytes
more left to
data left to transfer‐just
transfer - just
-‐y=x2‐6x‐20
y = x2 - 6x - 20 like car's speed
like a car's speed would
would bebe 0 when it stops
Owhen stops at
at
the end
the end of a a trip.
trip. Therefore, most likely
Therefore, b most likely
N o w divide
Now divide the middle term
the middle term by 2 to get -‐33
to get represents
represents the time at which
the time which thethe file transfer
transfer
and square
and square that
that result
result to get
get 9. We put
put the -‐33
the completed.
completed.
the parentheses
inside the
inside with x and
parentheses with and subtract the
subtract the
9 at the end.
at the end. 17. [g Since g(x)
Since g( x) is aa parabola and x =
parabola and =O0 and
and
x=
= c are its x-intercepts,
are its = ~g is the
x-intercepts, x = the
= (x‐3)2‐20‐9
-‐ y = (X - 3)2- 20 - 9
parabola’s axis of symmetry,
parabola's axis symmetry, thethe line along
line along
N o w simplify
Now and multiply
simplify and multiply everything
everything back
back which
which the vertex lies. In this
the vertex case, the
this case, the vertex
vertex
by
by -‐1.
1. is where
where the
the maximum since the
occurs, since
maximum occurs, the
y: ‐(x‐3)2+29 parabola
parabola opens
opens downwards
downwards in an an
upside-down U ” shape.
upside-down “"U" Therefore, ~
shape . Therefore, %is
is the
the
time at which
time the data
which the transfer speed
data transfer speed w a s at a
was
maximum..
maximum

315
CHAPTER 30 ANSWERS TO THE EXERCISES

18. [Q]One of the x-intercepts is 3. Since the


x-coordinate of the
x-coordinate vertex, 5,
the vertex, must lie at
5, must at the
the
midpoint
midpoint of the w o x-intercepts,
the ttwo the other
x-intercepts, the other
x-intercept Therefore , k = 7,
x-intercept is 7. Therefore, 7, giving
giving us
us
y== a(x -‐ 3)(x
3)( x -‐ 7). We
We can
can n o w plug
now plug in the
the
vertex as a point
vertex as point to solve
solve for a.

2 = a(5 -‐ 3)(
‐- 332 3)(55 -‐ 7)
2 = a(2)(‐2)
‐- 332 a(2)( - 2)
2 := ‐- 44a
‐- 332 a
a == 8B
a

19. [I] Substituting


Substituting the point (3,
the point (3, k) into
into both
both
equations,
equations,

( 3 ) +b
k :=22(3)
k=(3)2+3b+5
k = (3)2 + 3b + 5

This system of equations.


This is a system equations. Substituting
Substituting the
the
first equation the second,
into the
equation into second,

2 ( 3+
2(3) (3)2 + 3b + 5
) +bb=: (3)2+3b+5
+ b := 9 +
6+ + 3b
3b + 5
+bb 2=33b
6+ b ++114
4
-‐ ‐88=
: 22b
b
b=
: ‐- 4

From
From the first equation,
the first equation,

k = 6+
+ bb == 66 -‐ 4 = 2

316
THE COLLEGE
THE PANDA
COLLEGE PANDA

Chapter Synthetic Division


Chapter 18: Synthetic Division
CHAPTER EXERCISE:
CHAPTER EXERCISE:

l.
1. [g 5. [TIzz ‐- 11is
5. is aa factor only ifif the
factor only the polynomial
polynomial
yields
yields O when z2 =
0 when = 1 (the
(the remainder
remainder theorem).
theorem).
4 Therefore,
Therefore, we can set
we can up an
set up equation.
an equation.
X - 2 I4x
x ‐ 2 4x 2(1)3‐kx(1)2+5x(1)+2x‐2=0
2(1) 3 - kx (1) 2 + 5x (l ) + 2x - 2 = 0
4 x -‐ 8 8
4x l 2 ‐kx+5x+2x‐2=O
2 - kx + 5x + 2x - 2 = 0
3
8 -‐ kkxx +
+ 7x
7 x:= 0
0

This result can


This result be expressed
can be expressed as
as44+
+ _‐xfB_2 From
From here,
here, we can see
we can that k =
see that 2 7.
7'
x- 2
6. [2J
6. E ByBy the remainder theorem,
the remainder the remainder
theorem , the remainder
z- ‘ is(‐4)2+2(‐4)+1=16‐8+l=9.
is (- 4) + 2(- 4) + 1 = 16 - 8 + 1 = 9.
2

3 x ++ 1
3x 7'‑
7. []]

2 x++11I6x
2x 6 2x 2+ +5x
5 x++ 22 ‘ xX ‐
-
2
2
6x 2
6x + 33xx
+ 3x‐23x2 x -‐4 4
3x - 2 j 3x 2 ‐ 8Bx
2x +
2x + 2 3X2 _ 21
3x 2 - 2x
2x + 11 ‐ 6x ‐ 4
6x 4
11
-‐ 6x + 4
6x + 4
This result can
This result be expressed
c a n be as
expressed as , ‐ 8
8
1 .
3x + 1
1 + -m, , from Wthh Q
from which Q== 3x
3x +
+ 1.
1. This
This result can be
result can be expressed
expressed as
as
2x + l
8 ..
3.
3. @]
[E] This question is asking
This question asking you to divide
you to divide the
the xx -‐ 2 x _ , from
2 -‐ 3x
3 _ 2'
2
from which
Whmh A A= _ xx -_ 2.
2'
expression
expression by 2x -‐ 1l and
and write result in
the result
write the in . ' _ _ . _
the form of
the form of 8. !iD
8. El This
This question is asking
question 15 asking you diVide the
to divide
you to the
Dividend
Dividend =: Quotient
Quotient xx Divisor
Divisor + Remainder. expression by xx +
expression by + 11 and
and write the result in the
write the result in the
+ Remainder.
‘ form
form of
2x +
2x + 11 , Dividend
Dividend =: Quotient
Quotient x><Divisor + Remainder
Divisor + Remainder..
2 X- ‐ 11j 4x
4X 2 -l-5 2 x ‐ 6
2x + 2x - 6
2 r
4x 2
4x‐‘2x - 2x
xx ++ 11j 2x
2 2x 2- ‐4x
4 x- ‐ 33
2x +
2x + 55 2x22 + 2x
2x
2x +
2x
2x -_ 11 -‐ 6x ‐ 3
3
6
6 1 -‐ 6x ‐
6x 6
6
Therefore, 4x22 +
Therefore, 4x +5= +1)(2x
= (2x + 1) (2x -‐ 1)
1) +
+ 66.. . 3
3
R2= 66..
Therefore, 2x22 -‐ 44xx -‐ 3 = (2x -‐ 6)(x
Therefore, 2x 6)(x +
+ 11)) +
+ 3.
3.
4.
4. [I] Using the remainder
Using the the
theorem, the
remainder theorem,
remainder when
remainder when g(x)g(x) is divided
divided by x +
+ 33 is
is
equal to g ((-‐ 33)) =
equal tog = 2.
2.

317
317
CHAPTER 30 ANSWERS TO THE EXERCISES

9. ~ This question is asking you to divide the 14. [I] From the remainder
From the theorem, 3x -‐
remainder theorem, 11 must
must
expression
expression by x ‐- 22 and the result
write the
and write in the
result in the
form
form of be factor of p(x) ifif p
be a factor (1)= 0.0.
(g)
Dividend
Dividend == Quotient
Quotient ><x Divisor Remainder,
Divisor + Remainder,
where ax + b
where is the
bis the quotient and c
quotient and is the
c is the
remainder..
remainder

x
X +
+ 6
2
xX -‐ 221xx 2 + 4x
4 x- - 99
+
xx22 -‐ 22x
x
6x x - ‐ 9
6
6
6xx -‐ 112
2
3

Therefore, x2 + 4x - 9 = (x
Therefore,x2+4x‐9= ( x ++ 66)() (xx-‐ 22)) +
+ 33..
F a=
i n a l l y,
Finally, a =1,l ,bb== 6,
6 ,cc=
=3 a n daa++bb++ cc ==110.
3,,and 0.
10. ~ Using the remainder
Using the theorem, p(2) =
remainder theorem, =0
means that
means that x -‐ 2 is a factor of p(x).
a factor

11. ~ Use the remainder


Use the theorem to test
remainder theorem test each
each
option for a
option a remainder
remainder of 0.
p(2) = 23 + 22 - 5(2) + 3 =
=23+22‐5(2)+3 = 55..
p(1) = 13 + 12 - 5(1) + 3 = 0.
: 13+12‐5(1)+3=0.
p(‐3) (- 3)3 +
p(- 3) = (‐3)3 + (‐3)2 5(- 3) +
(- 3)2 -‐ 5(-3) + 3 =z o.
0.
Therefore, p(x) is divisible
Therefore, divisible by x -‐ 1 and
and x + 3.

12. [E]
[ill ItIf p(x) is divisible then p(2)
2, then
divisible by x -‐
must equal
must equal 0 (the
(the remainder
remainder theorem).
theorem).
Testing
Testing each
each answer choice, only
answer choice, only choice
choice (D)
results when x =
results in 0 when : 2.
13. ~ Using the remainder
Using the theorem, we
remainder theorem, can set
we can set
up
up a system of equations
a system equations.. When
When the
the
polynomial
polynomial is divided
divided by
by x ‐- 1or
1 or x + 1,
1,the
the
remainder
remainder is 0, which means
which that if we
means that we let
let
p(x) denote
denote the polynomial, p(1) = 00 and
the polynomial, and
p(- 1)== 0-
p(‐1) 0.

a(1)4 + b(1) 3 - 3(1)2 + 5(1)


a(1)4+b(1)3‐3(1)2+5(1) =o
=0
{ a(‐1)4 + b(‐1)3
a(- 1)4 + b(- 1)3 -‐ 3(‐1)2 + 5(-
3(- 1)2 + 5(‐1) =o
1) = 0

+ b ‐- 3 +
a+ + 5 =
= 0
{ a ‐- b ‐- 3 ‐- 5 z= 0

Adding the equations


Adding the equations together,
together,

2 a- ‐ 66=: 0
2a
a= 3

318
THE COLLEGE PANDA
THE COLLEGE PANDA

Chapter19:Complex Numbers
CHAPTER EXERCISE:

.I(5‐3i)‐(‐2+5i)=5‐3i+2‐5i=7‐8i
1. [g (5 - 3i) - (- 2 + Si) = 5 - 3i + 2 - Si = 7 - Bi
2. I[[j
i (ii(i++11)) == ii22++i =i :-‐1l + ii
.Ii4+3i2+2=1‐3+2:0
3. [g i4+ 3i + 2 = 1 - 3 + 2 = 0
2

0 2 + 3i + 4i + 5i + 6i = 22 ++33ii ++44(-( ‐ 11))++ 5(-


4...2+3i+4i2+5i3+6i4 2 3 4
5 ( ‐i)i )++ 6(1)
6 ( 1 =) :44-‐ 22ii
S n daa++bb=
Sooaa=: 44,, bb == ‐-22,, aand : 22..
0
5.. (6+21)(2+5i)=12+30i+4i+10i2=12+34i+10(‐1)=2+34i
(6 + 2i)( 2 + Si) = 12 + 30i + 4i + 10i2 = 12 + 34i + 10(- 1) = 2 + 34i
Therefore, a 2
Therefore,a =22..
6..@3(i+2)‐2(5‐4i) : 33ii ++66 ‐- 110
[g 3(i + 2) - 2(5 - 4i) = 0+8 = -‐ 44 ++11
Bii = lli i

7..I3i(i+2)‐i(i‐1)=3i2+6i‐i2+i=
[[l 3i(i + 2) - i(i - 1) = 3i2 + 6i - i2 + i = -‐ 3 ++ 66ii-‐ ((-‐ 11))++ ii = ‐- 22++77ii
8..Ei93=(i4)23-i: (1)23. i = i
@]i93 = (i4)23. i = (1)23-i=i

0 (3 - i) 2 = 32 - 6i + ;2 = 9 - 6i - 1 = 8 - 6i
.-(3‐i)2=32‐6i+i2=9‐6i‐1=8‐6i
9.

10. 0 Deal with the exponents first: (- i)2 - (- i)4 = 1'2‐i4


1Z'Dealwiththeexponentsfirst:(‐i)2‐(‐i)4 = ‐-
i2 - i4 = 1 -‐ 11=: -‐22

11. E(5‐2i)(4‐3i)zzo‐151‐81+6i2:20‐231‐6=14‐23i
[[j (5 - 2i)( 4 - 3i) = 20 - 15i - Bi+ 6i2 = 20 - 23i - 6 = 14 - 23i

12
12.
~ i
!1 + ..!_
1
i2
1
.'Z|7+i‐2+i‐4_7‐1+1_7
G\l + ..!_
i4
1
i
1
= ! - 1+1 = !
i

Now multiply
Now both top
multiply both top and bottom b
and bottom byy i ttoo get~·~ getygz
I I
~ = ‐i- i
= 1L= I

.(1-3i)(3‐1’)_3‐i‐9i‐1~3i2
!Al (1 - 3i) (3 - i) _ 3 - i - 9i + 3;2 _ 3‐101'+3i2
13 L.:2J-- · -- - -----=-
3 - lOi +
- ----=--- 2
3
3 -‐ 1lOi
0 i-‐ 3
- ---,----,--
3_ 3i2 _
_ ‐ -1lOi
- --
0 i -2_ ‐- 1/.
+ ii)) (3‐i)_9‐3i+3i‐i2:
9 - 3i + 3i - i 10
_ ‐ _ ‐ _

. ((33+ (3 - i) 2 9-12
9- i “ 9 9‐ -( ‐(-11)) 10

14 .(2‐i).(2‐i)_22‐2i‐2i+i2_4‐4i+i2_4‐4i‐1_3~4i
0 2 2
(2 - i) ' (2 - i) = 2 - 2i - 2i + i2 = 4 - 4i + i = 4 - 4i - 1 = 3 - 4i = ~ -_ 2
§ !i
. ((22++ ii)) ((22- ‐ i) 2 + 22i i-‐ i 2 _
i ) _ 44 ‐- 22ii + 2 4‐12
4- i ‘ 44‐- ( (‐- 1
1) ) " 5
5 “5 5 5 5

IThe
15.. [[] The common
common denominator i)(1 + i).
denominator is (1 -‐ i)(1 i).

((44++ ii)( ) (11++ ii)) (2


( 2-‐ ii)( ( 4++ii ++ 4
) (11-‐ ii)) _ (4 4ii++ i 22) )++(2( 2- ‐ 2i 5 i- ‐ 1l ++ 2 -‐ 33ii -‐ 1 _ 44 ++ 22ii
2 i-‐ ii++i 2i2 )) _ 44 + Si
((1l ‐- i )i)(1 ) (ll ‐- i i)(1
( 1 + ii)+( ) ( 1++i)i )=‐ 11++ ii -‐ ii ‐- i i2
2 =
‘ ‐ ( ‐ 11))
1 -(- '
=- 2
2-
: 22 ++ ii
=

319
CHAPTER 30 ANSWERS
CHAPTER 30 ANSWERS TO
TO THE EXERCISES
THE EXERCISES

Chapter20: Absolute Value


Chapter Value
CHAPTER
CHAPTEREXERCISE:
EXERCISE:

1.‑
1. []]

1|f(1)|
/ (l ) I = 1- 2(1) 2 - 3 (1) + 11 = l- 4 1 = 4
=l‐2(1)2‐3(1)+1|:|_4|=4
2. ~ The
2. The best
best way to solve
way to solve this question is
this question is trial error.. If
and error
trial and If x
x := 3, for example,
3, for example, 1|2 2 -‐ 33|J =
= 1,
1, which
which isis not
not
greater than
greater than 5. This result indicates
This result indicates that we should
that we try larger
should try numbers . If
larger numbers. we continue
If we continue to to work o u r way
work our way
up, we
up, we would at the
arrive at
would arrive possible value
minimum possible
the minimum value xx =
= 8, in J2
results in
which results
8, which |2 -‐ BJ
8| =: 6.
6.

3. []J Only the expression


Only the expression in answer
answer (B) can equal -‐55 (when
can equal (when x = 11 or
= Because the
or 3). Because value of
absolute value
the absolute of
anything is always greater than or equal to O, the other answer choices can never reach ‐
anything is always greater than or equal to 0, the other answer choices can never reach - 5.5 .

Recall that
4. []] Recall that the graph of y = Jx
the graph V‐shape centered
[x]l is aa V-shape centered at the origin.
at the The graph
origin. The pictured is
graph pictured is also
also
V‐shaped but
V-shaped converges at
but converges at y =
= -‐ 22,, which
which means
means it hashas shifted down. Therefore,
units down.
w o units
shifted ttwo the equation
Therefore, the equation
the graph
of the graph is y =: lx Note that
le l -‐ 2. Note that y = Ix |x -‐ 21
2| shifts the graph
shifts the graph ttwo
w o units the right,
units to the right , N O T ttwo
NOT units
w o units
down..
down
5. [ill
5. IE Test each
each of thethe answer making sure
choices, making
answer choices, sure to include
include the the negative For example,
possibilities. For
negative possibilities. the
example , the
answer n o t (A) because
answer is not because when
when x =z 2 or -‐ 2,
2, Ix
[x -‐ 33]J is n o t greater
not than 10. However,
greater than However, Ix greater
[x -‐ 33]1is greater
than 10 when
than 10 when x = : -~8.
8.
6. @]Smart
Smart trial
trial and
and error
error is the way to find
fastest way
the fastest the bounds
find the bounds for x. The lower bound
The lower and the
for xxisis -‐88 and
bound for the
upper bound
upper bound is -‐ 44.. There
There are integers between
are 5 integers between -‐44 and (inclusive). If we wanted
and -‐88 (inclusive). this problem
do this
wanted to do problem
more mathematically, we
more mathematically, we could
could set up the
set up following equation:
the following equation:

-‐3 3 <
< +6< 3
x -+- 6
X <
Subtracting 6,
Subtracting 6,
-‐ 99 <
< Xx << -‐ 3
Since xx is an
Since an integer,
integer,
~ Xx ~g -‐ 4
-‐ 88 S

7. [ill
El In the graph of 1|f/ (x)I,
the graph (x)|, all
all points
points with negative y-values
with negative y-values (below the x-axis)
(below the are flipped
x‐axis) are the
across the
flipped across
x-axis. All points with positive y-values stay
points with positive y-values stay the same.. Graph
the same (D) is the
Graph (D) one that
the one shows this
that shows correctly..
this correctly

8.
8. [J] If nn is positive,
positive,

n -‐ 22 =
: 110
0
n = 12
n = 1 2

negative,
IIff n iiss negative,

n -~ 22=-: ‐ 110
0
n
11 == -‐ 8
s u m of these
The sum
The w o possible
these ttwo n is 12
values of 11
possible values 12+ = 4.
+ ( -‐ 88)) =

320
THE COLLEGE PANDA
THE COLLEGE PANDA

9. II] Make up a
Make up a number
number for x. Let's say x =
Let’s say = 3. Then b =
3. Then = 13
|3 ‐- 10] = 7,
101 = 7, and
and b = 77 -‐ 33 =
b -‐ x = 4. Using
= 4. Using o ur
our
numbers,
numbers, we’re looking for an
we're looking that gives
choice that
answer choice
an answer gives 4 when b = 7. The only that does so is
when b = 7. The only one that does so is (C).
one (C).
To do this question
do this mathematically, we
question mathematically, we have
have to realize that
to realize when x < 10,
that when 10, x -‐ 10
10 is always negative.
is always negative .
Therefore,
Therefore,

10 == -‐bb
x -‐ 10
x z= 10
lO ‐- b

Using substitution, b -‐ x becomes


Using substitution, becomes b -‐ (10 -‐ b) = 2b
b) = 2b ‐- 10.
. .
10. II] The
The midpoint 6!41 and
midpomt of 61
3.
and 6~
62 is
4
15the average: (6;
the average: (6!4 ++ 62)
6~)
4
//22 = 6!.
= 6;
2
The midpoint
The is !
midpoint is %away from the
away from the
4
boundaries of the
boundaries accepted range
the accepted range for the length of a
the length hot dog.
a hot So whatever
dog. So whatever h is, it must
h is, within !
be within
m u s t be %of
of the
the
4
midpoint:
midpoint:
1
}162‘<
_ _

[Q]
Smart trial
11. El Smart trial and
and error the fastest
error is the find the
way to find
fastest way the bounds
bounds for n. The lower bound
The lower bound for is -‐22 and
for n is and the
the
upper bound is 6.
upper bound 6. There are 9 integers
There are integers between and 6 (inclusive).
between -‐22 and (inclusive). If we wanted
If we wanted to do this problem
do this problem
more mathematically, we
more mathematically, we could set up
could set the following
up the equation:
following equation:

‐5 < n ‐- 2 < 5
-5

Adding
Adding 2,
< n << 7
-‐33 <
Since 71is an
Since n an integer,
integer,
-~22 g
:S n 5:S 6

[Q]The
12. E] The midpoint is the
and 410 is
midpoint of 400 and average: ((400
the average: 400 +
+ 410) /2=
410) /2 z 405. The midpoint is
The midpoint is 55 away from the
away from the
boundaries of the
boundaries accepted range
the accepted range for the length of a
the length roll of tape.
a roll So whatever
tape. So whatever Il is,
is, it must be within
m u s t be within 5 of
the midpoint:
the midpoint:
ll|l ‐- 405|
405 1 < 5
5

13. @ There
There are
are ttwo possible values
w o possible x, 33 and
values of x, and -‐ 11.. There w o possible
are ttwo
There are possible values
values ofof y, 11 and
and -‐ 55.. We get
We get
the smallest possible
the smallest possible value when x =
value of xy when and y =
= 33 and z -‐ 55,, in which case xy =
which case = -‐ 115.
5.

14. II] If |a|


laI << 1,
1, then
then by definition,
definition,
‐1
- 1 < a < l1
This means that
This means I I I is true.
that III Because a must
true . Because be aa fraction,
m u s t be a 2 < 1,
fraction, a2 1,so also true.
so II is also However, I is n
true. However, o t always
not always
1
true because when
true because negati ve, !
when a is negative, 5 is
is not than 1.
greater than
n o t greater 1.
a
.. ..
1o7 The m1dpomt 3
1 3 and
1 .. 3 1 . . i 1
15. ~ The midpomt of 11 2 1 1s
and 25 is the average: ( 1113 +
the average: 2 1) //22 =
+ 21 = 2. The midpomt is
The midpoint 1 away
15Z away from the
from the
4 4 4 4 4
boundaries
boundaries of the accepted range
the accepted range for the weight of a
the weight muffin . So
a muffin. whatever m
So whatever is, it must
m is, must be
1
within 1 of
be within of the
the i
midpoint:
midpoint:
1
lm-_ 221
Im | << _-4
4

321
CHAPTER 30 ANSWERS
CHAPTER THE EXERCISES
ANSWERS TO THE EXERCISES

Chapter 21: Angles


Chapter21:
CHAPTER EXERCISE:
CHAPTER EXERCISE:

@]
Using the exterior
1. E] Using angle theorem,
exterior angle theorem, 5. [}] Because
Because alternate angles are
interior angles
alternate interior are
equal, one of the
equal, one missing angles
the missing angles of the
the lower
lower
kk = ii +
+ jj triangle
triangle is also a:
also a:
1 4 0=: 50
140 5 0++j j
9 0==jj
90

2. [}] The missing


2. angle in the
missing angle the left triangle
triangle isis
180° -‐ 60° -‐ 50° = 70°
180° angle is an
70°.. This angle an
angle to the
exterior angle
exterior triangle on the
the triangle right.. So,
the right
using the
using the exterior angle theorem
exterior angle theorem,,

7 0== y ++440
70 0
3 0=: y
30

Since x is an exterior angle


an exterior angle to the lower
the lower
.a a + b +
3. []] + c + d is
isequal to the
equal to thesum ofthe
sum of the
triangle,
triangle,
angles of the
angles quadrilateral, as
the quadrilateral, below..
shown below
as shown
x = aa +
x= + bb

6. []] The angle at the


angle at top of the
the top triangle is
the triangle
180 -‐ 70
180 30 =
70 -‐ 30 = 80.
80. If we look at
we look the larger
at the larger
triangle, taking away
triangle, taking the top
away the angle gives
top angle gives
a + b.
a+
a +b=
= 180
180 -‐ 80 = 100
100

7. [f] The angles form a circle, which


angles form which means
means
they sum
they sum to 360
360°.
°.

angles of a
Because the angles
Because quadrilateral sum
a quadrilateral to
sum to +y =
xX + 360 ° - 45 ° - 80° =
=360°‐45°‐80° =235°
235°
360, the answer
360, answer is 360.
4 0. ‑
4. 8. [fJ Filling
Filling out
out the bottom triangle,
the bottom the
triangle, the
missing angle
missing 180° -‐ 60° -‐ 40° = 80°,
angle is 180°
x+
X +yy ++( (x
x ++yy))==1 180
80 which
which means the angle
means the across from
angle across the
from it in the
40 + + (40 y) = 180
4 0 + y + ( 4 0 + y ) = 1 80 upper triangle is also
upper triangle also 80°. Finally,
80°. Finally,
2y + 80 = 180
2y+80=180 2
z= 180° -‐ 445°
2180° 5 ° -‐ 880°
0 ° == 55°
2yz100
2y = 100
y z=S50
O The ttwo
9. [fil The angles form
w o angles form aa line, which means
line, which means
they sum 180°..
sum to 180°

((xx + 4400))+
+ xX ==1180
80
2 x+ 4400 z=1180
2x 80
2x=
2x =1140
40
xX ==7700

322
THE
THE COLLEGE PANDA
COLLEGE PANDA

10. [Q]We can figure out two angles within the 15. -Angleais
I260 IAngle a is equal to180 -‐ 60
equal to 60 = 120.
triangle: 100°
triangle: 100° and Because yy is an
and 50°. Because an Angle
Angle bbisis equal 40 = 140. Finally,
equal to 180 -‐ 40 Finally,
angle, we can
exterior angle,
exterior use the
can use the exterior angle
exterior angle a+ b = 120 + 140 =
a+b=120+140 =260.
260.
theorem to get
theorem get its value:
its value:

= 100 + 50 = 150
yy:100+50=150

n . III
Angles = Angles
Shaded Angles
Shaded Angles of Rectangle
Rectangle
+ Angles Quadrilateral
Angles of Quadrilateral
= 360 +
= + 360
=
= 720

12. @]
E] The angles of any
The angles any polygon
polygon sum
s u m to
180(n -‐‐ 2), where
where n is sides.
the number of sides.
the number
angles a hexagon (6 sides)
The angles of a hexagon sides) sum
s u m to
180(6 -‐ 2) =
180(6 hexagon is
the hexagon
= 720. Because the
regular, all angles
regular, angles have
have the same
same measure
measure..
Therefore, each
Therefore, angle is 720 7+ 6 =
each angle 120°..
= 120°
Finally,
FinalJy,
x = 120 -‐ 90 = 30
X 30

13. [Elb is an
[Q] an alternate
alternate interior angle to the
interior angle the 45°
angle, which
angle, means they’re
which means equal: bb =
they 're equal: = 45. aa
and c are
and alternate interior
also alternate
are also angles so
interior angles so
a=z c = 180 -‐‐ 45 Using these
45 = 135. Using values,,
these values
can see that
we can all three
that all three are
are true
true..
14. IIIThe two missing angles
two missing angles in the smaller
the smaller
triangle add
triangle up to 80°. The two
add up bottom
two bottom
angles larger triangle
angles in the larger triangle add up to
add up
70 =
180 -‐ 70 = 110. If
If we
we take thethe two missing
t w o missing
angles smaller triangle
angles of the smaller away from
triangle away from the
the
two bottom angles
two bottom angles of the larger triangle, we’ll
larger triangle, we 'll
end up
end with x +
up with + y.

x + yy ==11
x+ ll00 -‐ 880
0 ==330
0
i,
r.
I
I
I
t_

\ I

"'·
)
I.

' 323
323
\
CHAPTER ANSWERS TO THE
CHAPTER 30 ANSWERS EXERCISES
THE EXERCISES

Chapter 22: Triangles


Triangles
CHAPTER
CHAPTER EXERCISE:
EXERCISE:

1.
1. [f] Because
Because the hypotenuse
hypotenuse is always
always the 7. 0 side length
The side length of the square v'4:= 2.
square is fl
largest side, x +
largest side, + 5m u s t be
must be the hypotenuse
hypotenuse Draw height of the triangle
Draw the height triangle to create
create ttwo
wo
while
while atand
x and x ‐- 2 m be the legs. Using
u s t be
must Using the 30-60-90 triangles:
30‐60‐90 triangles :
pythagorean
pythagorean theorem,
theorem,

x2+(x‐2)2:(x+5)2

2. @ Using 30--60-90 triangle


Using the 30‐60‐90 triangle relationship,
relationsrup, 1 1
1 1
DC _ 53C
DC = _ E(10)
BC = _ 5.
(10) =
2 2
3. [f] Using
3. Using the triangle relationship,
45‐45‐90 triangle
the 45-45-90 relationslup,
xX = 6\/2.
6y1. The area of the
the triangle
triangle is then
then
4. [I]Triangles
Triangles ABE
ABE and
and DCE are similar.
DCE are similar . 1
1§(2)(\/§) : v'3
(2)( ./3) = \/5
Therefore,
Therefore,

CD
CD AB
AB 8. @ Let thethe height
height of the
the bottom piece be
bottom piece x.
be x.
5CE == BE
E The height
height of the cone and
the cone radii of the
the radii
and the
similar triangles
triangles as
circles form w o similar
form ttwo as shown
shown
6 AB
AB
below.
5 =Z 4T
- below.
3
8 = AB
AB

5.. Iss IIfIf ttwo


w o angles
angles have
have the
the same measure,
same measure, I?
then sides opposite
then the sides opposite them
them have
have the same
the same
length.
length. To get the largest
largest perimeter,
perimeter, we
we
choose the third
choose side to be
third side be 20. The perimeter
perimeter
then 15 + 20 + 20 =
is then = 55.
6 .‑
6. lli] Using the similarity,
Using similarity,

1 1+X
-
2 6
Cross multiplying,
Cross multiplying,

M 4 4 0 2x = 6
22 ++ 2x

Drawing
Drawing the height splits
the height the base
splits the into ttwo
base into wo xx 2
=2
equal parts of length
equal parts length 4. From 3‐4‐5
From the 3-4-5
pythagorean triple,
pythagorean triple, we know height is 3.
know the height
The area is then %(8)(3)
is then : 12.
~ (8)(3) =

324
THE COLLEGE PANDA
COLLEGE PANDA

9. I2.5 ITriangles
Triangles GEF and GHC are
and GHC similar.
are similar. 02]
13. [E We
We can the pythagorean
use the
can use theorem to
pythagorean theorem to
Solving
Solving for EF, find
find BC:

EF HC Ac2 + A32
AC 2 + AB 2 = BC2
: 13C2
EZFTG
EG HG
1222 + 9922 =
12 BC22
= BC
EF_10
EF 10
T2 ‘ 5? BC 2
225 := BC2
E
EFF== 4 15 =
= BC

Triangles A D P and
Triangles ADF are also
and GEF are similar. So,
also similar. Note that
Note this is
that this is a
a multiple
multiple of the
the 3‐4‐5
3-4-5
triangle.
triangle.
AD_ D _GE
A §
D
OFF _ EEF
F C

AD
A_D_z 2
= -
5 ‘4 4
5
5
AD‐§_2.5
AD =
2 = 2.5

10.‑
[g
10.
0 X _!!_
7T _57t
= 547T
225°
225 x180°_
180° A 9 B
4
N o w ACDE
Now similar to ACAB.
.6.CDE is similar .6.CAB .
sides of triangle
The sides
11. [f] The triangle DEF are 9
DEF are 9+6 = 1.5
6= 1.5
times longer
times longer than the respective
than the respective sides
sides of
triangle ABC.
triangle Therefore, EF = 9 x 1.5 = 13.5
ABC. Therefore, 13.5 2CE_ CB
CB
DE
DE ‐ AB
AB
and
and DF
OF = = 5 xx 1.5 == 7.5. The
The perimeter of
perimeter of
triangle
triangle DEF then 9 +
DEF is then + 7.5 =
+ 13.5 + : 30. g
CE_ 15
15
6 ' 9
12. [g IfIf W
BC is the side in the
shortest side
the shortest the isosceles
isosceles
triangle,
triangle, then AB == AC
then AB and [A
AC and L A is
is the
the Cross multiplying,
Cross multiplying,
smallest angle.
smallest angle . At the same time,
the same time, we to
w a n t to
want
maximize L
maximize AA so that AB is minimized.
A so that L B minimized . Noww N o 9(CE) =
: (15)(
(15>(6)
6)
the angles
if all the were 60°, then
angles were the triangle
then the triangle
CE = 10
10
would
would be beequilateral and EE
equilateral and wouldn't be
BC wouldn’t the
be the
shortest
shortest side.
side . So need to decrease
So we need decrease L [AA to
Draw the
14. []] Draw extra lines
the extra lines shown below and
shown below and
the next highest
the next highest option,
option, 50°, which minimizes
which minimizes
use the
use 8-15-17 right
the 8‐15‐17 triangle.
right triangle.
L B to 130 + 2 =
£3 = 65°.
B w
W
17
65° 8

1--------- z
20 15

65° 50° 12
12
CL---------~A

X 15 y

325
CHAPTER
CHAPTER 30 ANSWERS THE EXERCISES
ANSWERS TO THE EXERCISES

15. [f] Draw


Draw an line to
extra line
an extra to complete the
complete the [fJThe
19. a smaller triangle
The smaller triangle in first quadrant
the first
in the quadrant
rectangle . Then use the 7-24- 25 right triangle. is a 3-4-5 triangle
a 3‐4‐5 triangle and is similar
and is similar to
to triangle
triangle
AOB. Using
AOB. the similarity,
Using the similarity,
28
3
OB _ 3
@ =-
15 ‘ 5
15
24
24 24
24 OB == 9

Therefore, n =
Therefore, = -‐ 99..
28 7 20. 0 radii extending
The radii extending to to the comers of
the comers of the
the
triangle split
triangle the circle into
split the three equal
into three equal parts,
parts,
244+
2 +2288+
+2255+ +2288 ==11
+7+ 2
112
so the
so measure of angle
the measure ADB is
angle ADB
16. ~ Using the pythagorean
Using the pythagorean theorem,
theorem, 360 ++ 3 = 120°. In radians, this is
radians , this
o
200 n
TT: _ 271'
2n:
120
l x 17305 _ ?.
X 180° = 3·
82+x2
2
=(x+2)2
8 + x2 = (x + 2) 2
64+x2=x2+4x+4
64 + x 2 = x 2 + 4x + 4
21. [fJ Because
Because triangle ABC is 45‐45‐90,
triangle ABC 45-45-90,
AB = 2\/2.
AB 2/2 . Because triangle ABO
Because triangle ABD is
is
644=
6 =4 4xx ++ 4
6 0=
60 : 44xx 2
30‐60‐90,
30--60-90, AD = ii;; and DB is twice
and twice that:
that:
1 5== Xx
15 \/§

DB _ 4_fl
4
= /2
17. [fJ Label what you
Label what know..
you know $3
v'3
B 2 XX 1 C

2ELl~
We can
We can rationalize the fraction
rationalize the fraction by multiplying
multiplying
1 top and
both the top
both bottom by $3:
and bottom y'3:
2 Y
W 2 zflxéz‘h/E
W
1
2
D B fi fi s
A 11ZZ 2 D
22. [i] Because the triangles
Because the are 45‐45‐90,
triangles are 45-45-90,
All triangles
triangles in the diagram
diagram are 45‐45‐90,
are 45-45-90,
which means wz
which means WZ = : XY = = Jz‘
/2 and
and = i.~. The
BC = the circle
radius of the
The radius is half
circle is BC:
half BC:
wx
WX = zr ZY = N2.2/2. The perimeter of wxrz
The perimeter WXYZ is \/§
then fi + + fi/2+ +
then /2 2 \2/2
/ § ++22/2
\ / § :=6 6/2.
\/§. ( ~) (é)
(é) ( ~) 2
= L2
~. Finally, the area of the
Finally, the area of the
18. [i] From the
From coordinates, AB =
the coordinates, = 7 and
and
circle is
BC = 7. Because
Because [LAABC right angle,
B C is a right angle,
triangle BC is aa 45-45-90
ABC
triangle A 45‐45‐90 triangle.
triangle .
the measure
Therefore, the
Therefore, measure of L BAC =
ABAC = 45°, 2
which
180°
= !:
182° =
which is 45° x _!!_ g radians.
radians .
4

326
THE COLLEGE PANDA
THE COLLEGE PANDA

23. GI]Outer
Outer AXYZ
6 XYZ and w WXZ
and 6 a 2 both
both have
have 36° [Q]Because
25. ED] equilateral triangle
the equilateral
Because the lies on
triangle lies on aa
angles and
angles and share 42. Therefore,
share L.Z. Therefore, they
they are
are side of the
side the square,
square, all their
their sides are equal,
sides are equal,
similar. Label
similar. Label the angles with
the angles marks ifif
with tick marks which means AA
which means BE and
6 ABE and A 6DDCEC E are
are isosceles.
isosceles.
you want to see
you want this similarity
see this clearly.
more clearly.
similarity more
B C
W and W
WZ and XZ in AWXZ
6 WXZ correspond with W
correspond with XZ
and W
and YZ in A6X respectively (sides
Y Z , respectively
XYZ, (sides
opposite
opposite the angles correspond
the 36° angles with each
correspond with each
other and
other and sides
sides opposite angles
largest angles
opposite the largest
correspond with each
correspond with So,
other) . 50,
each other).

W2 from AWXZ
WZ from 6 WXZ _ XZ from AWXZ
from 6 WXZ
xz
XZ from AXYZ
from 6 XYZ _ yz from AXYZ
YZ from 6 XYZ
A D
.. wz ‐ k,
WZ xz
XZ
Smce XZ = k, fiYZ must
Smee E also be
must also be equal
equal to k, [ A E D=
L.AED 2 60°, L.BAE ZCDE =
[ B A E = L.CDE : 30°, which
which
means
means
which means XE
which means ~~
XZ (the reciprocal) is equal
(the reciprocal) equal to A B E =
[L.ABE = é
L.AEB L.DCE =
A E B == ADCE D E C =
= [L.DEC : 75°.
75° .
Finally,
Finally,
1
1
k'
k. L.BEC = 360° ‐- 75° -‐ 75° -‐ 60° = 150°
ZBEC 150°..

24. [[I Let


Let AD = x. Because ADE is a
Because ADE 30‐60‐90
a 30-60-90 I I
Since TC
26. 3.75 Since is parallel
AC is parallel to fii
OD,
B A Cg
[L.BAC B O D and
~ [L.B00 L.BCA E
and ZBCA ~ ZBDO.
L.BOO.
triangle,
triangle, DE = xy13
DE = x 3 and AE = 2x. Note
and AE Note that
that Therefore,
Therefore, A
6 BBAC and ABOD_are
A C and 6 BOD are similar.
similar.
AADE,
6 ADE, 6 ABEF, and 6
BEF, and ADCF are all
DCF are congruent.
all congruent. Notice that
Notice that CD and that
OD = 8 and that AC and
and W
OD areare
B
corresponding sides. The
corresponding sides. The given lengths imply
given lengths imply

that
that the sides of ABAC
the sides are~2 the
6 BAC are sides of
the sides
ABOD.
6 BOD. N o w finding
Now finding the length of 35
the length BO will
will
allow
allow us
us to determine the length
determine the length of W.
BC.
Draw a vertical
Draw vertical line from point
line from down to the
point B down the
2x x-axis to form
form a right triangle
a right with 35
triangle with as the
BO as the
hypotenuse.
hypotenuse . The The base this triangle
base of this triangle is
8 ‐- 2 z= 6 and height is 8 (the y-coordinate
its height
and its (they-coordinate
of point This is aa 6‐8‐10
point B). This 6-8-10 triangle
triangle (a(a
A xX D
0 C
multiple of the
multiple 3‐4~5 triangle).
the 3~5 triangle) . If you didn’t
If you didn't
know this, you
know this, you could’ve also used
could've also used the
the
length of outer
side length
The side
The outer triangle ABC is 3x.
triangle ABC pythagorean theorem to find BD.
pythagorean theorem BD. Since
The
The side length of inner
side length inner triangle
triangle DEF x\/3.
DEF is xy13.
BD : 10, BC= gum) =
3
é(10) == 3.75.
BO = BC = ~(BD) = ~(10)
Because the
Because w o triangles
the ttwo are similar,
triangles are the ratio
similar, the ratio
of their
their areas
areas is equal
equal to the
the square
square of the
the
ratio of their
ratio sides:
their sides:

Area ofA
Area of 6D E F _ (x/3)
DEF (x\/3)22 __ 3x
3x22 _a 1
1
Area of AABC
Area 6 ABC _ (3x)2 " 9x2 _
(3x) 2 - 9x 2 - 3

327
CHAPTER
CHAPTE R 30 ANSWERS THE EXERCISES
ANSWERS TO THE

IE]Draw
27. ~ Draw a a straight
straight line down the middle.
line down middle . 29. ~
length of this line
The length
The because the
line is 9 because top
the top
part simply a radius
part is simply radius of the
the semicircle,
semicircle, y
whose length
whose half the side
length is half side of the square,
the square,
+ 2 = 3.
6 --c-
A

Ah
3 B Draw the extra line shown above to form a
30‐60‐90 triangle
30-60-90 (the sides
triangle (the are in aa ratio
sides are ratio of
Using the
Using pythagorean theorem,
the pythagorean theorem,
11:: x/3
v'3:: 2). The acute angle
The acute the line
angle the segment
line segment
92 + 3§2 =
¥ = AAB§2 forms
forms with
with the x-axjs is 30°, which
the x-axis which makes
makes
0 =
9 z 360 -‐ 30 30 =: 330°. Inin radians,
radians, this
this is
A §2
90 = AB
% H _ 117r
3300
330° X x _!!__ = 117r_
180° ‘ “ 6Z “
180°
JT=AB
V90= AB
square, DB = 3 and
DBCE is a square,
Because DBCE
30. j 3.75 j Because and
3JE=AB
3M = AB
triangles
triangles ABDABO and
and DEO similar (their
are simjlar
DEO are (their
angles
angles areare the same) . Using
the same). pythagorean
the pythagorean
Using the
28. @]
E' Draw height from
Draw the height A as
from A asshown below.
shown below. theorem, D0
theorem, DO = 5.5. Using
Using the similarity,
the simjlarity,
AADB turns
.0.ADB out to bea
turns out triangle.
30‐60‐90 triangle.
be a 30-60-90
AD _ DO
£
A D = @
DBB _ O
OEE
£3
AD 5
3
3 ” 44
5
15
A D =z 44‐ =
AD z 33B
..75

'---- -- -----""'- C
D 3 B 6

The area
The is ~%(6)(3\/3)
area is (6)(3v'3) 9\/3
= 9v'3

328
THE COLLEGE PANDA
THE COLLEGE PANDA

31. [g 34. [g Notice that both outer triangle RQT and


triangle
triangle QST
QST have right angles
have right and share
angles and 4 T.
share L T.
A B Because they have the same angle measures,
Because they have the same angle measures,

I 4 F
they are similar.
they are

R
similar.

12

D cC Q
Q 15
15 T
T

Triangles ADE and


Triangles ADE and FBEFBE are are similar. The sides
similar. The sides Since and W
ST and
Since 5‐7“ QT in triangle QST correspond
triangle QST correspond
of triangle
triangle ADE
ADE are times longer
are 3 times longer than
than the
the with CT
with and W
QT and RT in outer
outer triangle
triangle RQT,
RQT,
respective sides of triangle
respective sides triangle F BE.. Because
FBE Because respectively,
respectively, we can equate
we can the following
equate the following
triangle ABD a 45‐45‐90 triangle, the
triangle ABO is a 45-45-90 triangle, the ratios:
ratios:
length
length of BD l z fi . ifIf we
BO is 12\1'2. let BE
we let BE == x, then
then
DE
DE =: 3x.
3x. 2ST _ QTg
QT
QT -‘ RT
RT
x+3x:12f2
x+3x = 12v'2 ST _ 15
$1 E
15 _ 17
15 17
4x = 12v'2
4x=12\/f_2
15
15
xX ==33V2
\/§ ST
ST =
z‐ 17(15) 13.2
(15) z~ 13.2
17

I I
32. 7.2 Triangle ABC is a 5-12-13 triangle 30, 31, 32, 33, 34, 35, 36, 37, 38, 39, or 40 ISince
35. I30,31, Since
(BC =
(BC = 5). Triangle ABC is similar
Triangle ABC similar to triangle
triangle DE
DE is parallel
parallel to
to E,
AC, ABDE
L'..BDE ~ L'..BAC and
’‐‘_~’ ABAC and
AED (the angles
AED (the are equal).
angles are Using this
equal). Using this BBED
[L'.. E ABCA.
E D~ L'..BCA . Therefore, 6 B0£ and
Therefore , ABDE and
similarity is tricky
similarity because the
tricky because w o triangles
the ttwo triangles A
6 BBAC are similar.
A C are The sides
similar. The sides of 6ABAC are
BAC are
have different
have different orientations. The following
orientations . The following is 6+4
one example
one example of a correct setup:
a correct setup: :4 = 149~ the
the sides 6 8D£, which
sides of ABDE, which means
means

AE AB the perimeter
the perimeter of ABAC,
6 BAC, p,
p, must also be
must also ~O as
be ‐14‐0 as
A_E_A_B
D
DEE ‐ BBC
C long
long as
as the perimeter of ABDE.
the perimeter 6 80£. Thus,
Thus,
AE 12
A_E_e
33 ‐ 55 10 10
4( 12) ~ p ~ 4(16)
(16)
AE
AE = 36
36
‐5 == 7.2
z2
5 30 ~ p ~ 40

where p is an
where an integer.
integer.
33. [fil Since AC and OF cut through three
parallel lines , TC
parallel lines, and D?
AC and are divided
OF are divided into
into
proportional parts. We
proportional parts. can then
We can set up
then set up the
the
following equation
following equation and cross multiply.
and cross multiply .

xX 2
4 X
x2
x2 = 8

xX = 2v'2

329
CHAPTER ANSWERS TO THE EXERCISES
CHAPTER 30 ANSWERS EXERCISES

Chapter23: Circles
Chapter Circles
CHAPTER EXERCISE:
CHAPTER EXERCISE:

l. [[)
1. E The circumference of the
The circumference the circle is 2m. The
2 m . The 6. I6O IBecause A G is formed
Because [LBBAC from the
formed from the
square divides
square divides the circle into
the circle into four equal arcs.
four equal arcs. endpoints
endpoints of aa diameter, its measure is 90°.
diameter, its measure 90° .
27rr
. 2rrr rrr
7rr Since n d AC = 2, AABC
Since AB = l aand .6.ABC is a
Therefore, the
Therefore, length of arc AP
the length APD is T = 7
D 1s
30-60-90 triangle
30‐60‐90 triangle and BAC = 60°..
= 60°
4 2 and LABAC

2. [Q]Finding
2. El Finding the radius of each
the radius each of the
the small
small 7.‑
7. [I]
circles,
circles,
nr2=367r
rrr 2
= 36rr
7rr2:97r
rrr 2 = 9rr
r =: 6
rr =
=3
The
The circumference
circumference of the circle is
the circle
The radius
The outer circle is equivalent
the outer
radius of the equivalent toto 2m = 27r(6)
2m = 12rr.
2rr (6) ~_‐ 127r. Because the equilateral
Because the equilateral
three
three radii the smaller
radii of the circles, 3 x 3 =
smaller circles, : 9. triangle splits
triangle the circumference
splits the circumference of thethe circle
The area
The area is then 7t(9)22 =
then rr(9) : 8lrr.
817T.
into 3 equal
into equal pieces, arc AB
pieces, arc fl is one-third
one-third of the
the
3. [I] First, find the
First, find the radius.
radius.
circumference: ~
circumference: %x 127r =
x 12rr : 4m
4rr .
m22 = 36rr
m 3671
8. ~ TheThe area the circle
area of the circle is
rr =
= 6 mr r =
7 = rr(6 )2 =
7'r(6)2 = 36rr. shaded sector
The shaded
3671. The sector is
circumference of the
The circumference
The circle is
the circle lOrr” =
1 0n: o f the
‐ i S of t h e en . circle,
entire
tire . 1e, which
cue w h 1.ch means
means
2m =
2rrr : 2rr (6) =
27r(6) = 12rr. The perimeter
127r. The perimeter of one
one
if?
36 _ 18
18
5
region is made
region made up of ttwo
w o radii
radii and one-eighth
and one-eighth central angle
central angle AC
ACBB m u s t be
must be % of 360.
18
of the circumference.
the circumference.

1
6+6+;(12n)
6 + 6 + (12rr ) = 12 + l.Srr
=12+1.57t ~5 x 360°
1‐8 X 360o =
_ 100o
‐ 100°
18
8
Converting
Converting this to radians,
this to radians,
4.@
4.@]
0 rr Srr
7rr2:497r
rrr2 = 49rr lOO X 180 =9
rr2=49
2 = 49 We could've gotten
Wecould’ve gotten this answer directly
this answer directly by
r=: 7 sticking
sticking to radians. area of aa sector
The area
radians. The sector is
1
érze
r 2 8 when 9, the
when 8, measure of the
the measure central
the central
standard form
The standard
The a circle with
form of a with center
center 2
(h,k)) and
(h,k h) 2 +
radius r is (x -‐ h)2
and radius k) 2 =
+ (y -‐ k)2 = rr2.
2
. angle,
angle, is in radians
radians..
So the equation
So the equation of the
the circle
circle is
(x + 2)2 +y2
2)2 + y 2 = 49. 1
-$39
2
r 8=
=107r
lOn:
2
5. [g The measure of AB
The arc measure XE is twice the
twice the 1
measure of the
measure inscribed angle.
the inscribed Therefore,
angle . Therefore, 2
1(6)26
-( ‐_1 lOrr
6)28 = 07r
2
-A 60°
60° 11
AB =
AB : 60,wh1chis
° which
60°, ' is' ‐360°

3600
= ‐60 f t he
6 of the
189 = lOrr
188 = 1071
circumference.
circumference. 5
80=: gn
- 1[
9

330
330
THE COLLEGE PANDA
THE COLLEGE PANDA

9. I4, 5, 6, or 71The arc length can be @]Circle


12. E] Circle P and
and circle U LI each have an
each have an area
area of
determine
determined by rOwhen
r0 when 0,
0, the measure of the
the measure the 7r(3)22 =
rr(3) = 9rr. get the
971. To get shaded region,
the shaded we
region , we
angle, is expressed
central angle,
central expressed in radians.
radians. need to subtract
need subtract out
o u t the unshaded portions
the unshaded portions of
Therefore, the arc length
Therefore, the m u s t be
length must greater than
be greater than both
both circles. Because
Because b. APH
PHU U is equilateral,
equilateral,
5 (g) ~z 3.92 and
(i) less than
and less i) ~z 7.85.
than 5 ((g)
AH
L PU and
HPU
the
and AH
the unshaded
L HUP are both
UP are both 60°, which
are each
sectors are
unshaded sectors
which means
each one-sixth
means
one-sixth of
Wecould’ve
We done this
could've done question by converting
this question converting their respective circles (60° is one‐sixth
their respective one-sixth of
radians degrees but
back to degrees
radians back but the process
the process 360°).
would’ve taken aa lot longer.
would've taken longer .
1 1
97r 9rr ‐-
+971
9rr + g(97T)‐%(97t)
(9rr) - (9rr) =
=157r
15rr
@]Draw
10. E] Draw aa square
square connecting centers of
the centers
connecting the 6 6
each circle:
each
13. [!] Let y be the angle at the top of the triangle .
7rr -_ _Jf._
_y rrr 22:= 247f
360m
2
rrr 247r
360

rr(6) firms?
7r(6)22 -‐ L
360
rr(6) 2 =
= 2471
24rr

36 -_ y_
36 l = 24
10 24
10
To get
get the
the shaded region, we
shaded region, we need
need to subtract
subtract
out the
out four quarter-circles
the four quarter -circles from
from the square..
the square _ y_
12 = 1
The square
The has an
square has area of 8 x 8 =
an area = 64. The
The 12 _ 10
10
four quarter-circles
four make up
quarter-circles make up one with
one circle with 120 =
: y
an area
an 7r(4)22 =
area of rr(4) = 16rr. The area
167i. The the
area of the
shaded region is then
shaded region then 6464 -‐ 167f.
l67r. If
If y is 120, then and x have
then x and have to add
add up to 60.
Therefore, x =
Therefore, = 30.
11. [£] Unraveling the cylinder
Unraveling the gives aa rectangle
cylinder gives rectangle 14. ~ From information given,
the information
From the given, AB =: 8,
with aa base
with equal to the
base equal circumference and
the circumference and a
a BC == 4, and
and because tangent to circle B,
because AC is tangent
height equal
height to the
equal to the height
height of the cylinder:
the cylinder: L ACB is aa right
ZACB angle. Using
right angle. the
Using the
pythagorean theorem
pythagorean theorem to find A C,
AC,

AC2 + 42 =
+42 = g2
82
h
AC2=48
AC 2 = 48
27rr
2rrr A c :=44/3
AC \/§
The surface area
The surface area of the cylinder is
the cylinder is equal
equal to the
the The area
area of
area of this
area rectangle plus
this rectangle plus the areas of the
the areas two
the two
at either
circles at either end.
end . A
b.A B C = %(AC)(BC)
ABC = !l (4v'3
!( AC )( BC) = (4\/§)(4) = 88\/§
)( 4) = v'3
2 2
2m 2 = 27r(4)(5)
27rrh + 27'tr2
2mh 27r(4)22
2rr(4)(5) + 2rr(4) 15. [!]The
The circle has center ( -‐ 22,, -‐ 44)) and
has center and radius
radius
= 4071+
= 40 7f + 327r
327f 2. If
If you
you draw this circle oout,
draw this you’ll
u t , you 'll see that
that
it’s tangent only
it's tangent only to the y‐axis.
the y-axis.
=
= 727f
7271

331
331
CHAPTER THE EXERCISES
ANSWERS TO THE
CHAPTER 30 ANSWERS EXERCISES

24: Trigonometry
Chapter 24: Trigonometry
CHAPTER EXERCISE:
CHAPTER EXERCISE:

1.
1. 0 cos x = sin
Since cos
Since (90 -‐ xx),
sin(90 ), 5. [f] After
5. After drawing right triangle,
the right
drawing the triangle, we let
we let
cos 40°° = sin50
cos40 sin 50°° = a. the opposite
the be m
side be
opposite side m and the adjacent
and the side
adjacent side
‘ be 1.
be 1.
tan x = 0.75 =
Since tan
2. []] Since z ~,
2, we can draw aa
can draw
right triangle such
right triangle such that opposite side
the opposite
that the is 33
side is
and the
and adjacent side
the adjacent 4.
side is 4. m
m

I
1
3
Using the
Using pythago;ean
the pyth✓ theorem, the
orean theorem, the
l hypotenuse
hypotenuse is \/ m + 1.
m2+ 1. Therefore,
Therefore,
4 . ,. m
ss1nx=
m x ~= ‐ .-
vm 2 + 1
Using the
Using pythagorean theorem,
the pythagorean theorem, thethe Vm2+ 1
hypotenuse
hypotenuse is 5 (this is a 3 4 ‐ 5 triangle)..
is 5 (this is a 3-4-5 triangle) 6. [I!] The
The fact that AB = 55 is irrelevant
that AB irrelevant since
since
Therefore, cos
Therefore, 2 4f =
cos xx = = 0.8
0.8 the ratios
the ratios of the sides will
the sides will always bebe the same
the same
5 for proportional triangles. Instead
proportional triangles. Instead of actually
actually
3, [QJ
3.
El Since
Since sin 9 2 cos (90 _ 9) and
sin 0 = cos (90 - 0) and
trying
trying to to figure thellengths
out the
figure out lengths of
of the Sides,
the sides,
_ . let 5 use
let's use aa triangle that'ss easier
triangle that to work
ea51er to With..
work with
cost) =
cos0 sm(90
‐ sin 9),
(90 -‐ 0),
B
sine0 + cos(90
sin cos (90 -‐ 0) + cos
cosG0+
+sin(90 9) =
sin (90 -‐ 0)
0 + sin 0 + cos 0 + cos 0 =
sin0+sin9+c056+cose
sin
sin 0 + 2 cos 0
225in9+2cos€ 3
3

4. I25 IDrawing
4. Drawing the triangle,
the triangle,
A 4
4 C
A
Using the
Using pythagorean theorem,
the pythagorean BC =
theorem, BC : 5 (it's
(it’s
a 3‐4‐5 triangle).
a 3-4-5 triangle) .
30
..mBB + c o sBB ‐ 4§+§
4 33 ‐ 5
77 ‐ 1 . 4
sSm + COS = S + S =S = 1.4
C
C B
7‑ [IT]
7.

5
cosA = 5
cosA = ‐
6 l
.
S 'l n x =
smx
1
= -_
4
6 4
AC z §5
A_C i3 _ 11
30 6 BC -4
BC 4
AC =
= 25
25
Cross multiply
Cross multiply to BC:= 12.
get BC
to get

332
COLLEGE PANDA
THE COLLEGE PANDA

r:;-i ..
5
8. , 1 / Since tan B = 2.4 =
12
, we can let 7 SmcecosxO=
13. L.:JSmcecosx 0 ‐3
= 4'AB
AC
AC = ‐3 .
- , ‐- Since AC z= 6,
=-.4.SinceAC 6,
3 5 4 AB 4
4 4
AC
AC = 12and
= 12 AB = 5. Using
and AB Using thethe pythagorean
pythagorean AB
AB =
= §(AC) = §(6)
(AC ) = (6) = 8. Using
= 8. Using the
the
theorem, BC
theorem, BC 2 = 13. Since the w o triangles
the ttwo triangles are
are 3 3
pythagorean
pythagorean theorem,
theorem,
similar,
similar,
./28= 2/7.
JB 2 - 62 = \/64‐3
BC = \/82-62
BC= ,/~64--- 3-6 := \/2_8=2\/7.
AB
AB 5 Therefore, = 7.
Therefore, k =
cos N
N = :cosB
cos B =z -BC
‐ =z -13‐
BC 13 14. j 12.s INotice that triangles
Notice that triangles ABC and DBE
ABC and are
DBE are
similar,
similar, which means angle
which means angle BAC equivalent
BAC is equivalent
to angle the tangent
BOE . Since the
angle BDE. tangent of L'. A C is
[ BBAC
9. [!Ij Since COS X = sin (90 - x),
the tangent
1.25, the tangent of é B D E is also
L'.BDE also 1.25.
cos 32°° = sin
cos32 Setting up
sin 58°. Setting up an
an equation,
equation,

sin 58 =
: sin(5m
sin (5m -‐ 12) Opp
adj =: 1.25
op~
adJ
588 =
5 = 5Sm
m -‐ 112
2
BE
BE
700 =5Sm
7 m BE
DE '‐
= 1.25
1.25
m= = 144
1
BE (1.25)( DE) := (1.25)(1o)
BE = (1.25)(DE) (1.25)( 10) = 12.5

10. @]From the coordinates, AB = 5 - ( - 3) = 8


@][L'.AABC
15. [E measures 90° because
B C measures because it’s
it's
and BC
and 12 - ((-‐ 33)) =
BC := 12‐ : 15. Using the
Using the
inscribed in aa semicircle.
inscribed Therefore, triangle
semicircle. Therefore, triangle
pythagorean theorem to find AC,
pythagorean theorem AC ,
ABC is a
ABC a right triangle. Let the height
right triangle. height be AB
be AB
AC 2 =
AC2 AB 2 +
= A32 BC2
+ 3c2 and the base be
and the base be BC. Since the hypotenuse
hypotenuse
AC
AC = 1, 1,
AC 2 = 82 + 152
AC2=82+152
sine
sin0 == AB
AB
AC 2 =
Ac2 : 2289
89
ccos0 =
ost) = B
BCC
A C =: 117
AC 7

. BC
BC _ 15
Finally,cosC
Finally, cos C ‐= E
AC ‐= 1‐7.. Area of
of triangle = ~%(BC)(AB)
triangle = ( BC )( AB )
17
. o . 0o . 0o 8 8 1 .
@]Since
11. @Smcecos(90°cos (90° ‐x- x 0 ))== ssin mx =
m x ,, ssin _‐_ fi' . = 1§(cosl9)(sm
= (cos0 )( sin09))
17
50
So we
we can the side
can let the opposite the
side opposite the angle
angle x bebe _ sin
sinGcosH
0cos 0
8 and the hypotenuse
and the hypotenuse be be 17. Using
Using the the _ 2
2
pythagorean theorem, the side
pythagorean theorem, side adjacent
adjacent to
angle
angle x has J17 2 -‐ 82
length \/172
has length 82 = 15. Finally,
Finally,
adjacent
adjacent 15
15
16. [g This question is basically
This question basically asking
asking for the
the
cosx 0o =-~--
: ‐ - : ‐ , quadrants
quadrants in which sin 9
which sin can equal
0 can equal cos 6. For
0. For
cosx hypotenuse
hypotenuse 17
17 them to beequal,
them be equal, they must
must have
have the
the same
same
12. [I) Draw a triangle . Since the sine of one of sign. That rules
sign. That out option
rules out option II since sine is
since sine
positive
positive in the second quadrant while cosine
second quadrant while cosine
the acute angles
the acute angle s is ?, J;, we can
can let the side
side is negative.
negative. 1n
both positive,
both
In quadrant
positive, and
sine and
quadrant I, sine and cosine
equal to cosine
sine is equal
and sine cosine
are
cosine are

opposite the angle


opposite the angle be v'3and
be \/3 the hypotenuse
and the hypotenuse when 0 := 45° (remember
when (remember your your 45‐45‐90
45-45-90
be
be 2. Using the pythagorean
Using the the side
theorem, the
pythagorean theorem, side triangle?).
triangle?). In
1n quadrant
quadrant III, sine and
lll, sine and cosine are
cosine are
adjacent the angle
adjacent to the has length
angle has length both negative,
both negative, and sine is equal
and sine equal to cosine
cosine
J2222‐- ((x/3)2
y'3)2 == l.1. Notice
Notice that this is a
that this a when 6
when 0 = 225° (this
(this is the third
third quadrant
quadrant
30-60-90 triangle.
30‐60‐90 triangle complete,
triangle. With the triangle complete , equivalent
equivalent of 45° in thethe first quadrant).
quadrant) .
the sine of the other
the sine acute angle
other acute then
angle is then
opposite
opposite _ 1
hypotenuse ‐- 2
hypotenuse 2'

333
CHAPTER 30 ANSWERS TO THE EXERCISES

Chapter 25: Reading Data


CHAPTER EXERCISE:

1. II] We estimate the total commute time for each point:

Point Commute Time


A 25 + 60 = 85
I B I
38 + 40 = 78 ‑
C 45 + 80 = 125
“‑ D 80 + 20 = 100

Even though
Even the times
though the times were
were estimated, it’s clear
estimated, it's that C represents
clear that represents the greatest commute
the greatest commute time.
time .

2. II] The vertical distance between the points at 2004 and 2006 is the smallest among the answer choices.
3. II] The points corresponding to July through September are the highest in both 2013 and 2014.

150 - 33 -
150 0
250 - 5 - 60 1/o

5. [I]
E San Diego is the
San Diego only city for which
the only which the
the estimated bar is lower
estimated bar than (to
lower than the left
(to the left of) the actual bar.
the actual bar.

6. [I]
1 Both graphs go
line graphs
Both line go downward
downward every year.
every year.

E The lowest
7. @The point with
lowest point respect to the
with respect the y-axis at a
y-axis is at a little 40 years
under 40
little under age.
years of age.

A The
8. [}] graph’s minimum,
The graph's minimum, 16, must
must be
be the weight of the
the weight the truck
truck when empty. The
when empty. graph’s maximum,
The graph's maximum,
be the
must be
30, must weight of the
the weight at maximum
truck at
the truck capacity. Subtract
maximum capacity. the two
Subtract the the truck’s
get the
t w o to get maximum
truck's maximum
capacity, 30
capacity, 16 = 14.
30 -‐ 16

9. II] From 2010 to 2011, the percent decrease was

~ -~ 1 .
¥
40
= ‐}1
: -
4 =z -‐25% (percent decreases
25% (percent negative)
are negative)
decreases are

From 2013 to 2014, the


From percent increase
the percent increase was
was

2 5-‐ 220
0 = !1 = 25ox
0
25 20
20
_Z_25/o
4 °

ag _z
l27 120
lO. li_J180 =
2
3
3 180 ‐ 3

11. IConsole
[[j A generated
Console A generated 250,000 :
250,000 x 100 = Console Bgenerated
$25, 000, 000. Console
$25,000,000. 2 5 , 000 x 150 =
B generated 225,000 : $33,750,000.
$33, 750,000.
Console D generated
Console 125,000 x 250 =
generated 125,000 Console E generated
$31,250,000. Console
: $31,250,000. 50,000 x 300 = $15,000,000.
generated 50,000 $15,000,000.
Console generated the
Console B generated the most revenue.
m o s t revenue.

12. [I) In
in Quarter
Quarter 3,
3, Company profit was
Company Y's profit about 66 million
was about million and Company X's
and Company X’s profit was about
profit was 12million
about 12 million
Company Y's).
(twice Company
(twice no other
Y’s). In no quarter was
other quarter Company X's
w a s Company profit as
X’s profit as close being twice
close to being twice Company
Company Y’s.
Y's.

334
THE COLLEGE
THE PANDA
COLLEGE PANDA
-
13.
13. [fJ Alabama spent a
Alabama spent a combined 15+
combined 15 + 2.5 =
2.5 = 17.5 billion. Alaska
17.5 billion. Alaska spent + 7.5
7.5 +
spent 7.5 = 15
7.5 = 15billion. Arizona spent
billion. Arizona spent
12.5 +
+ 7.5 =
= 20billion. Arkansas spent 10+ 5 = 15billion.
20 billion. Arkansas spent 10 + 5 = 15 billion. Arizona spent the most.
Arizona spent the most.

14. I44 / During first two


During the first two hours, Jeremy answered
hours, Jeremy answered 44 calls
calls per for aa total
hour for
per hour of 22 xx 4
total of calls. During
4 = 88 calls. During
the next three
the next Jeremy answered
hours, Jeremy
three hours, answered 8 calls per hour for a
per hour a total During the
total of 3 x 8 = 24 calls. During the final
t w o hours, Jeremy answered 6 calls
two hours, Jeremy answered per hour
calls per hour for a total of 2 x 6 =
a total z 1212 calls. He answered
answered a total of
a total
8+ 24 +
+ 24 12 =
+12 = 44
44 calls.
15. [}] From graph, we
From the graph, we can
can see that
that it takes Greg’s glucose
takes Greg's glucose level hours to return
level 2.5 hours return to its
its initial
initial value
value (140
mg/ dL) after breakfast and 8 - 4 = 4 hours
mg/dL) after breakfast and ‐ = hours return to return to its initial value
initial value (also 140 mg/dL) after lunch
mg/ dL) after lunch..

. 5=: 1.5
4 -~ 22.5

16. ~
16. E AtAt 30
30 miles per hour,
miles per hour, Car
Car X
X gets
gets 25 25 miles
miles per
per gallon
gallon.. Driving for 5
Driving for 5 hours
hours at
at 30
30 miles per hour
miles per hour covers
covers a
a
total distance
total distance of 5 x 30 = 150 miles
m i l e s .,

11gallon
gallon
150 miles
miles x . = 6 gallons
25 miles
25 miles : 6 gallons

335
ANSWERSTO THEEXERCISES
‘ f½PTER30
HAPTER 30 ANSWERS TO THE EXERCISES
\‘ t.

thapter 26:
‘/ 'Chaptet 2.6:Ptobability
Probability
CHAPTEREXERCISE:
CHAPTER EXERCISE:

1.‑ Stop sign violations committed by truck drivers = 39 ~ 0.433


M = 3.9. x 0.433
Stop sign
Stop violations
sign violations 90
90

2. ElThe
2. @] The percentage
percentage ofof silver
silver cars is 100
cars is 100 -‐ 20
20 -~ 33 = 23. Red
33 ‐- 10 ‐- 14 = and silver
Red and = 43
make up 20 + 23 =
silver make +
percent
percent of the
the cars.
cars.

3.@J
3. El
Plumbers with
Plumbers at least
with at least 4 years experience =
years of experience _ ‐ ‐083‐ +
_ 40,
40,083 + ‐45,376
45,‐376‐ z
~ 0.46
0.4
6
plumbers
All plumbers 183,885
183,885

4.‑
4. @
Plumbers with
Plumbers at least
with at least 4 years
with at least
Workers with
Workers
. _ _ ______ =
experience
years of experience
years of experience
least 4 years experience
= 40,
182,
083 + 45,
40,083
410 +
182,410 + 208,
376 z
45,376

757
208,757
~ 0.22

_
0 22 _ _ _ .

5.‑
5. Cg
Games won
Games asunderdogs
w o n as underdogs _ 10
10 _ 22
Games played
Games asunderdogs
played as underdogs 45 _ 9
45

6. CgFilling
6. Filling in the
the table,
table,

Week
W e e k l] Week2 \ Week3
Week2 Week3 Weel<4
Week4 Total
Total
springs
Box springs 35
35 40 20 55 150
Mattresses
Mattresses 47 61 68 22 198
Total 82 101 88 77 348

Box spring units sold during weeks 2 and 3 _ 40 + 20 _ 2


All box spring units
box spring units sold -_ 150 - 5

7.
7. @ For
For the
the USA,
. 15W
the probability
USA, the
. 29 x 0.28. For Russia, the probability is g z 0.39. For Great Britain,
probability is : ~ 0.28. For Russia, the probability is
1
~ 0.39. For Great Britain, !~
probability is
the probability
the
probability
probability is
!:
. . . 19
15@ ~
z 0.29.
is Russia.
Russia.
For Germany
0.29. For the probability
Germany,, the probability is
is ~ 2~ 0.32.
[11‐20.32. The country with
The country the highest
with the highest

8.
8. [I]
Cartilaginous fish
Cartilaginous fish species in the
species in Philippines Cartilaginous
the Philippines Cartilaginous fish species in
fish species in New
New Caledonia
Caledonia
Total fish
Total species in
fish species the Philippines
in the PhjJippines - Total fish
Total species in
fish species New Caledonia
in New Caledonia
: 400 _ 300
300 _ 1
1 1 1_ 22
-,--,-----------
400 + 800 3oo+1,2oo
400+8OO 300 + 1,200 ‘ 3 5 _55 _ E
15

336
336
‐‑
THE COLLEGE PANDA
THE COLLEGE PANDA

9. [g Filling
9. Filling in the
the table,
table,

_ Lightning-caused
Lightning-caused fires Human-caused
Human-caused fires Total
East Africa 55
ss 65
65 120
South Africa 30 70 100

‐_Total

Human-caused
Human
85

-caused fires in East


135

Africa _ 65 _ 13
East Africa 13
220

Fires East Africa


Fires in East Africa _ 120 _ 24
24

10. []]
Defective from Assembly
Defective from Assembly Line
Line A _@ -E
300 3
Defective
Defective ‘ 8800
00‘ 88

11. [Q]
Duplex with 2 family
Duplex with members or less_
family members less + 12_
22 + 12 17
17
=
Duplex
Duplex 46
46 23
23

The total
12. []] The number of samples
total number samples contaminated Chemical A is (450
with Chemical
contaminated with + (550 x 0.06) =
(450 x 0.08) + = 69.

Contaminated samples
Contaminated samples _
= ~ 69 = _
= 0.069
0 069
All samples
All samples _ 1,
1, 000

13. [fil The incorrect when


test is incorrect
The test gives positive
when it gives indicators for patients
positive indicators patients who don't have
who don’t the virus
have the and
virus and
negative indicators for patients
negative indicators do, a total
who do,
patients who = 80 occurrences.
total of 30 + 50 = occurrences.

80 8
‐ = =~=
~ ‐ : %
8%
1000 100 8

14. []] The number of patients


The number cured by
patients cured sugar pill
the sugar
by the 90 + 33 = 30. The
pill is 90+ number of patients
The number patients who weren't
who weren’t

cured sugar pill


the sugar
cured by the 30 x ;g =
pill is 30 z 75.

Cured
Cured Not cured
Not cured
Drug 90 25
Sugar Pill
Sugar Pill 30 75
Given aa sugar
Given and cured_
pill and
sugar pill cured _ 30
30 _2
=2
Given sugar pill
Given aa sugar pill =30
- 30 +
+ 75
75 7

I I
15. 240 Let
Let the number of seniors
the number who prefer
seniors who gym equipment
prefer gym equipment be
be x.

L X
_ 1
1
x
X + 160 _ 3
+

Cross multiplying,
Cross multiplying,
3x == xX ++ 160
2 =1160
2xx = 60
x :=880
X 0

There + 160 := 240 seniors


are 80 +
There are the school.
seniors at the school.

337
CHAPTER 30 ANSWERS TO THE EXERCISES

Chapter 27: Statistics I


CHAPTER EXERCISE:

1. [f] The sum


sum of the heights
heights in the class is 14
the first class 14 x 63 = 882. The
63 = sum of the heights
The sum second class
the second
heights in the is
class is
21 x 68 = 1,428. u m of the
1,428. The ssum heights in the
the heights the combined then 882 +
class is then
combined class 2,310. The
+ 1428 = 2,310. average
The average
height
height is
Sum of the
Sum the heights
heights = 2, 310 =
2,310
66
Total number +
students I 14+ 21 = 66
number of students 14

2. [f] The sum of all five of Kristie's test scores is 5 x 94 = 470. The sum of her last three test scores is
3 x 92 = 276. The difference between
The difference between these the sum
sums is the
w o sums
these ttwo her first ttwo
s u m of her w o test scores: 470 -‐ 276 =
test scores: =
. . 194
194. The average
average of her
her first ttwo scores is
test scores
w o test then ‐2- =
1s then = 97.
2
3. I]] A range of 3 days means the difference between the longest shelf life and the shortest shelf life among
the units is 3.
the units could be
3. This could be 10 days vs.
10days days or
13days
vs. 13 25days
or 25 28 days.
vs . 28
days vs. range says
The range
days. The nothing about
says nothing the
about the
mean or median.
mean median.
4. I]] Because there are 20 editors,
the median is the average of the 10th and 11th editors' number of books
read. From
read. the graph,
From the graph, the and 11th
10th and
the 10th editors both
11th editors both read
read between and 15
between 11 and books last
15 books which
year, which
last year,
means average must
the average
means the also be
must also and 15. The only
between 11 and
be between choice between
answer choice
only answer and 15
between 11 and is 12.
15is

5. [QJThe median score, designated by the line segment in the middle of the "box", is approximately 83.
The lowest individual
The lowest score, designated
individual score, segment at
line segment
designated by the line at the end of the
the left end plot, is approximately
the plot, approximately
difference between
72. The difference between them 83 -‐ 72 =
them is 83 = 11.
6. [fil
(18x6)+(19x3)+(20><5)+(21x4)+(22x2)+(23><3)+(24x1)
(18 X 6) + (19 X 3) + (20 X 5) + (21 X 4) + (22 X 2) + (23 X 3) + (24 X 1) :
24
24 = 24 ‐ 20.25
24 = 20.25

7. I67 IThe median is represented by the average of the 14th and 15th days, both of which are 67°F.

8. [QJThe standard deviation decreases the most when the outliers, the data points furthest away from the
mean, are removed.
mean, are The outliers
removed . The outliers here are the
here are Rhone and
the Rhone the Vosges.
and the

9. [QJBy definition, at least half the values are greater than or equal to the median and at least half the
values are
values are less than or equal
than equal to the median.
the median.

10. GI]Even though the frequencies


Even though are the
frequencies are the travel
same, the
the same, times themselves
travel times m o r e spread
are more
themselves are out for Bus
spread out
B. The travel times for Bus A are
travel times are much closer together.
much closer together . Therefore, the standard
Therefore, the deviation of travel
standard deviation times
travel times
for Bus A is smaller.
smaller .
11. [f] The median weight is represented
median weight represented by the kayak (47 pounds
10th kayak
the 10th pounds for both Company A and
both Company Company
and Company
B). The median weight
The median weight is the same for both
the same companies.
both companies.

12. I]] Arranging


Arranging the scores in order,
the scores order,
75, 83, 87, 87, 90, 91, 98
75,83,87,87,90,91,98

average iis
The average
The s‐7755‐++ 883
‐3++78877‐++ ‐88:7‐+
+ 9900 + 9911 +98
‐ +‐ ‐ +‐ 98‐ ~ mode iis 8
at: 87.3. The mode s7. The
87. i aalso
median is
The median slso 87. The
The
range
range is 98 -‐ 75 = = 23. From
From these
these numbers,
numbers, I is false, I] and IIll
lI is false, and I I is true.
true .

338
THE COLLEGE
THE PANDA
COLLEGE PANDA

13. I]]
(5x2)+(6x1)+(8x4)‐+‐(9><2)+(10><1)_§_
Mean = (5
Mean :
x 2) + (6 x 1) + (8 x 4) + (9 x 2) + (10 x 1) = 76 = 7.6
7.6
2 ++ 1 ++ 44 ++ 2 ++ 1 “ 1100 ‘
n g e= l10e- ‐5 =s 5t
Range

14. [E] Before the 900-calorie


[Q] 900‐calorie meal added, the
meal is added, the median the average
median is the average of the
the 5th 6th meals
and 6th
5th and (550), the
meals (550), the
mode is 550, and
mode and the range is 900 -‐ 500 =
the range z 400. After the 900-calorie
After the meal is
900-calorie meal added, the
is added, the median becomes
median becomes
the 6th meal
the 6th meal (still the mode
(still 550), the still 550, and
mode is still and the range is
the range is still
still 400. None of them
None of change.
them change.

15. [Q]
E] The median in School
The median School A is represented
represented by the class (4
10th class
the 10th (4 films) and and the median in
the median in School
School BB is
is
represented by the
represented 8th class
the 8th (also 44 films). The
class (also median is
The median is the same in
the same in both schools.. N
both schools o w we
Now we calculate
calculate the
the
means:
means:
Mean in School A = 1 x 2 + 2 x 3 + 3 xx 44 ++ 44 xx 5
5 ++ 55 xx s
5~
MeaninSchoolA : 1 x 2 + 2 x 3 + 3 19
19 z3.42
3 .42

MeaninSchoolB=1X1+2X2+31x53+4><4+5x5¢=1367
M . Sch oo l B = 1 x 1 + 2 x 2 + 3 x 3 + 4 x 4 + 5 x 5 ~ 67
ean m 3.
15
The mean is greater
The mean greater in School B.lntuitively,
School B. this makes
Intuitively, this sense because
makes sense because the for School
distribution for
the distribution A has
School A has aa
higher
higher proportion
proportion of the
the smaller numbers 1,
smaller numbers 1,2,
2, and asshown
and 3 as shown by the
the chart. These smaller
chart. These numbers pull
smaller numbers pull
down
down the
the mean.
mean .
16. I]] Before
Before the car
car is removed,
removed, the median is represented
the median represented by the
the 8th car (23 mpg).
8th car mpg). After
After the
the car is removed,
car is removed,
the median is represented
the median represented by the average of the
the average the 7th
7th and
and 8th (still 23
cars (still
8th cars mpg). So
23 mpg). So the
the median the
stays the
median stays
However, the
same. However,
same. mean and
the mean and the standard deviation
the standard both decrease.
deviation both removing aa data
We’re removing
decrease. We're data point higher
point higher
than all
than others so
the others
all the so the
the m e a n decreases.
mean decreases. We're also reducing
We’re also spread in
the spread
reducing the in the
the data so the
data so the standard
standard
deviation decreases.
deviation decreases.
17. [J] First, easy to see
it’s easy
First, it's that the
see that decrease since
will decrease
mean will
the mean we're replacing
since we're replacing the
the maximum data point
maximum data point with
with
a
a minimum.
minimum. Now N o w before the replacement,
before the range is
the range
replacement, the is 90
90 -‐ 45 = 45. After
45 = the replacement,
After the replacement, the range
the range
is 65
65 ‐- 20
20 = 45, so so the range remains
the range remains the same.
the same. Before the replacement, the median is represented by
replacement, the median is represented by
average of the
the average
the the 9th and 10th
9th and 10th cars After the
cars (57). After replacement, the
the replacement, the median is represented
median is represented by 10th car
by 10th car
(still 57, don't
(still forget to count
don’t forget count the replacement as
the replacement as the The median
value). The
the first value). median also remains the
also remains the same.
same.
Therefore, the
Therefore, mean changes
the mean changes the most.
the most.

18. ~ To construct
construct the box plot,
the box plot, we need know the
need to know minimum number
the minimum and maximum
number and number of lectures
maximum number lectures
given,
given, the median, the
the median, quartile, and
the first quartile, and the third quartile.
the third table indicates the
The table indicates that the minimum
quartile . The that minimum isis 12
12
and the maximum
and the maximum is 40.
The
The median
median is the average of the
the average 90+ 2 =
the 90-;- : 45th 46th professors.
and 46th
45th and professors. From the table,
From the we can
table, we up in
count up
can count in
order of the
order the number lectures given
number of lectures given and see that
and see that the 45th and
the 45th 46th professors
and 46th both gave
professors both 25 lectures,
gave 25 so
lectures, so
the median
the median is 25.
To find the quartiles,
find the quartiles, we use the
we use the median split the
median to split the data w o halves:
into ttwo
data into first 45
the first
halves: the 45 professors
professors and
and the
the
last 45
last 45 professors. Note that
professors . Note that since
since the median is
the median is the average of
the average the 45th
of the 45th and 46th professors,
and 46th it's already
professors, it’s already
"excluded” from either
"excluded" from either half.
half.
The first quartile
The quartile is the median of the
the median the first 45
45 professors,
professors, represented
represented by by the
the 45
45 7+ 2
2 : 22.5 ‐) 23rd
22.5 ➔ 23rd
professor.
professor. From the table,
From the the 23rd
table, the professor gave
23rd professor gave 15 lectures, so
15 lectures, the first quartile
so the quartile is 15.
is 15.
third quartile
The third the median
quartile is the median of thethe last 45 professors,
last 45 represented by
professors, represented the 45
by the 45 7+ 2
2 = 22.5
22.5 ‐+
➔ 23rd
23rd
professor
professor from 45th professor,
the 45th
from the professor, or the 45 + 23 = 68th professor overall . Again we count up in the table
or the 45+ 23 : 68th professor overall. Again we count up in the table
and see that
and see that the 68th professor
the 68th gave 28
professor gave lectures, so
28 lectures, so the third quartile
the third is 28.
quartile is
N
Now that we have
o w that have these
these statistics (min: 12, ql:
statistics (min: med: 25, q3:
q l : 15, med: q3: 28, max:
max: 40), we can see that
can see the correct
that the correct
boxplot
boxplot is the
the one
one in answer C.
choice C.
answer choice

339
CHAPTER ANSWERS TO THE
CHAPTER 30 ANSWERS THE EXERCISES

Chapter 28: Statistics


Statistics 1!
II
CHAPTER
CHAPTEREXERCISE:
EXERCISE:

1. [I] There are 2 points


There are above the line
points above line of best value along
when the value
best fit when the x-axis is 19.
along the

2. [I]Note that the sample


Note that size of 400 is irrelevant
sample size irrelevant information.
information. make things
To make easier, we’ll
things easier, be a
let xx be
we 'll let
decimal o w and
decimal for nnow convert itit to a
and convert percentage later,
a percentage later,

3, 300x =
3,300x : 66
x=
X z 0.02 = 2°/o
2%

3. Cf]The line
line of best gives a
best fit gives y‐value of 55
a y-value when the
55 when x-value is 75.
the x-value

4. Cf]First, be conducted
should be
survey should
First, the survey students from the university’s
with students
conducted with freshman class since
university's freshman that's
since that’s
intended target.
the intended target. Secondly, larger the sample,
Secondly, the larger more valid
sample, the more results.
the results.
valid the

5. GI]Using
Using proportions, Candidate A is expected
proportions, Candidate expected to receive ~~~ x 500,000
receive %3 500,000 =
: 220,000 votes. Candidate B
votes . Candidate

is expected
expected to receive ~:~ x><500,000 = 280,000 votes
receive £8 Candidate B is expected
So Candidate
votes.. So receive 280,000 ‑-
expected to receive
220, 000 := 60, 000 more
220,000 votes .
more votes.
6. GI]The y‐intercept is the value
They-intercept value of y when case, it's
value of x is 0. In this case,
the value
when the average shopping
it’s the average time
shopping time
when the
when 0% (no
discount is 0°/o
store discount
the store discount).
(no discount).
7. GI]The slope is rise
The slope rise over
over run. line of best
the line
Because the
run. Because has a
best fit has positive slope,
a positive slope, it’s the increase
it's the revenue
increase in revenue
for every
every dollar
dollar increase
increase in advertising
advertising expenses.
expenses. Note
Note that
that because
because both
both revenue
revenue and
and advertising
advertising expenses
expenses
are expressed
are expressed in thousands
thousands of dollars graph, they
the graph,
dollars in the they cancel out
out and no effect on the interpretation
have no
and have interpretation
of the slope. That's why
slope . That's why the answer isn’t
the answer (8).
isn't (B).

slope is rise over


8. []] The slope Because the line of best
run.. Because
over run best fit has a positive
fit has positive slope, increase in box office
slope, it's the increase
sales per minute
sales per minute increase length.
movie length.
increase in movie

[QJThe y‐intercept
9. @ y-intercept is the value when the
value of y when value of x is 0. ln
the value In this case, it’s the expected
case, it's number of
expected number
mistakes
mistakes made when the cash
made when prize 0 dollars (no cash prize).
cash prize is O dollars (no cash prize) .

10. [I]This question asking for the slope


question is asking slope of the best fit. At 20 grams
line of best
the line there are
grams of fat, there calories . At
are 340 calories.
25 grams of fat, there
25 grams there are calories. Calculating
are 380 calories. the slope
Calculating the w o points,
slope from ttwo points,

380 -‐ 340 _ 40 _
= 419 = 8
8
25 -‐ 20
20 _ 5 _

11. [I]Apply learn from the sample


you learn
what you
Apply what larger population.
sample to the larger From the sample
population . From size,
sample size,
Car Speeding
Car Speeding Violations _ 83 . .
Total Vi
li Violations
.
ota I 10 1ations
=
‐ @' . N
284
can apply
o w we can
Now apply this same proportion to the
same proportion total of 2,000:
state total
the state 2,000:

83
282 Xx2,000~585
2,000 ~ 585
284

12. [I]The oat


oat field whose yield is best
whose yield best predicted
predicted by the line
line of best
best fit is represented
represented by the point closest to
point closest
the line. That point
line. That has an
point has which is the amount
x-value of 350, which
an x-value nitrogen applied
amount of nitrogen that field.
applied to that

340
THE COLLEGE PANDA
COLLEGE PANDA

[Q]The point
13. IE point farthest
farthest from the line of best
the line best fit is at an x~value of 7. The
an x-value The total number of
total number seats at
of seats the food
at the food
court represented
court represented by this point 80 = 560.
point is 7 x 80

14. [Q]
[B To draw
draw a reliable conclusion
a reliable the effectiveness
about the
conclusion about effectiveness of the
the n vaccine, the patients
e w vaccine,
new be
m u s t be
patients must
randomly assigned to their
randomly assigned treatment. Only
their treatment. Only answer leads to
answer (D) leads random assignment.
to random assignment. NoteNote that answer
that answer
(C) does
does not because the patients
not because are allowed
patients are group themselves
allowed to group themselves asas they desire . For example,
they desire. example, three
three
friends might
friends want to remain
might want the same
remain in the group, leading
same group, leading to assignment that is
assignment that n o t random.
is not random.

15. [g Answer (A) is wrong


Answer wrong because possible that
it’s possible
because it's most of the
that most the basketballs produced in
basketballs produced in Week
Week 11 had an
had an
air pressure of over 8.2 psi.
air pressure Likewise for Week 2. We don’t
psi. Likewise know for sure
don't know Answer (B) is
sure.. Answer wrong because
is wrong because
it’s too definite.
it's too definite. Just because the
Just because sample means
the sample were 0.5 psi
means were apart doesn’t
psi apart mean the
doesn't mean the true
true means, which
means , which
would take
would into account
take into the basketballs
account all the produced in Week 1 and
basketballs produced and Week 2, were
were also also 0.5 psi
psi apart.
apart.
That’s why
That's there 's aa margin
why there’s margin of error samples.. Answer
the samples
error for the Answer (D) is is wrong because the
wrong because the samples suggest
samples suggest
the
the reverse: mean air
reverse: the mean afr pressure
pressure for Week 1 (8.2 psi)
psi) is greater
greater than
than the
the m a i r pressure
e a n air
mean pressure forfor Week 2
(7.7 psi).
psi). Answer (C) because the
correct because
(C) is correct greater the sample
the greater size,
sample size, the lower the margin
lower the margin of The
error. The
sample
sample from Week 11 had had a
a lower margin of error
lower margin error than
than the sample
sample from Week 2.
2.

16. lElThe
[Q]The lower
lower the standard deviation
the standard (variability), the
deviation (variability), the lower margin of
lower the margin error. Selecting
of error. Selecting students
students who
who
are following
are following the same daily
the same daily diet plan will
diet plan lead to the
will likely lead the lowest
lowest standard deviation because
standard deviation they are
because they are
likely to be
be eating
eating the number of servings
same number
the same servings of vegetables. other answer
The other
vegetables . The answer choices would result
choices would in
result in
much more variability.
much more variability .

17. [g Answer (C) best


Answer (C) expresses the
best expresses meaning of a
the meaning confidence interval,
a confidence which applies
interval, which applies only
only to the statistical
the statistical
mean
m and does
e a n and n o t say anything
does not about blue-spotted
anything about blue-spotted salamanders
salamanders themselves. Answer (D) is
themselves . Answer wrong
is wrong
because
because the
the study involved only
study involved blue-spotted
only blue o t all salamanders.
salamanders, nnot
-spotted salamanders, salamanders .

18. I]] The most that we can conclude


most that there is a negative
that there
conclude is that association between
negative association the price
between the food and
price of food and
population density
the population density in U S . cities (as one
U.S. goes up,
one goes other goes
the other
up, the goes down).
down). WeWeCANNOT
CANNOT conclude
conclude that
that
there is aa cause
there cause and relationship between
and effect relationship between the two. We can’t say that one causes the other.
the two . We can't say that one causes the other.

341
CHAPTER
CHAYfER 30 ANSWERS
ANSWERS TO THE EXERCISES
EXERCISES

Chapter
Chapter29: Volume
CHAPTER EXERCISE:
CHAPTER EXERCISE:

1.
1. 0 Each piece is half
Each piece the cylinder.
half the cylinder. 5. \ 32.5 IThe width
width of the brick is 6(1.25) =
the brick : 7.5
inches, and its height
inches, and height is 6 -‐ 2 = z 4 inches.
inches . The
brick’s volume is then
V‐_12m 2h‐_127r(2)2(5) ‐107r
_ brick's volume then
lwh == (6)(7.5)(4)
lwh (6)(7.5 )(4) =: 180 cubic inches. 1
inches. Since 1
kg = 1000 grams,
grams, the brick's
brick's mass
mass is 5,850
2. [!]The height grams. Finally, the
grams. Finally, brick's density
the brick’s density is
height of the + 225
the box is 100 ...;- 5=z 44
mass _ 5,850
mass 5,850 grams _
(dividing the
(dividing volume by
the volume by the area of the
the area the base
base volume
volume = - - ‐180 1grams
8 0 =‐ 32.5
in3
in3 32.5 grams per
grams per
gives us
gives us to the
the height).
height). The sides of
sides the the base
are x/E
are v'25= 5 inches
inches long. rectangular box
long . The rectangular cubic inch.
inch.
has dimensions
has dimensions 5 x 5 x 4. [illThis question
6. El question is asking volume of
asking for the volume
the cylinder.
the radius of the base
The radius
cylinder . The base is 2 and
and
since the diameter
since diameter of each tennis ball
each tenni s ball is 4, the
4
height the cylinder
height of the cylinder is 44 x 3
3 = 12.

vV =: rrr 2 h =
7Tr2h = n(2)2(12)
rr(2)2 (12) = 487r
487T
5
5

The top and bottom have


and bottom have a a surface area of
surface area 7. ~ The shortest way to do this question
shortest way question is to
2(5 x 5) = 50. The front and
and back have
have aa pretend that
pretend that the block
block is liquefied and poured
liquefied and poured
surface area
surface left and
area of 2(5 x 4) = 40. The left and into the aquarium.
into How high
aquarium. How high would
would the level
the level
right have a
right have a surface area of 2(5 x 4) =
surface area z 40. of the liquid
liquid rise?
The total surface area
total surface 50 +
area is 50 40 +
+ 40 40 = 130.
+ 40
V = lwh
lwh
3. [f] Let the side of the cube
the side cube be 5. The cube
be s. has
cube has 5,000 = (80)(25)h
(80) (25)11
six faces and
and the area 2.
each face is s$2.
area of each
Solving for ss in terms
Solving terms of a, 2.5 =
: h

The longer way to do this question


longer way is to
question is to find
6s2 = 24112
652 24a2
the original volume, add
original volume, block, find the
the block,
add the the
s522 = 4a 2
= 4112 new height,
new and then
height, and compare it to the
then compare
s = 2a
2a original height.
original height. While nnot the fastest
o t the fastest method,
method,
it is certainly viable.
certain ly viab le.
volume is then s3 = (2a)3 =: 8a3.
then 53=
The volume
8. 0 If you take
If you take away the cubes
away all the cubes with black
with black
4. E]
[[] The cylindrical
cylindrical tank that can
tank that be filled in 3
can be paint on them,
paint them, you
you are essentially uncovering
are essentially uncovering
hours has aa volume
hours has volume ofof rr(4) 2 (6) = 96rr.
7t(4)2(6) 967t. The an
an inner
inner cube with a
cube with side length
a side length of 3. A front
tank in question
tank has aa volume
question has volume of view is shown
shown below.
below .
7T(6)2(8)
rr(6)2(8) = 28871. Using the
288rr. Using the first tank asa
tank as a
conversion factor,
conversion
.-- - -
I
~
I

hours
3 hours I
3
I

28871 x ---
2887T = 9 hours
hours ,_- - - .,
I I

9671
967T
There 33 =
There are 33 z 27 cubes that are
cubes that unpainted..
are unpainted

342
THE
THE COLLEGE PANDA
COLLEGE PANDA

9. ~ Since
9. Since each small cube
each small has a
cube has volume of
a volume
22‘ =
= 8 and
and the volume of the
the volume the outer box is
outer box
8833
=
z 512, there +8=
be 512 -i-
m u s t be
there must cubes in
= 64 cubes 1
V rrr 2h
V = émzh
box. If you
the box.
the take away
you take all the
away all the cubes
cubes that
that 3
touching the
are touching
are the box,
box, youyou are essentially
are essentially 1
uncovering an
uncovering an inner rectangular box
inner rectangular with aa
box with 6na 4
67m4 rrr 2 (2a 2 )
= -énr2(2a2)
3
square base
square base of side
side 4 and height of 6. A
and aa height
187m44 =
18na 2 rrr 2
(2a 2 )
7rr2(2a2)
front view
front below.
shown below.
view is shown
4
187m4
18na =
= 2rra 2 2
27mzr2r
9a22 =
9a r2
: r2
8
I 3aa== r
3
61
I
I
4 I
•------~ 12. [I] Draw line down
Draw aa line the middle
down the middle of the
the cone
cone
8
8 to form
form a right triangle
a right with the
triangle with the radius
radius and
and
the slant
the slant height. This triangle
height. This is a
triangle is multiple of
a multiple
The volume of this
The volume this inner rectangular box
inner rectangular is
box is right triangle:
3‐4‐5 right
the 3--4-5
the triangle: 9‐12‐15.
9-12-15. You
4x4x6= : 96. Since each cube
Since each has aa volume
cube has volume could’ve
could've used pythagorean theorem
the pythagorean
used the theorem
of 8, there
there are 96 + 8 =
are 96...,... 12cubes
= 12 that are
cubes that not
are not instead if
instead you weren't
if you this. In any
aware of this.
weren’t aware any
touching
touching thethe box, which means
box, which there are
means there are case, the
case, height of the
the height cone is 12
the cone 12cm.
cm.
64‐- 12
64 12== 52
52 cubes
cubes that
that are touching. You
are touching.
could’ve taken
also could've
also taken the straight-forward
the straight-forward V=
: volume cone +
volume of cone -+- volume hemisphere
volume of hemisphere
approach of counting
approach counting up up the
the cubes along the
cubes along the
you took
sides. If you
sides. this route,
took this route, you should’ve
you should've V_1!rrr 22h + !1(4
V‐37rrh+§
= 5m33)
(~rrr )
3 2 3
gotten something along
gotten something along the lines of
the lines
1 6++16
16 1 6++88++88+ + 4 = 52. v = ~n(9)
V
2
%n(9)2(12) ~
(12) + %(§n(9)3>
3
(;rr (9) )
10. IE]The
[Q] only cubes
The only cubes that exactly one
have exactly
that have one face
painted black
painted black are the ones
are the ones in the middle of
the middle V = 3247T+
= 32471' 48671 =
+ 486n = 81071
810n
each side.. For
each side example, the
For example, the front side has
front side has
3x 1= = 3 of these
these cubes.
cubes.
13. ~ Since radius rr is 2 inches
the radius
Since the longer than
inches longer than
the height,
the height , the height is r -‐ 2.
the height Using the
2. Using the
3
volume formula for aacylinder,
volume formula cylinder, wewe get
get
:-------;1 V = nr
V 2h =
7rr2h r 2 (r -‐ 2) = 7rr3
: n7rr2(r nr 3 -‐ 2nr 2
27rr2..
, _ - - - - - _1

3 14. []] This question is essentially


This question essentially asking
asking for the the
4
5 volume, or the
volume, amount of rroom
the amount the crate.
o o m, in the crate.
The room in the
The room the crate
crate can
can be
be seen
seen asas a
The right side
The right has 2 x 1 = 2 of these
side has these cubes,
cubes, rectangular
rectangular box with a
box with length of
a length
and the
and the top has 3 x 2 =
top has = 6 of these
these cubes.
cubes. So
So 1 =i 8n inches,
100-‐ 11 ‐- 11: 8 c h e s ,aawidth h o8f-8 ‐11- ‐ 11 =
w i d t of : 66
have 3 +
far, we have + 2++6= z 11 of these
these cubes.
cubes. To inches, and
inches, height of 3 -‐ 1 = 2 inches.
and a height inches.
account
account for thethe back,
back, left, and bottom
left, and bottom sides,
sides,
we double
we double this
this to get 22 cubes.
get 22 cubes. V :=8 >
8 <X 66 >X<22=: 996
6

343
CHAPTER 30 ANSWERS
CHAPTER ANSWERS TO THE EXERCISES
THE EXERCISES

15. [I] Cut staircase vertically


Cut the staircase blocks .
into 3 blocks.
vertically into

staircase =
Volume of staircase block 1
= Volume of block
+
+ Volume of block
block 2
+
+ Volume of block
block 3

V=(5x2x0.2)+(5x2x0.4)+(5x2x0.6)
V = (5 X 2 X 0.2) + (5 X 2 X 0.4) + (5 X 2 X 0.6)
= 22 + 4 + 6
=
= 112
2

Mass =
Mass = Density
Density x Volume
= 130 x 12 =
= 1,
1,560
560 kg

344

You might also like